You are on page 1of 293

fiziks

Institute for NET/JRF, GATE, IIT‐JAM, JEST, TIFR and GRE in PHYSICAL SCIENCES 
 

fiziks
Forum for CSIR-UGC JRF/NET, GATE, IIT-JAM/IISc,
JEST, TIFR and GRE in
PHYSICS & PHYSICAL SCIENCES

Modern Physics

(IIT-JAM/JEST/TIFR/M.Sc Entrance)

                                                                                
Head office  Branch office 
 
fiziks, H.No. 23, G.F, Jia Sarai,  Anand Institute of Mathematics, 
 
Near IIT, Hauz Khas, New Delhi‐16  28‐B/6, Jia Sarai, Near IIT 
 
Phone: 011‐26865455/+91‐9871145498 Hauz Khas, New Delhi‐16 
                                                   
                                             Website: www.physicsbyfiziks.com                                                                                          
                                                           Email: fiziks.physics@gmail.com                                                                   
fiziks
Institute for NET/JRF, GATE, IIT‐JAM, JEST, TIFR and GRE in PHYSICAL SCIENCES 
 
Content Modern Physics
1. Special Theory of Relativity……………………………………………………..(1-36)
1.1 Galilean Transformations
1.2 Lorentz Transformation
1.2.1 Postulates of Special Theory of Relativity
1.2.2 Derivation of Lorentz Transformation
1.3 Consequences of Lorentz Transformation
1.3.1 Length Contraction
1.3.2 Time Dilation
1.4 Relative Velocity
1.5 Relativistic Mass and Momentum
1.6 Relativistic Second Law of Motion
1.7 Relativistic Energy
1.8 The Doppler Effect in Light
1.8.1 Transverse Doppler Effect in Light
1.8.2 Observer and Source Moving Apart
1.8.3 Observer and Source Moving Together
1.9 Four Vectors and Relativistic Invariance
2. Modern Physics…………………………………………………………..……..(37-87)
2.1 Black Body Radiation

2.1.1 Wien’s Distribution Law

2.1.2 Rayleigh’s Energy Density Distribution

2.1.3 Planks Radiation Formula

2.2 Photo Electric Effect

2.2.1 Threshold Frequency and Work Function

2.3 Compton Scattering


2.4 Bohr Atomic Model

                                                                                
Head office  Branch office 
 
fiziks, H.No. 23, G.F, Jia Sarai,  Anand Institute of Mathematics, 
 
Near IIT, Hauz Khas, New Delhi‐16  28‐B/6, Jia Sarai, Near IIT 
 
Phone: 011‐26865455/+91‐9871145498 Hauz Khas, New Delhi‐16 
                                                   
                                             Website: www.physicsbyfiziks.com                                                                                          
                                                           Email: fiziks.physics@gmail.com                                                                   i 
fiziks
Institute for NET/JRF, GATE, IIT‐JAM, JEST, TIFR and GRE in PHYSICAL SCIENCES 
 
2.5 Wave Particle Duality
2.5.1 De Broglie Wave
2.5.2 Uncertainty principle
2.5.3 Superposition Principle

Questions and Solutions

3. Tools and Postulates of Quantum Mechanics………………………………(88-133)


3.1 The Linear Vector Space
3.1.1 Scalar Product
3.1.2 Hilbert Space
3.1.3 Dimension and Basis of a Vectors.
3.1.4 Square Integrable Function
3.2 Dirac Notation
3.3 Operator
3.3.1 Linear Operator
3.3.2 Matrix Representation of Operator
3.3.3 Eigen Value of Operator
3.3.4 Correspondence Between Ket and Bra

3.3.5 Hermitian operator

3.3.6 Commutator

3.3.7 Set of Commuting Observables

3.3.8 Projection operator

3.4 Postulates of Quantum Mechanics

3.4.1 Expectation Value

3.4.2 Fourier transformation

Questions and Solutions

                                                                                
Head office  Branch office 
 
fiziks, H.No. 23, G.F, Jia Sarai,  Anand Institute of Mathematics, 
 
Near IIT, Hauz Khas, New Delhi‐16  28‐B/6, Jia Sarai, Near IIT 
 
Phone: 011‐26865455/+91‐9871145498 Hauz Khas, New Delhi‐16 
                                                   
                                             Website: www.physicsbyfiziks.com                                                                                          
                                                           Email: fiziks.physics@gmail.com                                                                   ii 
fiziks
Institute for NET/JRF, GATE, IIT‐JAM, JEST, TIFR and GRE in PHYSICAL SCIENCES 
 
4. Application of Quantum Mechanics in Cartesian coordinate…………….(134-183)

4.1 One Dimensional System


4.2 Current Density (J)
4.3 Free Particle in One Dimension
4.4 The Step Potential
4.5 Particle in a One Dimensional Box
4.6 Infinite Symmetric Potential Box
4.7 Square Well Finite Potential Box
4.8 Harmonic Oscillator (Parabolic potential)
4.9 Multiple Dimensional System
4.10 Two Dimensional Free Particle
4.11 Three Dimensional Free Particle
4.12 Particle in Two Dimensional Box
4.13 Particle in Three Dimensional Box
4.14 Two Dimensional Harmonic Oscillator
Questions and Solutions

5. Statistical Physics……………..……………………………………….……..(184-229)
5.1 Basic Definition, Mathematical Tools and Postulates of Statistical Mechanics
5.2 Postulates of statistical mechanics
5.2.1 Micro canonical ensemble  
5.2.1.1 Entropy
5.2.2 Canonical Ensemble
5.2.2.1 Partition Function for Canonical Ensemble
5.2.2.2 Relation between Macroscopic Variable and Canonical
Partition Function Z
5.2.2.3 Relation between Entropy and Probability 
5.2.3 Grand canonical ensemble
                                                                                
Head office  Branch office 
 
fiziks, H.No. 23, G.F, Jia Sarai,  Anand Institute of Mathematics, 
 
Near IIT, Hauz Khas, New Delhi‐16  28‐B/6, Jia Sarai, Near IIT 
 
Phone: 011‐26865455/+91‐9871145498 Hauz Khas, New Delhi‐16 
                                                   
                                             Website: www.physicsbyfiziks.com                                                                                          
                                                           Email: fiziks.physics@gmail.com                                                                   iii 
fiziks
Institute for NET/JRF, GATE, IIT‐JAM, JEST, TIFR and GRE in PHYSICAL SCIENCES 
 
5.3 Maxwell-Boltzmann distribution 
5.3.1 Energy Distribution Function
5.4 Bose Einstein distribution
Questions and Solutions

Nuclear Physics
6. Basic Nuclear Properties………………………………………………….....(230-242)
6.1 Size and Density
6.2 Spin and Magnetic Moment
6.3 Angular Momentum of Nucleus

6.4 Stable Nuclei

6.5 Binding Energy


6.6 Salient Features of Nuclear Forces
7. Radio Active Decay…………………………………………………………..(243-255)
7.1 Alpha Decay

7.2 Beta Decay

7.3 Gamma Decay

7.3.1 Various Processes by which -rays can lose its Energy


7.3.2 Internal Conversion

7.3.3 Pair Production (Energy into Matter)

7.3.4 Pair Annihilation


7.3.5 Massbauer Effect
7.4 Activity
7.4.1 Successive Growth and Decay Process
7.4.2 Branching
7.4.3 Determination of the Age of the Earth

                                                                                
Head office  Branch office 
 
fiziks, H.No. 23, G.F, Jia Sarai,  Anand Institute of Mathematics, 
 
Near IIT, Hauz Khas, New Delhi‐16  28‐B/6, Jia Sarai, Near IIT 
 
Phone: 011‐26865455/+91‐9871145498 Hauz Khas, New Delhi‐16 
                                                   
                                             Website: www.physicsbyfiziks.com                                                                                          
                                                           Email: fiziks.physics@gmail.com                                                                   iv 
fiziks
Institute for NET/JRF, GATE, IIT‐JAM, JEST, TIFR and GRE in PHYSICAL SCIENCES 
 
8. Nuclear Reaction……………………………………….……………...……..(256-264)
8.1 Conservation Laws
8.2 Nuclear Reaction Kinematics  Q  Value 
8.2.1 General Solution of Q -Equation
8.2.2 Exothermic Reaction  Q  0 

8.2.3 Endothermic Reaction  Q  0 


8.3 Nuclear Fission
8.4 Nuclear Fusion in Stars
8.4.1 Proton– Proton Cycle
8.4.2 Carbon-Cycle
Questions and Solutions

                                                                                
Head office  Branch office 
 
fiziks, H.No. 23, G.F, Jia Sarai,  Anand Institute of Mathematics, 
 
Near IIT, Hauz Khas, New Delhi‐16  28‐B/6, Jia Sarai, Near IIT 
 
Phone: 011‐26865455/+91‐9871145498 Hauz Khas, New Delhi‐16 
                                                   
                                             Website: www.physicsbyfiziks.com                                                                                          
                                                           Email: fiziks.physics@gmail.com                                                                   v 
fiziks
Institute for NET/JRF, GATE, IIT‐JAM, JEST, TIFR and GRE in PHYSICAL SCIENCES 
 
1. Special Theory of Relativity
1.1Galilean Transformations
S S′

v
( x, y , z , t )
A ( x′, y′, z ′, t ′ )
vt x′

A frame S' which is moving with constant velocity v relative to an inertial frames S ,
which is itself inertial.
r ' = r − vt , x ' = x − vt , y ' = y, z ' = z , t ' = t
The above transformation of co-ordinates from one inertial frame to another and they are
referred as Galilean transformations.
And inverse Galilean transformation is given by
x = x '+ vt , y = y ', z = z ', t = t '
The velocity transformation is given
dr dr '
r = r ′ + vt ⇒ =v+ ⇒u = v + u'
dt dt
d 2r d 2r'
The acceleration transformation is given = 2
dt 2 dt
It is found that acceleration measured on both frame is same. So it is inertial frame.
When velocity transformation is analyzed as u ' = c where c is velocity of light.
u = c+v.
It is seen velocity of light is depended on the reference frame which is physically not
accepted. So for high velocity v ≈ c Galilean transformation is not adequate.
So for velocity u ≈ c there is need for different transformation, which is given by Lorentz
transformation.
                                                                                
Head office  Branch office 
 
fiziks, H.No. 23, G.F, Jia Sarai,  Anand Institute of Mathematics, 
 
Near IIT, Hauz Khas, New Delhi‐16  28‐B/6, Jia Sarai, Near IIT 
 
Phone: 011‐26865455/+91‐9871145498 Hauz Khas, New Delhi‐16 
                                                   
                                             Website: www.physicsbyfiziks.com                                                                                          
                                                           Email: fiziks.physics@gmail.com                                                                   1 
fiziks
Institute for NET/JRF, GATE, IIT‐JAM, JEST, TIFR and GRE in PHYSICAL SCIENCES 
 
1.2 Lorentz Transformation
1.2.1 Postulates of Special Theory of Relativity
(i) There is no universal frame of reference pervading all of space, so there is no such
thing as “absolute motion”.
(ii) The law of physics are the same in all frames of reference moving at constant velocity
to one another.
(iii) The speed of light in free space has the same value for all inertial observers.
1.2.2 Derivation of Lorentz Transformation
Lorentz transformation have to such that
(a) It is linear in x and x′ so that a single event in frame S corresponds to a single event
in frame S' .
(b) For lower velocity it reduces to Galilean transformation.
(c) The inverse transformation exists.
Let us assume
x′ = k ( x − vt ) , x = k ( x ′ + vt ′ ) , y ′ = y and z ′ = z

Put the value x ′ = k ( x − vt ) in x = k ( x ′ + vt ) one will get

⎛1− k 2 ⎞
x = k 2 ( x − vt ) + kvt ′ and t ' = kt + ⎜⎜ ⎟⎟ x
⎝ kv ⎠
Now, x = ct in S frame and x ' = ct ' in S' frame
⎛ 1− k 2 ⎞
k ( x − vt ) = ckt + ⎜ ⎟ cx
⎝ kv ⎠
Solving these equation for k then
1
k=
v2
1− 2
c

                                                                                
Head office  Branch office 
 
fiziks, H.No. 23, G.F, Jia Sarai,  Anand Institute of Mathematics, 
 
Near IIT, Hauz Khas, New Delhi‐16  28‐B/6, Jia Sarai, Near IIT 
 
Phone: 011‐26865455/+91‐9871145498 Hauz Khas, New Delhi‐16 
                                                   
                                             Website: www.physicsbyfiziks.com                                                                                          
                                                           Email: fiziks.physics@gmail.com                                                                   2 
fiziks
Institute for NET/JRF, GATE, IIT‐JAM, JEST, TIFR and GRE in PHYSICAL SCIENCES 
 

Lorentz Transformation Inverse Lorentz Transformation


x − vt x '+ vt
x' = x=
2
v v2
1− 2 1− 2
c c
y' = y y = y'
z' = z z = z'
vx vx'
t− t '+
t' = c2 t= c2
v2 v2
1− 2 1− 2
c c

1.3 Consequences of Lorentz Transformation


1.3.1 Length Contraction
In order to measure the length of an object in motion, relative to observer, the position of
two end points recorded simultaneously, the length of object in direction of motion
y
appeared smaller to observer. y'
v

l0 = x '2 − x '1 , l = x2 − x1

x '2 − x '1 = γ ( x1 − vt ) − γ ( x2 − vt )

(x' 2 − x'1 ) = γ (x2 − x1 ) x x '1 x '2


x2 − x1
(x'2 − x'1 ) =
1 − v2 / c2 z z'

v2 v2
(x2 − x1 ) = 1− (x'2 − x'1 ) ⇒ l = lo 1 −
c2 c2
Thus l < l 0 , this means that the length of rod as measured by an observer relative to
which rod is in motion, is smaller than its proper length.
Such a contraction of length in direction of motion relative to observer is called Lorentz
Fitzgerald contradiction.

                                                                                
Head office  Branch office 
 
fiziks, H.No. 23, G.F, Jia Sarai,  Anand Institute of Mathematics, 
 
Near IIT, Hauz Khas, New Delhi‐16  28‐B/6, Jia Sarai, Near IIT 
 
Phone: 011‐26865455/+91‐9871145498 Hauz Khas, New Delhi‐16 
                                                   
                                             Website: www.physicsbyfiziks.com                                                                                          
                                                           Email: fiziks.physics@gmail.com                                                                   3 
fiziks
Institute for NET/JRF, GATE, IIT‐JAM, JEST, TIFR and GRE in PHYSICAL SCIENCES 
 
1.3.2 Time Dilation
When two observers are in relative uniform motion and uninfluenced by any gravitational
mass, the point of view of each will be that the other's (moving) clock is ticking at
a slower rate than the local clock. The faster the relative velocity, the greater the
magnitude of time dilation. This case is sometimes called special relativistic time
dilation.
A clock being at rest in the S' frame measures the time t '2 and t1' of two events

occurring at a fixed position x' . The time interval Δt measures from S frame appears
slow ( Δt0 ) from S' frame i.e. to the observer the moving clock will appear to go slow.

⎛ vx' ⎞ ⎛ vx' ⎞
Δt ' = t '2 − t '1 = Δt0 ⇒ Δt = t2 − t1 = γ ⎜ t '2 + 2 ⎟ − γ ⎜ t1 + 2 ⎟
⎝ c ⎠ ⎝ c ⎠
t '2 − t '1 Δ t0
⇒ Δt = =
v2 v2
1− 1−
c2 c2
Δt0 → Proper time, Δt → time interval measured from S frame Δt > Δt0

1.4 Relative Velocity


There is one inertial frame S , S ′ is another inertial frame moving with respect to S in
x -direction and A is another inertial frame which is moving with respect to S ′ with

(
velocity component u x′ , u ′y , u z′ . ) S S′ A
dx ' dy ' dz '
v (ux′ , u ′y , uz′ )
So, u ' x = u 'y = u 'z =
dt ' dt ' dt '
The velocity component of A from S frame is given by
dx dy dz x
ux = , uy = , uz =
dt dt dt
From the inverse Lorentz Transformation,
⎛ vx ' ⎞ 1
x = γ ( x '+ vt ' ) , y = y ', z = z ', t = γ ⎜ t '+ 2 ⎟ where γ =
⎝ c ⎠ v2
1−
c2

                                                                                
Head office  Branch office 
 
fiziks, H.No. 23, G.F, Jia Sarai,  Anand Institute of Mathematics, 
 
Near IIT, Hauz Khas, New Delhi‐16  28‐B/6, Jia Sarai, Near IIT 
 
Phone: 011‐26865455/+91‐9871145498 Hauz Khas, New Delhi‐16 
                                                   
                                             Website: www.physicsbyfiziks.com                                                                                          
                                                           Email: fiziks.physics@gmail.com                                                                   4 
fiziks
Institute for NET/JRF, GATE, IIT‐JAM, JEST, TIFR and GRE in PHYSICAL SCIENCES 
 
Differentiating both sides, we get
⎛ vdx ' ⎞
dx = γ ( dx '+ vdt ') , dy = dy ', dz = dz ', dt = γ ⎜ dt '+ 2 ⎟
⎝ c ⎠

⎛ ⎞ dx '
⎜ ⎟ +v
dx dx '+ vdt ' dt ' u 'x + v
ux = =γ ⎜ ⎟= ⇒ ux =
dt vdx ' v dx ' v
⎜⎜ dt '+ 2 ⎟⎟ 1 + 2 1 + 2 u 'x
⎝ c ⎠ c dt ' c
dy'
dy dy' dt ' u 'y
uy = = = ⇒ uy =
dt ⎛ vdx' ⎞ ⎛ v dx' ⎞ ⎛ vu x' ⎞
γ ⎜ dt '+ 2 ⎟ γ ⎜1 + 2 ⎟ γ ⎜1 + ⎟
⎝ c ⎠ ⎝ c dt ' ⎠ ⎝ c2 ⎠

dz '
dz dz ' dt ' u z'
uz = = = ⇒ uz =
dt ⎛ vdx ' ⎞ ⎛ v dx ' ⎞ ⎛ vu x' ⎞
γ ⎜ dt '+ 2 ⎟ γ ⎜ 1 + 2 ⎟ γ ⎜1 + 2 ⎟
⎝ c ⎠ ⎝ c dt ' ⎠ c ⎠

ux − v uy uz
Similarly u′x = , u′y = , u ′z =
vu x ⎛ vu x ⎞ ⎛ vu ⎞
1− 2 γ ⎜1 − 2 ⎟ γ ⎜1 − 2x ⎟
c ⎝ c ⎠ ⎝ c ⎠
1.5 Relativistic Mass
The mass of a body moving at the speed v relative to an observer is larger than its mass
1
when at rest relative to the observer by the factor .
v2
1− 2
c m
m0 m0
Thus m=
v2
1−
c2 v
v/c

1.0 c
where m0 is rest mass of body and m is observed mass.

Relativistic Momentum
m0 v
p = mv =
v2
1−
c2
                                                                                
Head office  Branch office 
 
fiziks, H.No. 23, G.F, Jia Sarai,  Anand Institute of Mathematics, 
 
Near IIT, Hauz Khas, New Delhi‐16  28‐B/6, Jia Sarai, Near IIT 
 
Phone: 011‐26865455/+91‐9871145498 Hauz Khas, New Delhi‐16 
                                                   
                                             Website: www.physicsbyfiziks.com                                                                                          
                                                           Email: fiziks.physics@gmail.com                                                                   5 
fiziks
Institute for NET/JRF, GATE, IIT‐JAM, JEST, TIFR and GRE in PHYSICAL SCIENCES 
 
1.6 Relativistic Second Law of Motion
⎛ ⎞
⎜ ⎟
dp d d ⎜ m0 v ⎟
F= = ( mv ) = ⎜ ⎟
dt dt dt ⎜ v2 ⎟
⎜ 1 − ⎟
⎝ c2 ⎠
1.7 Relativistic Energy
Einstein suggested if m is relativistic mass of body then relativistic energy E is given
by
m0 c 2
E = mc 2
and E=
v2
1− 2
c

Rest Energy
If rest mass of particle is m0 then rest mass energy is given by mo c 2

Relativistic Kinetic Energy


m0c 2
K = mc 2 − m0 c 2 ⇒K= − m0 c 2
2
v
1−
c2
Relationship between total Energy and Momentum
m0 c 2 m0 v
E= and p =
2
v v2
1− 1−
c2 c2

⇒ E 2 = m02 c 4 + p 2 c 2

( )
1/ 2
⇒ E = m02 c 4 + p 2 c 2

                                                                                
Head office  Branch office 
 
fiziks, H.No. 23, G.F, Jia Sarai,  Anand Institute of Mathematics, 
 
Near IIT, Hauz Khas, New Delhi‐16  28‐B/6, Jia Sarai, Near IIT 
 
Phone: 011‐26865455/+91‐9871145498 Hauz Khas, New Delhi‐16 
                                                   
                                             Website: www.physicsbyfiziks.com                                                                                          
                                                           Email: fiziks.physics@gmail.com                                                                   6 
fiziks
Institute for NET/JRF, GATE, IIT‐JAM, JEST, TIFR and GRE in PHYSICAL SCIENCES 
 
1.8 The Doppler Effect in Light
The Doppler Effect in sound evidently varies depending on whether the source, or the
observer, or both are moving, which appears to violate the principle of relativity: all that
should count is the relative motion of source and observer. But sound waves occur only
in a material medium such as air or water, and this medium is itself a frame of reference
with respect to which motions of source and observer are measurable. Hence there is no
contradiction. In the case of light, however, no medium is involved and only relative
motion of source and observer is meaningful. The Doppler Effect in light must therefore
differ from that in sound.
We can analyze the Doppler Effect in light by considering a light source as a clock that
ticks ν 0 times per second and emits a wave of light with each tick. We will examine the
three situations shown in figure given below.

1.8.1 Transverse Doppler Effect in Light


The observer is moving perpendicular to a line between him and the light source. The
proper time between ticks is t0 = 1/ν 0 , so between one tick and the next time

t = t0 / 1 − v 2 / c 2
elapses in the reference frame of the observer.
The frequency he finds is accordingly

1 1 − v2 / c2
ν ( transverse ) = =
t t0

The observed frequency ν is always less than the source frequencyν 0 .

                                                                                
Head office  Branch office 
 
fiziks, H.No. 23, G.F, Jia Sarai,  Anand Institute of Mathematics, 
 
Near IIT, Hauz Khas, New Delhi‐16  28‐B/6, Jia Sarai, Near IIT 
 
Phone: 011‐26865455/+91‐9871145498 Hauz Khas, New Delhi‐16 
                                                   
                                             Website: www.physicsbyfiziks.com                                                                                          
                                                           Email: fiziks.physics@gmail.com                                                                   7 
fiziks
Institute for NET/JRF, GATE, IIT‐JAM, JEST, TIFR and GRE in PHYSICAL SCIENCES 
 
1.8.2 Observer and Source Moving Apart
The observer is receding from the light source. Now the observer
travels the distance vt away from the source between ticks,
which mean that the light wave from a given tick takes vt / c
longer to reach him than the previous one. Hence the total time
between the arrivals of successive waves is

vt 1+ v / c 1+ v / c 1+ v / c 1+ v / c
T =t+ = t0 = t0 = t0
c 1 − v2 / c2 1+ v / c 1− v / c 1− v / c

and the observed frequency is


1 1 1− v / c 1− v / c
ν ( receding ) = = =ν0
T t0 1 + v / c 1+ v / c

the observed frequency ν is lower than the source frequency ν 0 . Unlike the case of
sound waves, which propagate relative to a material medium, it makes no difference
whether the observer is moving away from the source or the source is moving away from
the observer.

1.8.3 Observer and Source Moving Together


The observer is approaching the light source. The observer here
travels the distance ν t toward the source between ticks, so each
light wave takes ν t / c less time to arrive than the previous one.
In this case T = t −ν t / c and the result is

1 +ν / c
ν ( approaching ) = ν 0
1 −ν / c
The observed frequency is higher than the source frequency. Again, the same formula
holds for motion of the source toward the observer.

                                                                                
Head office  Branch office 
 
fiziks, H.No. 23, G.F, Jia Sarai,  Anand Institute of Mathematics, 
 
Near IIT, Hauz Khas, New Delhi‐16  28‐B/6, Jia Sarai, Near IIT 
 
Phone: 011‐26865455/+91‐9871145498 Hauz Khas, New Delhi‐16 
                                                   
                                             Website: www.physicsbyfiziks.com                                                                                          
                                                           Email: fiziks.physics@gmail.com                                                                   8 
fiziks
Institute for NET/JRF, GATE, IIT‐JAM, JEST, TIFR and GRE in PHYSICAL SCIENCES 
 
1.9 Four Vectors and Relativistic Invariance
• Four position vector ds = ( dx, dy , dz , icdt )

ds ⎛ dx dy dz icdt ⎞ ⎛ dx dy dz icdt ⎞
• Four velocity vector =⎜ , , , ⎟ u =γ⎜ , , , ⎟
dt ⎝ dτ dτ dτ dτ ⎠ ⎝ dt dt dt dt ⎠

u = γ (u x , u y , u z , ic ) ⇒ u = γ u, ic ( )
• Four momentum – Four Energy vector

(
⎛ imc 2
P = γ ( m0u , im0 c ) = mu, imc = ⎜⎜ P,
c
) ⎞
⎟⎟ ⇒ p = ⎛⎜ p, ⎞⎟
iE
⎝ c ⎠
⎝ ⎠
⎛ dm ⎞
• Four Force: F = γ ⎜ F , ic ⎟
⎝ dt ⎠
Four Dimensional Space Time Continuums
The square of interval is represented as
S122 = ( x 2 − x1 ) + ( y 2 − y1 ) + ( z 2 − z1 ) − c 2 (t 2 − t1 )
2 2 2 2

S122 = r122 − c 2 (t 2 − t1 )
2

(i) Space like Intervals: Time separation between the two events is less than the time
taken by light in covering the distance between them.
r12
> (t 2 − t1 )
c
(ii) Time like intervals: Time separation between two events is more than the time taken
by light in covering the distance between them
r12
< (t 2 − t1 )
c
(iii) Light-like intervals: Time separation between two events is equal to time taken by
light in covering the distance between them
r12
= (t 2 − t1 )
c

                                                                                
Head office  Branch office 
 
fiziks, H.No. 23, G.F, Jia Sarai,  Anand Institute of Mathematics, 
 
Near IIT, Hauz Khas, New Delhi‐16  28‐B/6, Jia Sarai, Near IIT 
 
Phone: 011‐26865455/+91‐9871145498 Hauz Khas, New Delhi‐16 
                                                   
                                             Website: www.physicsbyfiziks.com                                                                                          
                                                           Email: fiziks.physics@gmail.com                                                                   9 
fiziks
Institute for NET/JRF, GATE, IIT‐JAM, JEST, TIFR and GRE in PHYSICAL SCIENCES 
 
Example: Determine the length and the orientation of a rod of length l0 in a frame of

reference which is moving with v velocity in a direction making θ angle with rod.
Solution:
Proper length of the rod in the direction of moving frame l x0 = l 0 cos θ , the length

v2
measured in the moving frame l x = l0 x cos θ 1 − and l y = l0 sin θ
c2

⎛ v2 ⎞ ⎛ v2 ⎞
l = l0 2 cos 2 θ ⎜ 1 − 2 ⎟ + l02 sin 2 θ = l0 cos2 θ ⎜⎜1 − 2 ⎟⎟ + sin 2 θ
⎝ c ⎠ ⎝ c ⎠
ly l0 sin θ
tan θ ' = = ⇒ tan θ ' = γ tan θ ⇒ θ ' = tan −1 ( γ tan θ )
lx v 2
l0 cos θ 1 −
c2
Example: A cube of density ρ o in rest frame is moving with velocity v with respect to
observer parallel to one of its edge. What is density measured by observer?
Solution:
m0 mass m0
In rest frame = ρ0 and from moving frame ρ = where m =
V0 volume v2
1−
c2

v2 v2
and relativistic volume V = ( l0l0 ) ⋅ l0 1 − = V0 1 − .
c2 c2
m0
v2
1−
c2 m0 ρ0 m0
ρ= = ⇒ρ= ∵ = ρ0
v 2
⎛ 2 ⎞
2
v2 V0
V0 1 − 2 V ⎜ 1 − v ⎟ 1− 2
c 0
⎜ c 2 ⎟⎠ c

                                                                                
Head office  Branch office 
 
fiziks, H.No. 23, G.F, Jia Sarai,  Anand Institute of Mathematics, 
 
Near IIT, Hauz Khas, New Delhi‐16  28‐B/6, Jia Sarai, Near IIT 
 
Phone: 011‐26865455/+91‐9871145498 Hauz Khas, New Delhi‐16 
                                                   
                                             Website: www.physicsbyfiziks.com                                                                                          
                                                           Email: fiziks.physics@gmail.com                                                                   10 
fiziks
Institute for NET/JRF, GATE, IIT‐JAM, JEST, TIFR and GRE in PHYSICAL SCIENCES 
 
Example: A spaceship is moving away from the earth with velocity 0.5c fires a rocket
whose velocity relative to space is 0.5c (a) Away from earth (b) Towards the earth.
Calculate velocity of the rocket as observed from the earth in two cases.
Solution:
u '+ v
(a) u ' = 0.5c, v = 0.5c ⇒u = = 0.8c
u 'v
1+ 2
c
−0.5c + 0.5c
(b) u ' = −0.5c, v = 0.5c ⇒ u = =0
1−
( 0.5c )( 0.5c )
c2
Example: Show that the rest mass of particle of momentum p and kinetic energy

p 2c 2 − T 2
T given by m0 =
2Tc 2
Solution: E = EK + E0 ⇒ E = T + m0 c 2

p 2c 2 − T 2
E 2 = p 2 c 2 + m02 c 4 ⇒ m0 =
2Tc 2

Example: A π -meson at rest mass mπ decays into μ-meson of mass mμ and neutrino of

mass mν . Find the total energy of μ -meson.

Solution: Eπ = mπ c 2 , Eμ2 = pμ2 c 2 + mμ2 c 4 , Eν2 = pν2 c 2 + mν2 c 4

From conservation of energy Eπ =E μ + Eν and from conservation of momentum

Pν = − Pμ = P

Using Eμ2 − Eν2 = ( mμ2 − mν2 ) c 4 and Eπ = Eμ + Eν

One will get


Eμ2 − Eν2
=
(m 2
μ − mν2 ) c 2
⇒ Eμ − Eν =
(m 2
μ − mν2 ) c 2
Eμ + Eν mπ mπ

1
Eμ = ⎡ mπ2 + mμ2 − mν2 ⎤ c 2
2mπ ⎣ ⎦

                                                                                
Head office  Branch office 
 
fiziks, H.No. 23, G.F, Jia Sarai,  Anand Institute of Mathematics, 
 
Near IIT, Hauz Khas, New Delhi‐16  28‐B/6, Jia Sarai, Near IIT 
 
Phone: 011‐26865455/+91‐9871145498 Hauz Khas, New Delhi‐16 
                                                   
                                             Website: www.physicsbyfiziks.com                                                                                          
                                                           Email: fiziks.physics@gmail.com                                                                   11 
fiziks
Institute for NET/JRF, GATE, IIT‐JAM, JEST, TIFR and GRE in PHYSICAL SCIENCES 
 
3
Example: The rate of a clock in spaceship Suryashakti is observed from earth to be at
5
the rate of the clock on earth.
(a) What is the speed of spaceship Suryashakti relative to earth?
5
(b) If rate of clock in spaceship Akashganga is observed from earth to be at the rate of
13
the clocks on earth. If both Aakashganga and Suryashakti are moving in same direction
relative to someone on earth, then what is the speed of Aakashganga relative to
Suryashakti?
Solution: (a)
Δt01 5 1 25 1 4
From the time dilation Δt1 = ⇒ = ⇒ = ⇒ v1 = c
v12 3 v12 9 ⎛ v12 ⎞ 5
1− 1− ⎜1 − 2 ⎟
c2 c2 ⎝ c ⎠
4
Velocity of Suryashakti is c
5
(b) Similarly

Δt02 13 1 5 v2 25 v2 12
Δt2 = ⇒ = ⇒ = 1 − 22 ⇒ = 1 − 22 ⇒ v2 = c
v22 5 v22 13 c 169 c 13
1− 1−
c2 c2
v2 − v1
Velocity of Akashganga with respect to Suryashakti is given v=
vv
1 − 221
c
4 12
c− c
5 13 8
Hence both are moving in same direction = =− c
4 12 17
1−
5 13
8
Velocity of Aakashganga with respect to Suryashakti is − c
17

                                                                                
Head office  Branch office 
 
fiziks, H.No. 23, G.F, Jia Sarai,  Anand Institute of Mathematics, 
 
Near IIT, Hauz Khas, New Delhi‐16  28‐B/6, Jia Sarai, Near IIT 
 
Phone: 011‐26865455/+91‐9871145498 Hauz Khas, New Delhi‐16 
                                                   
                                             Website: www.physicsbyfiziks.com                                                                                          
                                                           Email: fiziks.physics@gmail.com                                                                   12 
fiziks
Institute for NET/JRF, GATE, IIT‐JAM, JEST, TIFR and GRE in PHYSICAL SCIENCES 
 
Example: In the laboratory frame a particle P at rest mass m0 is moving in the positive
5c
x-direction with speed of . It approaches an identical particle Q moving in the negative
19
2c
x-direction with a speed of .
5
(a) What is speed of the particle P in the rest frame of the particle Q.
(b) What is Energy of the particle P in the rest frame of the particle Q.
5 2 u ′x + v 3
Solution: (a) u ' x = c, v = c ⇒ ux = = c
19 5 u′x v 5
1+ 2
c
3c m0c 2 5
(b) v1 = ⇒E= = m0c 2
5 v 2 4
1− 1
2
c
2m0
Example: The mass m of moving particle is , where mo is its rest mass. Then what
3
is linear momentum of particle?
m0 2m0 m0 c
Solution: mass m = ⇒ = ⇒v=
v2 3 v2 2
1− 1−
c2 c2
m0 m0 1 mc
P= v= × c ⇒P= 0
v2 1 2 3
1− 1−
c2 4

Example: A distant galaxy in constellation Hydra is receding from the earth at 6.12 x 107
m/s by how much is a green spectral line of wavelength 500 nm emitted by this galaxy
shifted towards the red at spectrum.
v
1+
c 1 + 0.204
Solution: λ = λ0 ⇒ λ = 500 = 615nm ∵ v = 0.204c, λ0 = 500
v 1 − 0.204
1−
c
which is orange part of spectrum. The shift is λ – λ0 = 115 nm.

                                                                                
Head office  Branch office 
 
fiziks, H.No. 23, G.F, Jia Sarai,  Anand Institute of Mathematics, 
 
Near IIT, Hauz Khas, New Delhi‐16  28‐B/6, Jia Sarai, Near IIT 
 
Phone: 011‐26865455/+91‐9871145498 Hauz Khas, New Delhi‐16 
                                                   
                                             Website: www.physicsbyfiziks.com                                                                                          
                                                           Email: fiziks.physics@gmail.com                                                                   13 
fiziks
Institute for NET/JRF, GATE, IIT‐JAM, JEST, TIFR and GRE in PHYSICAL SCIENCES 
 
Multiple Choice Questions (MCQ)
Q1. In a system of units in which the velocity of light c = 1 , which of the following is a
Lorentz transformation?
(a) x′ = 4 x, y = y ′, z ′ = z, t ′ = 0.25 t
(b) x′ = x − 0.5 t , y = y ′, z ′ = z, t ′ = t + x
(c) x′ = 1.25x − 0.75 t , y ′ = y, z ′ = z, t ′ = 0.75 t − 1.25 x
(d) x′ = 1.25x − 0.75 t , y ′ = y, z ′ = z, t ′ = 1.25 t − 0.75 x

Q2. A circle of radius 5 m lies at rest in x - y plane in the laboratory. For an observer moving
with a uniform velocity v along the y direction, the circle appears to be an ellipse with an
equation
x2 y2
+ =1
25 9
The speed of the observer in terms of the velocity of light c is,
9c 3c 4c 16c
(a) (b) (c) (d)
25 5 5 25

Q3. An electron is moving with a velocity of 0.85c in the same direction as that of a moving
photon. The relative velocity of the electron with respect to photon is
(a) c (b) − c
(c) 0.15c (d) − 0.15c

Q4. The area of a disc in its rest frame S is equal to 1 (in some units). The disc will appear
distorted to an observer O moving with a speed u with respect to S along the plane of the
disc. The area of the disc measured in the rest frame of the observer O is ( c is the speed
of light in vacuum)
1/ 2 −1 / 2 −1
⎛ u2 ⎞ ⎛ u2 ⎞ ⎛ u2 ⎞ ⎛ u2 ⎞
(a) ⎜⎜1 − 2 ⎟⎟ (b) ⎜⎜1 − 2 ⎟⎟ (c) ⎜⎜1 − 2 ⎟⎟ (d) ⎜⎜1 − 2 ⎟⎟
⎝ c ⎠ ⎝ c ⎠ ⎝ c ⎠ ⎝ c ⎠

                                                                                
Head office  Branch office 
 
fiziks, H.No. 23, G.F, Jia Sarai,  Anand Institute of Mathematics, 
 
Near IIT, Hauz Khas, New Delhi‐16  28‐B/6, Jia Sarai, Near IIT 
 
Phone: 011‐26865455/+91‐9871145498 Hauz Khas, New Delhi‐16 
                                                   
                                             Website: www.physicsbyfiziks.com                                                                                          
                                                           Email: fiziks.physics@gmail.com                                                                   14 
fiziks
Institute for NET/JRF, GATE, IIT‐JAM, JEST, TIFR and GRE in PHYSICAL SCIENCES 
 
Q5. A light beam is propagating through a block of glass with index of refraction n . If the
glass is moving at constant velocity v in the same direction as the beam, the velocity of
the light in the glass block as measured by an observer in the laboratory is approximately
c ⎛ 1 ⎞ c ⎛ 1 ⎞
(a) u = + v ⎜1 − 2 ⎟ (b) u = − v ⎜1 − 2 ⎟
n ⎝ n ⎠ n ⎝ n ⎠
c ⎛ 1 ⎞ c
(c) u = + v ⎜1 + 2 ⎟ (d) u =
n ⎝ n ⎠ n
Q6. If fluid is moving with velocity v with respect to stationary narrow tube .If light pulse
enter into fluid in the direction of flow. What is speed of light pulse measured by
observer who is stationary with respect to tube?
⎛ nv ⎞ ⎛ v ⎞
1+ ⎟ 1+
c c⎜ c c ⎜ nc ⎟
(a) c (b) (c) ⎜ ⎟ (d) ⎜ ⎟
n n ⎜ 1+ v ⎟ n ⎜ 1 + nv ⎟
⎜ ⎟ ⎜ ⎟
⎝ nc ⎠ ⎝ c ⎠

Q7. A light beam is emitted at an angle θ 0 with respect to the x ' in S frame which is moving

with velocity uiˆ .Then the angle θ the beam makes with respect to x axis in S frame .
u cos θ 0
(a) θ = θ0 (b)
c
u u cos θ 0
cos θ0 + 1+
(c) cos θ = c (d) cos θ = c
u u
1 + cos θ 0 cos θ0 +
c c
Q8. The relativistic form of Newton’s second law of motion is

dv mc m c 2 − v 2 dv
(a) F = (b) F =
2 dt c dt
c −v
2

mc 3 dv c 2 − v 2 dv
(c) F = (d) F = m
(c )
3/ 2
2
− v2 dt c2 dt

                                                                                
Head office  Branch office 
 
fiziks, H.No. 23, G.F, Jia Sarai,  Anand Institute of Mathematics, 
 
Near IIT, Hauz Khas, New Delhi‐16  28‐B/6, Jia Sarai, Near IIT 
 
Phone: 011‐26865455/+91‐9871145498 Hauz Khas, New Delhi‐16 
                                                   
                                             Website: www.physicsbyfiziks.com                                                                                          
                                                           Email: fiziks.physics@gmail.com                                                                   15 
fiziks
Institute for NET/JRF, GATE, IIT‐JAM, JEST, TIFR and GRE in PHYSICAL SCIENCES 
 
Q9. Two particles each of rest mass m collide head-on and stick together. Before collision, the
speed of each mass was 0.6 times the speed of light in free space. The mass of the final
entity is
(A) 5m / 4 (B) 2m (C) 5m / 2 (D) 25 m / 8

Q10. According to the special theory of relativity, the speed v of a free particle of mass m and
total energy E is:

mc 2 2 E ⎛ mc 2 ⎞
(a) v = c 1 − (b) v = ⎜1 + ⎟
E m ⎜⎝ E ⎟⎠
2
⎛ mc 2 ⎞ ⎛ mc 2 ⎞
(c) v = c 1 − ⎜⎜ ⎟⎟ (d) v = c⎜⎜1 + ⎟⎟
⎝ E ⎠ ⎝ E ⎠

Q11. The velocity of a particle at which the kinetic energy is equal to its rest energy is (in
terms of c , the speed of light in vacuum)

(a) 3c / 2 (b) 3c / 4 (c) 3 / 5c (d) c / 2

Q12. In the laboratory frame, a particle P of rest mass m0 is moving in the positive x direction
5
with a speed of c. It approaches an identical particle Q, moving in the negative x
19
2
direction with a speed of c . The speed of the particle P in the rest frame of the particle
5
Q is
7 13 3 63
(a) c (b) c (c) c (d) c
95 85 5 95

Q13. The energy of the particle P in the rest frame of the particle Q is
1 5 19 11
(a) m0 c 2 (b) m0 c 2 (c) m0 c 2 (d) m0 c 2
2 4 13 9

                                                                                
Head office  Branch office 
 
fiziks, H.No. 23, G.F, Jia Sarai,  Anand Institute of Mathematics, 
 
Near IIT, Hauz Khas, New Delhi‐16  28‐B/6, Jia Sarai, Near IIT 
 
Phone: 011‐26865455/+91‐9871145498 Hauz Khas, New Delhi‐16 
                                                   
                                             Website: www.physicsbyfiziks.com                                                                                          
                                                           Email: fiziks.physics@gmail.com                                                                   16 
fiziks
Institute for NET/JRF, GATE, IIT‐JAM, JEST, TIFR and GRE in PHYSICAL SCIENCES 
 
Q14. If u ( x, y, z , t ) = f ( x + i β y − vt ) + g ( x − i β y − vt ) , where f and g are arbitrary and twice

differentiable functions, is a solution of the wave equation


∂ 2u ∂ 2u 1 ∂ 2u
+ = then β is
∂x 2 ∂y 2 c 2 ∂t 2
1/ 2 1/ 2
⎛ v⎞ ⎛ v⎞ ⎛ v2 ⎞ ⎛ v2 ⎞
(a) ⎜1 − ⎟ (b) ⎜1 − ⎟ (c) ⎜⎜1 − 2 ⎟⎟ (d) ⎜⎜1 − 2 ⎟⎟
⎝ c⎠ ⎝ c⎠ ⎝ c ⎠ ⎝ c ⎠
c
Q15. A relativistic particle of mass m and velocity zˆ is moving towards a wall. The wall is
2
c
moving with a velocity zˆ . The velocity of the particle after it suffers an elastic collision
3
is v zˆ with v equal to
(a) c / 2 (b) c / 5 (c) c / 7 (d) c / 15
(All the velocities refer to the laboratory frame of reference.)
Q16. The momentum of an electron (mass m ) which has the same kinetic energy as its rest
mass energy is (c is velocity of light)
(a) 3mc (b) 2mc (c) mc (d) mc / 2
Q17. A particle of mass M decays at rest into a mass less particle and another particle of mass
m . The magnitude of the momentum of each of these relativistic particles is:
c c
(a) M 2 − 4m 2 (b) M 2 + 4m 2
2 2

(c)
c
2M
(
M 2 − m2 ) (d)
c
2M
(
M 2 + m2 )
Q18. Consider a beam of relativistic particles of kinetic energy K at normal incidence upon a
perfectly absorbing surface. The particle flux (number of particles per unit area per unit
time) is J and each particle has rest mass m0. The pressure on the surface is

(a)
JK
(b)
(
J K K + m0 c 2 )
c c
J (K + m0 c ) 2
J K (K + 2m0 c 2 )
(c) (d)
c c
                                                                                
Head office  Branch office 
 
fiziks, H.No. 23, G.F, Jia Sarai,  Anand Institute of Mathematics, 
 
Near IIT, Hauz Khas, New Delhi‐16  28‐B/6, Jia Sarai, Near IIT 
 
Phone: 011‐26865455/+91‐9871145498 Hauz Khas, New Delhi‐16 
                                                   
                                             Website: www.physicsbyfiziks.com                                                                                          
                                                           Email: fiziks.physics@gmail.com                                                                   17 
fiziks
Institute for NET/JRF, GATE, IIT‐JAM, JEST, TIFR and GRE in PHYSICAL SCIENCES 
 
Multiple Select Type Questions (MSQ)

Q19. A particle of mass m1 = 3kg moving at velocity of u1 = +4m / sec along the x axis of

frame S , approaches a second Particle of mass m2 = 1kg , moving at velocity

u2 = −3m / sec along the axis after the collision the m2 has velocity U 2 = +3m / sec along

the x axis then which of the following is correct


(a) The velocity of m1 is U1 = +2m / sec

(b) The momentum of the system before collision and after collision is 9kg − m / sec

(c) If the observer S ' who has velocity v = +2 m / sec relative to S frame the momentum
measured before collision is +1kg − m / sec

(d) If the observer S ' who has velocity v = +2 m / sec relative to S frame the momentum
measured after collision is −1kg − m / sec
Q20. A rod of length l0 lies in plane with respect to inertial frame Κ . The length inclined at

angle of θ 0 with respect to horizontal as shown in figure below. If rod is moving in

1
horizontal direction with speed v with respect to observer if γ = .then
v2
y y′ 1− 2
B c
v
l0

A θ0
x′
x
l0 cos θ 0
(a) The length appeared to observer
γ

v 2 cos 2 θ
(b) The length of Rod appeared to observer is l0 1 −
c2
(c) The angle that rod makes with the horizontal is tan −1 ( γ tan θ 0 )

⎛ tan θ 0 ⎞
(d) The angle that rod makes with the horizontal is tan −1 ⎜ ⎟
⎝ γ ⎠

                                                                                
Head office  Branch office 
 
fiziks, H.No. 23, G.F, Jia Sarai,  Anand Institute of Mathematics, 
 
Near IIT, Hauz Khas, New Delhi‐16  28‐B/6, Jia Sarai, Near IIT 
 
Phone: 011‐26865455/+91‐9871145498 Hauz Khas, New Delhi‐16 
                                                   
                                             Website: www.physicsbyfiziks.com                                                                                          
                                                           Email: fiziks.physics@gmail.com                                                                   18 
fiziks
Institute for NET/JRF, GATE, IIT‐JAM, JEST, TIFR and GRE in PHYSICAL SCIENCES 
 
Q21. A train of rest length L0 and of velocity vxˆ is passes a station. At t ′A = tO′ , two light pulses

are sent from the train’s mid point, O , towards the points A and B . At the moment t ′ ,
two spectators sitting at points A and B receive the light pulses. The
(a)The time to reach light pulse O to A and O to B in the train frame is same and given
Lo
by
2c
(b) The distance traveled by the light pulse from O to A , as seen at the frame of
L0 1 + β v
reference of the station is where β =
2 1− β c
(c)The distance traveled by the light pulse from O to B , as seen at the frame of reference
L0 1 + β v
of the station is where β =
2 1− β c

(d) The time needed for the light pulses to travel from point O to point B , as seen in the

L0 1 − β v
station’s frame. Is ( Δt ) B = t B − tO = where β =
2c 1 + β c

Q22. The rod of proper length l0 and two toy train A and B moving with respect table with

speed 0.5c .rod and toy train A moving in same direction but toy train B moving in
opposite direction, then which one of the following observation is correct?
(a) The length of rod measured by observer attached the frame with toy train A is 0.866 l0

(b) The length of rod measured by observer attached the frame table is .866lo

(c) The length of rod measured by observer attached the frame table is lo

(d) The length of rod measured by observer attached the frame with toy train B is 0.6 l0

                                                                                
Head office  Branch office 
 
fiziks, H.No. 23, G.F, Jia Sarai,  Anand Institute of Mathematics, 
 
Near IIT, Hauz Khas, New Delhi‐16  28‐B/6, Jia Sarai, Near IIT 
 
Phone: 011‐26865455/+91‐9871145498 Hauz Khas, New Delhi‐16 
                                                   
                                             Website: www.physicsbyfiziks.com                                                                                          
                                                           Email: fiziks.physics@gmail.com                                                                   19 
fiziks
Institute for NET/JRF, GATE, IIT‐JAM, JEST, TIFR and GRE in PHYSICAL SCIENCES 
 
Q23. A particle with mass m and total energy E0 travels at a constant velocity V which may
approach the speed of light. It then collides with a stationary particle with the same mass
m , and they are seen to scatter elastically at the relative angle θ with equal kinetic
energies. then which of the following are correct.

⎛θ ⎞ E0 + mc 2
(a) cos ⎜ ⎟ =
⎝2⎠ E0 + 3mc 2

⎛θ ⎞ E0 + mc 2
(b) sin ⎜ ⎟ =
⎝2⎠ E0 + 3mc 2

π
(c) From point of view of classical mechanics θ ≈
2
(d) From point of view of relativistic mechanics θ = 0

Q24. If particle of rest mass m0 has momentum 2 2m0c then

(a) The velocity of particle is 2 2c

2 2c
(b) The velocity of particle is
3
(c) The total energy is 3m0 c 2

(d) Kinetic energy is 2m0 c 2

3
Q25. If The rate of a clock in spaceship “Fiziks” is observed from earth to be of the rate of
5
the clocks on earth then
4c
(a) The speed of spaceship “Fiziks ” relative to earth is
5
3c
(b) The speed of spaceship “Fiziks ” relative to earth is
5

                                                                                
Head office  Branch office 
 
fiziks, H.No. 23, G.F, Jia Sarai,  Anand Institute of Mathematics, 
 
Near IIT, Hauz Khas, New Delhi‐16  28‐B/6, Jia Sarai, Near IIT 
 
Phone: 011‐26865455/+91‐9871145498 Hauz Khas, New Delhi‐16 
                                                   
                                             Website: www.physicsbyfiziks.com                                                                                          
                                                           Email: fiziks.physics@gmail.com                                                                   20 
fiziks
Institute for NET/JRF, GATE, IIT‐JAM, JEST, TIFR and GRE in PHYSICAL SCIENCES 
 
5
(c) The rate of clock in spaceship “Akashganga” is observed from earth to be of the
13
rate of the clocks on earth. If both Aakashganga and “Fiziks” are moving in the same
direction relative to someone on earth, then the speed of Aakashganga relative to “Fiziks”
8
is c
17
5
(d) The rate of clock in spaceship “Akashganga” is observed from earth to be of the
13
rate of the clocks on earth. If both Aakashganga and “Fiziks” are moving in the opposite
direction relative to someone on earth, then the speed of Aakashganga relative to “Fiziks”
8
is c
17

Q26. Consider the decay process τ − → π − + ν τ in the rest frame of the τ-. The masses of the

τ − , π − and ν τ are Mτ M π and zero respectively. Then which of the following is correct?

(a) The energy of π −


is
(M τ
2
+ M π2 )c 2
2M τ
⎛ M 2 − M π2 ⎞
(b) The velocity of π − is ⎜ τ2 2 ⎟
c
⎝ Mτ + Mπ ⎠
⎛ M 2 − M τ2 ⎞
(c) The velocity of ν τ is ⎜ π2 2 ⎟
c
⎝ Mτ + Mπ ⎠

(d) The magnitude of energy of ν τ is


(M 2
τ − M π2 c) 2

2M τ

                                                                                
Head office  Branch office 
 
fiziks, H.No. 23, G.F, Jia Sarai,  Anand Institute of Mathematics, 
 
Near IIT, Hauz Khas, New Delhi‐16  28‐B/6, Jia Sarai, Near IIT 
 
Phone: 011‐26865455/+91‐9871145498 Hauz Khas, New Delhi‐16 
                                                   
                                             Website: www.physicsbyfiziks.com                                                                                          
                                                           Email: fiziks.physics@gmail.com                                                                   21 
fiziks
Institute for NET/JRF, GATE, IIT‐JAM, JEST, TIFR and GRE in PHYSICAL SCIENCES 
 
m c2
Q27. A particle of rest mass mo moves with kinetic energy o disintegrate into two photon
4
with momentum p1 and p2 and energy E1 and E2 respectively. Assume photon carries

momentum p1 moves in same direction and other photon in opposite direction. Then
which of the following is correct

(a) The magnitude of momentum p1 is mo c

(b) The magnitude of momentum p2 is mo c


mo c 2
(c) The total kinetic energy of both the photon is
4
mo c 2
(d) The value of energy E2 is
4

Q28. A shooter fires a bullet with velocity u in the x̂ direction at a target. The target is moving
with velocity v in the x̂ direction relative to the shooter and is at a distance L from him

at the instant the bullet is fired. If γ 1 = 1 1 − v 2 / c 2 and γ 2 = 1 1 − u 2 / c 2


(a) The time that bullet is fired will it take to hit the target in the target’s frame of
reference is γ 1 L(1 − uv / c 2 ) / (u − v )
(b) The time that bullet is fired will it take to hit the target in the target’s frame of
reference is γ 2 L(1 − uv / c 2 ) / (u − v )
(c) The time that bullet is fired will it take to hit the target in the bullet’s frame of
(
reference is γ 1 L 1 − uv / c 2 / (v − u ) )
(d) The time that bullet is fired will it take to hit the target in the bullet’s frame of
reference is γ 2 L(1 − uv / c 2 ) / (v − u )

                                                                                
Head office  Branch office 
 
fiziks, H.No. 23, G.F, Jia Sarai,  Anand Institute of Mathematics, 
 
Near IIT, Hauz Khas, New Delhi‐16  28‐B/6, Jia Sarai, Near IIT 
 
Phone: 011‐26865455/+91‐9871145498 Hauz Khas, New Delhi‐16 
                                                   
                                             Website: www.physicsbyfiziks.com                                                                                          
                                                           Email: fiziks.physics@gmail.com                                                                   22 
fiziks
Institute for NET/JRF, GATE, IIT‐JAM, JEST, TIFR and GRE in PHYSICAL SCIENCES 
 
Numerical Answer Type Questions (NAT)

An Event occurs in S frame at x = 6 ×10 m and in S x = 6 ×10 m , t = 4sec the relative


8 ' ' 8 '
Q29.
velocity is …………..

Q30. Two events separated by a (spatial) distance 9 × 109 m , are simultaneous in one inertial
frame. The time interval between these two events in a frame moving with a constant
speed 0.8 c (where the speed of light 3 × 108 m / sec ) is ……………
Q31. In a certain intertial frame two light pulses are emitted at point 5km apart and separated
in time by 5μ sec . An observe moving at a speed v along the line joining these points
notes that the pulses are simultaneous. Therefore v is………………… c .
Q32. A monochromatic wave propagates in a direction making an angle 60 o with the x -axis
4c
in the reference frame of source. The source moves at speed v = towards the
5
observer. The direction of the ( cos θ ) wave as seen by the observer is …………
Q33. A π0 meson at rest decays into two photons, which move along the x-axis. They are both
detected simultaneously after a time, t = 10 . In an inertial frame moving with a velocity
v = .6c in the direction of one of the photons, the time interval between the two
detections is ……….. sec
Q34. A rod of proper length l0 oriented parallel to the x-axis moves with speed 2c / 3 along the
x-axis in the S-frame, where c is the speed of the light in free space. The observer is also
moving along the x-axis with speed c / 2 with respect to the S-frame. The length of the
rod as measured by the observer is………………. l0
Q35. If the half-life of an elementary particle moving with speed 0.9c in the laboratory frame is
5 × 10 −8 s, then the proper half-life is _______________ ×10 −8 s. (c = 3 × 10 8 m / s )
Q36. The muon has mass 105 MeV/c2 and mean life time 2.2 μs in its rest frame. The mean
distance traversed by a muon of energy 315 MeV before decaying is approximately
…………. km

                                                                                
Head office  Branch office 
 
fiziks, H.No. 23, G.F, Jia Sarai,  Anand Institute of Mathematics, 
 
Near IIT, Hauz Khas, New Delhi‐16  28‐B/6, Jia Sarai, Near IIT 
 
Phone: 011‐26865455/+91‐9871145498 Hauz Khas, New Delhi‐16 
                                                   
                                             Website: www.physicsbyfiziks.com                                                                                          
                                                           Email: fiziks.physics@gmail.com                                                                   23 
fiziks
Institute for NET/JRF, GATE, IIT‐JAM, JEST, TIFR and GRE in PHYSICAL SCIENCES 
 
Q37. The recently-discovered Higgs boson at the LHC experiment has a decay mode into a
photon and a Z boson. If the rest masses of the Higgs and Z boson are 125 GeV/c2 and

90 GeV/c2 respectively, and the decaying Higgs particle is at rest, the energy of the
photon will approximately be ………………
Q38. A particle of rest mass m has momentum 5mc then velocity of the particle is
…………… c
Q39. The velocity of a particle at which the kinetic energy is equal to its rest energy is (in
terms of c , the speed of light in vacuum) ………………….. c
Q40. If particle of rest mass m0 have kinetic energy is m0 c 2 then total energy is given by

…………… m0 c 2

Q41. If particle of rest mass m0 have kinetic energy is m0 c 2 then velocity is given by

…………… c
Q42. A cosmic particle of rest mass m0 move with speed .5c with respect to Earth .A

spaceship moving in opposite direction with same speed .5c The mass of the cosmic
particle observed by the observer whose frame is attached to space ship is ………….. m0

c
Q43. A particle of rest mass m moving with speed decays into two particles of rest masses
2
2
m each. The daughter particles move in the same line as the direction of motion of the
5
original particle. The velocities of the daughter particles…………… c
Q44. It is found that pions are radioactive and they are brought to rest half life is measured to
be 1.77 ×10−8 sec. A collimated pion beam, leaving the accelerator target at speed of
0.99c , it is found to drop to half of its original intensity ………….. meter from target .
Q45. A rod is moving with a speed of 0.8c with respect to stationary observer in a direction
at 60 o to its own length. The length of the rod observed by the observer is __________ lo

                                                                                
Head office  Branch office 
 
fiziks, H.No. 23, G.F, Jia Sarai,  Anand Institute of Mathematics, 
 
Near IIT, Hauz Khas, New Delhi‐16  28‐B/6, Jia Sarai, Near IIT 
 
Phone: 011‐26865455/+91‐9871145498 Hauz Khas, New Delhi‐16 
                                                   
                                             Website: www.physicsbyfiziks.com                                                                                          
                                                           Email: fiziks.physics@gmail.com                                                                   24 
fiziks
Institute for NET/JRF, GATE, IIT‐JAM, JEST, TIFR and GRE in PHYSICAL SCIENCES 
 
Solution
MCQ
Ans. 1: (d)
Ans. 2: (c)

v2 4c
Solution: 3 = 5 1 − 2
⇒v=
c 5
Ans. 3: (b)
Ans. 4: (a)
Solution: Area of disc from S frame is 1 i.e. π a 2 = 1 or π a ⋅ a = 1

u2 u2 u2
Area of disc from S ′ frame is π a ⋅ b = π a ⋅ a 1 − = 1 ⋅ 1 − = 1 −
c2 c2 c2

u2
where b = a 1 − 2 .
c
Ans. 5: (a)
c
v+ −1
n = ⎛ v + c ⎞⎛1 + v ⎞ = ⎛ v + c ⎞ ⎛1 − v + v ⎞
2
Solution: Now u = ⎜ ⎟⎜ ⎟ ⎜ ⎟⎜ ⎟
v⋅c ⎝ n ⎠⎝ cn ⎠ ⎝ n ⎠ ⎝ cn c 2 n 2 ⎠
1+ 2
c ⋅n
v2 v3 c v v2 c ⎛ 1 ⎞
⇒v− + 2 2 + − 2 + 3 ⇒ u = + v ⎜1 − 2 ⎟
cn c n n n cn n ⎝ n ⎠
Ans. 6: (c)
c u' + v
Solution: u ' = and u =
n u 'v
1+ 2
c
⎛ nv ⎞
1+
c⎜ c ⎟
u= ⎜ ⎟
n ⎜ 1+ v ⎟
⎜ ⎟
⎝ nc ⎠

                                                                                
Head office  Branch office 
 
fiziks, H.No. 23, G.F, Jia Sarai,  Anand Institute of Mathematics, 
 
Near IIT, Hauz Khas, New Delhi‐16  28‐B/6, Jia Sarai, Near IIT 
 
Phone: 011‐26865455/+91‐9871145498 Hauz Khas, New Delhi‐16 
                                                   
                                             Website: www.physicsbyfiziks.com                                                                                          
                                                           Email: fiziks.physics@gmail.com                                                                   25 
fiziks
Institute for NET/JRF, GATE, IIT‐JAM, JEST, TIFR and GRE in PHYSICAL SCIENCES 
 
Ans. 7: (c)
u x' + v c cos θ 0 + u c cos θ 0 + u
Solution: u x = = = ∵ u x' = c cos θ 0 , v=u
1 + u x v / c 1 + c cos θ 0u / c
' 2 2
1 + cos θ 0u / c
u
cos θ 0 +
ux c
cos θ = ⇒ cos θ =
c u
1 + cos θ 0
c
Ans. 8: (c)
mv dP dv 1 ⎛ 1⎞ 1 −2v dv
Solution: P = ⇒F= =m ⋅ + mv ⎜ − ⎟ ⋅ 3/2
⋅ 2
v2 dt dt v2 ⎝ 2 ⎠ ⎛ v2 ⎞ c dt
1− 1− 2 ⎜ 1− 2 ⎟
c2 c ⎝ c ⎠
⎛ ⎞
⎜ v2 2 ⎟
dv 1 ⎜ c ⎟ mc 3 dv
⇒F =m 1+ =
v 2 ⎜ ⎛1 − v ⎞ ⎟ c 2 − v 2 ( )
2 3/ 2
dt dt
1 − 2 ⎜⎜ ⎜ 2 ⎟⎟
c ⎝ ⎝ c ⎠ ⎟⎠
Ans. 9: (c)
Solution: From conservation of energy
mc 2 mc 2 2mc 2
+ = m1c 2 ⇒ = m1c 2 Since v = 0.6c ⇒ m1 = 5m / 2
v 2
v 2
v2
1− 2 1− 2 1− 2
c c c
Ans. 10: (c)
2 2
v 2 ⎛ mc 2 ⎞
mc 2 v2 m2c 4 ⎛ mc 2 ⎞
Solution: E = ⇒ 1− 2 = ⎜ ⎟ ⇒ = 1 − ⇒ v = c 1 − ⎜ ⎟
v2 c ⎝ E ⎠ c2 E2 ⎝ E ⎠
1− 2
c
Ans. 11: (a)
Solution: K .E = mc 2 − m0 c 2 , rest mass energy = m0 c 2

K .E. = rest mass energy ⇒ mc 2 − m0 c 2 = m0 c 2 ⇒ mc 2 = 2m0 c 2

m0 1 ⎛ v2 ⎞ v2 3
c = 2m0 c ⇒
2 2
= 2 ⇒ 4⎜⎜1 − 2 ⎟⎟ = 1 ⇒ 4 2 = 3 ⇒ v = c
v2 v2 ⎝ c ⎠ c 2
1− 2 1− 2
c c
                                                                                
Head office  Branch office 
 
fiziks, H.No. 23, G.F, Jia Sarai,  Anand Institute of Mathematics, 
 
Near IIT, Hauz Khas, New Delhi‐16  28‐B/6, Jia Sarai, Near IIT 
 
Phone: 011‐26865455/+91‐9871145498 Hauz Khas, New Delhi‐16 
                                                   
                                             Website: www.physicsbyfiziks.com                                                                                          
                                                           Email: fiziks.physics@gmail.com                                                                   26 
fiziks
Institute for NET/JRF, GATE, IIT‐JAM, JEST, TIFR and GRE in PHYSICAL SCIENCES 
 
Ans. 12: (c)
5 2 u x′ + v 3
Solution: u ' x = c, v = c ux = = c
19 5 u x′ v 5
1+ 2
c
Ans. 13: (b)
m0 c 2 5
Solution: E = = m0 c 2
v2 4
1−
c2
Ans.14: (c)
∂ 2u ∂ 2u 1 ∂ 2u
Solution: 2 + 2 = 2 2 = ⎣⎡ f ′′ ( x + i β y − vt ) + g ′′ ( x − iβ y − vt ) ⎤⎦ 1 − β 2
∂x ∂y c ∂t
( )
1/ 2
v2 ⎛ v2 ⎞
= 2 f ′′ ( x + i β y − vt ) + g ′′ ( x − i β y − vt ) ⇒ β = ⎜1 − 2 ⎟
c ⎝ c ⎠
Ans. 15: (b)
c c
Solution: v = − zˆ u x' = 0, u 'y = 0, u z' = the speed of particle with respect to wall is
3 2
u z' + v c
uz = =
u z' v 5
1+ 2
c
Ans. 16: (a)
Solution: Kinetic energy T = mc 2 and T = E − mc 2 E = 2mc 2
E 2 = p 2c 2 + m2c 4

p = 3mc
Ans. 17: (c)
Solution: From conservation of momentum mass less particle and particle of mass m have same

momentum p and from conservation of energy. Mc 2 = p 2 c 2 + m 2 c 4 + pc

p=
c
2M
(
M 2 − m2 )

                                                                                
Head office  Branch office 
 
fiziks, H.No. 23, G.F, Jia Sarai,  Anand Institute of Mathematics, 
 
Near IIT, Hauz Khas, New Delhi‐16  28‐B/6, Jia Sarai, Near IIT 
 
Phone: 011‐26865455/+91‐9871145498 Hauz Khas, New Delhi‐16 
                                                   
                                             Website: www.physicsbyfiziks.com                                                                                          
                                                           Email: fiziks.physics@gmail.com                                                                   27 
fiziks
Institute for NET/JRF, GATE, IIT‐JAM, JEST, TIFR and GRE in PHYSICAL SCIENCES 
 
Ans. 18: (d)
Solution: From conservation of energy

K ( K + 2mo c 2
K + mo c 2 = p 2 c 2 + mo c 2 so momentum p =
c
If particle flux (number of particles per unit area per unit time) is J then pressure
F
P=
A

P=
(
J K K + 2m0 c 2 )
c

MSQ
Ans. 19: (a), (b) and (c)
Solution: m1u1 + m2u2 = m1U1 + m2U 2 = +9kg − m / sec So U1 = +2m / sec

The velocity of particle with respect to observer on S ' frame is u1' = u1 − v = 2m / sec
similarly
u2' = −5m / sec U1' = 0m / sec U 2' = 1m / sec

Hence S ' is inertial frame of reference so m1u1' + m2u2' = m1U1' + m2U 2' = 1kg − m / sec

Ans. 20: (b), (c)


l0 cos θ 0
Solution: Δx = , Δy = l0 sin θ 0
γ

v 2 cos 2 θ
( Δx ) + ( Δy )
2 2
l= = l0 1 −
c2
Δy
tan θ = = γ tan θ 0
Δx

                                                                                
Head office  Branch office 
 
fiziks, H.No. 23, G.F, Jia Sarai,  Anand Institute of Mathematics, 
 
Near IIT, Hauz Khas, New Delhi‐16  28‐B/6, Jia Sarai, Near IIT 
 
Phone: 011‐26865455/+91‐9871145498 Hauz Khas, New Delhi‐16 
                                                   
                                             Website: www.physicsbyfiziks.com                                                                                          
                                                           Email: fiziks.physics@gmail.com                                                                   28 
fiziks
Institute for NET/JRF, GATE, IIT‐JAM, JEST, TIFR and GRE in PHYSICAL SCIENCES 
 
Ans. 21: (a), (b) and (d)
Solution: (a) In the train’s frame of reference, Δt ′ = t A′ − tO′ where tO′ is the time point when the
pulse leaves point O , and t A′ is the time when the light pulse reaches A .
L0
Now, Δt ′ = t A′ − tO′ =
2c
Notice that the two light pulses are received simultaneously by A and B in the frame of
reference of the train.
(b) We define events in S - the station frame, and S ′ - the train’s frame.
Events S ′ frame S frame
Pulse left O ( tO′ , xO′ ) ( tO , xO )
Pulse reached A ⎛ L0 ⎞ ( t A , xA )
⎜ t A′ , xO′ + 2 ⎟
⎝ ⎠
Pulse reached B ⎛ L0 ⎞ ( t B , xB )
⎜ t A′ , xO′ − 2 ⎟
⎝ ⎠
⎧ x = γ ( x ′ + β ct ′ )
⎪ v
Using the Lorentz transformation, ⎨ ⎛ ′ β ′ ⎞ where β = .
⎪t = γ ⎜ t + x ⎟ c
⎩ ⎝ c ⎠
Therefore, the distance the light pulse has to travel in the laboratory frame is:
⎛ L0 L ⎞
( Δx ) A = xA − xO = γ ( x′A + β ct ′A ) − γ ( xO′ + β ctO′ ) = γ ( Δx′ + β cΔt ′) = γ ⎜ + βc 0 ⎟
⎝ 2 2c ⎠

L0 L 1+ β
=γ (1 + β ) = 0
2 2 1− β

L0 1 − β
By means of the same treatment, we find: ( Δx ) B = xB − xO =
2 1+ β
(d) The time needed for the light to travel from O to B in the station’s frame of
β ⎛ L β L0 ⎞
reference is ( Δt ) B = tB − tO = γ ⎢⎡tB′ − tO′ + ( x′B − xO′ )⎥⎤ =γ ⎜ 0 − ⎟=
⎣ c ⎦ ⎝ 2c c 2 ⎠
L0 1 − β
( Δt ) B = t B − tO =
2c 1 + β
                                                                                
Head office  Branch office 
 
fiziks, H.No. 23, G.F, Jia Sarai,  Anand Institute of Mathematics, 
 
Near IIT, Hauz Khas, New Delhi‐16  28‐B/6, Jia Sarai, Near IIT 
 
Phone: 011‐26865455/+91‐9871145498 Hauz Khas, New Delhi‐16 
                                                   
                                             Website: www.physicsbyfiziks.com                                                                                          
                                                           Email: fiziks.physics@gmail.com                                                                   29 
fiziks
Institute for NET/JRF, GATE, IIT‐JAM, JEST, TIFR and GRE in PHYSICAL SCIENCES 
 
Ans. 22: (b), (d)
Solution: The relative speed between toy train B and rod is zero so no change in length of rod
from observer attached with toy train B
The relative speed between rod and table is u = 0.5c so observer attached with table will

u2
see length contraction l = l0 1 − = .866
c2
u' + v
The relative speed between rod and toy train A is u = where u ' = 0.5c, v = 0.5c so
u 'v
1+ 2
c

4c u2
u= will see length contraction as l = l0 1 − 2 = 0.6 lo
5 c
Ans. 23: (a), (c), (d)
Solution: As the elastically scattered particles have the same mass and the same kinetic energy,
θ
their momenta must make the same angle with the incident direction and have the
2
same magnitude. Conservation of energy and of momentum given
⎛θ ⎞
mc 2 + E0 = 2 E , p0 = 2 p cos ⎜ ⎟
⎝2⎠
where E, p are the energy and momentum of each scattered particle. Squaring both sides

of the energy equation we have m 2 c 4 + E02 + 2 E0 mc 2 = 4 ( p 2 c 2 + m 2 c 4 )

p02 c 2 E02 − m 2 c 4
E02 + 2 E0 mc 2 − 3m 2 c 4 = =
⎛θ ⎞ ⎛θ ⎞
cos 2 ⎜ ⎟ cos 2 ⎜ ⎟
⎝2⎠ ⎝2⎠

⎛θ ⎞ E02 − m 2 c 4 E0 + mc 2
⇒ cos ⎜ ⎟ = =
⎝2⎠ (E 0 − mc 2 )( E0 + 3mc 2 ) E0 + 3mc 2

⎛θ ⎞ 2 1 π
(c) v << c, E0 ≈ mc 2 cos ⎜ ⎟ ≈ = giving θ≈
⎝2⎠ 4 2 2

⎛θ ⎞
(d) v → c, E0 >> mc 2 cos ⎜ ⎟ ≈ 1 giving θ = 0 .
⎝2⎠
                                                                                
Head office  Branch office 
 
fiziks, H.No. 23, G.F, Jia Sarai,  Anand Institute of Mathematics, 
 
Near IIT, Hauz Khas, New Delhi‐16  28‐B/6, Jia Sarai, Near IIT 
 
Phone: 011‐26865455/+91‐9871145498 Hauz Khas, New Delhi‐16 
                                                   
                                             Website: www.physicsbyfiziks.com                                                                                          
                                                           Email: fiziks.physics@gmail.com                                                                   30 
fiziks
Institute for NET/JRF, GATE, IIT‐JAM, JEST, TIFR and GRE in PHYSICAL SCIENCES 
 
Ans. 24: (b), (c), (d)
m0 v 2 2c
Solution: p = 2 2m0 c = v⇒
v2 3
1−
c2
E 2 = p 2 c 2 + m02 c 4 = 8m02 c 2 + m02 c 4 = 9m02 c 4 ⇒ E = 3m0 c 2

Kinetic T = 3m0 c 2 − m0 c 2 = 2m0 c 2


Ans. 25: (a), (c)
3 4c
Solution: For spaceship Fiziks 5 = so v =
v2 5
1−
c2
12 12c
Speed of Akashganga with respect to earth 13 = u=
u2 13
1−
c2
4c u + v 8c
For Velocity of Akashganga with respect to Fiziks v = − uf = =
5 uv
1 + 2 17
c
Ans. 26: (a), (b), (d)

Solution: τ − → π − +ντ
From conservation of energy M τ c 2 = E π + E ν .

E 2π = p 2 c 2 + M π2 c 4 and E ν2 = p 2 c 2 since momentum of π − and ν τ is same.

M 2πc4
M τc 2 = E π + E ν , M π 2 c 4 = E π2 − E ν2 ⇒ E π − E ν =
M τc2

Eπ − Eν =
M πc2
and E π + E ν = M τ c ⇒ E π =
2
M 2τ + Mπ2 c2
. and Eν =
(
( M τ2 − M π2 ) c 2 . )
Mτ 2Mτ 2Mτ

Velocity of π − , Eπ =
(M 2
τ + M π2 ) c 2
=
M πc2 ⎛ v2 ⎞
⇒ ⎜1 − 2 ⎟ =
4 M π2 M τ2
⎝ c ⎠ ( M τ2 + M π2 )
2
2M τ v2
1−
c2

v2 4M π2 M τ2 v 2 M τ4 + M π4 + 2 M τ2 M π2 − 4 M π2 M τ2 ⎛ M τ2 − M π2 ⎞
⇒ = 1 − ⇒ = ⇒ v = ⎜ 2 2 ⎟
c.
c2 ( τ π) 2 2
(M + M ) 2 2 +
2
M 2
+ M c 2
⎝ M τ M π ⎠
τ π

                                                                                
Head office  Branch office 
 
fiziks, H.No. 23, G.F, Jia Sarai,  Anand Institute of Mathematics, 
 
Near IIT, Hauz Khas, New Delhi‐16  28‐B/6, Jia Sarai, Near IIT 
 
Phone: 011‐26865455/+91‐9871145498 Hauz Khas, New Delhi‐16 
                                                   
                                             Website: www.physicsbyfiziks.com                                                                                          
                                                           Email: fiziks.physics@gmail.com                                                                   31 
fiziks
Institute for NET/JRF, GATE, IIT‐JAM, JEST, TIFR and GRE in PHYSICAL SCIENCES 
 
Ans. 27: (a), (d)
mo c 2 5m c 2
Solution: If kinetic energy of particle is T = then total energy is E = T + mo c 2 = 0 and
4 4
3mo c
momentum is E 2 = p 2 c 2 + m02 c 4 ⇒ p =
4
3mo c
Then from conservation of momentum p1 − p2 =
4
5mo c 2
And from conservation of energy E1 + E2 = p1c + p2 c = , p1 = m0 c
4
Ans. 28: (a), (d)
v−u
Solution: The speed of bullet with respect to target ut = and the bullet have to travel
vu
1− 2
c

v2
distance from target reference L1 = L 1 − = Lγ 1
c2
L1
So time of that bullet hit the target from reference of target
ut
( )
= γ 1 L 1 − uv / c 2 / ( u − v )

u−v
The speed of target with respect to bullet ub = and the bullet have to travel
vu
1− 2
c

u2
distance from target reference L2 = L 1 − = Lγ 2
c2
L2
So time of that bullet hit the target from reference of target
ut
( )
= γ 2 L 1 − uv / c 2 / ( u − v )

                                                                                
Head office  Branch office 
 
fiziks, H.No. 23, G.F, Jia Sarai,  Anand Institute of Mathematics, 
 
Near IIT, Hauz Khas, New Delhi‐16  28‐B/6, Jia Sarai, Near IIT 
 
Phone: 011‐26865455/+91‐9871145498 Hauz Khas, New Delhi‐16 
                                                   
                                             Website: www.physicsbyfiziks.com                                                                                          
                                                           Email: fiziks.physics@gmail.com                                                                   32 
fiziks
Institute for NET/JRF, GATE, IIT‐JAM, JEST, TIFR and GRE in PHYSICAL SCIENCES 
 
NAT
Ans. 29: 0
x ' + vt '
x= ⇒v=0
v2
1− 2
c
Ans. 30: 40

Solution: x2' − x1' = 9× 109 m and t 2' − t1' = 0 . Then

⎛ ⎞ ⎛ ⎞
⎜ t ' + v x' ⎟ ⎜ t1 + v x' ⎟
⎜ 2 c2 2
t 2 − t1 = ⎜
⎟ ⎜

1
c 2 ⎟ ⇒ t − t = t 2 − t1 + v x 2 − x1 = v x 2 − x1 .
1 ' ' ' ' ' '
( ) ( )
⎟ ⎜ 2 ⎟
2 1 2
c2
⎜ 1− v
2
⎟ ⎜ 1− v ⎟ v2 c v2 v2
⎜ ⎟ ⎜ ⎟ 1 − 1 − 1 −
⎝ c2 ⎠ ⎝ c2 ⎠ c2 c2 c2
Put v = 0.8c ⇒ t2 − t1 ≅ 40 sec
Ans. 31: 0.3
Solution: Δt = 0 , t 2′ − t1′ = 5μs , x′2 − x1′ = 5km v=?

⎛−v⎞ ⎛−v⎞ ⎡ ′ ′
t 2′ + ⎜ 2 ⎟ x 2′ t1′ + ⎜ 2 ⎟ x1′ ⎢(t 2 − t1 ) − v2 (x′2 − x1′ )⎤⎥
t 2 − t1 = ⎝c ⎠ − ⎝c ⎠ ⇒⎣ c ⎦ =0
v2 v2 v2
1− 2 1− 2 1− 2
c c c

v v 5 × 10 −6
⇒ 5 × 10 −6 − × 5 × 10 3
= 0 ⇒ = = 10 −9 ⇒ v = 3 ×108 × c × 10−9 = 0.3 c
c 2
c 2
5 × 10 3

Ans. 32: 0.928


4c c 3
Solution: v = , u x′ = c cos 60o = , u ′y = c sin 60o = c
5 2 2
c 4
+ c
2 5 13c 13
Now u x = = ⇒ cos θ =
c 4c 14 14
1+ ⋅ 2
2 5c

                                                                                
Head office  Branch office 
 
fiziks, H.No. 23, G.F, Jia Sarai,  Anand Institute of Mathematics, 
 
Near IIT, Hauz Khas, New Delhi‐16  28‐B/6, Jia Sarai, Near IIT 
 
Phone: 011‐26865455/+91‐9871145498 Hauz Khas, New Delhi‐16 
                                                   
                                             Website: www.physicsbyfiziks.com                                                                                          
                                                           Email: fiziks.physics@gmail.com                                                                   33 
fiziks
Institute for NET/JRF, GATE, IIT‐JAM, JEST, TIFR and GRE in PHYSICAL SCIENCES 
 
Ans. 33: 15
v
1+
Solution: t1 = t 0 c = 10 1 + 0.6 = 10 × 2 = 20sec
,
v 1 − 0.6
1−
c

v
1−
t2 = t0 c = 10 1 − 0.6 = 10 × 1 = 5sec ⇒ t − t = 15sec
1 2
v 1 + 0.6 2
1+
c
Ans. 34: 0.97
u x' + v 2c c c
Solution: u x = u x' = ,v = − ux =
u' v 3 2 4
1 + x2
c

u2x u2
l = l0 1 − = 0.87 l0 l = l0 1 − = 0.97
c2 c2
Ans. 35: 2.18

t0 v2
Solution: t = , t0 = t × 1 − ⇒ t0 = 5 ×10−8 × .19 ⇒ 2.18 ×10−8 s
v2 c2
1−
c2
Ans. 36: 1.86
MeV
Solution: Since E = 315 MeV and m0 = 105 .
c2
m0 c 2 m0 c 2 105
E = mc 2 ⇒ E = ⇒ 315 = ⇒ 315 = ⇒ v = 0.94 c .
v2 v2 v2
1− 2 1− 2 1− 2
c c c
t0 2.2 × 10 −6
Now, t = , t 0 = 2.2μs ⇒ t = ⇒ t = 6.6 μs
v2 8
1− 2 1−
c 9

Now the distance traversed by muon is vt = 0.94c × 6.6 × 10 −6 = = 1.86 km .

                                                                                
Head office  Branch office 
 
fiziks, H.No. 23, G.F, Jia Sarai,  Anand Institute of Mathematics, 
 
Near IIT, Hauz Khas, New Delhi‐16  28‐B/6, Jia Sarai, Near IIT 
 
Phone: 011‐26865455/+91‐9871145498 Hauz Khas, New Delhi‐16 
                                                   
                                             Website: www.physicsbyfiziks.com                                                                                          
                                                           Email: fiziks.physics@gmail.com                                                                   34 
fiziks
Institute for NET/JRF, GATE, IIT‐JAM, JEST, TIFR and GRE in PHYSICAL SCIENCES 
 
Ans. 37: 30.1

Solution: H B → PH + Z B

From conservation of momentum 0 = P1 + P2 ⇒ P1 = − P2 ⇒ P1 = P2

Now EH B = EPH + EZ B ⇒ EPH + EZ B = M H B c 2

E P2H = P12 c 2 + 0 and EZ2B = P22c 2 + M Z2B c 4

(
⇒ EZ B − E PH )( E ZB )
+ E PH = M Z2B c 4 ∵ P1 = P2

M Z2B c 4 M Z2B c 2
⇒ EZ B − EPH = 2
= ∵ EZ B + EPH = M H B c 2
M HB c M HB

⇒ 2 EPH = M H B c − 2
M z2B c 2
⇒ EPH =
(M 2
HB )
− M z2B c 2
M HB M HB
⎛ 125 ×125 − 90 × 90 ⎞ c
4
⇒ EPH =⎜ ⎟ × 4 = 30.1GeV
⎝ 2 ×125 ⎠ c
Ans. 38: 0.99

mv 100
= 5mc v=
v2 101
1−
c2
Ans. 39: 0.866
Solution: K .E = mc 2 − m0 c 2 , rest mass energy = m0 c 2

K .E . = rest mass energy


mc 2 − m0 c 2 = m0 c 2

mc 2 = 2m0 c 2

m0 1 ⎛ v2 ⎞ v2 3
c 2 = 2m0 c 2 ⇒ = 2 ⇒ 4⎜⎜1 − 2 ⎟⎟ = 1 ⇒ 4 2 = 3 ⇒ v = c
v2 v2 ⎝ c ⎠ c 2
1− 1−
c2 c2
Ans. 40: 2
Solution: E = T + m0 c 2 , E = m0 c 2 + m0 c 2 = 2m0c 2

                                                                                
Head office  Branch office 
 
fiziks, H.No. 23, G.F, Jia Sarai,  Anand Institute of Mathematics, 
 
Near IIT, Hauz Khas, New Delhi‐16  28‐B/6, Jia Sarai, Near IIT 
 
Phone: 011‐26865455/+91‐9871145498 Hauz Khas, New Delhi‐16 
                                                   
                                             Website: www.physicsbyfiziks.com                                                                                          
                                                           Email: fiziks.physics@gmail.com                                                                   35 
fiziks
Institute for NET/JRF, GATE, IIT‐JAM, JEST, TIFR and GRE in PHYSICAL SCIENCES 
 
Ans. 41: 0.866
m0 c 2 3
Solution: E = m0 c + m0 c = 2m0 c =
2 2 2
, v= c
v 2 2
1−
c2
Ans. 42: 1.66
u x' + v 0.5c + 0.5c c 4c
Solution: Speed of cosmic particle with respect u x = = = = ,
'
ux v (0.5) × (0.5)c 2
1 + .25 5
1+ 2 1+
c c2
4c mo 5mo
u 'y = 0 , u z' = 0 u = u x' 2 + u 'y 2 + u 'y 2 = ,m = =
5 u 2 3
1−
c2
Ans. 43: 0.33
Solution: Using conservation of momentum
c 2m
given m1 = m, v = , m2 = , v1 = ?
2 5
mv 2.m1v1 25c
= 2 p1 = ⇒ v1 = = .33c
v 2
v2 73
1− 1− 1
c2 c2

Ans. 44: 39
Δτ
Solution: Δt = , Δτ = 1.77 ×10−8 sec v = 0.99c ,
2
v
1−
c2
Δt = 1.3 ×10−7 sec v × Δt = 0.99c ×1.3 ×10−7 sec = 39m.
Ans. 45: 0.916

v2 1 l 3
= l0 cos θ 1 − ( 0.8) ⇒ lx = l0 × × 0.6 = 0.3 l0 and l y = l0 sin θ = 0
2
Solution: lx = l0 x 1 − 2
c 2 2
2
⎛ 3l0 ⎞
( 0.3l0 )
2
New length l = +⎜ = l0 0.09 + 0.75 = 0.916 l0
⎜ 2 ⎟⎟
⎝ ⎠

                                                                                
Head office  Branch office 
 
fiziks, H.No. 23, G.F, Jia Sarai,  Anand Institute of Mathematics, 
 
Near IIT, Hauz Khas, New Delhi‐16  28‐B/6, Jia Sarai, Near IIT 
 
Phone: 011‐26865455/+91‐9871145498 Hauz Khas, New Delhi‐16 
                                                   
                                             Website: www.physicsbyfiziks.com                                                                                          
                                                           Email: fiziks.physics@gmail.com                                                                   36 
fiziks
Institute for NET/JRF, GATE, IIT‐JAM, JEST, TIFR and GRE in PHYSICAL SCIENCES 
 
2. Modern Physics
2.1 Black Body Radiation

A black body is an idealized physical body that absorbs all incident electromagnetic
radiation, regardless of frequency or angle of incidence.

A black body in thermal equilibrium (that is, at a constant temperature) emits


electromagnetic radiation called black body radiation.

When an object is heated, it radiated electromagnetic energy as result of thermal agitation


of electrons in its surface .the intensity of radiation depends on its frequency and on the
temperature, the light it emits ranges over the entire spectrum.

An object in thermal equilibrium with its surrounding radiates as much energy it


absorbers. A Black body is perfect absorber as well perfect emitter of radiation.

2.1.1 Wien’s Distribution Law

Wien's approximation (also sometimes called Wien's law or the Wien distribution
law) is a law of used to describe the spectrum of thermal radiation (frequently called
the blackbody function). The equation does accurately describe the short wavelength
(high frequency) spectrum of thermal emission from objects, but it fails to accurately fit
the experimental data for long wavelengths (low frequency) emission.

The law may be written as


h
2h 3  kBT
I ( , T )  2 e
c

where I ( , T ) is the amount of energy per unit surface area per unit time per unit solid
angle per unit frequency emitted at a frequency  .

T is the temperature of the black body, h is planks constant, c is the speed of light,

k B is Boltzmann’s constant .

                     
Head office  Branch office 
 
fiziks, H.No. 23, G.F, Jia Sarai,  Anand Institute of Mathematics, 
 
Near IIT, Hauz Khas, New Delhi‐16  28‐B/6, Jia Sarai, Near IIT 
 
Phone: 011‐26865455/+91‐9871145498 Hauz Khas, New Delhi‐16 
              
                   Website: www.physicsbyfiziks.com                        
                           Email: fiziks.physics@gmail.com                       37 
fiziks
Institute for NET/JRF, GATE, IIT‐JAM, JEST, TIFR and GRE in PHYSICAL SCIENCES 
 
hc
2hc 2 
 k BT
This equation may also be written as I ( , T )  e
5

Where I ( , T ) is the amount of energy per unit surface area per unit time per unit solid
angle per unit wavelength emitted at a wavelength λ.

2.1.2 Rayleigh’s Energy Density Distribution

Total energy per unit volume in the cavity in the frequency interval from  to   d is
8 k BT 2
given by u ( ) d   d
c3

But above relation does not describe the experimental trend at higher frequency. The
equation does accurately describe the long wavelength (short frequency) spectrum of
thermal emission from objects, but it fails to accurately fit the experimental data for
short wavelengths (high frequency) emission.

8 k BT
In term of wavelength the energy density is given by u ( ) d   d
4
Spectral energy density, u(v)dv

Rayleigh - Jeans

Observed

11014 21014 31014 41014


Frequency, vHz
Fig: Comparison of the Rayleigh-Jeans formula for the spectrum of the radiation from
a blackbody at 1500 K with the observed spectrum. The discrepancy is known as the
ultraviolet catastrophe because it increases with increase frequency. This failure of
classical physics led Planck to the discovery that radiation is emitted in quanta whose
energy is h .

                     
Head office  Branch office 
 
fiziks, H.No. 23, G.F, Jia Sarai,  Anand Institute of Mathematics, 
 
Near IIT, Hauz Khas, New Delhi‐16  28‐B/6, Jia Sarai, Near IIT 
 
Phone: 011‐26865455/+91‐9871145498 Hauz Khas, New Delhi‐16 
              
                   Website: www.physicsbyfiziks.com                        
                           Email: fiziks.physics@gmail.com                       38 
fiziks
Institute for NET/JRF, GATE, IIT‐JAM, JEST, TIFR and GRE in PHYSICAL SCIENCES 
 
2.1.3 Planks Radiation Formula

T  1800 

Spectral energy density,


T  1200 

0 2  10 14 4  10 14 6  10 14 Hz

Visible light
Frequency, v
Fig: Blackbody spectra. The spectral distribution of energy in the radiation
depends only on the temperature of the body.

With assumption that radiation has discrete energy analogous to oscillator which is given
by

En  nh where n  0,1, 2...

Total energy per unit volume in the cavity in the frequency interval from  to   d is
given by

8 h 3
u ( ) d 
c 3 e h / kBT  1

where for high frequency (low wavelength) ii will approach to Wien’s distribution and
for low frequency(high wavelength) it will approach to Raleigh- Jeans formula.

                     
Head office  Branch office 
 
fiziks, H.No. 23, G.F, Jia Sarai,  Anand Institute of Mathematics, 
 
Near IIT, Hauz Khas, New Delhi‐16  28‐B/6, Jia Sarai, Near IIT 
 
Phone: 011‐26865455/+91‐9871145498 Hauz Khas, New Delhi‐16 
              
                   Website: www.physicsbyfiziks.com                        
                           Email: fiziks.physics@gmail.com                       39 
fiziks
Institute for NET/JRF, GATE, IIT‐JAM, JEST, TIFR and GRE in PHYSICAL SCIENCES 
 
2.2 Photo Electric Effect

In the photoelectric effect, electrons are emitted from solids, liquids or gases when they
absorb energy from light. Electrons emitted in this manner may be called photoelectrons.

The photoelectric effect requires photons with energies from a few electron volts to over
1MeV in high atomic number elements. Study of the photoelectric effect led to important
steps in understanding the quantum nature of light and electrons and influenced the
formation of the concept of wave-particle duality. It also led to Max Planck's discovery of
quantized energy and the Planck Relation  E  h  , which links a photon's frequency

with its energy. The factor h is known as the Planck constant.

Experimental observation

Light

 Electrons 

Evacuated quartz tube


V

Fig: Experimental observation of the photoelectric effect

1. Because electromagnetic wave is concentrated in photons and not spread out, there
should be no delay in the emission of photoelectron as light falls on the matter.

2. All photons of frequency  have the same energy so changing the intensity of
monochromatic light beam will change the number of photoelectrons not their
energies.

3. The higher the frequency , the greater the photon energy  E  h  and so the more

energy the photoelectron have.

                     
Head office  Branch office 
 
fiziks, H.No. 23, G.F, Jia Sarai,  Anand Institute of Mathematics, 
 
Near IIT, Hauz Khas, New Delhi‐16  28‐B/6, Jia Sarai, Near IIT 
 
Phone: 011‐26865455/+91‐9871145498 Hauz Khas, New Delhi‐16 
              
                   Website: www.physicsbyfiziks.com                        
                           Email: fiziks.physics@gmail.com                       40 
fiziks
Institute for NET/JRF, GATE, IIT‐JAM, JEST, TIFR and GRE in PHYSICAL SCIENCES 
 
2.2.1 Threshold Frequency and Work Function

For a given metal, there exists a certain minimum frequency of incident radiation below
which no photoelectrons are emitted. This frequency is called the threshold frequency or
critical frequency ( 0 ).

There must be minimum energy  for an electron to escape from a particular metal
surface; this energy is known as work function which is given by   h 0 .

Einstein equation of photoelectric effect


E  hv0 E  hv

KE max  hv  hv0

KE  0

Metal

Fig. If the energy hv 0 (the work function of the surface) is needed to remove
an electron from a metal surface, the maximum electron kinetic energy will
be hv  hv 0 when light of frequency v is directed at the surface.

When a metal is irradiated with light, electron may get emitted. Kinetic energy of
photoelectron observed when irradiated with a light of frequency    o , where  o is

threshold frequency is given by KEmax where KEmax  h  h 0

This maximum kinetic energy is equivalent to Stopping potential Vs which is energy


required to stop electron which contain maximum kinetic energy.
Then eVs  h  h 0 which is known as Einstein equation.

                     
Head office  Branch office 
 
fiziks, H.No. 23, G.F, Jia Sarai,  Anand Institute of Mathematics, 
 
Near IIT, Hauz Khas, New Delhi‐16  28‐B/6, Jia Sarai, Near IIT 
 
Phone: 011‐26865455/+91‐9871145498 Hauz Khas, New Delhi‐16 
              
                   Website: www.physicsbyfiziks.com                        
                           Email: fiziks.physics@gmail.com                       41 
fiziks
Institute for NET/JRF, GATE, IIT‐JAM, JEST, TIFR and GRE in PHYSICAL SCIENCES 
 

Maximum photoelectron
3

Energy, eV 2 Sodium
Cesium

1
Calcium

v0 v0 v0
0 2 4 6 8 10 121014
Frequency,Hz
Fig: Maximum photoelectron kinetic energy KEmax versus
frequency of incident light for three metal surfaces.

31 Frequency  v Light intensity


 Constant  Constant
Photoelectron current

v1  v2  v3
21
Photoelectron

v1
v2
1 v3

0 V0 V 0 V0 3 V0 2 V0 1 V


Retarding Retarding potential
Fig: Photoelectron current is proportional to Fig: The extinction voltage V 0 , and hence the
light intensity I for all retarding voltages. The maximum photoelectron energy, depends on
extinction voltage v0 which corresponds to the the frequency of the light. When the
maximum photoelectron energy, is the same retarding potential is V = 0, the photoelectron
for all intensities of light of the same current is the same for light of a given
frequency v. intensity regardless of its frequency.

                     
Head office  Branch office 
 
fiziks, H.No. 23, G.F, Jia Sarai,  Anand Institute of Mathematics, 
 
Near IIT, Hauz Khas, New Delhi‐16  28‐B/6, Jia Sarai, Near IIT 
 
Phone: 011‐26865455/+91‐9871145498 Hauz Khas, New Delhi‐16 
              
                   Website: www.physicsbyfiziks.com                        
                           Email: fiziks.physics@gmail.com                       42 
fiziks
Institute for NET/JRF, GATE, IIT‐JAM, JEST, TIFR and GRE in PHYSICAL SCIENCES 
 
Example: When light of a given wavelength is incident on metallic surface ,the stopping
potential for the photoelectrons is 3.2V .if a second light source whose wavelength is
double that of the first is used , the stopping potential drop to 0.8V .
(a) Calculate the wavelength of first radiation
(b) The work function and the cutoff frequency of the metal.
Solution: Let us assume work function of metal is given by  .
For wavelength 1 and 2 stopping potential is given by Vs1  3.2V and Vs2  0.8 where
2  21 .
hc hc hc
(a) eVs1    ……..(i) eVs2     eVs2    ………...(ii)
1 2 21
hc
Solving equation (i) and (ii) 1   2.6  106
2e(Vs1  Vs2 )

(b) Eliminating 1 from (i) and (ii)   e(Vs  2Vs ) = 3.84  10 19 J
1 2


Cutoff frequency   = 5.8  1014
h
Example: Ultraviolet light of wavelength 350nm and intensity 1 W / m 2 is directed at potassium
surface.
(a) Find the maximum kinetic of photoelectrons
(b) If 0 .5% of incident photons produce photoelectrons, how many are emitted per
second if the potassium surface has area of 1cm 2
Solution: It is given hc  1.24  10 6 eV .m and work function of potassium is 2.2eV
hc 1.24  10 6 eV .m
(a) Energy of photon is   3.5eV  5.68  10 19 J
 350  10 9
hc
Maximum kinetic energy is given by Emax    = 3.5eV  2.2eV  1.3eV

(b) No. of photon that reach the surface per second is given by
E / t ( p / A). A (1W / m 2 )1 104 m 2
np   = 19
 1.76 1014
Ep Ep 5.68  10 J / photon

No of photo electron is ne  n p  0.005 = 8.8  1011 photo electrons/second .


                     
Head office  Branch office 
 
fiziks, H.No. 23, G.F, Jia Sarai,  Anand Institute of Mathematics, 
 
Near IIT, Hauz Khas, New Delhi‐16  28‐B/6, Jia Sarai, Near IIT 
 
Phone: 011‐26865455/+91‐9871145498 Hauz Khas, New Delhi‐16 
              
                   Website: www.physicsbyfiziks.com                        
                           Email: fiziks.physics@gmail.com                       43 
fiziks
Institute for NET/JRF, GATE, IIT‐JAM, JEST, TIFR and GRE in PHYSICAL SCIENCES 
 
Example: In a photoelectric experiment, it was found that the stopping potential decreases from
1.85 V to 0.82 V as the wavelength of the incident light is varied from 300 nm to 400 nm.
Calculate the value of the Planck constant from these data.
hc
Solution: The maximum kinetic energy of a photoelectron is K max  

K max hc 
and the stopping potential is V   
e e e
If V1 ,V2 are the stopping potentials at wavelengths 1 and 2 respectively,

hc  hc  hc  1 1 
V1   and V2    V1  V2    
1e e 2 e e e  1 2 

eV1  V2  e1.85V  0.82V 


or, h 
1 1   1 1 
c   c  
 1 2 
9 9
 300  10 m 400  10 m 
1.03eV
  4.12  10 15 eV  s
 1
 
3  10 8 m / s   10 7 m 1 
 12 
Example: A monochromatic light of wavelength  is incident on an isolated metallic sphere of
radius a . The threshold wavelength is  0 which is larger than  . Find the number of
photoelectrons emitted before the emission of photoelectrons will stop.
Solution: As the metallic sphere is isolated, it becomes positively charged when electrons are
ejected from it. There is an extra attractive force on the photoelectrons. If the potential of
the sphere is raised to V, the electrons should have a minimum energy   eV to be able
to come out. Thus, emission of photoelectrons will stop when

hc hc hc  1 1 
   eV   eV or, V    
 0 e   0 

The charge on the sphere needed to take its potential to V is Q  4 0 a V .

Q 4 0 aV 4 0 aV 1 1 
The number of electrons emitted is, therefore, n     
e e e   0 

                     
Head office  Branch office 
 
fiziks, H.No. 23, G.F, Jia Sarai,  Anand Institute of Mathematics, 
 
Near IIT, Hauz Khas, New Delhi‐16  28‐B/6, Jia Sarai, Near IIT 
 
Phone: 011‐26865455/+91‐9871145498 Hauz Khas, New Delhi‐16 
              
                   Website: www.physicsbyfiziks.com                        
                           Email: fiziks.physics@gmail.com                       44 
fiziks
Institute for NET/JRF, GATE, IIT‐JAM, JEST, TIFR and GRE in PHYSICAL SCIENCES 
 
2.3 Compton Scattering
Compton scattering is an inelastic scattering of a photon by a free charged particle,
usually an electron. It results in a decrease in energy (increase in wavelength) of the
photon (which may be an X-ray or gamma ray photon), called the Compton Effect. Part
of the energy of the photon is transferred to the scattering electron. Inverse Compton
scattering also exists, in which a charged particle transfers part of its energy to a photon.
This experiment give experimental prove of particle aspect of photon (light).

E  m0 c 2 E  hv 
Incident photon p0  p  hv  / c

E  hv Target
p  hv / c electron
E  m02 c 4  p 2 c 2
p p
Scattered
electron
a 
The scattering of photon of energy h by an electron rest mass m0 . After scattering

photon scattered at angle  and electron scattered at angle  . The scattered photon have

energy h ' and electron have energy E .


The vector diagram of the momenta and their components of the incident and scattered
photon and scattered electron shown in vector diagram
From the conservation of energy hv hv
sin 
c hv  c
loss in photon energy = gain in electron energy cos 
 c
hv  hv  KE hv / c

E hv p p cos 
The momentum of photon is given p  
c c p sin 
From conservation of momentum in x direction
hv hv 
0 cos   p cos 
c c

                     
Head office  Branch office 
 
fiziks, H.No. 23, G.F, Jia Sarai,  Anand Institute of Mathematics, 
 
Near IIT, Hauz Khas, New Delhi‐16  28‐B/6, Jia Sarai, Near IIT 
 
Phone: 011‐26865455/+91‐9871145498 Hauz Khas, New Delhi‐16 
              
                   Website: www.physicsbyfiziks.com                        
                           Email: fiziks.physics@gmail.com                       45 
fiziks
Institute for NET/JRF, GATE, IIT‐JAM, JEST, TIFR and GRE in PHYSICAL SCIENCES 
 
hv 
Conservation of momentum in y direction 0  sin   p sin 
c
From above two equation
hv hv hv
p cos    cos  and p sin   sin 
c c c
Squaring and adding one will get p 2 c 2  hv   2hv hv  cos   hv  …….(i)
2 2

Kinetic energy of electron is given hv  hv  = p 2 c 2  m02 c 4  m0 c 2

p 2 c 2  hv   2hv hv   hv   2m0 c 2 hv  hv  …….(ii)


2 2

Equating equation (i) and (ii) 2m0 c 2 hv  hv   2hv hv 1  cos  

h  1 ' 1
    1  cos   where  and  '
m0 c c  c 
h
     C 1 cos  where C  = c  2.46  1012 m
m0 c
Value of    '   for different scattering angle  shown in figure


  0o
  45 o
Relative
Relative

Waveleng1th Waveleng1th

 
  90 o
  135 o
Relative

Relative

Waveleng1th Waveleng1th

                     
Head office  Branch office 
 
fiziks, H.No. 23, G.F, Jia Sarai,  Anand Institute of Mathematics, 
 
Near IIT, Hauz Khas, New Delhi‐16  28‐B/6, Jia Sarai, Near IIT 
 
Phone: 011‐26865455/+91‐9871145498 Hauz Khas, New Delhi‐16 
              
                   Website: www.physicsbyfiziks.com                        
                           Email: fiziks.physics@gmail.com                       46 
fiziks
Institute for NET/JRF, GATE, IIT‐JAM, JEST, TIFR and GRE in PHYSICAL SCIENCES 
 
Example: X-ray of wavelength 10.0 pm are scattered from a target

(a) Find the wavelength of the X-rays scattered through 450


(b) Find the maximum wavelength present in the scattered X-rays
(c) Find the maximum kinetic energy of the recoil electrons .

h
It is given that (Where C  = 2.46  10 12 m )
m0 c

Solution: (a)  '    c (1  cos  )  '    c (1  cos  ) = 10.7 pm

(b)  '    c (1  cos  ) is maximum for    max


'
   2c = 14.9 pm

hc hc
(c) Kinetic energy of electron is given by hv  hv  KE = 
 '

hc hc
For maximum kinetic energy KEmax   = 6.54  10 15 J
 
'
max

sin 
Example: In a Compton scattering prove that tan   where  and  are the angle of
'
 cos 

recoil of electron and scattering angle of photon .
Solution: From conservation of momentum

hv
We will get p sin   sin  ..……..(A)
c
hv hv
p cos    cos  ……….(B)
c c
(h ' / c) sin  sin  sin 
tan   = tan   = tan   ' proved.
(h / c)  (h / c) cos 
'
 
 cos   cos 
' 

                     
Head office  Branch office 
 
fiziks, H.No. 23, G.F, Jia Sarai,  Anand Institute of Mathematics, 
 
Near IIT, Hauz Khas, New Delhi‐16  28‐B/6, Jia Sarai, Near IIT 
 
Phone: 011‐26865455/+91‐9871145498 Hauz Khas, New Delhi‐16 
              
                   Website: www.physicsbyfiziks.com                        
                           Email: fiziks.physics@gmail.com                       47 
fiziks
Institute for NET/JRF, GATE, IIT‐JAM, JEST, TIFR and GRE in PHYSICAL SCIENCES 
 
Example: Show that the maximum kinetic energy transferred to proton when hit by photon of
h
energy h is kmax 
mpc2
1
2h
1 1 h(1  cos  )
Solution:  
 '
 mpc2

h h
Kinetic energy KE  h  h ' = h  =
h (1  cos  ) mpc2
1 1
mpc2 h (1  cos  )
h
For maximum kinetic energy     , K max 
mpc2
1
2h
Example: High energy photons (   rays ) are scattered from electrons initially at rest. Assume
the photons are backscattered and their energies are much larger than the electron’s rest-
mass energy, E  me c 2 .
(a) Calculate the wavelength shift
(b) Show that the energy of the scattered photons is half the rest mass energy of the
electron, regardless of the energy of the incident photons.
(c) Calculate the electron’s recoil kinetic energy if the energy of the incident photons is
150 MeV.
Solution: (a) In the case where the photons backscatter (i.e.,    ), the wave length shift
becomes
h
       1  cos    2h
me c me c

The numerical value of  is easy to obtain by making use of c  197.33  10 15 MeV m

and me c 2  0.511 MeV :

4 4  3.14  197.33  10 15 MeVm


  2
  4.8  10 12 m
me c 0.511 MeV

                     
Head office  Branch office 
 
fiziks, H.No. 23, G.F, Jia Sarai,  Anand Institute of Mathematics, 
 
Near IIT, Hauz Khas, New Delhi‐16  28‐B/6, Jia Sarai, Near IIT 
 
Phone: 011‐26865455/+91‐9871145498 Hauz Khas, New Delhi‐16 
              
                   Website: www.physicsbyfiziks.com                        
                           Email: fiziks.physics@gmail.com                       48 
fiziks
Institute for NET/JRF, GATE, IIT‐JAM, JEST, TIFR and GRE in PHYSICAL SCIENCES 
 
(b) Since the energy of the scattered photons E  is related to the wavelength   by
hc
E  , equation ) yields

hc hc me c 2 me c 2
E     ,
    2h / me c  me c 2  / hc   2 me c 2 / E  2
c
where E  is the energy of the incident photons. If E  me c 2 we can approximate by


E 
me c 2  2m e c 2  me c 2
1   

me c 2


me c 2
 0.25 MeV
 
2  E  2 E 2

(c) If E  15 MeV the kinetic energy of the recoiling electrons can be obtained from the
conservation of energy
K e  E  E '  150 MeV  0.25 MeV  149.75 MeV

                     
Head office  Branch office 
 
fiziks, H.No. 23, G.F, Jia Sarai,  Anand Institute of Mathematics, 
 
Near IIT, Hauz Khas, New Delhi‐16  28‐B/6, Jia Sarai, Near IIT 
 
Phone: 011‐26865455/+91‐9871145498 Hauz Khas, New Delhi‐16 
              
                   Website: www.physicsbyfiziks.com                        
                           Email: fiziks.physics@gmail.com                       49 
fiziks
Institute for NET/JRF, GATE, IIT‐JAM, JEST, TIFR and GRE in PHYSICAL SCIENCES 
 
2.4 Bohr Atomic Model
In atomic physics, the Bohr model, introduced by Niles Bohr in 1913, depicts the atom
as small, with a positively charged nucleus surrounded by electrons that travel in circular
orbits around the nucleus-similar in structure to the solar system, but with attraction
provided by electrostatic forces rather than gravity.
He suggested that electrons could only have certain classical motions:

1. Electrons in atoms orbit the nucleus.

2. The electrons can only orbit stably, without radiating, in certain orbits (called by Bohr
the "stationary orbits"): at a certain discrete set of distances from the nucleus. These
orbits are associated with definite energies and are also called energy shells or energy
levels. In these orbits, the electron's acceleration does not result in radiation and energy
loss as required by classical electromagnetic.

3. Electrons can only gain and lose energy by jumping from one allowed orbit to another,
absorbing or emitting electromagnetic radiation with a frequency ν determined by the
energy difference of the levels according to the Planck relation:

E  E2  E1  h

where h is Planck’s constant . The frequency of the radiation emitted at an orbit of


period T is as it would be in classical mechanics; it is the reciprocal of the classical orbit
1
period:  
T

2.4.1 Bohr Quantization Rule

The significance of the Bohr model is that the laws of classical mechanics apply to the
motion of the electron about the nucleus only when restricted by a quantum rule. The
angular momentum L is restricted to be an integer multiple of a fixed unit:

L  n where n  1, 2, 3... and

h
n is called the principle quantum number , and   . The lowest value of n is 1this
2
gives a smallest possible orbital radius of 0.592 A0 known as the Bohr radius.

                     
Head office  Branch office 
 
fiziks, H.No. 23, G.F, Jia Sarai,  Anand Institute of Mathematics, 
 
Near IIT, Hauz Khas, New Delhi‐16  28‐B/6, Jia Sarai, Near IIT 
 
Phone: 011‐26865455/+91‐9871145498 Hauz Khas, New Delhi‐16 
              
                   Website: www.physicsbyfiziks.com                        
                           Email: fiziks.physics@gmail.com                       50 
fiziks
Institute for NET/JRF, GATE, IIT‐JAM, JEST, TIFR and GRE in PHYSICAL SCIENCES 
 
2.4.2 Hydrogen Atom

A hydrogen atom is an atom of the chemical element hydrogen. The electrically neutral
atom contains a single positively charged proton and a single negatively charged electron
bound to the nucleus by the Coulomb force. According to Bohr electron revolve about the
nucleus in different quantized circular orbits whose angular momentum is given by
L  n where n  1, 2, 3... .The electron is held in a circular orbit by electrostatic
attraction. The centripetal force is equal to the Coulomb force.

me v 2 ke e 2
 2
r r
1
where me is the electron's mass, e is the charge of the electron, ke  is Coulomb's
4 o
constant and v is velocity of electrons in orbit.

ke e 2
This equation determines the electron's speed at any radius: v 
me r

It also determines the electron's total energy at any radius:


me v 2 ke e 2
E 
2 r

ke e 2
Putting the value of v one will get E  
2r
The total energy is negative and inversely proportional to r. This means that it takes
energy to pull the orbiting electron away from the proton. For infinite values of r, the
energy is zero, corresponding to a motionless electron infinitely far from the proton. The
total energy is half the potential energy,
From the quantization the angular momentum

L  n  me vr  n

                     
Head office  Branch office 
 
fiziks, H.No. 23, G.F, Jia Sarai,  Anand Institute of Mathematics, 
 
Near IIT, Hauz Khas, New Delhi‐16  28‐B/6, Jia Sarai, Near IIT 
 
Phone: 011‐26865455/+91‐9871145498 Hauz Khas, New Delhi‐16 
              
                   Website: www.physicsbyfiziks.com                        
                           Email: fiziks.physics@gmail.com                       51 
fiziks
Institute for NET/JRF, GATE, IIT‐JAM, JEST, TIFR and GRE in PHYSICAL SCIENCES 
 
Substituting the expression for the velocity gives an equation for r in terms of n :

ke e2 me r  n

So that the allowed orbit radius at any n is:

n2 2
rn  2
 r1  0.53 1010 m (For n  1 )
ke e me

The smallest possible value of r in the hydrogen atom is called the Bohr radius r1 .

The energy of the nth level for any atom is determined by the radius and quantum number:

 
2
ke e 2 ke e 2 me 13.6
En   =  eV
2rn 2 2 n 2 n2

The combination of natural constants in the energy formula is called the Rydberg energy

k e 
2
2
e me
RE which is given by RE 
2 2

This expression is clarified by interpreting it in combinations which form more natural


units: me c 2 is the rest mass energy of the electron (511 keV ) .

ke e 2 1
  is the fine structure constant .
c 137

1
R
2
 
me c 2  2 = ( 1.097  107 m 1 )

For nuclei with Z protons, the energy levels are (to a rough approximation):

Z 2R
En   2
n

                     
Head office  Branch office 
 
fiziks, H.No. 23, G.F, Jia Sarai,  Anand Institute of Mathematics, 
 
Near IIT, Hauz Khas, New Delhi‐16  28‐B/6, Jia Sarai, Near IIT 
 
Phone: 011‐26865455/+91‐9871145498 Hauz Khas, New Delhi‐16 
              
                   Website: www.physicsbyfiziks.com                        
                           Email: fiziks.physics@gmail.com                       52 
fiziks
Institute for NET/JRF, GATE, IIT‐JAM, JEST, TIFR and GRE in PHYSICAL SCIENCES 
 
2.4.3 The Structure and Spectra of Hydrogenic Atoms
Atomic Spectra
The spectrum of atomic hydrogen arises from transitions between its permitted states.
 Each element has a characteristic line spectrum

 When an atomic gas is excited by passing electric current, it emits radiation. The
radiation has a spectrum which contains certain specific wavelength, called Emission
line spectrum.

 When while light is passed through a gas, gas absorb light of certain wavelength
present in its emission spectrum. Resulting spectrum is called Absorption line
spectrum.

 The number, intensity and exact wavelength of the lines in the spectrum depend on
Temperature, Pressure, Presence of Electric field, Magnetic field, and the motion of
the source.

Spectral series

When an electric discharge is passed through gaseous hydrogen, the H2 molecules


dissociate and the energetically excited H atoms that are produced emit light of discrete
frequencies, producing a spectrum of a series of 'lines'.

1 1 1
(i) Lyman Series:  R 2  2  ; n  2, 3, 4 ... (In U.V. region)
 1 n 
Where, R is Rydberg constant ( 1.097  107 m 1 )

1 1 1
(ii) Balmer Series:  R 2  2  ; n  3, 4, 5... (In Visible region)
 2 n 
n  3 for H  Line, n  4 for H  Line, n  5 for H  Line,

1 1 1 
(iii) Paschen series:  R 2  2  ; n  4, 5, 6 (Near Infra Red)
 3 n 

                     
Head office  Branch office 
 
fiziks, H.No. 23, G.F, Jia Sarai,  Anand Institute of Mathematics, 
 
Near IIT, Hauz Khas, New Delhi‐16  28‐B/6, Jia Sarai, Near IIT 
 
Phone: 011‐26865455/+91‐9871145498 Hauz Khas, New Delhi‐16 
              
                   Website: www.physicsbyfiziks.com                        
                           Email: fiziks.physics@gmail.com                       53 
fiziks
Institute for NET/JRF, GATE, IIT‐JAM, JEST, TIFR and GRE in PHYSICAL SCIENCES 
 
1  1 1 
(iv) Bracket Series:  R 2  2  ; n  5, 6, 7... (Infra Red)
 4 n 
1 1 1 
(v) Pfund Series:  R 2  2  ; n  6, 7, 8... (Far Infra Red)
 5 n 
If mass of nucleus is not considered as very heavy then reduce mass will take in to
mn me
account which is given by   where me and mn are mass of electron and
mn  me
nucleus respectively.
 Correction in Energy due to Reduced Mass

 (13.6)
En 
me n 2

 Correction in Rradius of the orbit

me n 2 (a0 )
rn 

 Variation of Rydberg constant with respect to Atomic Mass


R
2 2 me4
R
R for infinitely heavy nucleus:  
h3 
16
2 2  e 4  e
R for nucleus of mass M: Rm  Rm
h3
R  '
 Rm 
m
1 e 
mn

 Spectral Wavelength for Hydrogen like atoms

1  1 1 
 RZ 2  2  2  where R is Red berg and Z is atomic number.
 n 
 f ni 

                     
Head office  Branch office 
 
fiziks, H.No. 23, G.F, Jia Sarai,  Anand Institute of Mathematics, 
 
Near IIT, Hauz Khas, New Delhi‐16  28‐B/6, Jia Sarai, Near IIT 
 
Phone: 011‐26865455/+91‐9871145498 Hauz Khas, New Delhi‐16 
              
                   Website: www.physicsbyfiziks.com                        
                           Email: fiziks.physics@gmail.com                       54 
fiziks
Institute for NET/JRF, GATE, IIT‐JAM, JEST, TIFR and GRE in PHYSICAL SCIENCES 
 
2.4 X-radiation (composed of X-rays) is a form of electromagnetic radiation. X-rays
have a wavelength in the range of 0.01 to 10 nanometers, corresponding to frequencies in
the range (3×1016 Hz to 3×1019 Hz) and energies in the range 100 eV to 100 keV. The
wavelengths are shorter than those of UV rays and longer than of gamma rays.

When the electrons hit the target, X-rays are created by two different atomic processes:

1. X-ray fluorescence: If the electron has enough energy it can knock an orbital
electron out of the inner electron shell of a metal atom, and as a result electrons
from higher energy levels then fill up the vacancy and X-ray photons are emitted.
This process produces an emission spectrum of X-rays at a few discrete
frequencies, sometimes referred to as the spectral lines.

The spectral lines generated depend on the target element used and thus are called
characteristic lines. Usually these are transitions from upper shells into K shell
(called K lines), into L shell (called L lines) and so on.

2. Bremsstrahlung (breaking radiation): electromagnetic theory predicts that an


accelerated electric charge will radiate electromagnetic waves, and a rapidly
moving electron suddenly brought to rest is certainly accelerated.

                     
Head office  Branch office 
 
fiziks, H.No. 23, G.F, Jia Sarai,  Anand Institute of Mathematics, 
 
Near IIT, Hauz Khas, New Delhi‐16  28‐B/6, Jia Sarai, Near IIT 
 
Phone: 011‐26865455/+91‐9871145498 Hauz Khas, New Delhi‐16 
              
                   Website: www.physicsbyfiziks.com                        
                           Email: fiziks.physics@gmail.com                       55 
fiziks
Institute for NET/JRF, GATE, IIT‐JAM, JEST, TIFR and GRE in PHYSICAL SCIENCES 
 
X- ray spectra of tungsten and molybdenum at 35KV accelerating potential.

tungsten, 35 kV

molybdenum, 35 kV

Analysis:
An electron of initial kinetic energy K is decelerated during an encounter with heavy
target nucleus .the electron interacts with the charge nucleus via the coulomb field ,
transferring momentum to the nucleus . the accompanying deceleration of the electron
lead to photon emission . the target nucleus is so massive that the energy it acquires
during the collision can safely be neglected .
If K ' is the kinetic energy of the electron after the electron then energy of photon is given
hc
by h  K  K ' and photon (X- ray ) wavelength is given by  K  K'

The shortest wavelength photon would be emitted when an electron loses all the kinetic
hc
energy in one deceleration process so K '  0 and K
min
Since K  eV the energy acquired by the electron in being accelerated through the
hc
potential difference V so  K  eV
min

hc 1.24 106
And min   min  V .m
eV V

                     
Head office  Branch office 
 
fiziks, H.No. 23, G.F, Jia Sarai,  Anand Institute of Mathematics, 
 
Near IIT, Hauz Khas, New Delhi‐16  28‐B/6, Jia Sarai, Near IIT 
 
Phone: 011‐26865455/+91‐9871145498 Hauz Khas, New Delhi‐16 
              
                   Website: www.physicsbyfiziks.com                        
                           Email: fiziks.physics@gmail.com                       56 
fiziks
Institute for NET/JRF, GATE, IIT‐JAM, JEST, TIFR and GRE in PHYSICAL SCIENCES 
 
Example: Find the maximum frequency in the radiation from an x-ray machine whose
accelerating potential 50,000V.
1.24  106 1.24  106
Solution: min  V .m min  V .m  2.48  10 11m
V 50  10 V
4

c 3  108
 max    1.21 1019 Hz
min 2.48  10 11

Example: Show that the frequency of K  X-ray of a material equals the sum of the frequencies

of K  and L X-rays of the same material.


Solution:
EK
The energy level diagram of an atom with one
K K
electron knocked out is shown in figure .
EL
Energy of K  X-ray is E K   E K  E L L
EM
Energy of K  X-ray is E K   E K  E M , Figure
and Energy of L X-ray is E L  E L  E M ,

Thus, E K   E K  E L or, hv K   hv K  hv L or, v K   v K  v L

2.5 Wave Particle Duality


Wave-Particle Duality: Postulates that all particles exhibit both wave and particle
properties. A central concept of quantum mechanics, this duality addresses the inability
of classical concepts like "particle" and "wave" to fully describe the behavior of
quantum-scale object.
List of experiments which explain particle nature of light wave .
Photoelectric effect
Compton Effect
Pair production
List of experiments which explain wave nature of particle
Davisson –Germen effect (diffraction due to electrons)
Young double slit Interference due to electrons

                     
Head office  Branch office 
 
fiziks, H.No. 23, G.F, Jia Sarai,  Anand Institute of Mathematics, 
 
Near IIT, Hauz Khas, New Delhi‐16  28‐B/6, Jia Sarai, Near IIT 
 
Phone: 011‐26865455/+91‐9871145498 Hauz Khas, New Delhi‐16 
              
                   Website: www.physicsbyfiziks.com                        
                           Email: fiziks.physics@gmail.com                       57 
fiziks
Institute for NET/JRF, GATE, IIT‐JAM, JEST, TIFR and GRE in PHYSICAL SCIENCES 
 
2.5.1 De Broglie Wave
In quantum mechanics, the concept of matter waves or de Broglie waves reflects
the wave–particle duality of matterThe de Broglie relations show that
the wavelength is inversely proportional to the momentum of a particle and is also
called de Broglie wavelength.
The wavelength of the wave associated with a particle as given by the de Broglie relation
h h
 
p mv
m0
For relativistic case, the mass becomes m  where m0 is rest mass and v is
v2
1 2
c
velocity of body.
2.5.2 Uncertainty principle
“It is impossible to determine two canonical variable simultaneously for microscopic

particle”. If q and pq are two canonical variable then qp q 
2
where, q is the error in measurement at q and pq is error in measurement at pq and

h is Planck’s constant (   h / 2 ) .
Important uncertainty relations:

 X  PX  ( X is position and px is momentum in x direction )
2

 E   t  ( E is energy and t is time).
2

 L    ( L is angular momentum,  is angle measured)
2

                     
Head office  Branch office 
 
fiziks, H.No. 23, G.F, Jia Sarai,  Anand Institute of Mathematics, 
 
Near IIT, Hauz Khas, New Delhi‐16  28‐B/6, Jia Sarai, Near IIT 
 
Phone: 011‐26865455/+91‐9871145498 Hauz Khas, New Delhi‐16 
              
                   Website: www.physicsbyfiziks.com                        
                           Email: fiziks.physics@gmail.com                       58 
fiziks
Institute for NET/JRF, GATE, IIT‐JAM, JEST, TIFR and GRE in PHYSICAL SCIENCES 
 
2.5.3 Superposition Principle
According to de Broglie, matter waves are associated with every moving body. These
matter waves moves in a group of different waves having slightly different wavelength.
The formation of group is due to superposition of individual wave.
Analogy: If  1 x, t  and  2  x, t  are two waves of slightly different wavelength and
frequency.
 1  A sin kx   t 
 2  A sink  dk x    d t 
   1  2

 dk d t 
 2 A cos   sin kx   t  → The velocity of individual wave is known as Phase
 2 2 

velocity which is given as v p 
k
d
The velocity of amplitude is given by group velocity vg which is given by .
dk
d
vg 
dk
The relationship between group and phase velocity is given by
vg
d dv p
v ph
vg   vp  k
dk dk
dv p
vg  v p  
d
t

Due to superposition of different wave of slightly different wavelength resultant wave


moves like a wave packet with velocity equal to group velocity.

                     
Head office  Branch office 
 
fiziks, H.No. 23, G.F, Jia Sarai,  Anand Institute of Mathematics, 
 
Near IIT, Hauz Khas, New Delhi‐16  28‐B/6, Jia Sarai, Near IIT 
 
Phone: 011‐26865455/+91‐9871145498 Hauz Khas, New Delhi‐16 
              
                   Website: www.physicsbyfiziks.com                        
                           Email: fiziks.physics@gmail.com                       59 
fiziks
Institute for NET/JRF, GATE, IIT‐JAM, JEST, TIFR and GRE in PHYSICAL SCIENCES 
 
Example: Calculate the group and phase velocities for the wave packet corresponding to a
relativistic particle.
Solution: Recall that the energy and momentum of ta relativistic particle are given by
m0 c 2 m0 v
E  mc  2
, p  mv 
1 v / c
2 2
1 v2 / c2
where m0 is the rest mass of the particle and c is the speed of light in a vacuum.

Squaring and adding the expressions of E and p , we obtain E 2  p 2 c 2  m02 c 4 , hence

E  c p 2  m02 c 2

 
Using this relation along with p 2  m02 c 2 / 1  v 2 / c 2 and , we can show that the group
velocity is given as follows

vg 
dE d

dp dp

c p 2  m02 c 2   pc
p 2  m02 c 2
v

The group velocity is thus equal to the speed of the particle,


vg  v

The phase velocity can be found from and:

v p  E / p  c 1  m02 c 2 / p 2

which, when combined with p  m0 v / 1  v 2 / c 2 , leads to 1  m02 c 2 / p 2  c / v,

hence

E m2 c2
vp   c 1  02 
p p v

                     
Head office  Branch office 
 
fiziks, H.No. 23, G.F, Jia Sarai,  Anand Institute of Mathematics, 
 
Near IIT, Hauz Khas, New Delhi‐16  28‐B/6, Jia Sarai, Near IIT 
 
Phone: 011‐26865455/+91‐9871145498 Hauz Khas, New Delhi‐16 
              
                   Website: www.physicsbyfiziks.com                        
                           Email: fiziks.physics@gmail.com                       60 
fiziks
Institute for NET/JRF, GATE, IIT‐JAM, JEST, TIFR and GRE in PHYSICAL SCIENCES 
 
Example: Use the uncertainty principle to estimate: (a) the ground state radius of the hydrogen
atom
(b) the ground state energy of the hydrogen atom
Solution: (a) According to the uncertainty principle, the electron’s momentum and the radius of
its orbit are related by rp ~  , hence p ~  / r . To find the ground state radius, we simply
need to minimize the electron-proton energy
p2 e2 2 e2
E r     
2me 4 0 r 2me r 2 4 0 r
with respect to r :
dE 2 e
0  
dr me r0 4 0 r 2
3

This leads to the Bohr radius


4 0  2
r0   5.3 nm
me e 2
(b) Inserting we obtain the Bohr energy;
2
2 e2 m  e2 
E r0     2    13.6 eV
2mr0 4 0 r0
2
2  4 0 
The results obtained for r0 and E r0  , as shown in , are indeed impressively accurate
given the crudeness of the approximation

                     
Head office  Branch office 
 
fiziks, H.No. 23, G.F, Jia Sarai,  Anand Institute of Mathematics, 
 
Near IIT, Hauz Khas, New Delhi‐16  28‐B/6, Jia Sarai, Near IIT 
 
Phone: 011‐26865455/+91‐9871145498 Hauz Khas, New Delhi‐16 
              
                   Website: www.physicsbyfiziks.com                        
                           Email: fiziks.physics@gmail.com                       61 
fiziks
Institute for NET/JRF, GATE, IIT‐JAM, JEST, TIFR and GRE in PHYSICAL SCIENCES 
 
MCQ (Multiple Choice Questions)
Q1. How is de-Broglie wavelength    of an electron in the nth Bohr orbit related to the

radius R of the orbit?


3 R
(a) n   R (b) n  (c) n  2 R (d) n  4 R
2
Q2. The correct expression for the de-Broglie wavelength  of a particle ( E is the kinetic
energy) is
hc hc
(a)   (b)  
 E  2m c 
0
2

E E  m0 c 2 
hc hc
(c)   (d)  

E E  2m0 c 2
 E  m c  0
2

Q3. A particle of mass M at rest decays into two particles of masses m1 and m2 , having
nonzero velocities. The ratio of the de Broglie wavelengths of the particles, 1 / 2 , is

(a) m1 / m2 (b) m2 / m1 (c) 1 (d) m2 / m1

Q4. A proton has kinetic energy E which is equal to that of a photon. The wavelength of
2
photon is 2 and that of proton is 1 . The ratio is proportional to
1

(a) E 2 (b) E1/ 2 (c) E 1 (d) E 1/ 2


Q5. Electrons with de-Broglie wavelength  fall on the target in an X-ray tube. The cut-off
wavelength of the emitted X-rays is
2mc2 2h 2 m 2 c 2 3
(a) 0  (b) 0  (c) 0  (d) 0  
h mc h2
Q6. In a Compton - scattering experiment, photons with incoming momentum mc ( m is the
mass of the electron) are scattered at an angle 90o . What is the magnitude of the
momentum of the scattered photon?
mc mc mc
(a) mc (b) (c) (d)
2 3 4

                     
Head office  Branch office 
 
fiziks, H.No. 23, G.F, Jia Sarai,  Anand Institute of Mathematics, 
 
Near IIT, Hauz Khas, New Delhi‐16  28‐B/6, Jia Sarai, Near IIT 
 
Phone: 011‐26865455/+91‐9871145498 Hauz Khas, New Delhi‐16 
              
                   Website: www.physicsbyfiziks.com                        
                           Email: fiziks.physics@gmail.com                       62 
fiziks
Institute for NET/JRF, GATE, IIT‐JAM, JEST, TIFR and GRE in PHYSICAL SCIENCES 
 
Q7. In photoelectric effect with incoming radiation of frequency v0 with hv0  8eV , electrons

of energy 3eV are emitted from a metal surface. The energy of the electrons emitted
from this surface when radiation with frequency 1.2 v0 is incident, is:

(a) 4.2 eV (b) 5.2 eV (c) 3.6 eV (d) 4.6 eV


Q8. Diagram shown here corresponds to observations made in photoelectric effect observed
with radiation of frequency v , and wavelength  , resulting in K max as the maximum

kinetic energy of photoelectrons. The quantities shown on the x and y -axes in the
diagram are:
(a) x-axis :  ; y -axis:E max (b) x-axis : Emax ; y -axis: 

(c) x-axis : v; y -axis:E max (d) x-axis : Emax ; y -axis: v


Q9. The maximum kinetic energy of photoelectrons emitted from a surface when photons of
energy 6eV fall on it is 4eV . The stopping potential, in volt, is
(a) 2 (b) 4 (c) 6 (d) 10
Q10. In a photoelectric experiment anode potential is plotted against
plate current I
(a) A and B will have same intensities while B and C will have
different frequencies C
B
(b) B and C will have different intensities while A and B will A
have different frequencies
V
(c) A and B will have different intensities while B and C will have equal frequencies
(d) B and C will have equal intensities while A and B will have same frequencies.
Q11. In the following electronic transitions in a hydrogen atom, which transition emits the
minimum wavelength?
(a) n  2 to n  1level (b) n  3 to n  2level
(c) n  4 to n  3level (d) n  5 to n  4 level

                     
Head office  Branch office 
 
fiziks, H.No. 23, G.F, Jia Sarai,  Anand Institute of Mathematics, 
 
Near IIT, Hauz Khas, New Delhi‐16  28‐B/6, Jia Sarai, Near IIT 
 
Phone: 011‐26865455/+91‐9871145498 Hauz Khas, New Delhi‐16 
              
                   Website: www.physicsbyfiziks.com                        
                           Email: fiziks.physics@gmail.com                       63 
fiziks
Institute for NET/JRF, GATE, IIT‐JAM, JEST, TIFR and GRE in PHYSICAL SCIENCES 
 
Q12. What is the speed vn of the electron in the nth Bohr orbit of hydrogen atom, if v1 is the
speed of the electron in the first Bohr orbit?
v1 v
(a) v1n (b) v1n3 (c) (d) 13
n n
Q13. If elements with principal quantum number n  4 were not allowed in nature, the number
of possible elements would be
(a) 60 (b) 32 (c) 4 (d) 64
Q14. Consider the spectral line resulting from the transition n  2  n  1 in the atoms and
ions given below. The shortest wavelength is produced by
(a) hydrogen atom (b) deuterium atom
(c) singly ionized helium (d) doubly ionised lithium
Q15. A hydrogen atom and a Li  ion are both in the second excited state. If lH and lLi are

their respective electronic angular momenta, and EH and ELi their respective energies,
then
(a) lH  lLi and EH  ELi (b) lH  lLi and E H  ELi
(c) lH  lLi and E H  ELi (d) lH  lLi and E H  ELi

Q16. X  rays are produced in an X  ray tube operating at a give accelerating voltage. The
wavelength of the continuous X  rays has values from
(a) 0 to  (b) min to  where min  0

(c) 0 to max where max   (d) min to max where 0  min  max  

Q17. Given that for an atom with nuclear charge Z1 the X -ray frequency for transition

between two low-lying states is v1. According to Moseley’s law, what is the

corresponding frequency v2 for an atom with nuclear charge Z 2 approximately equal to


(ignoring shielding factor)?
Z2 Z1
(a) v2  v1 (b) v2  v1
Z1 Z2
Z12 Z 22
(c) v2  v1 (d) v2  v1
z22 Z12
                     
Head office  Branch office 
 
fiziks, H.No. 23, G.F, Jia Sarai,  Anand Institute of Mathematics, 
 
Near IIT, Hauz Khas, New Delhi‐16  28‐B/6, Jia Sarai, Near IIT 
 
Phone: 011‐26865455/+91‐9871145498 Hauz Khas, New Delhi‐16 
              
                   Website: www.physicsbyfiziks.com                        
                           Email: fiziks.physics@gmail.com                       64 
fiziks
Institute for NET/JRF, GATE, IIT‐JAM, JEST, TIFR and GRE in PHYSICAL SCIENCES 
 
Q18. Which one is the single characteristic of the target element which occurs in Moseley’s
law for the frequencies of emitted X -rays?
(a) Density (b) Atomic weight
(c) Atomic number (d) Spacing between the atomic planes
Q19. For the X -ray spectrum due to transition between n  2 and n  1 states for large nuclear
charge Ze , we have frequencies v0 , v1 , v2 for Z 0  Z 0 , Z1  Z 0  1, Z 2  Z 0  2 respectively.
Moseley’s law implies which one of the following equations?

(a) v1 
 v0  v2  (b) v1   v0 v2  2
1

(c) v1 
v0  v2
(d) v1 
 v0v2 
2  v0  v2 
NAT (Numerical Answer Type)
Q20. The de-Broglie wavelengths of a proton and an  - particle are equal. The ratio of their
velocities is…………..
Q21. The potential energy of a particle of mass in is given by
 E ; 0  x  1
V  x   0 
 0; x 1 

1 and 2 are the de-Broglie wavelengths of the particle, when 0  x  1 and x  1


1
respectively. ‘If the total energy of particle is 2 E0 , then ratio …………...
2
h
Q22. In Compton scattering, an incoming photon of wavelength 0  ( h  Planck’s
2mc
constant, m  mass of electron, c  speed of light) is scattered by an electron at rest. If
the photon is scattered backward at angle of 180o , the momentum of the corresponding
scattered electron is………….. mc
Q23. In the Compton scattering process, at which scattering angle……… does the maximum
energy transfer to the electron occur

                     
Head office  Branch office 
 
fiziks, H.No. 23, G.F, Jia Sarai,  Anand Institute of Mathematics, 
 
Near IIT, Hauz Khas, New Delhi‐16  28‐B/6, Jia Sarai, Near IIT 
 
Phone: 011‐26865455/+91‐9871145498 Hauz Khas, New Delhi‐16 
              
                   Website: www.physicsbyfiziks.com                        
                           Email: fiziks.physics@gmail.com                       65 
fiziks
Institute for NET/JRF, GATE, IIT‐JAM, JEST, TIFR and GRE in PHYSICAL SCIENCES 
 
Q24. X -ray of energy 50 keV strikes an electron initially at rest. The change in wavelength of

the X -ray scattered at angle 90o is, approximately…………. 1012 meter .


(Given, h  6.63  1034 J  s, m  9.11 1031 kg )
Q25. In a photoelectric effect experiment, for radiation with frequency v0 with hv0  8eV ,

electrons are emitted with energy 2 eV . if the incoming radiation of frequency 1.25 v0
then the energy of photo electron is ……………….
o
Q26. A threshold wavelength of a metal is 7000 A . The work function is ………….
(Given, velocity of light c  3  108 m / s and Planck’s constant  6.624 1034 J  S )
Q27. The work function of a substance is 4eV The longest wavelength of light that can cause
photoelectron emission from this substance is approximately……………. 109 m
0 0 0
Q28. A beam of light has three wavelengths 4144 A, 4972 A and 6216 A with a total intensity

of 3.6  10 3 Wm 2 equally distributed amongst the three wavelengths. The beam falls
normally on an area 1.0cm 2 of a clean metallic surface of work function 2.3 eV . Assume
that there is no loss of light by reflection and that each energetically capable photon
ejects one electron. The number of photo electrons liberated in per
second ……………… 1011
Q29. When a beam of 10.6 eV photons of intensity 2.0 W / m 2 falls on a platinum surface of

area 1.0  10 4 m 2 and workfunction 5.6 eV , 0.53% of the incident photons eject

photoelectrons. Take 1 eV  1.6 1019 J

(a) The number of photoelectrons emitted per second…………… 1011 .


(b) Minimum energy of photoelectron………………. eV .
(c) Maximum energies ………………………. eV
Q30. For He  which has one electron and a nuclear charge 2e , then the binding energy of the
first excited state with principal quantum number n  2 is ………( eV )
Q31. The shortest wavelength in Lyman series of hydrogen spectra is 91.2 nm , the longest
wavelength in this series must be ………………. nm
                     
Head office  Branch office 
 
fiziks, H.No. 23, G.F, Jia Sarai,  Anand Institute of Mathematics, 
 
Near IIT, Hauz Khas, New Delhi‐16  28‐B/6, Jia Sarai, Near IIT 
 
Phone: 011‐26865455/+91‐9871145498 Hauz Khas, New Delhi‐16 
              
                   Website: www.physicsbyfiziks.com                        
                           Email: fiziks.physics@gmail.com                       66 
fiziks
Institute for NET/JRF, GATE, IIT‐JAM, JEST, TIFR and GRE in PHYSICAL SCIENCES 
 
Q32. Ionisation potential for a hydrogen atom is 13.6 eV . The ionisation potential for a
positronium atom where an electron revolves round a positron, is……….. eV
Q33. An energy of 24.6 eV is required to remove one of the electrons from a neutral helium
atom. The energy in  eV  required to remove both the electrons from a neutral helium

atom is…………..
Q34. The wavelength of the characteristic X-ray K  line emitted by a hydrogen like element is
0
0.32  . The wavelength of the K  line emitted by the same element will be……………

Q35. The recoil speed of a hydrogen atom after ii emits a photon in going from n  5 state to
n  1 state is ……. m / s .
Q36. As per Bohr model, the minimum energy in eV required to remove an electron from the
ground state of doubly ionized Li atom  Z  3 is…………

Q37. K wavelength emitted by an atom of atomic number Z1  11 is  . For atomic number

Z 2  ………… for an atom that emits K radiation with wavelength 4 .

MSQ (Multiple Select Questions)


Q38. Which of the following is correct for Compton effect
(a) The energy of incoming X ray is approximately 20KeV
(b) The Compton shift is not dependent on energy of incoming X ray
(c) The wavelength of scattered X ray is less than wavelength of incoming X ray
(d) Maximum energy is transfer from X ray to electron at scattering angle 900 with
horizontal
Q39. For which of the following cases is the de-Brogile wavelength is same?
(a) Particle of mass m , kinetic energy K
(b) Particle of mass 2m , kinetic energy 2K
K
(c) Particle of mass 2m , kinetic energy
2
m
(d) particle of and kinetic energy K
2
                     
Head office  Branch office 
 
fiziks, H.No. 23, G.F, Jia Sarai,  Anand Institute of Mathematics, 
 
Near IIT, Hauz Khas, New Delhi‐16  28‐B/6, Jia Sarai, Near IIT 
 
Phone: 011‐26865455/+91‐9871145498 Hauz Khas, New Delhi‐16 
              
                   Website: www.physicsbyfiziks.com                        
                           Email: fiziks.physics@gmail.com                       67 
fiziks
Institute for NET/JRF, GATE, IIT‐JAM, JEST, TIFR and GRE in PHYSICAL SCIENCES 
 
Q40. Consider the following statements:
The maximum kinetic energy of a photoelectron depends on:
(a) frequency of incident radiation
(b) nature of photo emitter
(c) intensity of incident radiation
(d) on plate potential
Which of these statements are correct?
Q41. When photons of energy 4.25 eV strike the surface of metal A ,, the ejected
photoelectrons have maximum kinetic energy TAeV and de Broglie wavelength  A . The
maximum kinetic energy of photoelectrons liberated from another metal B by photons of
energy 4.70eV is TB  TA  1.50  eV . If the de Broglie wavelength of these

photoelectrons is B  2 A , then

(a) The work function of A is 2.25eV (b) the work function of B is 3.95 eV
(c) TA  2.00 eV (d) TB  2.75 eV

1
Q42. The graph between the stopping potential V0  and   is shown in the figure 1 , 2 and

3 are work functions. Which of the following is/are correct?
V0
Metal1 Metal 2 Metal3

1
1 3


(a) 1 : 2 : 3  1: 2 : 4 1

nm  1

0.001 0.002 0.004


(b) 1 : 2 : 3  4 : 2 :1

hc
(c) tan  is proportional to where h is Planck’s constant and c is the speed of light
e
(d) Ultraviolet light can be used to emit photoelectrons from metal 2 and metal 3 only

                     
Head office  Branch office 
 
fiziks, H.No. 23, G.F, Jia Sarai,  Anand Institute of Mathematics, 
 
Near IIT, Hauz Khas, New Delhi‐16  28‐B/6, Jia Sarai, Near IIT 
 
Phone: 011‐26865455/+91‐9871145498 Hauz Khas, New Delhi‐16 
              
                   Website: www.physicsbyfiziks.com                        
                           Email: fiziks.physics@gmail.com                       68 
fiziks
Institute for NET/JRF, GATE, IIT‐JAM, JEST, TIFR and GRE in PHYSICAL SCIENCES 
 
Q43. In the Bohr model of the hydrogen atom, if kinetic energy is T potential energy is V and
total energy is E then which of following is /are correct
E E
(a) V  (b) V  2 E (c) T   E (d) T  
2 2
Q44. The transition from the state n  4 to n  3 in a hydrogen-like atom results in ultraviolet
radiation which of the following is /are not Infrared radiation obtained in the transition
(a) 2  1 (b) 3  2 (c) 4  2 (d) 5  4
Q45. In Bohr’s model of the hydrogen atom
(a) the radius of the nth orbit is proportional to n2
(b) the total energy of the electron in nth orbit is inversely proportional to n2 .
h
(c) the angular momentum of electron in an nth orbit is an integral multiple of .
2
(d) the magnitude of potential energy of the electron in any orbit is greater than its kinetic
energy
Q46. The electron in a hydrogen atom makes a transition n1  n2 , where n1 and n2 are the
principal quantum numbers of the two states. Assume the Bohr model to be valid. The
time period of the electron in the initial state is eight times that in the final state. The
possible values of n1 and n2 are

(a) n1  4, n2  2 (b) n1  8, n2  2

(c) n1  8, n2  1 (d) n1  6, n2  3
Q47. The shortest wavelength of X  rays emitted from an X  ray tube depends on
(a) the current in the tube (b) the voltage applied to the tube
(c) the nature of the gas in tube (d) the atomic number of the target material.
Q48. The potential difference applied to an X - ray tube is increased. As a result, in the emitted
radiation
(a) the intensity increases (b) the minimum wavelength increases
(c) the intensity remains unchanged (d) the minimum wavelength decreases

                     
Head office  Branch office 
 
fiziks, H.No. 23, G.F, Jia Sarai,  Anand Institute of Mathematics, 
 
Near IIT, Hauz Khas, New Delhi‐16  28‐B/6, Jia Sarai, Near IIT 
 
Phone: 011‐26865455/+91‐9871145498 Hauz Khas, New Delhi‐16 
              
                   Website: www.physicsbyfiziks.com                        
                           Email: fiziks.physics@gmail.com                       69 
fiziks
Institute for NET/JRF, GATE, IIT‐JAM, JEST, TIFR and GRE in PHYSICAL SCIENCES 
 
Solutions (MCQ)
Ans. 1: (c)
Solution: According to Bohr Quantization condition, the electron wave can be
adjusted around an orbit only when the circumference of the orbit is
equal to an integral multiple of the wavelength i.e., 2 R  n
Ans. 2: (c)
h
Solution: The de-Broglie wavelength   (i)
mv
m0
Where relativistic mass m  m0  rest mass
v2
1 2
c
v2 m02
 2
 1  2  mv  c m2  m02 (ii)
c m
hc
Thus, equation (i) and (ii), we get    (iii)
c 2 m 2  m02
1
Now, c 2 m 2  m02  c 4  m  m0  m  m0    m  m0  m  m0  c 2  2m0 c 2  2


 E E  2m0 c 2  [Since E   m  m0  c 2 ]
hc
So, by equation (iii)  

E E  2m0 c 2 
Ans. 3: (c)
h 1 p1
Solution: de-Broglie wavelength    
p 2 P2
1
Since momentum p is conserved in the decay process, p2  p1  1
2

                     
Head office  Branch office 
 
fiziks, H.No. 23, G.F, Jia Sarai,  Anand Institute of Mathematics, 
 
Near IIT, Hauz Khas, New Delhi‐16  28‐B/6, Jia Sarai, Near IIT 
 
Phone: 011‐26865455/+91‐9871145498 Hauz Khas, New Delhi‐16 
              
                   Website: www.physicsbyfiziks.com                        
                           Email: fiziks.physics@gmail.com                       70 
fiziks
Institute for NET/JRF, GATE, IIT‐JAM, JEST, TIFR and GRE in PHYSICAL SCIENCES 
 
Ans. 4: (d)
hc hc
Solution: For photon, E  or 2  (i)
2 E
1 h h
For proton kinetic energy K  m p v 2p or 1   (ii)
2 2m p K 2m p E

1 hc 2m p E  c  2m p 
From (i) and (ii),   or 2   c 2m p  E 1/ 2 or 2  E 1/ 2
2 E h 1 E 1
Ans. 5: (a)
Solution: Let K be the kinetic energy of the incident electron. Its linear momentum p  2mK .
The de-Broglie wavelength is related to the linear momentum as
h h h2
  or K 
p 2mK 2m 2
The cut-off wavelength of the emitted X  ray is related to the kinetic energy of incident
hc h2 2mc 2
electron as K   
0 2m 2
0
h
Ans. 6: (b)
y y Scattered
Solution: electron photon
 x
x
v0  Recoiling
electron
incident photon
After collision
Before collision

h
The change in wavelength of scattered photon is given as   1  cos   where  is
m0 c

scattered angle. Hence,   90o (given)


h h h
So,  
m0c

1  cos 90o 
m0c

1  0    
m0 c
h
if  is wavelength of incident photon then wavelength of photon    (i)
mc

                     
Head office  Branch office 
 
fiziks, H.No. 23, G.F, Jia Sarai,  Anand Institute of Mathematics, 
 
Near IIT, Hauz Khas, New Delhi‐16  28‐B/6, Jia Sarai, Near IIT 
 
Phone: 011‐26865455/+91‐9871145498 Hauz Khas, New Delhi‐16 
              
                   Website: www.physicsbyfiziks.com                        
                           Email: fiziks.physics@gmail.com                       71 
fiziks
Institute for NET/JRF, GATE, IIT‐JAM, JEST, TIFR and GRE in PHYSICAL SCIENCES 
 
h
But moment of incident photon mc (given)  
mc
h h 2h
So, wavelength of scattered photon   by equation (i) s 
mc mc mc
h h 1
Corresponding momentum  Momentum   mc
s 2h 2
mc
Ans. 7: (d)
Solution: In photoelectric effect, the energy of electron is given as
mv 2
 h  W  3 eV  8 eV  W
2
work function W   8  3 eV , W  5 eV

Now, if energy corresponding to v0 is 8eV , then energy corresponding to 1.2 v0 .


 1.2  8 eV  9.6 eV

so, energy of electron = h '  W   9.6  5  eV  4.6 eV

Ans. 8: (c)
Solution: If the threshold frequency v0 is given as then y

1 2
mv  hv  hv0 where h is Planck’s constant i.e.,  KE   KE e
2
of electron  hv  hv0

thus, y -axis is KE of electron and x -axis is frequency of x


v
incident photon.
Ans. 9: (b)
Solution: Stopping potential is the negative potential which stops the emission of
 K .E max electrons when applied.  Stopping potential = 4 volt

                     
Head office  Branch office 
 
fiziks, H.No. 23, G.F, Jia Sarai,  Anand Institute of Mathematics, 
 
Near IIT, Hauz Khas, New Delhi‐16  28‐B/6, Jia Sarai, Near IIT 
 
Phone: 011‐26865455/+91‐9871145498 Hauz Khas, New Delhi‐16 
              
                   Website: www.physicsbyfiziks.com                        
                           Email: fiziks.physics@gmail.com                       72 
fiziks
Institute for NET/JRF, GATE, IIT‐JAM, JEST, TIFR and GRE in PHYSICAL SCIENCES 
 
Ans. 10: (d)
I Same intensity
Solution: At stopping potential, photoelectric current is zero. It
Saturation current
is same for A and B .
B
 A and B will have equal frequencies C
Saturation current is proportional to intensity. B and
A
C will have equal intensity
Option (d) represents correct answer. V
Ans. 11: (a)
Solution: The wavelength in hydrogen atom’s transition is given as

ni
nf
1 1 1
 R 2  2 
  n f ni 
where R in Rydberg constant clearly  will be minimum if n  2  n  1 .

Ans. 12: (c)


Solution: The electrostatic force = centripetal force
Electron
mv 2 Ze 2
i.e.,  2
r kr
Ze 2
 mv 2 r  (i)
k
Nucleus
By Bohr theory mv r  n (ii)
Dividing equation (i) by, equation (ii), we get

Ze 2 1  Ze 2  1 v1 n2 n v
v v    v    v2  1 v1  vn  1
kn n  k  n v2 n1 n2 n

                     
Head office  Branch office 
 
fiziks, H.No. 23, G.F, Jia Sarai,  Anand Institute of Mathematics, 
 
Near IIT, Hauz Khas, New Delhi‐16  28‐B/6, Jia Sarai, Near IIT 
 
Phone: 011‐26865455/+91‐9871145498 Hauz Khas, New Delhi‐16 
              
                   Website: www.physicsbyfiziks.com                        
                           Email: fiziks.physics@gmail.com                       73 
fiziks
Institute for NET/JRF, GATE, IIT‐JAM, JEST, TIFR and GRE in PHYSICAL SCIENCES 
 
Ans. 13: (a)
Solution: The maximum number of electrons in an orbit are 2n 2 . If n  4 is not allowed, the
maximum number of electrons that can lie in first four orbits are

2 1  2  2   2  3  2  4   2  8  18  32  60  Possible elements can be 60 .


2 2 2 2

Ans. 14: (d)

1  1 1  1
Solution:  RZ 2  2  2    Z2.
  n2 n1  

 is shortest if Z is largest. Z is largest for doubly ionised lithium atom  Z  3 among

the given elements.


Hence wavelength for doubly ionised lithium will be the least.
Ans. 15: (b)
Solution: In the second excited state, n  3
 h 
lH  lLi  3   Z H  1, Z Li  3, E  Z 2  ELi  9 EH  EH  ELi
 2 
Ans. 16: (b)
12375
Solution: In X  ray tube, min  where min is in Ǻ
V  volt 

All wavelengths greater than min are found.


Option (b) is correct
Ans. 17: (d)
Solution: Moseley’s law states that the square root of the frequency of a K -line is closely
proportional to the atomic number of the element and may be expressed as
vZ  v  kZ 2 where k is a constant Thus, v1  kZ12 and v2  kZ 22

v2 Z 22 Z 22
  v2  v1
v1 Z12 Z12

                     
Head office  Branch office 
 
fiziks, H.No. 23, G.F, Jia Sarai,  Anand Institute of Mathematics, 
 
Near IIT, Hauz Khas, New Delhi‐16  28‐B/6, Jia Sarai, Near IIT 
 
Phone: 011‐26865455/+91‐9871145498 Hauz Khas, New Delhi‐16 
              
                   Website: www.physicsbyfiziks.com                        
                           Email: fiziks.physics@gmail.com                       74 
fiziks
Institute for NET/JRF, GATE, IIT‐JAM, JEST, TIFR and GRE in PHYSICAL SCIENCES 
 
Ans. 18: (c)
Solution: Moseley’s law state that the square root of frequency of H  is directly proportional to

the atomic number is vZ.


Ans. 19: (c)
Solution: Moseley law states that the square root of frequency for K -lines is directly
proportional to the atomic number of the element i.e.,
we have given as   K  Z  1
(1)
 0  K  Z 0  1 (2)

 1  K  Z 0  1  1  1  KZ 0 (3)

 2  K  Z 0  2  1  2  K  Z 0  1 (4)

Adding equations (2) and (4)


0  2
 0   2  K  Z 0  1  Z 0  1  2 KZ 0  2  1   1
2

                     
Head office  Branch office 
 
fiziks, H.No. 23, G.F, Jia Sarai,  Anand Institute of Mathematics, 
 
Near IIT, Hauz Khas, New Delhi‐16  28‐B/6, Jia Sarai, Near IIT 
 
Phone: 011‐26865455/+91‐9871145498 Hauz Khas, New Delhi‐16 
              
                   Website: www.physicsbyfiziks.com                        
                           Email: fiziks.physics@gmail.com                       75 
fiziks
Institute for NET/JRF, GATE, IIT‐JAM, JEST, TIFR and GRE in PHYSICAL SCIENCES 
 
NAT
Ans. 20: 4
Solution: Every particle of mass m moving with velocity v is associated with a wave of
h
wavelength given as   h  Planck’s constant
mv
h
the wavelength for proton p  (i)
mpvp

h
and the wavelength for He   (ii)
m v

h h v m
Since,    p    p   (iii)
m p v p m v v m p

Since, 2 He 4 has four nucleons so m  4m p

vp
Thus, (iii)  4  v p : v  4 :1
v
Ans. 21: 1.414
h
Solution: de Broglie wavelength   where K denotes kinetic energy of particle
2mK
Case (I): 0  x  1
Given: potential energy  E0 , Given: Total energy  2E 0

h
 Kinetic energy  2 E 0  E 0  E 0  1 
2mE0

Case (II): x  1
Given: potential energy V  x   0 , given: Total energy  2E 0

 Kinetic energy  2E 0

h 1 2 E0 1
2     2   2
2m2 E0  2 E0 2

                     
Head office  Branch office 
 
fiziks, H.No. 23, G.F, Jia Sarai,  Anand Institute of Mathematics, 
 
Near IIT, Hauz Khas, New Delhi‐16  28‐B/6, Jia Sarai, Near IIT 
 
Phone: 011‐26865455/+91‐9871145498 Hauz Khas, New Delhi‐16 
              
                   Website: www.physicsbyfiziks.com                        
                           Email: fiziks.physics@gmail.com                       76 
fiziks
Institute for NET/JRF, GATE, IIT‐JAM, JEST, TIFR and GRE in PHYSICAL SCIENCES 
 
Ans. 22: 2.4
Solution: Applying the conservation law of linear momentum along the horizontal and vertical
component, we have
h h
Horizontal  cos   p cos  (i)
  scattered photon

h Linear momentum
h

wavelength  ' 
Vertical component sin   p sin  (ii) 
 Incident photon 
h  wavelength  
By equation   ;   180o Before collision
2mc
Putting these values in equations (i) and (ii), we get
After collision
h h
 2mc  cos180o  p cos   2mc  p cos   (iii)
 
12mc
by equation (ii)  0  p sin  By equations (iii) and (ii), we get p
5
Ans. 23: 3.14
h
Solution: In Compton effect the range in wavelength is given as   1  cos   where
m0 c

m0  electron rest mass,   scattering angle.

The maximum energy is transferred to electron, if  is maximum


i.e., 1  cos   is maximum    180o Thus, scattering angle is 180o or  .
Ans. 24: 2.4
Solution: When a photon of wavelength  strike a stationary  photon  

electron the wavelength of the photon increases. This effect photon electron
is known as Compton effect. before strike
after strike
h
The change b wavelength is given as   1  cos  
m0 c
The change in wavelength does not depend on the energy of the incident photon.
h h
Here,   90o  
m0 c
1  cos 90o  
m0 c
 2.4 1012 m

                     
Head office  Branch office 
 
fiziks, H.No. 23, G.F, Jia Sarai,  Anand Institute of Mathematics, 
 
Near IIT, Hauz Khas, New Delhi‐16  28‐B/6, Jia Sarai, Near IIT 
 
Phone: 011‐26865455/+91‐9871145498 Hauz Khas, New Delhi‐16 
              
                   Website: www.physicsbyfiziks.com                        
                           Email: fiziks.physics@gmail.com                       77 
fiziks
Institute for NET/JRF, GATE, IIT‐JAM, JEST, TIFR and GRE in PHYSICAL SCIENCES 
 
Ans. 25: 4 eV
Solution: The kinetic energy of photoelectron is given as
KE  hv  W
where v  frequency of incident photon W  work function of the substance h
Planck constant
Now, hv0  8 eV , KE  2 eV

So, work function   8  2  eV W  6 eV

Now, if hv0  8 eV  1.25 hv0  10 eV

Thus, hv  10 eV , W  6 eV  Kinetic energy  hv  W  10 eV  6 eV  4 eV


Ans. 26: 1.775 eV
Solution: The maximum wavelength  of a photon which can emit electron from a metal is
known as threshold wavelength of the metal. The energy corresponding to this threshold
wavelength is equal to work function of the metal. Hence, Work function = Threshold
energy
hc 6.624  10 34  3  108 12375
    eV  1.775 eV
0 7000  1010 7000
Ans. 27: 310
hc
Solution: max 
work function

max 
 6.63 10    3 10 
34 8

or max  310  109 m


19
4  1.6  10
Ans. 28: 5.5
12375
Solution: Energy of photon in eV 
  A 
0

 
12375 12375 12375
 E1  eV  2.99 eV , E 2  eV  2.49 eV , E3  eV  1.99eV
4144 4972 6216
work function  2.3 eV
                     
Head office  Branch office 
 
fiziks, H.No. 23, G.F, Jia Sarai,  Anand Institute of Mathematics, 
 
Near IIT, Hauz Khas, New Delhi‐16  28‐B/6, Jia Sarai, Near IIT 
 
Phone: 011‐26865455/+91‐9871145498 Hauz Khas, New Delhi‐16 
              
                   Website: www.physicsbyfiziks.com                        
                           Email: fiziks.physics@gmail.com                       78 
fiziks
Institute for NET/JRF, GATE, IIT‐JAM, JEST, TIFR and GRE in PHYSICAL SCIENCES 
 
First two wavelengths have energies great than work function of metallic surface. Hence
they can eject photoelectrons.
Total intensity  3.6  10 3 Wm 2
Number of wavelengths = 3
3.6 103
Intensity per wavelength   1.2  103 Wm 2 Area  104 m 2
3
 Energy falling per second  1.2  10 3  10 4  1.2  10 7 J / S

 Let number of photons of first wavelength  n1 and number of photons of second


wavelength  n2

1.2  10 7 1.2  10 7
 n1   2.5  1011 n2   3.0  1011
2.99  1.6  10 19
  2.49  1.6  10 19

 Total photons per second  2.5  3.01011  5.5  1011  Each capable photon ejects
an electron
 Photoelectrons liberated in sec 5.5  1011
Ans. 29: (a) 6.25 , (b) 0 , (c) 5
Solution: (a) Incident energy E  10.6 eV  10.6  1.6  10 19 J  16.96  10 19 J
Energy incident
 2 W / m2
area  time
Number of incident photons 2
  19
 1.18 1018
area  time 16.96  10


Incident photons
time

 1.18  1018  area 
 
 1.18  1018  1.0  10 4  1.18  1014


Number of photoelectrons  0.53 
time
 
  1.18  10  n  6.25 10
14 11
 (ii)
 100 
Minimum energy = zero
Maximum energy  E1  work function
or K max  10.6  5.6 eV or K max  5.0 eV (iii)
                     
Head office  Branch office 
 
fiziks, H.No. 23, G.F, Jia Sarai,  Anand Institute of Mathematics, 
 
Near IIT, Hauz Khas, New Delhi‐16  28‐B/6, Jia Sarai, Near IIT 
 
Phone: 011‐26865455/+91‐9871145498 Hauz Khas, New Delhi‐16 
              
                   Website: www.physicsbyfiziks.com                        
                           Email: fiziks.physics@gmail.com                       79 
fiziks
Institute for NET/JRF, GATE, IIT‐JAM, JEST, TIFR and GRE in PHYSICAL SCIENCES 
 
Ans. 30: 13.6eV
 13.6  4 13.6
Solution: Since, energy  Z 2 Energy for He    4   2  eV   eV
 n  n2
The first excited state with principal quantum number n  2
4  13.6
So, binding energy    13.6 eV
4
Ans. 31: 68.4nm
n
Solution: When electron moving in ni the orbit transited

to n f orbit the frequency of radiation and so


n4
1  1 1  n3
wavelength is given by  R 2  2 
  n f ni  n2
 
where R is Rydberg’s constant. n 1

when ni  1 , then the series of spectral lines are known as Lyman series.

1 1 1 
Thus, Lyman series is given as

 R 2  2  n f  1
 1 ni 
1 R 1  1 1  1  1  4 
   max  and  R 2  2    R  1    min   
max 12
R min 1 2  min  4  3R 
max 4 3 3
By equation (i) and (ii), we have   max   min   91.2 nm  68.4 nm
min 3 4 4
Ans. 32: 6.8eV
Solution: The amount of energy required to an electron moving in ground state of the atom, so
that electron go off from the atom is called ionisation energy and potential corresponding
to this energy is called ionisation potential for hydrogen this is given as E  13.6 eV
Since, positronium is a system in which electron revolves round the positron hence
M eM p 1
reduced mass of the positroniuin    Me (Since M p  M e )
Me  M p 2

13.6
and E is proportional to reduce mass so for positronium E  eV  6.8 eV
2
                     
Head office  Branch office 
 
fiziks, H.No. 23, G.F, Jia Sarai,  Anand Institute of Mathematics, 
 
Near IIT, Hauz Khas, New Delhi‐16  28‐B/6, Jia Sarai, Near IIT 
 
Phone: 011‐26865455/+91‐9871145498 Hauz Khas, New Delhi‐16 
              
                   Website: www.physicsbyfiziks.com                        
                           Email: fiziks.physics@gmail.com                       80 
fiziks
Institute for NET/JRF, GATE, IIT‐JAM, JEST, TIFR and GRE in PHYSICAL SCIENCES 
 
Ans. 33: 79eV
Solution: When one of the electrons is removed from a neutral helium atom, energy is given by
13.6 Z 2
En   eV per atom
n2
For helium ion, Z  2 , when doubly ionized
13.6
  2   54.4 eV
2
For first orbit, n  1  E1  
1
2

 Energy required removing it 54.4 eV  Total energy required  54.4  24.6  79eV
0
Ans. 34: 0.27 A
Solution: K corresponds to: n  2 to n  1 K  corresponds to: n  3 to n  1

1 1 1  1 3R 1  1 1  8R
 R    or    R 2  2  
 1 4 
0
0.32 A 4  1 3  9

 3R 9 27 0 0
 0
  or    0.32 A or   0.27 A
0.32 A 4 8R 32

Ans. 35: 4.33 m / s


Solution: Linear momentum is conserved in the recoil process.
Momentum of recoil hydrogen atom  mv
E
Momentum of emitted photon 
c
1 1  24  13.6  24
E  E5  E1  13.6  2  2  eV
5 1 
 13.6    eV 
 25  25

 1.6  1019 J 
E E
E  20.8  1018 J  mv  or v
c mc
20.8  1018
v  4.33 m / s
1.67 1027    3 108 

                     
Head office  Branch office 
 
fiziks, H.No. 23, G.F, Jia Sarai,  Anand Institute of Mathematics, 
 
Near IIT, Hauz Khas, New Delhi‐16  28‐B/6, Jia Sarai, Near IIT 
 
Phone: 011‐26865455/+91‐9871145498 Hauz Khas, New Delhi‐16 
              
                   Website: www.physicsbyfiziks.com                        
                           Email: fiziks.physics@gmail.com                       81 
fiziks
Institute for NET/JRF, GATE, IIT‐JAM, JEST, TIFR and GRE in PHYSICAL SCIENCES 
 
Ans. 36: 122.4eV
Solution: For hydrogen atom and hydrogen like atoms
13.6 z 2
En   eV Therefore, ground state energy of doubly ionized lithium atom
n2

13.6   3
2

 Z  3, n  1 will be  E1   13.6  9 or E1  122.4 eV


1
2

Ans. 37: 6
Solution: According to Moseley’s law,
c
f  a 2  Z  b  where f  frequency =
2


c
 a 2  z  1
2
For K line, b  1 

c
 a 2  Z1  1
2
For one atom,
1

 a  Z  1
2 2
c
 a  Z 2  1
2
For other atom, 2
or 1  2 2
2 2 a  Z1  12

  Z 2  1  Z 2  1
2 2

or  Z 2  1  25
2
or  
4 11  1 2
100

or Z 2  1  5 or Z 2  6

                     
Head office  Branch office 
 
fiziks, H.No. 23, G.F, Jia Sarai,  Anand Institute of Mathematics, 
 
Near IIT, Hauz Khas, New Delhi‐16  28‐B/6, Jia Sarai, Near IIT 
 
Phone: 011‐26865455/+91‐9871145498 Hauz Khas, New Delhi‐16 
              
                   Website: www.physicsbyfiziks.com                        
                           Email: fiziks.physics@gmail.com                       82 
fiziks
Institute for NET/JRF, GATE, IIT‐JAM, JEST, TIFR and GRE in PHYSICAL SCIENCES 
 
MSQ
Ans. 38: (a) and (b)
Ans. 39: (a), (c) and (d)
Solution: Every moving particle is associated with wave, the wavelength of this wave is called
h
de-Broglie wavelength given as   where h is Planck’s constant
mv

 p
2

The kinetic energy and momentum is related as E


2m
For constant wavelength momentum should be equal
h
 p  2mE  
2mE
h
For m  m, E  K 1 
2mK
h
m  2m, E  2 K 2 
2 mK
K h
m  2m, E  3 
2 2mK
m h
m E  2 K 4 
2 2mK
Ans. 40: (a) and (b)
Solution: When a photon of frequency  incident on a target metal of threshold frequency v0 and

if v  v0 the electron from metal is ejected. These electron are known as photoelectrons
and this effect is called photo electric effect.
The kinetic energy of the ejected electron  hv  hv0

 KE depends on incident frequency v and v0 i.e., nature of photo emitter.

                     
Head office  Branch office 
 
fiziks, H.No. 23, G.F, Jia Sarai,  Anand Institute of Mathematics, 
 
Near IIT, Hauz Khas, New Delhi‐16  28‐B/6, Jia Sarai, Near IIT 
 
Phone: 011‐26865455/+91‐9871145498 Hauz Khas, New Delhi‐16 
              
                   Website: www.physicsbyfiziks.com                        
                           Email: fiziks.physics@gmail.com                       83 
fiziks
Institute for NET/JRF, GATE, IIT‐JAM, JEST, TIFR and GRE in PHYSICAL SCIENCES 
 
Ans. 41: (a), (b) and (c)
Solution: Consider metal A
Incident energy = work function + Kinetic energy of photoelectrons
 4.25  eV   WA  TA (i)

P2
Kinetic energy = where P  momentum
2m
2 2
P2 1  h  1  h 
 TA  A    , by de Broglie equation TA    (ii)
2m 2m   A  2m  A 

consider metal B
4.7  TA  1.5   WB (iii)
2 2
1  h  1  h 
Also TB       (iv)
2 m  B  2 m  B 

From (iv) and (ii), we get


2 2
TB  A  T  1.5  A 
  or A   TB  TA  1.5
TA  B  TA  B 
2
TA  1.5   A 
or    B  2A 
TA  2 A 
or 4TA  6.0  TA or 3TA  6

TA  2.00eV (v)

from (i), WA  4.25  TA


 4.25  2  2.25 eV
From (iii),
WB  4.7  TA  1.5   4.7   2  1.5   3.95 eV (vii)

Again TB  TA  1.5  2.25  1.5  0.5 eV (viii)


Option (a), (b), (c) are correct as depicted in equation (vi) (vii) and (v) above

                     
Head office  Branch office 
 
fiziks, H.No. 23, G.F, Jia Sarai,  Anand Institute of Mathematics, 
 
Near IIT, Hauz Khas, New Delhi‐16  28‐B/6, Jia Sarai, Near IIT 
 
Phone: 011‐26865455/+91‐9871145498 Hauz Khas, New Delhi‐16 
              
                   Website: www.physicsbyfiziks.com                        
                           Email: fiziks.physics@gmail.com                       84 
fiziks
Institute for NET/JRF, GATE, IIT‐JAM, JEST, TIFR and GRE in PHYSICAL SCIENCES 
 
Ans. 42: (a) and (c)
hc
Solution: According to Einstein’s equation,    eV where   work function

hc   hc  1 
or V   or V    
e e  e  e
1
V and   relation represents a straight line.

hc hc
 Slope of line  or tan  
e e
 Option (c) is correct (i)
At V0  0
V0 Metal1 Metal 2 Metal3
hc hc hc
1 : 2 : 3  : :
01 02 03
1
  0.001 hc  :  0.002 hc  :  0.004 hc   1: 2 : 4 1 3
 Option (a) is correct (ii)
Option (b) is obviously incorrect when (a) is
0.001 0.002 0.004
correct.
From graph
1

nm  1

1
 0.001 nm 1  01  1000nm for metal 1.
01
1
 0.002 nm1  02  500nm for metal 2
02
1
 0.004 nm1  03  250nm for metal 3.
03
 of ultraviolet < 400nm.
The ultraviolet light can be used to emit photoelectrons from metal 1 and metal 2. It
cannot emit electrons from metal 3.
Option (d) is incorrect.
Options (a) and (c) are correct.

                     
Head office  Branch office 
 
fiziks, H.No. 23, G.F, Jia Sarai,  Anand Institute of Mathematics, 
 
Near IIT, Hauz Khas, New Delhi‐16  28‐B/6, Jia Sarai, Near IIT 
 
Phone: 011‐26865455/+91‐9871145498 Hauz Khas, New Delhi‐16 
              
                   Website: www.physicsbyfiziks.com                        
                           Email: fiziks.physics@gmail.com                       85 
fiziks
Institute for NET/JRF, GATE, IIT‐JAM, JEST, TIFR and GRE in PHYSICAL SCIENCES 
 
Ans. 43: (b) and c
Solution: The kinetic energy of an electron in nth orbit of hydrogen atom is
me 4 e2 me 4
K  2 2 2 , V  E 2 2 2
8 0 h n 4 0 r 8 0 h n

th me 4 K
The total energy of an electron in n orbit of hydrogen atom is E  2 2 2   1
8 0 h n E
Ans. 44: (a), (b) and (c)
1  1 1 
Solution: In hydrogen like atoms:  R 2  2 
  n1 n2 
1
Transition of electron occurs from n2 to n1 , is proportional to energy

1  1 1  7R
From n  4 to n  3 , ultraviolet radiation is obtained  R 2  2    0.048 R
  3 4  144
 1 1  3R
1 1  1 1  5R
(a)  R 2  2    0.75R (b)  R 2  2    0.14 R
 1 2  4   2 3  36
1  1 1  3R 1  1 1  9R
(c)  R  2  2    0.2 R (d)  R  2  2    0.02
  2 4  16   4 5  400
 is smaller than ultra violet in (a), (b) and (c)
 is greater than ultra violet in (d). greater the  , less the energy of radiation
Ans. 45: (a), (b), (c) and (d)
Solution: (a) rn  n 2 . Option (a) is correct

13.6 Z 2
(b) Total energy of electron T .E . T .E  . Option (b) is correct
n2
nh
(c) Angular momentum of electron  . Option (c) is correct
2
 27.2 
(d) Potential energy of electron   2  eV for hydrogen atom.
 n 
 13.6 
Kinetic energy of electrons   2  eV
 n 
27.2
 P.E.  2  K .E.  P.E.  2 The option (d) is correct
n
                     
Head office  Branch office 
 
fiziks, H.No. 23, G.F, Jia Sarai,  Anand Institute of Mathematics, 
 
Near IIT, Hauz Khas, New Delhi‐16  28‐B/6, Jia Sarai, Near IIT 
 
Phone: 011‐26865455/+91‐9871145498 Hauz Khas, New Delhi‐16 
              
                   Website: www.physicsbyfiziks.com                        
                           Email: fiziks.physics@gmail.com                       86 
fiziks
Institute for NET/JRF, GATE, IIT‐JAM, JEST, TIFR and GRE in PHYSICAL SCIENCES 
 
Ans. 46: (a) and (d)
1
Solution: According to Bohr model, rn  n 2 (i) vn  (ii)
n
2 2 rn r n2
now Tn   or Tn  n or Tn   Tn  n3
 vn vn 1/ n

Tn 1 n3 n 
3

  13 or 8   1  or n1  2n2
Tn 2 n2  n2 
Option (a): n1  4, n2  2 It fulfils condition

Option (d): n1  6, n2  3 It fulfils condition


Options (a) and (d) are correct
Ans. 47: (b) and (d)
Solution: X  rays emitted from an X  ray tube depend upon:
(i) The accelerating voltage applied to tube. When accelerated, the electrons acquire
greater energy before striking the target.
X  rays emitted from target therefore possess greater energy. X  rays with shorter
wavelength possess greater energy. Hence wavelength of emitted X  rays depends on
the voltage applied to tube.

(ii) According to Moseley’s law, frequency   a 2  Z  b  . Frequency depends upon


2

atomic number of target from which X  ray are emitted.


Ans. 48: (c) and (d)
Solution: for X  ray tube,

m  A  
12375
0

  V
As accelerating voltage is increased, m will decrease.
Number of electrons bombarding the target determine the intensity (or quantity) of
emitted radiation. Accelerating voltage does not change the intensity of X  rays emitted.

                     
Head office  Branch office 
 
fiziks, H.No. 23, G.F, Jia Sarai,  Anand Institute of Mathematics, 
 
Near IIT, Hauz Khas, New Delhi‐16  28‐B/6, Jia Sarai, Near IIT 
 
Phone: 011‐26865455/+91‐9871145498 Hauz Khas, New Delhi‐16 
              
                   Website: www.physicsbyfiziks.com                        
                           Email: fiziks.physics@gmail.com                       87 
fiziks
Institute for NET/JRF, GATE, IIT‐JAM, JEST, TIFR and GRE in PHYSICAL SCIENCES 
 
3. Tools and Postulates of Quantum Mechanics
3.1 The Linear Vector Space
A set of vectors      , … and set of scalars a, b, c defined vector spaces which will
follow A rule for vector addition and rule for scalar multiplication
(i) Addition rule:
If  and  are vectors of elements of a space, their sum    is also vector of the same
space.
(i) Law of Commutative:        

(ii) Law of Associativity:             

(iii) Law of Existence of a null vector and inverse vector   ( )       0

(ii) Multiplication rule:


 The product of scalar with a vector gives another vector, If  and  are two
vectors of the space, any linear combination a  b is also a vector of the space,
where a and b being scalars.
 Distributive with respect to addition:
a (   )  a  a  ( a  b)  a  b

 Associativity with respect to multiplication of scalars. a(b ) = (ab)


 For each element  there must exist a unitary element I and a no. zero scalar
such that I     I   , O     O  O
3.1.1 Scalar Product
The scalar product of two functions   x  and   x  is given by ( ,  )   * ( x) ( x)dx

where  ( x) and  ( x ) are two complex function of variable x.  * ( x ) and  * ( x) are

complex conjugate of   x  and   x  respectively.

The scalar product of two function   x, y, z  and  ( x, y, z ) is 3 dimensional is defined or

(* , )   * dx dy dz

                     
Head office  Branch office 
 
fiziks, H.No. 23, G.F, Jia Sarai,  Anand Institute of Mathematics, 
 
Near IIT, Hauz Khas, New Delhi‐16  28‐B/6, Jia Sarai, Near IIT 
 
Phone: 011‐26865455/+91‐9871145498 Hauz Khas, New Delhi‐16 
              
                   Website: www.physicsbyfiziks.com                        
                           Email: fiziks.physics@gmail.com                       88 
fiziks
Institute for NET/JRF, GATE, IIT‐JAM, JEST, TIFR and GRE in PHYSICAL SCIENCES 
 
3.1.2 Hilbert Space
The Hilbert space H consists of a set of vectors      and set of scalar a,b,c which
satisfied the following four properties.
(i) H is a linear space
(ii) H is a linear space that defined scalar product that is strictly positive.
  ,     ,  *
  , a1  b2   a  , 1   b  , 2 
  ,   
2
0

(iii) H is separable i.e.,


    n  0

(iv H is complete   m  0 when m  , n  

3.1.3 Dimension And Basis of a Vectors.


Linear independency:
A set of N vectors 1 , 2 , 3 ......n , is said to be linearly independent if and only if the
N
solution of the equation.  ai i  0 is a1 = a 2 = a 3 = a 4 = 0 other wise 1 , 2 , 3 ......n
i 1

is said to be linear dependent.


The dimension of a space vector is given by the maximum number of linearly
independent vectors the space can have.
The maximum number of linearly independent vectors a space has is N
1 , 2 , 3 ......N , this space is said to be N dimensional. In this case any vector  of the
N
vector space can be expressed as linear combination.   aii
i 1

Ortho Normal Basis


Two vector i ,  j is said to be orthonormal if their scalar product i ,  j    i , j

where  i , j is kronikar delta that means  i , j  0 where i  j and  i , j  1 if i  j

                     
Head office  Branch office 
 
fiziks, H.No. 23, G.F, Jia Sarai,  Anand Institute of Mathematics, 
 
Near IIT, Hauz Khas, New Delhi‐16  28‐B/6, Jia Sarai, Near IIT 
 
Phone: 011‐26865455/+91‐9871145498 Hauz Khas, New Delhi‐16 
              
                   Website: www.physicsbyfiziks.com                        
                           Email: fiziks.physics@gmail.com                       89 
fiziks
Institute for NET/JRF, GATE, IIT‐JAM, JEST, TIFR and GRE in PHYSICAL SCIENCES 
 
3.1.4 Square Integrable Function

If scalar product  ,     * dx     x  dx   where  is positive finite number
2



Then   x  (x) is said to be square integrable.

The square intergable function can be treated as probability distribution function if


  1 and  is said to be normalized
3.2 Dirac Notation
Dirac introduced what was to become an invaluable notation in quantum mechanics;
state vector  which is square integrable function by what he called a ket vector  .

And its conjugate  by a bra  and scalar product  ,  bra-ket   ( In summery

 |  , *   | and (,  )   |  ) Where ,    * (r , t ) ( r , t )d 3 r .


Properties of kets, bras and bra-kets.
 (|  )*   |

 (a |  )*  a*  |

 | a  a | 

  a | a*  |

  |  *   | 

 a11  a22 | b1 1  b2 2   a1*b1 1 |  1   a1*b2 1 |  2   a2*b1 2 |  1   a2*b2 2 |  2 

  is normalized if    1

 Schwarz inequality
 
|  |   |2   |  |   , which is analogically derived from | A  B || A |2 | B |2

 Triangular inequality    |       |     |  

 Orthogonal states  |    0
 Orthonormal state  |    0,  |    1,  |    1

                     
Head office  Branch office 
 
fiziks, H.No. 23, G.F, Jia Sarai,  Anand Institute of Mathematics, 
 
Near IIT, Hauz Khas, New Delhi‐16  28‐B/6, Jia Sarai, Near IIT 
 
Phone: 011‐26865455/+91‐9871145498 Hauz Khas, New Delhi‐16 
              
                   Website: www.physicsbyfiziks.com                        
                           Email: fiziks.physics@gmail.com                       90 
fiziks
Institute for NET/JRF, GATE, IIT‐JAM, JEST, TIFR and GRE in PHYSICAL SCIENCES 
 
 Forbidden quantities: If  and  belong to same vector space product of the

type   and   are forbidden. They are nonsensical.

 If  and  belong, however to different vector space then    represent

tensor product of  and 

3.3 Operator
An operator A is the mathematical rule that when applied to a ket  will transformed

into another  of the same space and when it acts on a any bra  it transforms it into

another bra  that means A    and  A 


Example of Operator:
Identity operator I |   |  
Pairity operator  | (r )  [ (r )]

Gradiant operator  r ) and Linear momentum operator P (  i r )
3.3.1 Linear Operator: A is linear operator if
 A(1 |  1   2 |  2   1 A |  1   2 A |  2 

 Product of two linear operator A and B written AB which is defined


( AB) |    A( B |   )
3.3.2 Matrix Representation of Operator:
If  is in orthonormal basis of ui is defined as

|    c1 | u2   c2 | u2   ... |    ci | ui  , where ci  ui | 

 c1 
 u1 |     
   c2  which is column vector.
The ket | is defined as  ui |    or | is defined as
 c3 
    
 
 

 The corresponding bra | is defined as (u1 |  * u2 |  * ...) or (c1* , c2* ...c*j ) which

is row matrix.

                     
Head office  Branch office 
 
fiziks, H.No. 23, G.F, Jia Sarai,  Anand Institute of Mathematics, 
 
Near IIT, Hauz Khas, New Delhi‐16  28‐B/6, Jia Sarai, Near IIT 
 
Phone: 011‐26865455/+91‐9871145498 Hauz Khas, New Delhi‐16 
              
                   Website: www.physicsbyfiziks.com                        
                           Email: fiziks.physics@gmail.com                       91 
fiziks
Institute for NET/JRF, GATE, IIT‐JAM, JEST, TIFR and GRE in PHYSICAL SCIENCES 
 
 Operator A is represented as Matrix A whose Matrix element Aij is defined as
ui | A | u j  in ui the basis.

 Transpose of operator A is represented as Matrix AT whose matrix element is


defined as AijT  u j | A | ui 

 Hermitian Adjoint of Matrix A is represent as A† whose matrix element is defined


as A†  u j | A | ui 

Hermitian conjugate A† of matrix A can be find in two step.


Step I: Find transpose of A i.e,.convert row into column ie AT
Step II: Then take complex conjugate to each element of AT .
Properties of Hermitian Adjoint A†
 ( A† )†  A

 (A)†  * A†

 ( A  B)†  A†  B†

 ( AB )†  B † A†
3.3.3Eigen Value of Operator:
If A operator is defined such that A |    |  then

 is said to be eigen value and  is said to be eigen vector corresponding to


operator.
3.3.4 Correspondence between Ket and Bra
If A |  |  then  | A†   | where A† is Hermitian Adjoint of matrix or
operator A.
3.3.5 Hermitian operator: A operator is said to be Hermitian if
A†  A i.e., Matrix element ui | A | u j   (u j | A | ui  )*

 The eigen values of Hermitian matrix is real


 The eigen vectors corresponding to different eigen values are orthogonal.

                     
Head office  Branch office 
 
fiziks, H.No. 23, G.F, Jia Sarai,  Anand Institute of Mathematics, 
 
Near IIT, Hauz Khas, New Delhi‐16  28‐B/6, Jia Sarai, Near IIT 
 
Phone: 011‐26865455/+91‐9871145498 Hauz Khas, New Delhi‐16 
              
                   Website: www.physicsbyfiziks.com                        
                           Email: fiziks.physics@gmail.com                       92 
fiziks
Institute for NET/JRF, GATE, IIT‐JAM, JEST, TIFR and GRE in PHYSICAL SCIENCES 
 
3.3.6 Commutator: if A and B are two operators then the commentator  A, B  is

defined as AB  BA
If  A, B   0 then it is said to be A and B operator commute to each other.

Properties of commutator:
 Antisymmetry [ A, B ]  [ B, A]

 Linearity  A, B  C  D    A, B    A, C    A, D 

 Distributive :  AB, C    A, B  C  B  A, C 
 Jacobi Identity  A,  B,C   B, C , A  C ,  A, B   0

3.3.7 Set of Commuting Observables:


 If two operator A and B commute and if | is eigen vector o A then B| is also
an eigen vector of A, with the same eigen value.
 If two operator A and B commute and if |1 and |2 are two eigen vector of A
with different eigen values then Matrix element 1 | B |  2  is zero.

 If two operator A and B commute one can construct an orthonormal basis of state
space with eigen vectors common to A and B.
3.3.8 Projection operator: The operator P is said to be operator if
P †= P P2 = P
 The product of two commutating projection operator P1 and P2 is also projection
operator.
 The sum of two projection operator is generally not a projection operator.
 The sum of projector P1 + P2 + P2 + .... is projector if Pi Pj are mutually

orthogonal.

                     
Head office  Branch office 
 
fiziks, H.No. 23, G.F, Jia Sarai,  Anand Institute of Mathematics, 
 
Near IIT, Hauz Khas, New Delhi‐16  28‐B/6, Jia Sarai, Near IIT 
 
Phone: 011‐26865455/+91‐9871145498 Hauz Khas, New Delhi‐16 
              
                   Website: www.physicsbyfiziks.com                        
                           Email: fiziks.physics@gmail.com                       93 
fiziks
Institute for NET/JRF, GATE, IIT‐JAM, JEST, TIFR and GRE in PHYSICAL SCIENCES 
 
Example: Prove that f ( x)  x, g ( x)  x , h( x)  x are linear independent
2 3

Solution: For linear independency


a1 f ( x)  a2 g ( x)  a3 h( x)  0  a1 x  a2 x 2  a3 x 3  0
Equating the coefficient of x, x2 and x3 both side one can get.
a1  0 a2  0 a3  0 so f(x), g(x) and h(x) f(x), g(x) are linearly independent.
 
Example: Prove that vector A  6iˆ  9 ˆj and B  2iˆ  3 ˆj are linear dependent.
 
Solution: a1 A  a2 B  0  a1 (6iˆ  9 ˆj )  a2 (2iˆ  3 ˆj )  0  (6a1  2a2 )iˆ  (9a1  3a2 ) ˆj  0

a2
6a1 – 2a2 = 0 or 9a1  3a2  0  a1 
3
 
So A and B are linearly dependent.

Example: If f ( x)  Ae x , g ( x)  Bxe x


2 2
/2 /2
then prove that f ( x) and g ( x) are orthogonal as well
as linearly independent.

Solution: For linearly independency: a1 f ( x)  a2 g ( x)  0  a1e  x  a2 xe  x 0


2 2
/2 /2

(a1  a2 x)  0  a1  0, a2  0

So f ( x) and g ( x ) are linearly independent.

For orthogonality:  f ( x) g ( x)    f * ( x) g ( x)dx



 ( f ( x) g ( x))  AB  xe x dx  0 Scalar product of f ( x) and g ( x ) is zero orthogonal.
2



Example: If f(x) = 0 x<0


f ( x)  Ae  x / a ikx x>0
Then
(a) Find f * ( x)
(b) Find value of A such that f(x) is normalized.

                     
Head office  Branch office 
 
fiziks, H.No. 23, G.F, Jia Sarai,  Anand Institute of Mathematics, 
 
Near IIT, Hauz Khas, New Delhi‐16  28‐B/6, Jia Sarai, Near IIT 
 
Phone: 011‐26865455/+91‐9871145498 Hauz Khas, New Delhi‐16 
              
                   Website: www.physicsbyfiziks.com                        
                           Email: fiziks.physics@gmail.com                       94 
fiziks
Institute for NET/JRF, GATE, IIT‐JAM, JEST, TIFR and GRE in PHYSICAL SCIENCES 
 
Solution: (a) f(x) = 0 x<0
f*(x) = 0 x<0
f ( x)  Ae  x / a ikx x>0

f * ( x)  A*e  x / a ikx x  0

(b) For normalization 
f * ( x) f ( x)dx  1

0   2
 
0dx   A*e x / a ikx Ae x / a ikx dx  1 | A |2
0 0
e 2 x / a dx  1  A 
a

1 
 
Example: (a) If |    A  i  , find 
0
 
(b) Find the value of A such that | is normalized.
1 
Solution: (a): If |    A  i   | A* (1  i 0)
0
 
(b) For normalization condition
  1

1 
 
A * 1  i 0  A  i   1 A
2
1  1  0   1
0
 
1 1
| A |2   A
2 2
Example: If |  1   a1 | 1   a2 | 2  And |  2   b1 | 1   b2 | 2 

It is given that 1 | 2   ij then

(a) Find condition for  1 and  2 are normalized.

(b) Find condition for  1 and  2 are orthogonal.

                     
Head office  Branch office 
 
fiziks, H.No. 23, G.F, Jia Sarai,  Anand Institute of Mathematics, 
 
Near IIT, Hauz Khas, New Delhi‐16  28‐B/6, Jia Sarai, Near IIT 
 
Phone: 011‐26865455/+91‐9871145498 Hauz Khas, New Delhi‐16 
              
                   Website: www.physicsbyfiziks.com                        
                           Email: fiziks.physics@gmail.com                       95 
fiziks
Institute for NET/JRF, GATE, IIT‐JAM, JEST, TIFR and GRE in PHYSICAL SCIENCES 
 
Solution: (a)
If  1 is normalized Then 1 | 1   1

a *
1   a2* 2 a1   a 2 2  1
| a1 |2 1 | 2   a1*a2  1| 2   a2* a1 2 | 1   a2 2 | 2   1
2

It is given that 1 | 1   1 , 2 | 2   1 , 1 | 2   0 , 2 | 1   0 ,

So | a1 |2  | a2 |2  1

Similarly for  2 is normalized

 2 |  2   1  b1 1 | 2   b1*b2 1 | 2   b2*b1 2 | 1   b2 2 | 2   1


2 2

 | b1 |2  | b2 |2  1 condition for  2 is normalized.

(b) For  1 and  2 are orthogonal.

1 |  2   0 or  2 | 1   0

 a 
*
1 1 |  a2* 2 | |  b1 | 1   b2 | 2  

a1*b1 1 | 1   a1*b2 1 | 2   a2*b1 2 | 1   a2*b2 2 | 2 

 a1*b1  a2*b2  0

Similarly from  2 | 1   0  b1*a1  b2*a2  0

Example: If S operator is defined as


S | u1  | u3  S | u2  | u2  S | u3  | u1 

ui | u j   ij i,j = 1, 2, 3

(a) Construct S matrix


(b) Prove that S is hermitian matrix
 S11 S12 S13 
 
Sol.: The Matrix S   S 21 S 22 S23 
S S33 
 31 S32

Where matrix element Sij  ui | S | u j 

                     
Head office  Branch office 
 
fiziks, H.No. 23, G.F, Jia Sarai,  Anand Institute of Mathematics, 
 
Near IIT, Hauz Khas, New Delhi‐16  28‐B/6, Jia Sarai, Near IIT 
 
Phone: 011‐26865455/+91‐9871145498 Hauz Khas, New Delhi‐16 
              
                   Website: www.physicsbyfiziks.com                        
                           Email: fiziks.physics@gmail.com                       96 
fiziks
Institute for NET/JRF, GATE, IIT‐JAM, JEST, TIFR and GRE in PHYSICAL SCIENCES 
 
S11  u1 | S | u1   u1 | u3   0 S12  u1 | S | u2   u1 | u2   0

S13  u1 | S | u3   u1 | u3   1 S21  u2 | S | u1   u2 | u3   0

S22  u2 | S | u2   u2 | u2   1 S23  u2 | S | u3   u2 | u1   0

S31  u3 | S | u1   u3 | u3   1 S32  u3 | S | u2   u3 | u2   0

S33  u3 | S | u3   u3 | u1   0

0 0 1 0 0 1
   
So S matrix is  0 1 0  S†  0 1 0
1 0 0 1 0 0
   
S  S† so S matrix Hermitian.

 nx
Example: If Dx is defined as and  ( x )  A sin
x a
(a) operate D x and  ( x )

(b) operate Dx2 on  ( x )


(c) which one of the above given eigen value problem.
 nx n nx
Solution: (a) Dx  ( x )  A sin  A cos
x a a a

2 nx  n 2 2  nx
(b) Dx2 ( x)  A sin  A   2  sin
x 2
a  a  a

n 2 2 nx
Dx2 ( x)   2
A sin
a a
n 2 2
(c) when Dx2 operate on Dx2  ( x)   ( x)
a2
nx
So operation of Dx2 ( x) on  ( x )  A sin give eigen value problem with eigen
a
n 2 2
value 
a2

                     
Head office  Branch office 
 
fiziks, H.No. 23, G.F, Jia Sarai,  Anand Institute of Mathematics, 
 
Near IIT, Hauz Khas, New Delhi‐16  28‐B/6, Jia Sarai, Near IIT 
 
Phone: 011‐26865455/+91‐9871145498 Hauz Khas, New Delhi‐16 
              
                   Website: www.physicsbyfiziks.com                        
                           Email: fiziks.physics@gmail.com                       97 
fiziks
Institute for NET/JRF, GATE, IIT‐JAM, JEST, TIFR and GRE in PHYSICAL SCIENCES 
 
0 1
Example: If A operator is given by A   
1 0
(a) find eigen value and eigen vector of A .
(b) normalized there eigen vector.
(c) prove both eigen vector are orthogonal.
0 1
Solution: (a) A  for eigen value
1 0
| A  I | 0

0 1 1 0  1
   0   0    1, and   1,
1 0 0 1 1 
The eigen vector corresponding to   1,
A | u1    | u1 

 0 1  a   a 
       a  b
 1 0  b   b 
a
so eigen vector corresponds to   1, | u1    
a
eigen vector corresponds to   1
A | u2    | u 2 

 0 1  a  a a 
    1   a  b | u2    
 1 0  b  b   a 
(b) For normalised eigen vector.
u1 | u1   1 u2 | u2   1

a
| u1     u1 |  a a 
a
1 1 1
u1 | u1   a 2  a 2  1  a  | u1    
2 2 1

                     
Head office  Branch office 
 
fiziks, H.No. 23, G.F, Jia Sarai,  Anand Institute of Mathematics, 
 
Near IIT, Hauz Khas, New Delhi‐16  28‐B/6, Jia Sarai, Near IIT 
 
Phone: 011‐26865455/+91‐9871145498 Hauz Khas, New Delhi‐16 
              
                   Website: www.physicsbyfiziks.com                        
                           Email: fiziks.physics@gmail.com                       98 
fiziks
Institute for NET/JRF, GATE, IIT‐JAM, JEST, TIFR and GRE in PHYSICAL SCIENCES 
 
Similarly,
a 
| u2     and u2 |  (a  a)
 a 
a  1 1 1 
 u 2 | u2   ( a  a )    1  a2  a2  1  a  | u2    
 a  2 2  1
(c) for orthogonality u1 | u2   u2 | u1   0

1 1 1  1 1  1
(1 1)  0 (1  1)  0
2 2  1 2 2  1

Example: If momentum operator Px is defined as i and position operator X is defined as
x
X  x   x  x 

(a) Find the value of commutator [X, Px ]

(b) Find the value of [X 2 , Px ]

(c) Find the value of [X, Px2 ]

Solution: (a): [ X , Px ]  ( X Px  Px X )

Operate both side with   XPx  ( x)  Px X  ( x)

  x 
X  i   Px x ( x)
x
  x  
X  i   i x( x)
x x
  x    x  x
X  i   x i  i  ( x )
x x x
[ X , Px ]  i ( x) [ X , Px ]  i

(b) [ X 2 , Px ]  [ X  X , Px ]  X [ X , Px ]  [ X , Px ] X  Xi  iX  2iX

(c) [ X , Px2 ]  [ X , Px  Px ]

Px [ X Px ]  [ X , Px ]Px  Px i  iPx [ X , Px2 ]  2iPx

                     
Head office  Branch office 
 
fiziks, H.No. 23, G.F, Jia Sarai,  Anand Institute of Mathematics, 
 
Near IIT, Hauz Khas, New Delhi‐16  28‐B/6, Jia Sarai, Near IIT 
 
Phone: 011‐26865455/+91‐9871145498 Hauz Khas, New Delhi‐16 
              
                   Website: www.physicsbyfiziks.com                        
                           Email: fiziks.physics@gmail.com                       99 
fiziks
Institute for NET/JRF, GATE, IIT‐JAM, JEST, TIFR and GRE in PHYSICAL SCIENCES 
 
Example: (a) Prove that P |  | is projection operator

(b) Operate P on 

(c) Operate P on 

(d) Operate P on  and 

(e) Find the eigen value of any projection operator.


Solution: (a) P* |   P P2  P  

       P2 |  | P

So P is projection operator.
(b) P |  |       |    | 

(c)  | P    |    |   |  |

(d) P |  |    |    |   1

 | P        |

(e) P2  P P2  P  0 P ( P  I )  0

P  0, P  I so eigen value of P  0 or 1

1 0 0 1
Example: If A  a   B  b 
0 1 1 0
(a) Find the value of [A, B]
(b) Write down eigen vector of B in the basis of eigen vector of A .
Solution (a): [A, B] = AB - BA

1 0 0 1 0 1 1 0
 a b   b a 
 0 1 1 0 1 0 0 1
0 1 0 1 0 0
 ab    ba     [ A, B ]  0 so A and B commute.
1 0 1 0 0 0

                     
Head office  Branch office 
 
fiziks, H.No. 23, G.F, Jia Sarai,  Anand Institute of Mathematics, 
 
Near IIT, Hauz Khas, New Delhi‐16  28‐B/6, Jia Sarai, Near IIT 
 
Phone: 011‐26865455/+91‐9871145498 Hauz Khas, New Delhi‐16 
              
                   Website: www.physicsbyfiziks.com                        
                           Email: fiziks.physics@gmail.com                       100 
fiziks
Institute for NET/JRF, GATE, IIT‐JAM, JEST, TIFR and GRE in PHYSICAL SCIENCES 
 
1 
(b) Eigen vector of A is a1    eigen value 1  a
0
0
|a2 =   eigen value 2  a
1 
1  1
Eigen vector of B is | b1     eigen value 1  b
2  1

1 1 
| b2     eigen value 2   b
2  1

1 1  1  0  1 1
| b1        | a1   | a2 
2 0 2 1  2 2

1 1  1  0  1 1
| b2       | b2   | a1   | a2 
2 0 2 1  2 2

1 0 0   0 i 3i 
   
Example: A   0 7 3i  B   i 0 i 
 0 3i 5   3i i 0 
   
(a) find A† (b) find B †
(c) which one of among A and B have real eigen value.
1 0 0  1 0 0 
   
Solution: (a) A   0 7 3i  A   0 7 3i 

 0 3i 5   0 3i 5 
   
A†  A so A is Hermitian.
 0 i 3i   0 i 3i 
   
(b) B   i 0 i  B  i

0 i 
 3i i 0   3i i 0 
  
B†   B
So it is not Hermitian rather it is Anti-Hermitian.
(c) The eigen value of A matrix is real because A is Hermitian.

                     
Head office  Branch office 
 
fiziks, H.No. 23, G.F, Jia Sarai,  Anand Institute of Mathematics, 
 
Near IIT, Hauz Khas, New Delhi‐16  28‐B/6, Jia Sarai, Near IIT 
 
Phone: 011‐26865455/+91‐9871145498 Hauz Khas, New Delhi‐16 
              
                   Website: www.physicsbyfiziks.com                        
                           Email: fiziks.physics@gmail.com                       101 
fiziks
Institute for NET/JRF, GATE, IIT‐JAM, JEST, TIFR and GRE in PHYSICAL SCIENCES 
 
3.4 Postulates of Quantum Mechanics
The quantum mechanical postulates enable us to understand.
 how a quantum state is described mathematically at a given time t.
 how to calculate the various physical quantities from this quantum state.
 Knowing the system’s state at a time t, how to find the state at any later time t. i.e.,
how to describe the time evolution of a system.
There are following set of postulates.
Postulate 1: The state of any physical system is specified, at each time t, by a state
vector | (t ) in the Hilbert space. | (t ) contains all the needed information about the
system. Any superposition of state vectors is also a state vector.

Postulate 2: To every measurable quantity A to be called an observable or dynamical


variable. There corresponds a linear Hermitian  whose eigen vectors form a complete
basis. A n =a n n

Postulate 3: The measurement of an observable A many be represented formally by an


action of  on a state vector   t  .

The state of the system immediately after the measurement is the normalized projection
Pn | 
of |   onto the eigen subspace associate with an.
 | Pn |  

Postulate 4 (a): When the physical quantity A is measured on a system in the state |
the probability P(an) of obtaining the non-degenerate eigen value an of the corresponding
| n |   |2
observable A is P(an )  where A n  an n
 |  

                     
Head office  Branch office 
 
fiziks, H.No. 23, G.F, Jia Sarai,  Anand Institute of Mathematics, 
 
Near IIT, Hauz Khas, New Delhi‐16  28‐B/6, Jia Sarai, Near IIT 
 
Phone: 011‐26865455/+91‐9871145498 Hauz Khas, New Delhi‐16 
              
                   Website: www.physicsbyfiziks.com                        
                           Email: fiziks.physics@gmail.com                       102 
fiziks
Institute for NET/JRF, GATE, IIT‐JAM, JEST, TIFR and GRE in PHYSICAL SCIENCES 
 
Postulate 4 (b): When the physical quantity A is measured on a system in the state  .

The probability P(an) of the obtaining the eigen value an of the corresponding observable
gn

 |  i
n |   |2
A is P(an )  i 1

 | 

Where gn is the degree of degeneracy of an and | ni  (i = 1, 2, 3, &, g n ) is orthonormal


set of vector which forms a basis in the eigen subspace en associated with eigen value an
of A.
Postulate 5: The time evaluation of the state vector |  (t ) is governed by schrodinger
d
equation i |  (t )  H (t ) |  
dt
Where H is Hamiltonion of the system.
The solution of schodinger equation must be
(a) Single valued and value must be finite
(b) Continuous
(c) Differentiable
(d) Square integrable.
3.4.1 Expectation Value: The expectation value of operator A is given
 | A |  an | n |   |
  A   A  
 |   n  |  

Where  m | A |  n   an mn   A   an Pn (an )


n

For continuous variable


 a |  (a) |
2
da 
  A  

  adP(a)
 |  (a) | 
2
da


 Error in measurement of A is A  A2  A where A  0


2

                     
Head office  Branch office 
 
fiziks, H.No. 23, G.F, Jia Sarai,  Anand Institute of Mathematics, 
 
Near IIT, Hauz Khas, New Delhi‐16  28‐B/6, Jia Sarai, Near IIT 
 
Phone: 011‐26865455/+91‐9871145498 Hauz Khas, New Delhi‐16 
              
                   Website: www.physicsbyfiziks.com                        
                           Email: fiziks.physics@gmail.com                       103 
fiziks
Institute for NET/JRF, GATE, IIT‐JAM, JEST, TIFR and GRE in PHYSICAL SCIENCES 
 
3.4.2 Fourier transformation: Change in basis from one representation to another
representation.
|p is defined as
1
| p  e i / p x
2 
The expansion of (x) in terms of |p can be written as.
 
1
 ( x)   a( p ) | p dp  ( x)   a ( p )e
ipx / 
dp
 2  

Where a(p) can be find



1
  ( x )e
 ipx / 
a( p)  dx
2  

In 3D
3/ 2 
 1   

  ( r )e d r
 ip .r /  3
a( p)   
 2   

Where a ( p ) being expansion coefficient of | p .


 If any function  ( x ) can be expressed as a linear combination of state function
n

i.e.,  ( x)   cnn ( x) then where  m* n dx   mn then cn   n* ( x ) f ( x ) dx


n

which is popularly derived from fourier trick.


Parity operator: The parity operator  defined by its action on the basis.
 | r  |  r   r |  |     ( r )
If  (  r )   ( r ) then state have even parity and
If  (  r )   ( r ) then state have odd parity.
So parity operator have 1 and 1 eigen value.
Representation of postulate (4) in continuous basis.

                     
Head office  Branch office 
 
fiziks, H.No. 23, G.F, Jia Sarai,  Anand Institute of Mathematics, 
 
Near IIT, Hauz Khas, New Delhi‐16  28‐B/6, Jia Sarai, Near IIT 
 
Phone: 011‐26865455/+91‐9871145498 Hauz Khas, New Delhi‐16 
              
                   Website: www.physicsbyfiziks.com                        
                           Email: fiziks.physics@gmail.com                       104 
fiziks
Institute for NET/JRF, GATE, IIT‐JAM, JEST, TIFR and GRE in PHYSICAL SCIENCES 
 
1
Example: A state function is given by |   | 1   | 2  It is given that i |  j    ij
2
(a) check | is normalized or not
(b) write down normalized wavefunction |.
(c) It is given H | n   (n  1) | n  n = 0, 1, 2, 3, …

If H will measured on |. what will be measurement with what probability.


(d) Find the expectation value at H i.e., H
(e) Find the error in the measurement in H.
Solution: (a) To check normalization one should verify.
 |    1
1
|   | 1   | 2 
2
2
1 1  1 
 |    1 | 1   1 | 2   2 | 1   2 2  
2 2  2
1 3
 1 0  0  
2 2
3
The value of  |    so | is not normalized.
2

(b) Now we need to find normalized | let A be normalization constant.


1
|   A | 1   | 2 
2

A2 3 A2 2
 |   A  1 
2
1  A 
2 2 3

2 1
So |   | 1   | 2 
3 3

2 1
 | 1 |  2 |
3 2

                     
Head office  Branch office 
 
fiziks, H.No. 23, G.F, Jia Sarai,  Anand Institute of Mathematics, 
 
Near IIT, Hauz Khas, New Delhi‐16  28‐B/6, Jia Sarai, Near IIT 
 
Phone: 011‐26865455/+91‐9871145498 Hauz Khas, New Delhi‐16 
              
                   Website: www.physicsbyfiziks.com                        
                           Email: fiziks.physics@gmail.com                       105 
fiziks
Institute for NET/JRF, GATE, IIT‐JAM, JEST, TIFR and GRE in PHYSICAL SCIENCES 
 
(c) It is given that
H | n   (n  1) n = 0, 1, 2, 3…

H | 1   2 H | 2   3

When H will measured | it will measured either 2 or 3


The probability of measured 2 is P(2) is given by

| 1 |   |2 2 | 2 |  |2 1
P(2 )   P(3 )  
 |   3  |   3
So when H will measure state | the following outcome will come.
Measurement of H on state : |1 |2
Measurement : 2 3
Probability : 2/3 1/3
 | H |  
(d) H     Pn (an )an
 |  n

2 1 7 
 2   3  H  
3 3 3
 | H 2 |   2 1
H  
2
 Pn (an )an2   (2 ) 2   (3 ) 
 |   3 3

8 2 2 9 2 2 17 2 2
  
3 3 3
(e) The error in measurement in H is given as
17 2 2
H   H 2    H  2 H 2 
3

 7   9 2 2
2
17 49
H    2
  H   
 3  9 3 9

51  49 2
H    
9 3

                     
Head office  Branch office 
 
fiziks, H.No. 23, G.F, Jia Sarai,  Anand Institute of Mathematics, 
 
Near IIT, Hauz Khas, New Delhi‐16  28‐B/6, Jia Sarai, Near IIT 
 
Phone: 011‐26865455/+91‐9871145498 Hauz Khas, New Delhi‐16 
              
                   Website: www.physicsbyfiziks.com                        
                           Email: fiziks.physics@gmail.com                       106 
fiziks
Institute for NET/JRF, GATE, IIT‐JAM, JEST, TIFR and GRE in PHYSICAL SCIENCES 
 
 B 
Example: The wave function of a particle is given by    0  Bi1  , where 0 and 1 are
 2 
the normalised eigenfunctions with energy E0 and E1 corresponding to ground state and
first exited state .
(a) Find the value of B such that  is normalised.
(b) What is measurement
(c) What is the probability getting E1

(d) What is E

B
Solution: (a)   0  Bi1
2

For normalized    1

B2 2
   B2  1  B2 
2 3

1 2
Now   0  i1
3 3
(b) Measurement are E0 , E1
2
1  2
(c) Probability getting E1 , P  E1   
 3

1 2 E  2 E1
(d) E   En P  En    E0  E1  0
n 0 3 3 3

Example: (a) Plot  I ( x )  A1e | x|   x  

Plot  II ( x)  A2 e x   x  
2
(b)

(c) discuss why I is not solution of Schrödinger wave function rather   is solution

of Schrödinger wave function.

                     
Head office  Branch office 
 
fiziks, H.No. 23, G.F, Jia Sarai,  Anand Institute of Mathematics, 
 
Near IIT, Hauz Khas, New Delhi‐16  28‐B/6, Jia Sarai, Near IIT 
 
Phone: 011‐26865455/+91‐9871145498 Hauz Khas, New Delhi‐16 
              
                   Website: www.physicsbyfiziks.com                        
                           Email: fiziks.physics@gmail.com                       107 
fiziks
Institute for NET/JRF, GATE, IIT‐JAM, JEST, TIFR and GRE in PHYSICAL SCIENCES 
 
Solution: (a)  I ( x )  A1e x  0
x   x

 II ( x )  A1e  x x  0 A1

The plot is given by

O x

 II ( x)  A2 e x   x  
2
(b) A2

The plot is given by

(c) Both the function I and II are single value, continuous, square integrable by  I is

not differentiable of x = 0 rather  II is differentiable at x  0

So  II can be solution of Schrödinger wave function but I is not solution of

Schrödinger wave function.


1
Example: A time t = 0 the state vector |  (0) as |  (0)  | 1  | 2  
2
It is given as Hamiltonian is defined as H | n   n 2 0 | n 

(a) What is wave function |(t) at later time t.


(b) Write down expression of evolution of |(x, t)|2
(c) Find H
(d) Find the value of H  t

1  
 i0 t  i 40 t
Solution: (a) |  (t )  e

| 1   e 
| 2  
2 
|  (t )  | 1   e 21t | 2  

E2  E1 3 0
Where 21  
 

                     
Head office  Branch office 
 
fiziks, H.No. 23, G.F, Jia Sarai,  Anand Institute of Mathematics, 
 
Near IIT, Hauz Khas, New Delhi‐16  28‐B/6, Jia Sarai, Near IIT 
 
Phone: 011‐26865455/+91‐9871145498 Hauz Khas, New Delhi‐16 
              
                   Website: www.physicsbyfiziks.com                        
                           Email: fiziks.physics@gmail.com                       108 
fiziks
Institute for NET/JRF, GATE, IIT‐JAM, JEST, TIFR and GRE in PHYSICAL SCIENCES 
 
(b) Evolution of shape of the wave packet
1 1
|  ( x, t ) |2  | 1 ( x) |2  | 2  x  |2 12 cos 21t
2 2

H    H 2    H  2 
1/ 2
(c)

1 1 5 1 2 1 2 17 2
H   E1  E2  E1 H 2  E1  E2  E1
2 2 2 2 2 2
3 3
H  E1 H  0
2 2
3 1  3  
(d) H  0 t  t  H  t  0  
2 21 3 0 2 3 0 2


H  t 
2
Example: Consider a one-dimensional particle which is confined within the region 0  x  a and
  x  it
whose wave function is  ( x, t )  sin   e . Find the potential V  x  .
 a 
Solution: From the fifth postulate.
 P2 2  2
H  i H  V ( x)    V ( x)
t 2m 2m x 2
 2  2 
 V ( x)  i
2m x 2
t
 2 2 x x x
2
sin eit  V ( x) sin eit  i sin (i )eia
2ma a a a

 22  2 2   2 2 
 V ( x)   V ( x )     V ( x)      
2ma 2 2ma 2  2ma 2 

                     
Head office  Branch office 
 
fiziks, H.No. 23, G.F, Jia Sarai,  Anand Institute of Mathematics, 
 
Near IIT, Hauz Khas, New Delhi‐16  28‐B/6, Jia Sarai, Near IIT 
 
Phone: 011‐26865455/+91‐9871145498 Hauz Khas, New Delhi‐16 
              
                   Website: www.physicsbyfiziks.com                        
                           Email: fiziks.physics@gmail.com                       109 
fiziks
Institute for NET/JRF, GATE, IIT‐JAM, JEST, TIFR and GRE in PHYSICAL SCIENCES 
 
Example: If eigen value of operator A is 0, 2a0, 2a0 and corresponding normalized eigen vector
0  0 1  1 
1   1     1 
is i , i and  0  respectively t system is in state 6  0  then
2   2   0  4
1  1     

1 
1 
(a) When A is measured on system in state  0  then what is probability to getting
6 
 4
value 0, 2a0 , respectively.
(b) What is expectation value of A ?
0  0
1   1  
Solution: Let | 1   i  1  0, | 2   i   2  2a0
2   2  
1  1 
2 = 3 = 2a0 i.e.,  = 2a0 is doubly degenerate.
| 1 |  (t ) |2 8
P(0)  
 |   17

| 2 |  (t ) |2 | 3 |  (t ) |2
P(2a0 )  
 |    |  

 1  
2  1  
2

 1      1  

 2 (0  i 1  0   (1 0 0)  0  
  6     6   1 1
 16
1
 
4  4 
    2 36  36
1  1  1 17
1 1  1 1   36  (1  16) 36
(1 0 4)  0  (1 0 4)  0 
6 6  6 6 
 4  4
2
1 9 8 9 18a0
 9     A  0   2a0    A  (Average value)
17 17 17 17 17 17
36

                     
Head office  Branch office 
 
fiziks, H.No. 23, G.F, Jia Sarai,  Anand Institute of Mathematics, 
 
Near IIT, Hauz Khas, New Delhi‐16  28‐B/6, Jia Sarai, Near IIT 
 
Phone: 011‐26865455/+91‐9871145498 Hauz Khas, New Delhi‐16 
              
                   Website: www.physicsbyfiziks.com                        
                           Email: fiziks.physics@gmail.com                       110 
fiziks
Institute for NET/JRF, GATE, IIT‐JAM, JEST, TIFR and GRE in PHYSICAL SCIENCES 
 
Example: A free particle which is initially localized in the range  a  x  a is released at
time t  0 .
A if  a  x  a
 ( x)  
0 otherwise

Find (a) A such that   x  is normalized.

(b) Find   x  i.e., wave function in momentum space.

(c) Find   x, t  i.e., wave function after t time.


 a
1
 |  ( x, t ) | dx  A  dx  1  A 
2 2
Solution: (a)
 a 2a
a a
1 1 1 1 sin ka
(b)  ( x)   a e  ( x)dx  2 e
 ikx  ikx

2 2a a 2 k
 k 2 

1 sin ka i kx  2 m t 
(c)  ( x, t ) 
 2a  k e

dk

                     
Head office  Branch office 
 
fiziks, H.No. 23, G.F, Jia Sarai,  Anand Institute of Mathematics, 
 
Near IIT, Hauz Khas, New Delhi‐16  28‐B/6, Jia Sarai, Near IIT 
 
Phone: 011‐26865455/+91‐9871145498 Hauz Khas, New Delhi‐16 
              
                   Website: www.physicsbyfiziks.com                        
                           Email: fiziks.physics@gmail.com                       111 
fiziks
Institute for NET/JRF, GATE, IIT‐JAM, JEST, TIFR and GRE in PHYSICAL SCIENCES 
 
Questions
MCQ (Multiple Choice Questions)
Q1. The quantum mechanical operator for the momentum of a particle moving in one
dimension is given by
d d  2 d 2
(a) i (b)  i (c) i (d) 
dx dx t 2m dx 2

Q2. If the distribution function of x is f  x   xe  x /  over the interval 0 < x < , the mean
value of x is

(a)  (b) 2 (c) (d) 0
2
1 
Q3. If  0 is written in orthonormal basis of n as  0  A 1  3 2  i 3  4 
7
Then the value of A Such that  0 is normalized.

(a) 2 (b) 2 (c) 12 (d) 2


Q4. If an operator A is defined as , A n   n  1 a0 n . n  1, 2, 3 &Where

m n   m ,n .If A is measured on state  0   1  2  what is measurement


(a) 2a0 (b) a0 , 2a0 (c) 2a0 ,3a0 (d) 4a0

Q5. If an operator A is defined as, A n   n  1 a0 n . & n  1, 2, 3 Where m n   m ,n . If

A is measured on state  0   2 1  3 2  what is probability of measurement of 3a 0

1 4 9
(a) (b) (c) (d) 0
4 13 13
Q6. If an operator A is defined as , A n   n  1 a0 n & n  1, 2, 3 Where m n   m ,n .If

A is measured on state  0   2 1  3 2  what will average value of measurement of


A
35a0 3a0 10a0
(a) 5a0 (b) (c) (d)
13 5 13

                     
Head office  Branch office 
 
fiziks, H.No. 23, G.F, Jia Sarai,  Anand Institute of Mathematics, 
 
Near IIT, Hauz Khas, New Delhi‐16  28‐B/6, Jia Sarai, Near IIT 
 
Phone: 011‐26865455/+91‐9871145498 Hauz Khas, New Delhi‐16 
              
                   Website: www.physicsbyfiziks.com                        
                           Email: fiziks.physics@gmail.com                       112 
fiziks
Institute for NET/JRF, GATE, IIT‐JAM, JEST, TIFR and GRE in PHYSICAL SCIENCES 
 
Q7. If an operator A is defined as , A n  na0 n n  1, 2, 3 where m n   m ,n .If A is

measured on state  0   2 1  3 2  what will average value of measurement of A

35a0 3a0 22a0


(a) 5a0 (b) (c) (d)
13 5 13
Q8. If potential energy of system is the attractive delta function potential defined
as V  x   b  x  , where b  0 , and wave function is defined

 x 
 A cos , for a  x  a 
as  x    2a  . The average value of energy is given by
 0, otherwise 

 2 2 b  2 2 b
(a) E  2
 (b) E  2

8ma a 8ma a
 2 2 b  2 2 b
(c) E  2
 (d) E  2

8mb a 8mb a
 1 
Q9. The wavefunction of a particle is given by    0  i1  , where 0 and 1 are the
 2 
normalized eigenfunctions with energies E0 and E1 corresponding to the ground state
and first excited state, respectively. The expectation value of the Hamiltonian in the state
 is
E0 E0 E0  2 E1 E0  2 E1
(a)  E1 (b)  E1 (c) (d)
2 2 3 3
 1 
Q10. The wave function of a particle is given by    0  i1  , where 0 and 1 are the
 2 
normalized eigenfunctions with energies E0 and E1 corresponding to the ground state

and first excited state, respectively. Then H 2 in the state  is

E02 E02 E02  2 E12 E02  2 E12


(a)  E12 (b)  E12 (c) (d)
2 2 3 3

                     
Head office  Branch office 
 
fiziks, H.No. 23, G.F, Jia Sarai,  Anand Institute of Mathematics, 
 
Near IIT, Hauz Khas, New Delhi‐16  28‐B/6, Jia Sarai, Near IIT 
 
Phone: 011‐26865455/+91‐9871145498 Hauz Khas, New Delhi‐16 
              
                   Website: www.physicsbyfiziks.com                        
                           Email: fiziks.physics@gmail.com                       113 
fiziks
Institute for NET/JRF, GATE, IIT‐JAM, JEST, TIFR and GRE in PHYSICAL SCIENCES 
 
Q11. The wave function of a particle at time t = 0 is given by  0 
1
 u1  u 2  , where
2
u1 and u2 are the normalized eigenstates with eigenvalues E1 and E2

respectively, E2  E1  . Find the expression  t  after later time t


i ( E1  E2 ) t i ( E1  E2 ) t

 u1  u2  e  u1  u2  e
1  1
(a)   t   2
(b)   t   2
2 2

1   1  
 iE1t  iE2 t  iE1t  iE2 t
(c)   t    u1 e 
 u2 e 
 (d)   t    u1 e 
 u2 e 

2  2 

Q12. The wave function of a particle at time t = 0 is given by  0 


1
u
1  u 2  , where
2
u1 and u 2 are the normalized eigenstates with eigenvalues E1  E0 and E2  2 E0

respectively, E2  E1  . The shortest time after which  t  will become orthogonal to

 0  is
 2   3 
(a) (b) (c) (d)
E0 E0 2 E0 2 E0

Q13. If the expectation value of the momentum is p for the wavefunction   x  , then the

expectation value of momentum for the wavefunction ei k x /   x  is

(a) k (b) p  k (c) p  k (d) p

Q14. If a particle is represented by the normalized wave function


 15 a 2  x 2

 for  a  x  a
  x   4a 5 / 2

0 otherwise

Find P2

5 2 5 2 2 2 2
(a) (b) (c) (d)
2a 2 a2 5a 2 5a 2

                     
Head office  Branch office 
 
fiziks, H.No. 23, G.F, Jia Sarai,  Anand Institute of Mathematics, 
 
Near IIT, Hauz Khas, New Delhi‐16  28‐B/6, Jia Sarai, Near IIT 
 
Phone: 011‐26865455/+91‐9871145498 Hauz Khas, New Delhi‐16 
              
                   Website: www.physicsbyfiziks.com                        
                           Email: fiziks.physics@gmail.com                       114 
fiziks
Institute for NET/JRF, GATE, IIT‐JAM, JEST, TIFR and GRE in PHYSICAL SCIENCES 
 
Q15. A quantum mechanical particle in has the initial wave function    0  x    1  x ,

where  0 and  1 are the real wave functions in the ground and first excited state of the

Hamiltonian with energy E0 and E1 respectively . What is expression of probability

density at time t .
t
(a)   x, t    0  x    1  x   2 0  x  1  x  cos  E2  E1 
2 2 2


t
(b)   x, t    0  x    1  x   2 0  x  1  x  cos  E2  E1 
2 2 2


t
(c)   x, t    0  x    1  x   2 0  x  1  x  cos  E2  E1  (c)
2 2 2


t
(d)   x, t    0  x    1  x   2 0  x  1  x  cos  E2  E1 
2 2 2


Q16. A quantum mechanical particle in has the initial wave function    0 x   1  x , where

 0 and  1 are the real wave functions in the ground and first excited state of the
Hamiltonian with energy E0 and E1 respectively . What is expression of probability

density at time t   if E1  E0   assume   1,   1

(a)  1 x   0  x  (b)  1 x    0  x 
2 2 2

(c)  1 x   0 x  (d)  1  x    0  x 
2 2 2

d  d 
Q17. The operator   x    x  is equivalent to
 dx   dx 
d2 d2
(a)  x2 (b)  x2 1
dx 2
dx 2
d2 d d2 d
(c) 2
 x x2 1 (d) 2
 2x  x 2
dx dx dx dx

                     
Head office  Branch office 
 
fiziks, H.No. 23, G.F, Jia Sarai,  Anand Institute of Mathematics, 
 
Near IIT, Hauz Khas, New Delhi‐16  28‐B/6, Jia Sarai, Near IIT 
 
Phone: 011‐26865455/+91‐9871145498 Hauz Khas, New Delhi‐16 
              
                   Website: www.physicsbyfiziks.com                        
                           Email: fiziks.physics@gmail.com                       115 
fiziks
Institute for NET/JRF, GATE, IIT‐JAM, JEST, TIFR and GRE in PHYSICAL SCIENCES 
 
Q18. Suppose Hamiltonian of a conservative system in classical mechanics is H  xp , where
 is a constant and x and p are the position and momentum respectively. The
corresponding Hamiltonian in quantum mechanics, in the coordinate representation, is
  1   1
(a)  i  x   (b)  i  x  
 x 2   x 2 
 i 
(c)  ix (d)  x
x 2 x
Q19. The commentator x 2 , p 2 is  
(a) 2ixp (b) 2i ( xp  px ) (c) 2ipx (d) 2i ( xp  px )
Q20. Given the usual canonical commutation relations, the commentator A, B  of
A  i xp y  yp x  and B   yp z  zp y  is

(a)  xp z  p x z  (b)   xp z  p x z 

(c)  xp z  p x z  (d)   xp z  p x z 

Q21. If   x   A exp x 4  is the eigenfunction of a one dimensional Hamiltonian with


eigenvalue E  0 , the potential V x  (in units where   2m  1 ) is

(a) 12x 2 (b) 16x 6 (c) 16 x 6  12 x 2 (d) 16 x 6  12 x 2


MSQ (Multiple Select Questions)
Q22. If a operator A associated with defined as A n  an n where n  1, 2,3, 4..... . if A is

measured on state  then which of the following is /are correct.

(a) The measurement of A on state  is an

n 
(b) The probability of measurement of an on state  is


(c) The expectation value of measurement of A on state  is  A


2
n 
(d) The average value of measurement of A on state  is a
n
n


                     
Head office  Branch office 
 
fiziks, H.No. 23, G.F, Jia Sarai,  Anand Institute of Mathematics, 
 
Near IIT, Hauz Khas, New Delhi‐16  28‐B/6, Jia Sarai, Near IIT 
 
Phone: 011‐26865455/+91‐9871145498 Hauz Khas, New Delhi‐16 
              
                   Website: www.physicsbyfiziks.com                        
                           Email: fiziks.physics@gmail.com                       116 
fiziks
Institute for NET/JRF, GATE, IIT‐JAM, JEST, TIFR and GRE in PHYSICAL SCIENCES 
 
 1 
Q23. The wave function of a particle is given by   B  0  i1  , where 0 and 1 are the
 2 
orthonormal eigenfunctions with energy E0 and E1 corresponding to ground state and
first exited state.
2
(a) The value of B such that  is normalized is
3
1
(b) The probability of measurement to getting E1 on state  is
2
E0  E1
(c) The average value of energy E 
2
E0  2 E1
(d) The average value of energy E 
3
1 0  2i 
Q24. If Hamiltonian H is defined as H   0   measure on state     .
0 4 3 
(a) if H will measure on state  measurement is 5 0

(b) if H will measure on state  measurement is 1 0 , 4 0


1
(c) Expectation value of H on eigen state   is 1 0
0
40 0
(d) Expectation value of H on eigen state  is
13
Q25. The wave function  of a quantum mechanical system described by a Hamiltonian
H can be written as a linear combination of 1 and 2 which are eigenfunction of
4 3
H with eigenvalues E1 and E2 respectively at t  0 ,  0  1  2 and then allowed to
5 5
evolve with time,
h 1  4 3
(a) The magnitude of wave function after time t  T    is  0  1  2
2  E1  E2  5 5
h 1  4 3
(b) The magnitude of wave function after time t  T    is  0  1  2
2  E1  E2  5 5
h 1 
2
4 3 
 is   r , t    1  2 
2
(c) The probability density after time t  T  
2  E1  E2  5 5 
h 1 
2
4 3 
 is   r , t    1  2 
2
(d) The probability density after time t  T  
2  E1  E2  5 5 

                     
Head office  Branch office 
 
fiziks, H.No. 23, G.F, Jia Sarai,  Anand Institute of Mathematics, 
 
Near IIT, Hauz Khas, New Delhi‐16  28‐B/6, Jia Sarai, Near IIT 
 
Phone: 011‐26865455/+91‐9871145498 Hauz Khas, New Delhi‐16 
              
                   Website: www.physicsbyfiziks.com                        
                           Email: fiziks.physics@gmail.com                       117 
fiziks
Institute for NET/JRF, GATE, IIT‐JAM, JEST, TIFR and GRE in PHYSICAL SCIENCES 
 
4 3
Q26. If A state of particle   r , t  0   1  2 having energy E1 and E2 energy eigen
5 5
value with 1 and 2 are normalized eigenfunction of Hamiltonian H .

4 iE1t 3 iE2t
(a) then   r , t   1 exp  2 exp
5  5 
 9
(b) after time t  cos 1     r , t  0  and   r , t  became orthogonal
 E2  E1   16 
4 E1  3E2
(c) Average value of energy at time t  0 is
5
16 E1  9 E2
(d) Average value of energy at time t  t is
25
Q27. A quantum mechanical state of a system is given by   Ax(a  x) if 0  x  a and
 ( x)  0 other wise then

30
(a) The value of A is for normalized  .
a5
a
(b) The average value of position on state  is .
2
(c) The average value of momentum on state  is not equal to zero

5 2
(d) The average value of kinetic energy is
ma 2

                     
Head office  Branch office 
 
fiziks, H.No. 23, G.F, Jia Sarai,  Anand Institute of Mathematics, 
 
Near IIT, Hauz Khas, New Delhi‐16  28‐B/6, Jia Sarai, Near IIT 
 
Phone: 011‐26865455/+91‐9871145498 Hauz Khas, New Delhi‐16 
              
                   Website: www.physicsbyfiziks.com                        
                           Email: fiziks.physics@gmail.com                       118 
fiziks
Institute for NET/JRF, GATE, IIT‐JAM, JEST, TIFR and GRE in PHYSICAL SCIENCES 
 
NAT (Numerical Answer Type)
C 3
Q28. If   1  2 the value of C is ………….. such that  is normalized .
15 15

Q29. If a state   C  1   3  4i  2  . Find the value of C such that  is normalised.

Q30. A operator is defined as An   3n  1 a0n is if A will measure on state  which is

defined as orthonormal basis of n


1
1 
 3  4i   . Measurement of A is
as   2
26 26
 a0 and  a0 then value of  and  respectively .........and ………. (    )
Q31. A operator is defined as An   3n  1 a0n is if A will measure on state  which is

defined as orthonormal basis of n as 


1
1 
 3  4i   . Then probability of
2
26 26
measurement of 5a0 is ……………
Q32. A operator is defined as An  na0n is if A will measure on state  which is defined as

orthonormal basis of n as   1  (3  4i )2 . Then probability of measurement of

2a0 is ………

Q33. Assume operator A defined as An  na0n and operator Bn  (n  1)b0n where

2 1
n  1, 2,3... and if A state  is defined   1  2 if some one measure A on state
3 3
 he measure a0 at the same time there is measurement of operator B the measurement
is ……….. b0

2 1
Q34. If state of a system at time t  0 is defined by   1  2 . If Hamiltonian H is
3 3
 1
defined as H n  n 2 E0n after time t  cos 1  then value of  is …………
 E0 2
1 4
Q35. If state of a system at time t  0 is defined by   1  3 if Hamiltonian H is
5 5

defined as H n  nE0n where n  1, 2,3.. after time t  cos 1   then value of  is
 E0
…………and  ………
                     
Head office  Branch office 
 
fiziks, H.No. 23, G.F, Jia Sarai,  Anand Institute of Mathematics, 
 
Near IIT, Hauz Khas, New Delhi‐16  28‐B/6, Jia Sarai, Near IIT 
 
Phone: 011‐26865455/+91‐9871145498 Hauz Khas, New Delhi‐16 
              
                   Website: www.physicsbyfiziks.com                        
                           Email: fiziks.physics@gmail.com                       119 
fiziks
Institute for NET/JRF, GATE, IIT‐JAM, JEST, TIFR and GRE in PHYSICAL SCIENCES 
 
MSQ (Multiple Select Questions)
Q36. Which of the following statement is /are correct about quantum mechanical system
(a) Solution of Schrödinger wave equation must single valued.
(b) Solution of Schrödinger wave equation must be continuous.
(c) Solution of Schrödinger wave equation must be differentiable.
(d) Solution of Schrödinger wave equation must vanish at x   but x   it may be
infinite
Q37. Which of the following must be correct about quantum mechanical system?
(a) There is Hermitian operator associated with any physical measurable quantities.
(b) Physical measurable quantities are Eigen values of operator associated with physical
measurable quantities which can be real or imaginary.
(c) Eigen values of operator associated with physical measurable quantities must be non
degenerate in nature.
(d) Any state of system can be written in orthonormal basis of eigen state .
Q38. Which one of the following is correct about quantum mechanical operator.
(a) If A( x) is operator  is unnormalised state then average value of A is

A    * A dx


(b) Physical measurable quantities in quantum mechanical system are discrete as well as
continuous.
(c) Energy can be measured by Hamiltonian operator which is always discrete.
(d) Momentum operator has always continuous in eigen value .

                     
Head office  Branch office 
 
fiziks, H.No. 23, G.F, Jia Sarai,  Anand Institute of Mathematics, 
 
Near IIT, Hauz Khas, New Delhi‐16  28‐B/6, Jia Sarai, Near IIT 
 
Phone: 011‐26865455/+91‐9871145498 Hauz Khas, New Delhi‐16 
              
                   Website: www.physicsbyfiziks.com                        
                           Email: fiziks.physics@gmail.com                       120 
fiziks
Institute for NET/JRF, GATE, IIT‐JAM, JEST, TIFR and GRE in PHYSICAL SCIENCES 
 
1 4
Q39. If state of a system at time t  0 is defined by   1  3 if Hamiltonian H is
5 5
defined as H n  nE0n where n  1, 2,3.. then which of following is correct

(a)  is normalized state .

(b) The measurement of H on state  is E0 and 3E0

1
(c) the probability of measurement of E0 is and the probability of measurement of
5
4
3E0 is
5
13
(d) the average value of H is E0
5
  Ax  a  x  0 xa
Q40. If , then which of the following is correct
0 otherwise
(a) The given function is square integrable

30
(b) If  is normalized then value of A 
a5
a
(c) The average value of position on state  is
2
a
(d) Maximum probability to find the particle is at x 
2
Q41. Which of the following/are correct
d
(a) i is Hermitian operator .
dx
d
(b) is Hermitian operator .
dx
(c) For any operator A the operator B  AA† is Hermitian
(d) 1  i  AB  1  i  BA is Hermitian irrespective of A and B are Hermitian

                     
Head office  Branch office 
 
fiziks, H.No. 23, G.F, Jia Sarai,  Anand Institute of Mathematics, 
 
Near IIT, Hauz Khas, New Delhi‐16  28‐B/6, Jia Sarai, Near IIT 
 
Phone: 011‐26865455/+91‐9871145498 Hauz Khas, New Delhi‐16 
              
                   Website: www.physicsbyfiziks.com                        
                           Email: fiziks.physics@gmail.com                       121 
fiziks
Institute for NET/JRF, GATE, IIT‐JAM, JEST, TIFR and GRE in PHYSICAL SCIENCES 
 
Q42. Which of following is correct?
 ( x)  Ae  x / a x  0
(a) can be solution of Schrödinger wave equation
  Ae x / a x0

(b)  ( x)  Ae  x can be solution of Schrödinger wave equation


2

 ( x)  Axe x  0
(c) can be solution of Schrödinger wave equation
0 0

(d)  ( x)  Axe  x can be solution of Schrödinger wave equation


2

                     
Head office  Branch office 
 
fiziks, H.No. 23, G.F, Jia Sarai,  Anand Institute of Mathematics, 
 
Near IIT, Hauz Khas, New Delhi‐16  28‐B/6, Jia Sarai, Near IIT 
 
Phone: 011‐26865455/+91‐9871145498 Hauz Khas, New Delhi‐16 
              
                   Website: www.physicsbyfiziks.com                        
                           Email: fiziks.physics@gmail.com                       122 
fiziks
Institute for NET/JRF, GATE, IIT‐JAM, JEST, TIFR and GRE in PHYSICAL SCIENCES 
 
Solutions
MCQ (Multiple Choice Questions)
Ans. 1: (b)
Ans. 2: (b)
x x
   
 xf  x dx  x.xe  dx  x 2 e  dx
Solution:  it is distribution function so x  

 0
 0
x
 2
f  x  dx
x
  

0

xe dx 

0
xe dx 

Ans. 3: (a)
Solution: For normalized
1
 0  0  1   A2 1 1  3 2 2  3 3  4 4   1
7
1 2
 A  3  1  1  1  A2  7  5  A  2
7
Ans. 4: (c)
Solution: Now  0   1  2 , A n   n  1 a0 n   n  1 a0 is eigen value of A with

eigen state n so measurement of A on state  0 is 2a0 and 3a0

Ans. 5: (c)
Solution: Now  0   2 1  3 2  , A n   n  1 a0 n

Now measurement of A on state  0 is 2a0 and 3a0 with eigen state 1 and 2

respectively
Ans. 6: (b)
Solution: Now  0   2 1  3 2  , A n   n  1 a0 n n  1, 2, 3

Now measurement of A on state  0 is 2a0 and 3a0 with eigen state 1 and 2
respectively
2  0 4
Probability of measurement of 2a0 P (2a0 )   ,
 0  0 13

                     
Head office  Branch office 
 
fiziks, H.No. 23, G.F, Jia Sarai,  Anand Institute of Mathematics, 
 
Near IIT, Hauz Khas, New Delhi‐16  28‐B/6, Jia Sarai, Near IIT 
 
Phone: 011‐26865455/+91‐9871145498 Hauz Khas, New Delhi‐16 
              
                   Website: www.physicsbyfiziks.com                        
                           Email: fiziks.physics@gmail.com                       123 
fiziks
Institute for NET/JRF, GATE, IIT‐JAM, JEST, TIFR and GRE in PHYSICAL SCIENCES 
 
  9
Probability of measurement of 3a0 P (3a0 )  2 0 
 0  0 13
Average value of measurement of A is
0 A 0 4 9 35a0
A    an P ( an )  2a0   3a0  
0 0 n 13 13 13

Ans. 7: (c)
Solution: Now  0   2 1  3 2  , A n  na0 n

Now measurement of A on state  0 is a0 and 2a0 with eigen state 1 and 2


respectively
2  0 4
Probability of measurement of a0 P (a0 )   ,
 0  0 13

2  0 9
Probability of measurement of 2a0 P (2a0 )  
 0  0 13
Average value of measurement of A is
0 A 0 4 9 22a0
A 
0 0
 a P (a )
n
n n  a0 
13
 2a0  
13 13

Ans. 8: (a)
 x
Solution: V  x   b  x  ; b  0 and   x    A cos ; a  x  a
 2a

2 x
Normalized   cos
2a 2a
2 2  2 2
T   *
a
 dx 
a 2m x 2 8ma 2
2 b
V    *  b  x  dx   b   
a

a 2a a
 2 2 b
E  2

8ma a

                     
Head office  Branch office 
 
fiziks, H.No. 23, G.F, Jia Sarai,  Anand Institute of Mathematics, 
 
Near IIT, Hauz Khas, New Delhi‐16  28‐B/6, Jia Sarai, Near IIT 
 
Phone: 011‐26865455/+91‐9871145498 Hauz Khas, New Delhi‐16 
              
                   Website: www.physicsbyfiziks.com                        
                           Email: fiziks.physics@gmail.com                       124 
fiziks
Institute for NET/JRF, GATE, IIT‐JAM, JEST, TIFR and GRE in PHYSICAL SCIENCES 
 
Ans. 9: (d)

1  H E0  2 E1
Solution:   0  i1 and H  
2  3

Ans. 10: (d)

1  H 2 E02  2 E12
Solution:   0  i1 and H    E n2 P ( E n ) 
2  n 3

Ans. 11: (c)

1  
 iE1t  iE2t
Solution:   0 
1
2
 u1  u2     0  
2
u1 e 
 u 2 e 


Ans. 12: (a)

1  
 iE1t  iE2t
Solution:   0 
1
2
u 1  u2     0 
2
u
 1 e 
 u 2 e 


 iE1t  iE 2 t
1 1
  t  is orthogonal to   0     0   t   0  e 
 e 
0
2 2
 iE1t  iE2t  iE1t  iE2t  E2  E1 
i
e  e  0e   e  e 
 1

 cos
 E2  E1  t  cos   t    

 E2  E1 E0
Ans. 13: (c)
   
Solution:   *  x   i    x  dx  p

 x 
Now
 
ikx
   ikx 
 ikx
 ikx  ik ikx 
 e 
 *  x   i   e   x  dx   e 
 *
 x   i    e   x   e   x 

 x  
 x  
 
ikx
  
ikx
 ik  ikx
 e 
 *  x   i   x   e    i. e   *  x   x  dx

 x   
    
   *  x   i   x    k   *  x   x   P  k

 x  

                     
Head office  Branch office 
 
fiziks, H.No. 23, G.F, Jia Sarai,  Anand Institute of Mathematics, 
 
Near IIT, Hauz Khas, New Delhi‐16  28‐B/6, Jia Sarai, Near IIT 
 
Phone: 011‐26865455/+91‐9871145498 Hauz Khas, New Delhi‐16 
              
                   Website: www.physicsbyfiziks.com                        
                           Email: fiziks.physics@gmail.com                       125 
fiziks
Institute for NET/JRF, GATE, IIT‐JAM, JEST, TIFR and GRE in PHYSICAL SCIENCES 
 
Ans. 14: (a)

2 2 2
aa  x  x 2 a  x dx
a
15
Solution: p   2
 p 2   2  2 2 2 2

x 2
16a 5
a
 2 x3 
 
a
  
15
2
 2   a 2
 x 2
dx   2

15
 2  a  x  
16a 5 a 16a 5  3  a

15  3 2a 3  15 3 1  15 2 2 5 2
   2  2a       2  2a 1      2
2 2 2
p
16a 5   3  4a
2
3  16 3 2a

Ans.15: (a)
Solution:   x    0  x    1  x 
 iE0 t  iE1t
  x, t    0  x  e 
 1  x  e 

iE0 t iE1t
  x, t    0  x  e  1  x  e  0  x    0  x  , 1  x    1  x 
* * * * *
 

t
  x, t    *  x, t   x, t    0  x    1  x   2 Re 0*  x  1  x  cos  E1  E0 
2 2 2


Ans. 16: (a)
Solution:   x    0  x    1  x 
 iE0 t  iE1t
  x, t    0  x  e 
 1  x  e 

iE0 t iE1t
  x, t    0  x  e  1  x  e  0  x    0  x  , 1  x    1  x 
* * * * *
 

t
  x, t    *  x, t   x, t    0  x    1  x   2 Re 0*  x  1  x  cos  E1  E0 
2 2 2


putting t  

  x, t    0  x    1  x   2 Re 0*  x  1  x  cos 
2 2 2
 E1  E0    1

  x, t    0  x    1  x   2 Re 0*  x  1  x    1  x   0  x  
2 2 2 2

                     
Head office  Branch office 
 
fiziks, H.No. 23, G.F, Jia Sarai,  Anand Institute of Mathematics, 
 
Near IIT, Hauz Khas, New Delhi‐16  28‐B/6, Jia Sarai, Near IIT 
 
Phone: 011‐26865455/+91‐9871145498 Hauz Khas, New Delhi‐16 
              
                   Website: www.physicsbyfiziks.com                        
                           Email: fiziks.physics@gmail.com                       126 
fiziks
Institute for NET/JRF, GATE, IIT‐JAM, JEST, TIFR and GRE in PHYSICAL SCIENCES 
 
Ans. 17: (b)
d  d  d  d 
Solution:    x   x  f  x     x   f  x   xf  x 
 dx  dx   dx   dx 
d d 
f x   xf x   x f x   x 2 f x 
d
 
dx  dx  dx

d2 df  x  d
 2 f  x  f  x  x x f  x   x2 f  x 
dx dx dx

d2  d2 
 2
f  x   x 2
f  x   f  x    2  x 2  1 f x 
dx  dx 
Ans. 18: (b)
Solution: Classically H   xp , quantum mechanically H must be Hermitian,
 
So, H   xp  px  and H   xp  px 
2 2
  i  x       
 H   x  i       i    x x  
2 x x  2  x x 

i    i     
 H   2x     2 x  1
2  x  2  x 

  1
 H    i   x   
 x 2 

Ans. 19: (b)


Solution:  x 2 , p 2   x  x, p 2    x, p 2  x  xp  x, p   x  x, p  p  p  x, p  x   x, p  px

x 2

, p 2  xpi   xi  p  pi x  i  px  2i xp  px .
Ans. 20: (c)
Solution: A, B   ixp y  iyp x ,  yp z  zp y 

A, B   ixp y , yp z   i yp x , yp z   ixp y , zp y   iyp x , zp y 


A, B   ixp y , yp z   0  0  iyp x , zp y   ixp y , yp z   iyp x , zp y 
A, B   ix p y , yp z   ix, yp z  p y  iy p x , zp y   iy, zp y p x

                     
Head office  Branch office 
 
fiziks, H.No. 23, G.F, Jia Sarai,  Anand Institute of Mathematics, 
 
Near IIT, Hauz Khas, New Delhi‐16  28‐B/6, Jia Sarai, Near IIT 
 
Phone: 011‐26865455/+91‐9871145498 Hauz Khas, New Delhi‐16 
              
                   Website: www.physicsbyfiziks.com                        
                           Email: fiziks.physics@gmail.com                       127 
fiziks
Institute for NET/JRF, GATE, IIT‐JAM, JEST, TIFR and GRE in PHYSICAL SCIENCES 
 
A, B   ix p y , yp z   0  0  iy, zp y p x  ix p y , yp z   iy, zp y p x
A, B  ix   i  p z  izi  p x
A, B  xp z  p x z 
Ans. 21: (d)
Solution: Schrodinger equation   2  V  0 (where   2m  1 and E  0 )


2
x 2
 4

4 
Ae  x  VAe  x  0   e x  4 x3   Ve x  0
x 
4


4

 4

4 3x 2 e  x  x 3  4 x 3 e  x
4
 Ve  x4
 0  12 x 2 e  x  16 x 6 e  x  Ve  x  0
4 4 4

 V  16 x 6  12 x 2
MSQ (Multiple Select Questions)
Ans. 22: (a) and (d)
Solution: The expectation value of measurement of A on state  is
2
 A n 
  an
 n 

Ans. 23: (a) and (d)


Solution: (a) For normalized    1

B2 2
   B2  1  B 
2 3

1 2
Now   0  i1 Measurement are E0 , E1
3 3
2
1  2
Probability getting E1 , P  E1   
 3
2
0  1
Probability getting E0 , P  E0   
  3

1 2 E  2 E1
So E   En P  En    E0  E1  0
n 0 3 3 3
                     
Head office  Branch office 
 
fiziks, H.No. 23, G.F, Jia Sarai,  Anand Institute of Mathematics, 
 
Near IIT, Hauz Khas, New Delhi‐16  28‐B/6, Jia Sarai, Near IIT 
 
Phone: 011‐26865455/+91‐9871145498 Hauz Khas, New Delhi‐16 
              
                   Website: www.physicsbyfiziks.com                        
                           Email: fiziks.physics@gmail.com                       128 
fiziks
Institute for NET/JRF, GATE, IIT‐JAM, JEST, TIFR and GRE in PHYSICAL SCIENCES 
 
Ans. 24: (a), (c) and (d)
1 0
Solution: H   0  
0 4
1 0
E1   0 , E2  4 0 and corresponding eigenvector 1    , 2   
0 1
1
Hence 1    is eigen state with eigen value 1 0 ,so measurement on state 1 is 1 0
0
but  is not eigen state so measurement is both of eigen value E1   0 , E2  4 0 and
2 2
1  4 2  9
Probability P  0    , Probability P  4 0   
 13  13

4 9 40 0
H  0   4 0  
13 13 13
Ans. 25: (c)
4 E1t 3 Et
Solution:  (t  t )  1 exp i  2 exp i 2
5  5 

E1t  4 3 ( E1  E2 )t  h 1 
 (t  t )  exp i  1  2 exp  i  put T   
 5 5   2  E1  E2 

E1t  4 3 
  r , t   exp  i  1  2 
 5 5 

  r, t  
2 16 2 9 4 3 E  E t
1  2  2 Re 1*2  cos 2 1
2

25 25 5 5 

  r, t  
2 16 2 9 4 3 E  E t
1  2  Re 1*2   2 cos 2 1  2 .
2 h
25 25 5 5 h 2  E1  E2 

16 2 9 4 3
  r, t  
2
1  2  2 Re 1*2 .
2

25 25 5 5
2
4 3 
  r , t    1  2 
2

5 5 

                     
Head office  Branch office 
 
fiziks, H.No. 23, G.F, Jia Sarai,  Anand Institute of Mathematics, 
 
Near IIT, Hauz Khas, New Delhi‐16  28‐B/6, Jia Sarai, Near IIT 
 
Phone: 011‐26865455/+91‐9871145498 Hauz Khas, New Delhi‐16 
              
                   Website: www.physicsbyfiziks.com                        
                           Email: fiziks.physics@gmail.com                       129 
fiziks
Institute for NET/JRF, GATE, IIT‐JAM, JEST, TIFR and GRE in PHYSICAL SCIENCES 
 
Ans. 26: (a), (b) and (d)
4 3 4 iE1t 3 iE2t
Solution:   r , t  0   1  2 from postulates five   r , t   1 exp  2 exp
5 5 5  5 
Now   r , t  0    r , t  t   0

16 iE1t 9 iE2t 16 iE1t 9 iE2 t


1 1 exp  2 2 exp 0 exp   exp
25  25  25  25 
i  E2  E1  t 9  E  E1  t 9  9
exp   cos 2  t  cos 1  
 16  16  E2  E1   16 
16 9
Probability of measurement of E1 and E2 are , respectively at t  0 and at t  t
25 25
from postulates 2 and 3.
Ans. 27: (a,b,d)
a a
30
Solution:   dx  1  A
* 2
 x (a  x) dx  1
2 2
 A
0 0
a5
a a

 X dx A  xx (a  x)
* 2 2 2
dx
a
X  0
a
0
a

2
  dx  x (a  x)
* 2 2 2
A dx
0 0

 
a a

0   i x   x(a  x)  i x x(a  x)
* 2
A
px  a
 0
a
0
  dx  x (a  x)
* 2 2 2
A dx
0 0

2  2 2  2
a a

   A2  x ( a  x )  x(a  x)
*

px2 2m x 2 2m x 2 5 2
T   0
a
 0
a

2m ma 2
  dx A2  x 2 (a  x) 2 dx
*

0 0

                     
Head office  Branch office 
 
fiziks, H.No. 23, G.F, Jia Sarai,  Anand Institute of Mathematics, 
 
Near IIT, Hauz Khas, New Delhi‐16  28‐B/6, Jia Sarai, Near IIT 
 
Phone: 011‐26865455/+91‐9871145498 Hauz Khas, New Delhi‐16 
              
                   Website: www.physicsbyfiziks.com                        
                           Email: fiziks.physics@gmail.com                       130 
fiziks
Institute for NET/JRF, GATE, IIT‐JAM, JEST, TIFR and GRE in PHYSICAL SCIENCES 
 
NAT (Numerical Answer Type)
Ans. 28: 2.45
C2 9 C2 6
Solution: For  is normalised    1   1   C  6
15 15 15 15
Ans. 29: 0.196

Solution: For normalized     1

 C 2  1 1    3  4i  3  4i  2 2  1

1
C 2 1  9  16  1  C   .196
26
Ans. 30: 2 and 5
Solution: Measurement of A on any state are eigen value 2a0 ,5a0

Ans. 31: 0.96


2
2  25
Solution: Probability getting P  5a0     0.96
 26

Ans. 32: 0.03


2
2  1
Solution: Probability getting P  a0     .03
 26

Ans. 33: 2
Solution: If A will measure state  and get the eigen value a0 means  is projected in the

direction of 1 if again B will measure on state  it will measure eigen value associate

with 1 so measurement is 2b0 .

Ans. 34: 3
iE1t iE2 t
2 1 2  1 
Solution:   t  0   1  2 ,  (t  t )  1e 
 2 e 
3 3 3 3
 (t  0) and  (t  t ) became orthogonal so their scalar product vanish

 *
(t  0), (t  t  0
                     
Head office  Branch office 
 
fiziks, H.No. 23, G.F, Jia Sarai,  Anand Institute of Mathematics, 
 
Near IIT, Hauz Khas, New Delhi‐16  28‐B/6, Jia Sarai, Near IIT 
 
Phone: 011‐26865455/+91‐9871145498 Hauz Khas, New Delhi‐16 
              
                   Website: www.physicsbyfiziks.com                        
                           Email: fiziks.physics@gmail.com                       131 
fiziks
Institute for NET/JRF, GATE, IIT‐JAM, JEST, TIFR and GRE in PHYSICAL SCIENCES 
 
 iE1t  iE2t
2  1  1
e  e  0 t  cos 1  where E2  4 E0 and E1  E0 and
3 3 E2  E1 2

 1
E2  E1  3E0 comparing with t  cos 1    3
 E0 2
Ans. 35: 2 and 0.25
iE2t iE3t
1 4 1  4 
Solution:   1  3   (t  t )  1e 
 3e 
5 5 5 5
 (t  0) and  (t  t ) became orthogonal so their scalar product vanish

 *
(t  0), (t  t   0
 iE3t
1 iE 2t 4  1
e  e 
0 t cos 1  where E1  E0 and E3  3E0 and
5 5 E2  E1 4


E2  E1  2 E0 comparing with t  cos 1    2   0.25
 E0
MSQ (Multiple Select Questions)
Ans. 36: (a), (b) and (c)
Solution: Solution of Schrödinger wave equation must single valued, continuous, differentiable
and it must vanish at x   but x  
Ans. 37: (a) and (d)
Solution: Eigen values of operator associated with physical measurable quantities which can be
real it may be non degenerate or degenerate.
Ans. 38: (b)
Ans. 39: (a), (b), (c) and (d)
1 4
Solution:   1 so it is normalized, the measurement of H on state  is E0 and 3E0 With
5 5
1 4 1 4 13
the probability and and H  E0   3E0   E0 .
5 5 5 5 5

                     
Head office  Branch office 
 
fiziks, H.No. 23, G.F, Jia Sarai,  Anand Institute of Mathematics, 
 
Near IIT, Hauz Khas, New Delhi‐16  28‐B/6, Jia Sarai, Near IIT 
 
Phone: 011‐26865455/+91‐9871145498 Hauz Khas, New Delhi‐16 
              
                   Website: www.physicsbyfiziks.com                        
                           Email: fiziks.physics@gmail.com                       132 
fiziks
Institute for NET/JRF, GATE, IIT‐JAM, JEST, TIFR and GRE in PHYSICAL SCIENCES 
 
Ans. 40: (a), (b), (c) and (d)
Solution: the function is vanish at x   x   so it is square integrable .
a a
30 a 30 a
  dx  1  A    * x dx  ,   5  x(a  x)  is max at
2 2
*
5
, x 
0
a 0
2 a 2

Ans. 41: (a) and (c)


Solution: Any operator A is Hermitian if A†  A
Ans. 42: (b), (c) and (d)
Solution: (a) is not continuous so it can not be solution of Schrödinger wave equation.

                     
Head office  Branch office 
 
fiziks, H.No. 23, G.F, Jia Sarai,  Anand Institute of Mathematics, 
 
Near IIT, Hauz Khas, New Delhi‐16  28‐B/6, Jia Sarai, Near IIT 
 
Phone: 011‐26865455/+91‐9871145498 Hauz Khas, New Delhi‐16 
              
                   Website: www.physicsbyfiziks.com                        
                           Email: fiziks.physics@gmail.com                       133 
fiziks
Institute for NET/JRF, GATE, IIT‐JAM, JEST, TIFR and GRE in PHYSICAL SCIENCES 
 
4. Application of Quantum Mechanics in Cartesian Coordinate
4.1 One Dimensional System
Properties of one dimensional motion:

continumum V  x
state V2
V1
Bound E
state Vmin
x2 x1
(A) Bound states (quantum mechanical discrete spectrum)
Bound states occur whenever the particle cannot move to infinity and particle is confined
into limited region.
 From the figure the condition for bound states is Vmin  E  V1

 In a one dimensional potential energy level of a bound state system are


discrete and non degenerate.
 The wave function  n of one dimensional bound state system has n nodes i.e.

 n vanishes n times if n corresponds to n  0 the and n  1 nodes if n  1


corresponds to the ground state.

(B) Continuous spectrum (unbound states)


Unbound states occur in those case where the motion of the system is not confined in
above figure.
 V1  E  V2 : The energy spectrum is continuous and none of the energy eigen
values is degenerate.
 E >V2 : The energy spectrum is continuous and particles motion is infinite in
both directions. And this spectrum is doubly degenerate.

                     
Head office  Branch office 
 
fiziks, H.No. 23, G.F, Jia Sarai,  Anand Institute of Mathematics, 
 
Near IIT, Hauz Khas, New Delhi‐16  28‐B/6, Jia Sarai, Near IIT 
 
Phone: 011‐26865455/+91‐9871145498 Hauz Khas, New Delhi‐16 
              
                   Website: www.physicsbyfiziks.com                        
                           Email: fiziks.physics@gmail.com                       134 
fiziks
Institute for NET/JRF, GATE, IIT‐JAM, JEST, TIFR and GRE in PHYSICAL SCIENCES 
 
4.2 Current Density (J):
  *   * 
The probability current density is defined as J x    
2im  x x 

The current density can also be given as.


J  V
k
Where  is probability density. i.e.,   
2
and v is velocity of particle which is in
m
momentum.
k 2 k
So current density is J  (*) 
m m
The current density and probability density will be satisfied the continuity equation which

is given by J  0
t
4.3 Free Particle in One Dimension
The potential of free particle is defined as  V  x   0;   x  

The schrodinger wave function is given by


H  E

 2  2  2  2mE 2mE
 V ( x)  E   2   0 where  k2
2m x 2 x 2
  2

2k 2
the solution is given by   Aeikx  Be ikx and E
2m
2k 2
The energy eigen value of free particle is E  is continuous and wave function is
2m
   Aeikx and    Ae ikx where   moves from positive axis x and   moves from
negative x axis.
2k 2
Hence there is two eigen function for energy E  then wave function is doubly
2m
degenerate.

                     
Head office  Branch office 
 
fiziks, H.No. 23, G.F, Jia Sarai,  Anand Institute of Mathematics, 
 
Near IIT, Hauz Khas, New Delhi‐16  28‐B/6, Jia Sarai, Near IIT 
 
Phone: 011‐26865455/+91‐9871145498 Hauz Khas, New Delhi‐16 
              
                   Website: www.physicsbyfiziks.com                        
                           Email: fiziks.physics@gmail.com                       135 
fiziks
Institute for NET/JRF, GATE, IIT‐JAM, JEST, TIFR and GRE in PHYSICAL SCIENCES 
 
4.4 The Step Potential
If J i is incident current density , J r is reflection current density and J t is transmission

Jr
current density.then Then reflection coefficient is defined s is de R  and
Ji

Jt
transmission coefficient is defined as T 
Ji

The potential step is defined as


0 x  0 V0
V ( x)   x
V0 x  0 0
Case I: E  V
For x  0 . The schrodinger wave is given x, y

 2 d 2 d 2 2mE
 E  2  0
2m dx 2 dx 2 
  Aeik x  Be ik x
1 1
x0

Where Aeik1x is incoming wave. And Be ik1x is reflected wave.


In x  0 the schrodinger wave equation.
 2 d 2 d 2 2m( E  V ) d 2
 V  E  2    0  2  k22  0
2m dx 2
dx  2
dx
2m( E  V )
where k22   II  Ceik x  De ik x
2 2
( x  0)
2
D  0 be are no wave reflection reflected in region II i.e., x  0
 II  Ceik2 x which is transmitted wave i.e.,  t  Ceik2 x

  *  i *i  k1
Ji = 
 i   i   | A |2
2im  x x  m

  *  r *r   k1
Jr = 
 r   r  | B |2
2im  x x  m

                     
Head office  Branch office 
 
fiziks, H.No. 23, G.F, Jia Sarai,  Anand Institute of Mathematics, 
 
Near IIT, Hauz Khas, New Delhi‐16  28‐B/6, Jia Sarai, Near IIT 
 
Phone: 011‐26865455/+91‐9871145498 Hauz Khas, New Delhi‐16 
              
                   Website: www.physicsbyfiziks.com                        
                           Email: fiziks.physics@gmail.com                       136 
fiziks
Institute for NET/JRF, GATE, IIT‐JAM, JEST, TIFR and GRE in PHYSICAL SCIENCES 
 
  *  t  t  k2
*
Jt =  t  t  | C |2
2im  x x  m

J r | B |2 J t K 2 | C |2
R  and T  
J i | A |2 Ji K1 | A |2

Using Boundary condition at x  0 i.e.,


Wave must be continuous and differentiable at boundary. So
 1 ( x  0)   2 ( x  0)  A  B  C
d 1 d 2
  k1 ( A  B)  k2C
dx x 0 dx x 0

Solution of above two equation


 k k   2k1 
B=  1 2  A and C =  A
 k1 + k 2   k1 + k 2 
2
 k k  4k1k2
R=  1 2  and T= where R  T  1
 k1  k2  (k1  k2 ) 2

Case II: E < V0


Schrodinger wave equation for 0  x
H  E

2 d 2 d 2  2mE d 2
 0 ( x )  E (  )     0   k12   0
2m dx 2 dx 2 2 dx 2
Solution of the   Aeikx  Be ikx

If Aeik1x is incoming wave  i  Aeik1x then Be ik1x is reflected wave  r  Be ik1x

Schrodinger wave equation for x  0


 2 d 2  d 2  2m 2m
 V0   E    2 (V0  E )  0  2 (V0  E )  k22
2m dx 2
dx 2
 
d 2
 k22   0    Aek2 x  Be k2 x
dx 2
A  0 wave function must vanish at x  0 then  t  Be  k2 x

                     
Head office  Branch office 
 
fiziks, H.No. 23, G.F, Jia Sarai,  Anand Institute of Mathematics, 
 
Near IIT, Hauz Khas, New Delhi‐16  28‐B/6, Jia Sarai, Near IIT 
 
Phone: 011‐26865455/+91‐9871145498 Hauz Khas, New Delhi‐16 
              
                   Website: www.physicsbyfiziks.com                        
                           Email: fiziks.physics@gmail.com                       137 
fiziks
Institute for NET/JRF, GATE, IIT‐JAM, JEST, TIFR and GRE in PHYSICAL SCIENCES 
 
k k
J i | A |2 1 , J r | B |2 2 and J t  0
m m
2
J B J
R r  and T  t  0
Ji A Ji
Now we boundary condition.at x  0
 1 ( x  0)   2 ( x  0)  (A + B) = C
d 1 d 2 k2
and   ( A  B)  C
dx x 0 dx x 0 ik1
k1  ik2 2k1
B A and C  A so R  1 and R  0
k1  ik2 k1  ik2
V0
 1  A2  B 2  2 AB cos 2 K1 x
2

1
2

o
 2  c e2 K x
2 2
2

where  1  A2  B 2  2 AB cos 2 K1 x
2

 1  c e2 K x
2 2
2

when E  V0 , there is finite probability to find the particle at x  0 even if E  0 but

current density is zero in region x  0 .


Strange part of the problem is that even if transmission coefficient is zero there is finite
probability to find the particle x  0 .

                     
Head office  Branch office 
 
fiziks, H.No. 23, G.F, Jia Sarai,  Anand Institute of Mathematics, 
 
Near IIT, Hauz Khas, New Delhi‐16  28‐B/6, Jia Sarai, Near IIT 
 
Phone: 011‐26865455/+91‐9871145498 Hauz Khas, New Delhi‐16 
              
                   Website: www.physicsbyfiziks.com                        
                           Email: fiziks.physics@gmail.com                       138 
fiziks
Institute for NET/JRF, GATE, IIT‐JAM, JEST, TIFR and GRE in PHYSICAL SCIENCES 
 
4.5 Particle in a One Dimensional Box:  
The potential of one dimensional box is defined as
V ( x)  0 0 xa
 otherwise
0 a
Time independent schrodinger wave equation is given as H  E for region x  0 and
x  0 ,  ( x)  0 because in this region potential is infinity so probability to find the
particle in that region is zero.
Schrodinger wave equation in the region 0  x  a
H  E

  2 d 2 d 2 2mE d 2 2mE
 E      0
2m dx 2 dx 2 2 dx 2 2
d 2
 k 2  0  ( x)  A sin kx  B cos kx
dx 2

Now wave function must be continuous at the boundary


So  (0)   (a)  0  0  A sin 0  B cos 0  B  B  0
 ( x)  A sin kx
 (a)  0  A sin ka  0  ka  n where n  0,1, 2,3,....
But for n  0  ( x)  0 so n  0 is not possible
so n  1, 2,3,.... ka  n

2mE n n 2 2  2
 En  , where n  1, 2,3,....
2 a 2ma 2
En is energy eigen value which is discrete.

n x
 n ( x)  A sin , the value of A can be find with normalization condition which is
a
a 2
  n ( x) dx  1  A 
2
0 a

                     
Head office  Branch office 
 
fiziks, H.No. 23, G.F, Jia Sarai,  Anand Institute of Mathematics, 
 
Near IIT, Hauz Khas, New Delhi‐16  28‐B/6, Jia Sarai, Near IIT 
 
Phone: 011‐26865455/+91‐9871145498 Hauz Khas, New Delhi‐16 
              
                   Website: www.physicsbyfiziks.com                        
                           Email: fiziks.physics@gmail.com                       139 
fiziks
Institute for NET/JRF, GATE, IIT‐JAM, JEST, TIFR and GRE in PHYSICAL SCIENCES 
 
2 n x
So energy eigen function is given by  n ( x)  sin n  1, 2,3,....
a a

n 2 2  2
Energy eigen value is given as En  where n  1, 2,3,....
2ma 2

a
 dx   mn
*
The orthonormal condition is given by n m
0

9 2  2
E3 
2ma 2

4 2  2
E2 
2ma 2

 2 2
E1 
2ma 2
0 a a
2

Any function f(x) can be expressed in the term of n(x).



f ( x)   cn n ( x) where cn   n* ( x) f ( x)dx
n 1

                     
Head office  Branch office 
 
fiziks, H.No. 23, G.F, Jia Sarai,  Anand Institute of Mathematics, 
 
Near IIT, Hauz Khas, New Delhi‐16  28‐B/6, Jia Sarai, Near IIT 
 
Phone: 011‐26865455/+91‐9871145498 Hauz Khas, New Delhi‐16 
              
                   Website: www.physicsbyfiziks.com                        
                           Email: fiziks.physics@gmail.com                       140 
fiziks
Institute for NET/JRF, GATE, IIT‐JAM, JEST, TIFR and GRE in PHYSICAL SCIENCES 
 
4.6 Infinite Symmetric Potential Box
a a  
V  x   x   and x
2 2 V  x
a a
0  x
2 2

a a
0 
2 2
Wave function in region x < a/2 and x > a/2 is (x) = 0 because wave function is infinite
a a
in region  x  the solution of Schrödinger wave function is given by.
2 2
  A sin kx  B cos kx Parity operator will commute with Hamiltonian because so wave
function can have either even or odd symmetry .
Now wave function must vanish at boundary.
Case I Solution for even symmetry Case II Solution for odd symmetry
 ( x)   ( x)  ( x)   ( x)
 A sin kx  B cos kx  A sin kx  B cos kx  A sin kx  B cos kx   A sin kx  B cos kx
means A  0   ( x)  B cos kx means B  0   ( x)  A sin kx
Using boundary i.e. Using boundary i.e.
ka ka  3 5 ka ka
cos 0  , , ,... sin 0  , 2,3.....
2 2 2 2 2 2 2
n n
kn  where n  1,3,5.... kn  where n  2, 4, 6...
a a
So  n ( x)  Bn cos kn x So  n ( x)  An sin kn x

2 n x 2 n x
normalized wavefunction  n ( x)  cos normalized wavefunction  n ( x)  sin
a a a a
n n
kn   n  1,3,5,... kn  where n  2, 4, 6...
a a

                     
Head office  Branch office 
 
fiziks, H.No. 23, G.F, Jia Sarai,  Anand Institute of Mathematics, 
 
Near IIT, Hauz Khas, New Delhi‐16  28‐B/6, Jia Sarai, Near IIT 
 
Phone: 011‐26865455/+91‐9871145498 Hauz Khas, New Delhi‐16 
              
                   Website: www.physicsbyfiziks.com                        
                           Email: fiziks.physics@gmail.com                       141 
fiziks
Institute for NET/JRF, GATE, IIT‐JAM, JEST, TIFR and GRE in PHYSICAL SCIENCES 
 
Solution for infinite symmetric box as mentioned above is given by
 2 n x
 cos n  1,3,5,....
n   a a
 2 n x
sin n  2, 4, 6,....
 a a

n 2 2  2
and energy eigen value En  n  1, 2,3, 4,5, 6...
2ma 2

  x

9 2  2 2 3 x
E3   3  x  cos
2ma 2 a a

4 2  2 2 2 x
E2   2  x  sin
2 ma 2 a a

 22 x
E1  2
2 ma 2 1  x  cos
a a a a
x

First2three eigenstate are shown in the2above figure.

                     
Head office  Branch office 
 
fiziks, H.No. 23, G.F, Jia Sarai,  Anand Institute of Mathematics, 
 
Near IIT, Hauz Khas, New Delhi‐16  28‐B/6, Jia Sarai, Near IIT 
 
Phone: 011‐26865455/+91‐9871145498 Hauz Khas, New Delhi‐16 
              
                   Website: www.physicsbyfiziks.com                        
                           Email: fiziks.physics@gmail.com                       142 
fiziks
Institute for NET/JRF, GATE, IIT‐JAM, JEST, TIFR and GRE in PHYSICAL SCIENCES 
 
4.7 Square Well Finite Potential Box (graphical method)
Square well finite potential box is defined as.
a a
V ( x)  0  x   
2 2
V0 V0 V0
 V0 otherwise

For Bound state E  0 a /2 a /2


a
Schrodinger wave solution in region I, i.e., x  
2
1 ( x)  Ae x  Be  x x    wave function must be zero
so B = 0
a
where 1 ( x)  Ae x x
2
a a
Schrondinger wave solution in region II, i.e.,   x  is
2 2
I ( x)  B cos kx .

II ( x)  C sin kx for odd parity

(potential is symmetric about x  0 so parity must be conserve).


2mE
where k 2 
2
a
Schrodinger wave solution in region III i.e., x 
2
  De x  Ee x
The wave function must vanish at x  so E = 0
2m(V0  E )
III ( x)  De x where  2 
2

                     
Head office  Branch office 
 
fiziks, H.No. 23, G.F, Jia Sarai,  Anand Institute of Mathematics, 
 
Near IIT, Hauz Khas, New Delhi‐16  28‐B/6, Jia Sarai, Near IIT 
 
Phone: 011‐26865455/+91‐9871145498 Hauz Khas, New Delhi‐16 
              
                   Website: www.physicsbyfiziks.com                        
                           Email: fiziks.physics@gmail.com                       143 
fiziks
Institute for NET/JRF, GATE, IIT‐JAM, JEST, TIFR and GRE in PHYSICAL SCIENCES 
 
Solution for even parity Solution for odd parity
a a
 ( x)   ( x) | x |   ( x)   ( x) | x | 
2 2
a a
I ( x)  Ae x for x   I  Ae x x
2 2
a a a a
II ( x)  B cos kx for   x  II  C sin kx   x 
2 2 2 2
a a
III ( x)  De x for x  III  De  x x
2 2
The wave function must be continuous and The wave function must be continuous and
differentiable at boundary. So differentiable at boundary. So
 a  a  a  a
I  x     II  x    I  x     II  x   
 2 2    2  2  
a a
 ka  ka
Ae 2
 B cos …(X) Ae 2
 C sin …(X)
2 2
a a
 ka  ka
Ae 2
 kB sin …(Y)  Ae 2
 C k cos ….(Y)
2 2
Dividing on Y by X one can get. a ka ka
  cot
ka 2 2 2
  k tan
2  
   cot     tan    
a ka ka  2
 tan
2 2 2 ka a mV0 a 2
and  , ,   
2 2

ka a mV0 a 2 2 2 2 2
 , , 2  2 
2 2 2 2 The even bound state energy can be found
The even bound state energy can be found by mV0 a 2
by solution of equation 2  2  and
mV0 a 2 2 2
solution of equation 2  2  and
2 2    cot  can be found graphically.
   tan  can be found graphically.

                     
Head office  Branch office 
 
fiziks, H.No. 23, G.F, Jia Sarai,  Anand Institute of Mathematics, 
 
Near IIT, Hauz Khas, New Delhi‐16  28‐B/6, Jia Sarai, Near IIT 
 
Phone: 011‐26865455/+91‐9871145498 Hauz Khas, New Delhi‐16 
              
                   Website: www.physicsbyfiziks.com                        
                           Email: fiziks.physics@gmail.com                       144 
fiziks
Institute for NET/JRF, GATE, IIT‐JAM, JEST, TIFR and GRE in PHYSICAL SCIENCES 
 
First even bound state can be found if First odd bound state can be found if

 mV0 a 2 
1/ 2
 mV0 a 2 3
   
2  2 2 2 2
 2 
Second odd bound state can be found.
Second even bound state can be found
3 mV0 a 2 5
  ...
1/ 2
 mV0 a  2
 2 
 2 2 2 2 2
 2 

n0
n 1    tan 
odd bound state
(first excited bound state) E2 n2 2  2 
mV0 a 2
2 2
   cot 
n3
0   3 2 5 3
even bound state x 2 2 2
 ground state  a / 2 a/2 E1

In the table below shown the number of bound states for various range of V0 a 2 .
where R denotes the radius.
R V0 a 2 Even function Odd function No. of bound state
  2 2
 1 0 1
2 2m
 
2 2
4 2  2
R to 1 1 2
2 2m 2m
3 4 
2 2
9 2  2
 R to 2 1 3
2 2m 2m
3 9 
2 2
162  2
 R  2 to 2 2 4
2 2m 2m

                     
Head office  Branch office 
 
fiziks, H.No. 23, G.F, Jia Sarai,  Anand Institute of Mathematics, 
 
Near IIT, Hauz Khas, New Delhi‐16  28‐B/6, Jia Sarai, Near IIT 
 
Phone: 011‐26865455/+91‐9871145498 Hauz Khas, New Delhi‐16 
              
                   Website: www.physicsbyfiziks.com                        
                           Email: fiziks.physics@gmail.com                       145 
fiziks
Institute for NET/JRF, GATE, IIT‐JAM, JEST, TIFR and GRE in PHYSICAL SCIENCES 
 
4.8 Harmonic Oscillator (Parabolic potential)

V  x
The parabolic potential is defined as
1
V  x  m 2 X 2
2
P2 1
H  m2 X 2
2m 2 x
0
The Schrodinger wave function is given.
 2 d 2 1
 m 2 X 2  E
2m dx 2
2

m 2E
put   x, k 
 
d 2
 ( 2  k ) …(i)
d 2

2

For large k . Solution is  ( )  Ae 2

2

The general solution is given by  ()  H n ()e 2

d 2Hn dH n
So equation (i) reduce to  2  (k  1) H  0
dx 2
d

Solving series solution by putting H n ()   an n
n 0

(2n  1  k )
One can find an  2  an
(n  1)(n  2)
When k = 2n + 1 the equation (i) reduce to Hermite polynomial and
 1
En   n    where n = 0, 1, 2, …
 2

d 2Hn dH n
And  2  2nH n  0 …(ii)
d 2
d

Hermite polynomials and it is given as H 0  1, H1  2 and H 2  4 2  2

                     
Head office  Branch office 
 
fiziks, H.No. 23, G.F, Jia Sarai,  Anand Institute of Mathematics, 
 
Near IIT, Hauz Khas, New Delhi‐16  28‐B/6, Jia Sarai, Near IIT 
 
Phone: 011‐26865455/+91‐9871145498 Hauz Khas, New Delhi‐16 
              
                   Website: www.physicsbyfiziks.com                        
                           Email: fiziks.physics@gmail.com                       146 
fiziks
Institute for NET/JRF, GATE, IIT‐JAM, JEST, TIFR and GRE in PHYSICAL SCIENCES 
 
2
 m 
1/ 4
1 
Energy eigen function is  n ( )    H n ( )e 2

   2 n! n

 1
And eigen value is En   n    n = 0, 1, 2, …
 2

Ground state n = 0  E  is zero point energy
2

5
E2   2
2
3
E1   1
2

1
E0   0
2

The first three stationary state and corresponding eigen value for the harmonic oscillator.
2
 m  
1/ 4

The eigenfunction  0 ()    e 2
have energy E0 
   2
 2
 m  3
1/ 4
1 
The eigenfunction 1 ()    2e 2 have energy E1 
   2 2
2
 m 
1/ 4

 4  2 e
1 
The eigenfunction  2 ()   
2 2

   22 2!
 2
 m  5
1/ 4

 4 2  2  e 2
1 
  have energy E2 
   8 2

                     
Head office  Branch office 
 
fiziks, H.No. 23, G.F, Jia Sarai,  Anand Institute of Mathematics, 
 
Near IIT, Hauz Khas, New Delhi‐16  28‐B/6, Jia Sarai, Near IIT 
 
Phone: 011‐26865455/+91‐9871145498 Hauz Khas, New Delhi‐16 
              
                   Website: www.physicsbyfiziks.com                        
                           Email: fiziks.physics@gmail.com                       147 
fiziks
Institute for NET/JRF, GATE, IIT‐JAM, JEST, TIFR and GRE in PHYSICAL SCIENCES 
 
Problems on One Dimensional System:
Example: A particle of mass m is confined into a box of width a where potential is defined as .
V(x)=0 0<x<L
=  otherwise
Find
(a) X (b) X 2 (c)  Px  (d)  Px2  (e) x  Px

Solution: A normalized wave function in above potential is given by.

2 nx
n ( x)  sin
a L
 L
2 nx L
(a)  x    xn dx  
*
n k sin 2 dx 
 0
L L 2
 L
2 2 2 nx L2 L2
 x2    n ( x ) x n ( x)  L 0
x dx  
* 2
(b) sin

L 3 2n 2  2
 L
   2 n nx nx
(c)  Px    n ( x)  i x  n ( x)dx   L (i) L 0 sin L cos L dx  0

 2 2  n 2 2  2
(d)  P     ( x )  
x
2 *
n  n ( x)dx 
  x 2  L2

1 1
(e) x   x 2    x 2 x  L  2 2
12 2n 
n
Px    Px2    Px  2 
1/ 2
Px 
L
1 1
x  Px  n  2 2
12 2n 

                     
Head office  Branch office 
 
fiziks, H.No. 23, G.F, Jia Sarai,  Anand Institute of Mathematics, 
 
Near IIT, Hauz Khas, New Delhi‐16  28‐B/6, Jia Sarai, Near IIT 
 
Phone: 011‐26865455/+91‐9871145498 Hauz Khas, New Delhi‐16 
              
                   Website: www.physicsbyfiziks.com                        
                           Email: fiziks.physics@gmail.com                       148 
fiziks
Institute for NET/JRF, GATE, IIT‐JAM, JEST, TIFR and GRE in PHYSICAL SCIENCES 
 
Example: A particle of mass m which moves freely inside an infinite potential well of length a
has the following initial wave function.

A x  3  3 x  1  5 x 
 ( x, 0)  sin   sin   sin  
a  a  5a  a  5a  a 
(a) Find A so that  ( x, 0) is normalized.
(b) If measurement of energy carried out what are the values that will be found and what
are the corresponding probability.
(c) Calculate the average energy.
Solution: Particle of mass m confine into a box of width a.

2 n x
So n ( x) or | n   sin
a a

A 3 1
(a) |   | 1   | 3   | 5 
2 10 10

A2 3 1 6
 |   1   1 A
2 10 10 5

6 3 1
(b) |   | 1   | 3   | 5 
10 10 10
If energy will be manual on | state the measurement of | yields either
2  2 92  2 252  2
, , which is eigen function.
2ma 2 2ma 2 2ma 2
Associate with |1 |3 |5 respectively.
2  2 | 1 |   |2 3
Probability to measures is 
2ma 2  |   5

9 2  2 | 3 |   |2 3
Probability to measures  
2ma 2  |   10

25 2  2 | 5 |   |2 1
Probability to measures  
2ma 2  |   10

                     
Head office  Branch office 
 
fiziks, H.No. 23, G.F, Jia Sarai,  Anand Institute of Mathematics, 
 
Near IIT, Hauz Khas, New Delhi‐16  28‐B/6, Jia Sarai, Near IIT 
 
Phone: 011‐26865455/+91‐9871145498 Hauz Khas, New Delhi‐16 
              
                   Website: www.physicsbyfiziks.com                        
                           Email: fiziks.physics@gmail.com                       149 
fiziks
Institute for NET/JRF, GATE, IIT‐JAM, JEST, TIFR and GRE in PHYSICAL SCIENCES 
 
(c) Average energy is given by  E   Ei Pi

3 2  2 3 92  2 1 252  2 292  2


     
5 2ma 2 10 2ma 2 10 2ma 2 10ma 2
Example: Prove that for any normalized wave function of particle in of mass m in one

 
  
*  
dimensional  J ( x)dx     dx     *dx

2im   x 
x

  
  
 *  
Solution: 
2im 
J ( x ) dx   
2im   x
 dx  

 *  dx 
x 

  1  ( x) 

    1  *
    * ( x)  i  ( x)     i  
2im  i   x   i  x 

 2 Px   Px 

 J ( x)dx  2im i

m

 A
Example: Three dimensional wave function is given by  (r )    eikr
r
Find the current density.
 A 
Solution:  (r )    eikr
r
J
2im
 *  * 
 1  ˆ 1  ˆ
where   rˆ   
r r  r sin  

| A |2 k
J rˆ
r m
V  x   V0 0 xa
Example: A potential barrier is given as
V  x  0 otherwise

Prove that the expression of transmission probability for E < V0 is given as.
16 2mE 2m(V0  E )
T e 2 k2 L where k12  and k22 
k 
2
 2
2
4 2 
 k1 

                     
Head office  Branch office 
 
fiziks, H.No. 23, G.F, Jia Sarai,  Anand Institute of Mathematics, 
 
Near IIT, Hauz Khas, New Delhi‐16  28‐B/6, Jia Sarai, Near IIT 
 
Phone: 011‐26865455/+91‐9871145498 Hauz Khas, New Delhi‐16 
              
                   Website: www.physicsbyfiziks.com                        
                           Email: fiziks.physics@gmail.com                       150 
fiziks
Institute for NET/JRF, GATE, IIT‐JAM, JEST, TIFR and GRE in PHYSICAL SCIENCES 
 
Solution: The potential is given as V(x) = V0 0<x<a
= 0 otherwise   
V0
Case E < V0
The Schrödinger wave solution in I, II and III is given by o a

For region I x < 0;  I ( x)  Aeik1x  Be  ik1x

For region II 0 < x < a;  II ( x)  Ce  ik2 x  De k2 x

For region III x > a;  III ( x)  Feik1x  Ge  ik1x

d  I d  II
Boundary condition  I   II and  at x = 0 and at x = a
dx dx
d  II d  III
 II   III ;  at x = a
dx dx
A + B = C + D, ik1 A  ik1 B  k2C  k2 D

and Ce  k2a  Dek2 a  Feik1a ,  k2Ce  k2 a  k2 De k2 a  ik1 Feik1a


The transmission probability is given by

J T | F |2
 where  i  Aeik1x and  t  Feik1x
Ji | A |2

Solving the above four boundary condition

 A   1 i  k2 k1   (ik1  k2 ) a  1 i  k2 k1    (ik1  k2 ) a
        e       e
 F   2 4  k1 k2    2 4  k1 k2  
k2 k1 k2
For approximation V0 > E,   e k2 a  e  k2 a
k1 k2 k1

 A   1 ik2  (ik1  k2 ) a  A   1 ik2  (  ik1  k2 ) a AA*  1 k22  2 k2 a


*

      e        e     e
 F   2 4k1   F   2 4k1  FF *  4 16k12 

 
 
FF *  16  2 k2 a
T   2 
e  T  e 2 k2 a
AA *
 4   k2  
   
  k1  

                     
Head office  Branch office 
 
fiziks, H.No. 23, G.F, Jia Sarai,  Anand Institute of Mathematics, 
 
Near IIT, Hauz Khas, New Delhi‐16  28‐B/6, Jia Sarai, Near IIT 
 
Phone: 011‐26865455/+91‐9871145498 Hauz Khas, New Delhi‐16 
              
                   Website: www.physicsbyfiziks.com                        
                           Email: fiziks.physics@gmail.com                       151 
fiziks
Institute for NET/JRF, GATE, IIT‐JAM, JEST, TIFR and GRE in PHYSICAL SCIENCES 
 
Example: Consider a particle of mass m and charge q place in uniform field E  E0iˆ . Apart from

1
this a restoring force corresponding to the potential V1 ( x)  m2 X 2 acts. Find the lowest
2
energy eigen value. Consider the electric field is originated at origin.
Solution: The electric potential energy at the position x will be qEx . So the effective potential
is given by
2
1 1  qE  q2 E 2
V ( x)  m2 X 2  qEx  m2  X   
2 2  m2  2m2
So Hamiltonion is given by
2
p2 1  qE  q2 E 2
H  m2  X   
2m 2  m2  2m2

qE P2 1 q2 E 2
Put X   X H  
 m X 
2 2

m2 2m 2 2m2
So Energy is given by

 1 q2 E 2
E   n    
 2 2m2

Example: A particle of energy 9ev are sent towards a potential step 8eV high.
(a) What is reflection coefficient.
(b) What percentage will transmitted.
2
 E  E  V0   9  9  8   3  1 2 1
2

Solution: R           1 T  R
 E  E V     3  1  4
 0   9 9 8 
1 3 3
 1  %T  100  75%
4 4 4
Example: A particle in the infinite square well has the initial wave function.
 ( x, 0)  Ax(a  x) (0  x  a)
(a) Find the value of A such that  is normalized.

                     
Head office  Branch office 
 
fiziks, H.No. 23, G.F, Jia Sarai,  Anand Institute of Mathematics, 
 
Near IIT, Hauz Khas, New Delhi‐16  28‐B/6, Jia Sarai, Near IIT 
 
Phone: 011‐26865455/+91‐9871145498 Hauz Khas, New Delhi‐16 
              
                   Website: www.physicsbyfiziks.com                        
                           Email: fiziks.physics@gmail.com                       152 
fiziks
Institute for NET/JRF, GATE, IIT‐JAM, JEST, TIFR and GRE in PHYSICAL SCIENCES 
 
(b) Write down (x) in the basis of n ( x) where n ( x) is the eigen function of the nth

state (wave function) for the system confine into box whose potential is given as
V  x  0 0 xa
 otherwise
(c) Write down expression of  ( x, t )
Solution: (a)  ( x, 0)  Ax(a  x) 0  x  a

 | ( x, 0) | dx  1
2
For normalization


a
30
 A2  x 2 (a  x) 2 dx  1  A 
0
a5

30
(b)  n ( x)  cn n ( x) Where  n ( x)  x(a  x)
a5
And can be found with Fourier’s trick cn   *n ( x)dx
a
2  nx  30
So cn 
a0 sin   5 x(a  x)dx
 a  a

0 if n is even
 8 15 2 nx
cn   15   n ( x)  
n 1,3,... ( n) a
sin
a
8 (n)3 if n is odd
3


From Schrodinger wave equation
 n
H n ( x )  i n  En n ( x)  i  n ( x, t )  n ( x, 0)e  Ent / 
t t
n 2 2  2
Where En 
2ma 2

 nx   in2 2 t / 2 ma2


3
30  2  1
So  ( x, t )   
a 

1,3,5,... n
3
sin 
 a 
e

                     
Head office  Branch office 
 
fiziks, H.No. 23, G.F, Jia Sarai,  Anand Institute of Mathematics, 
 
Near IIT, Hauz Khas, New Delhi‐16  28‐B/6, Jia Sarai, Near IIT 
 
Phone: 011‐26865455/+91‐9871145498 Hauz Khas, New Delhi‐16 
              
                   Website: www.physicsbyfiziks.com                        
                           Email: fiziks.physics@gmail.com                       153 
fiziks
Institute for NET/JRF, GATE, IIT‐JAM, JEST, TIFR and GRE in PHYSICAL SCIENCES 
 
Example: n represent the energy eigen state of a linear harmonic oscillator and If

1
state |   (| 0  | 1  ) of harmonic state of angular 
2
(a) If energy is measured what will measurement with what probability
(b) Find average value of energy.
 3 1 1
Solution: (a) if energy is measured the measurement is and with probability and
2 2 2 2
1  1 3
E      
2 2 2 2
4.9 Multiple Dimensional Systems
If x, y, z are independent then   x, y, z  can be written as X  x  Y  y  Z  z  and Energy

Eigen value can be written as Ex , y , z  Ex  E y  Ez

Where H x X  x   E x X  x  , H y Y  y   E y Y  y  , H z Z z   E z Z z 

and H  H x  H y  H z and H  E ( )

4.10 Two Dimensional Free Particle


1 i kx x  k y y 
  x, y  
1 1
   x, y  
ik y y
e ik x x  e e
2 2 2

E  Ex  E y 
2m

2 2
k x  k y2  
2 2
2m
k , k  k x2  k y2 = constant

So two dimensional free particle is infinitely degenerate.


4.11Three Dimensional Free Particle
The wave function is   x, y, z  is defined as

  x, y , z  
1
e

i kx xk y ykz z 
E  Ex  E y  Ez  
2 2
k x  k y2  k z2 
2 3 / 2 2m

2 2
E k k 2  k x2  k y2  k z2
2m
so three dimensional free particle is infinitely degenerate.

                     
Head office  Branch office 
 
fiziks, H.No. 23, G.F, Jia Sarai,  Anand Institute of Mathematics, 
 
Near IIT, Hauz Khas, New Delhi‐16  28‐B/6, Jia Sarai, Near IIT 
 
Phone: 011‐26865455/+91‐9871145498 Hauz Khas, New Delhi‐16 
              
                   Website: www.physicsbyfiziks.com                        
                           Email: fiziks.physics@gmail.com                       154 
fiziks
Institute for NET/JRF, GATE, IIT‐JAM, JEST, TIFR and GRE in PHYSICAL SCIENCES 
 
4.12 Particle in Two Dimensional Box
The two dimensional system is defined as
0 0  x  a, 0 ya
V  x, y   
 otherwise

2 n x 2 n y y
The wave function is given as   x, y   sin x sin
a a a a

2
2  x
E n 2  n 2y  where nx  1, 2,3..... n y  1, 2,3,......
2ma
2 2 2
Ground state energy nx  1, n y  1 E 
2ma 2
5 2 2
First excited state nx  1, n y  2 E 
2ma 2
(Degeneracy of first excited state is doubly degenerate)
8 2 2
Second excited state nx  2, n y  2 , E 
2ma 2
10 2 2
Third excited state nx  1, n y  3 , E 
2ma 2
nx  3, n y  3

(Third excited state is doubly generate)


4.13 Particle in Three Dimensional Box
0 0  x  Lx , 0  y  Ly , 0  z  Lz
V  x, y , z   
 otherwise

2 n x 2 n y 2 n z
  x, y , z   sin x sin y sin z
Lx Lx Ly Ly Lz Lz

 2 2  nx2 n y nz2 
2

Ex , y , z      where nx  1, 2,3,...... n y  1, 2,3,...... , nz  1, 2,3,......


2m  L2x L2y L2z 

                     
Head office  Branch office 
 
fiziks, H.No. 23, G.F, Jia Sarai,  Anand Institute of Mathematics, 
 
Near IIT, Hauz Khas, New Delhi‐16  28‐B/6, Jia Sarai, Near IIT 
 
Phone: 011‐26865455/+91‐9871145498 Hauz Khas, New Delhi‐16 
              
                   Website: www.physicsbyfiziks.com                        
                           Email: fiziks.physics@gmail.com                       155 
fiziks
Institute for NET/JRF, GATE, IIT‐JAM, JEST, TIFR and GRE in PHYSICAL SCIENCES 
 
 2 2  nx2 n y nz2 
2

For cubic box Lx  Ly  Lz  L Ex , y , z     


2m  L2x L2y L2z 

where nx  1, 2,3,...., n y  1, 2,3,... nz  1, 2,3,....

3 2  2
Ground state nx  1, n y  1, nz  1 E Non degenerate
2mL2
6 2  2
First excited state nx  1, n y  1, nz  2 E Triple degenerate
2mL2
nx  1, n y  2, nz  1 , nx  2, n y  1, nz  1

9 2  2
Second excited state nx  2, n y  2, nz  1 E Triple degenerate
2mL2
nx  2, n y  1, nz  2 , nx  1, n y  2, nz  2

4.14 Two Dimensional Harmonic Oscillator

V  x, y  
1 1
m 2 x 2  m 2 y 2   nx ,ny  x, y   X nx  x  Yny  y 
2 2
 1  1
Enx ,ny  Enx  Eny   n x      n y   
 2  2

  n x  n y  1  n x  0,1, 2,3,.....
n y  0,1, 2,3,.....

  n  1  n  1, 2,3,........

Ground state energy nx  0, ny  0 E  

First excited energy nx  1, n y  0 E  2

nx  0, ny  1

Second excited energy nx  2, n y  0 E  3

nx  0, ny  2

nx  1, n y  1

So degeneracy of two dimensional harmonic oscillators for nth state is  n  1


                     
Head office  Branch office 
 
fiziks, H.No. 23, G.F, Jia Sarai,  Anand Institute of Mathematics, 
 
Near IIT, Hauz Khas, New Delhi‐16  28‐B/6, Jia Sarai, Near IIT 
 
Phone: 011‐26865455/+91‐9871145498 Hauz Khas, New Delhi‐16 
              
                   Website: www.physicsbyfiziks.com                        
                           Email: fiziks.physics@gmail.com                       156 
fiziks
Institute for NET/JRF, GATE, IIT‐JAM, JEST, TIFR and GRE in PHYSICAL SCIENCES 
 
Three Dimensional Harmonic Oscillators
1 1 1
V  x, y , z   m x2 x 2  m y2 y 2  m z2 z 2
2 2 2
 n ,n
x y , nz
 x, y, z   X n  x  Yn  y  Z n  z 
x y z

 1  1  1
E   n x      n y       n z     nx  n y  nz  0,1, 2,.........
 2  2  2
For isotropic harmonic oscillator is x   y  z

 3  3
Enx ,ny , nz   nx  ny  nz      n   
 2  2
1
The degeneracy of the isotropic harmonic oscillator is for nth state is g n   n  1 n  2 
2
where n = 0 corresponding ground state.
Example: If b  2a , write down ground, first, second and third excited state energy.

 2   nx 2 ny 2   2  2  nx 2
ny 2 
Solution: Enx ,ny      Enx ,ny   2    b  2a
2m  a 2 b 2  2m  a  2a  
2

 2  2  nx 2 ny 2 
Enx ,ny    
2m  a 2 4a 2 

5  22
For ground state: E1,1 
8 ma 2
8  22
For first excited state: E1,2 
8 ma 2
13  2  2
For second excited stste E1,3 
8 ma 2
17 2  2
For third excited stste E2,1 
8ma 2
20 2  2
For fourth excited stste E1,4  E2,2 
8ma 2

                     
Head office  Branch office 
 
fiziks, H.No. 23, G.F, Jia Sarai,  Anand Institute of Mathematics, 
 
Near IIT, Hauz Khas, New Delhi‐16  28‐B/6, Jia Sarai, Near IIT 
 
Phone: 011‐26865455/+91‐9871145498 Hauz Khas, New Delhi‐16 
              
                   Website: www.physicsbyfiziks.com                        
                           Email: fiziks.physics@gmail.com                       157 
fiziks
Institute for NET/JRF, GATE, IIT‐JAM, JEST, TIFR and GRE in PHYSICAL SCIENCES 
 
Example: V  x, y , z   0 0  x  a, 0  y  b, 0  z  c

 Otherwise
If b  2a and c  3a , then write down energy eigenvalue for ground state, first excited
state and second excited state.

 22  nz2 
n y2
Solution: E  nx , n y , nz    n 2
 
ma 2  4 9 
x

49  2  2
For ground state: nx  1, n y  1, nz  1  E1,1  
36 2ma 2
61  2  2
For first excited state: nx  1, ny  1, nz  2  E1,1,2  
36 2ma 2
76  2  2
For second excited state: nx  1, ny  2, nz  1  E1,2,1  
36 2ma 2
81  2  2
For third excited state: nx  1, n y  1, nz  3  E1,1,3  
36 2ma 2
Example: If the potential of two dimensional harmonic oscillator is
1 1
V  x, y   m 2 x 2  m4 2 y 2 , then find energy Eigen value.
2 2
1 1
Solution: V  x. y   m 2 x 2  m  2  y 2 i.e. 1   , 2  2
2

2 2
 1  1
 Enx ,ny   nx      n y   2
 2  2
3 5
For ground state: E0,0   , For first excited state: E1,0  
2 2
7
For second excited state: E0,1  E2,0   (Doubly degenerate)
2
9
For third excited state: E3,0  
2
11
For fourth excited state: E2,1  E4,0   (Doubly degenerate)
2

                     
Head office  Branch office 
 
fiziks, H.No. 23, G.F, Jia Sarai,  Anand Institute of Mathematics, 
 
Near IIT, Hauz Khas, New Delhi‐16  28‐B/6, Jia Sarai, Near IIT 
 
Phone: 011‐26865455/+91‐9871145498 Hauz Khas, New Delhi‐16 
              
                   Website: www.physicsbyfiziks.com                        
                           Email: fiziks.physics@gmail.com                       158 
fiziks
Institute for NET/JRF, GATE, IIT‐JAM, JEST, TIFR and GRE in PHYSICAL SCIENCES 
 
Example: If the potential of three dimensional harmonic oscillator is
1 1 1
V  x, y , z   m 2 x 2  4m 2 y 2  9m 2 z 2 ,
2 2 2
then write down unnormalised wavefunction for ground state, first excited state and
second excited state
m x 2 m y 2 m z 2
  
Solution: Wavefunction   n , n 
 H nx  x  e 2
 H ny  y  e 2
 H nz  z  e 2
x y , nz

For ground state:  nx , ny , nz    0, 0, 0  , then wavefunction

m x 2 2 m y 2 3 m z 2
  
  0,0,0  e 2
.e 2
.e 2

For first excited state:  nx , n y , nz   1, 0, 0  , then wavefunction

m x 2 m y 2 3 m z 2
  
 1,0,0  x e 2
.e 
.e 2

For second excited state:  nx , n y , nz   1, 0,1 ,  2, 0, 0  , then wavefunction

m x 2 m y 2 3 m z 2
  
 1,0,1  xz e 2
.e 
.e 2

m x 2 m y 2 3 m z 2
  2,0,0  4 x  2  e
  
2 2
.e 
.e 2

                     
Head office  Branch office 
 
fiziks, H.No. 23, G.F, Jia Sarai,  Anand Institute of Mathematics, 
 
Near IIT, Hauz Khas, New Delhi‐16  28‐B/6, Jia Sarai, Near IIT 
 
Phone: 011‐26865455/+91‐9871145498 Hauz Khas, New Delhi‐16 
              
                   Website: www.physicsbyfiziks.com                        
                           Email: fiziks.physics@gmail.com                       159 
fiziks
Institute for NET/JRF, GATE, IIT‐JAM, JEST, TIFR and GRE in PHYSICAL SCIENCES 
 
MCQ (Multiple Choice Questions)
Q1. An electron with energy E is incident from left on a potential
V x 
barrier, given by
V ( x)  0 x0 V0

V ( x)  V0 x0 E
   
as shown in the figure. For E  V0 the space part of the x
0
wavefunction for x  0 is of the form
(a) exp  x (b) exp   x (c) exp i x (d) exp i x

Q2. The wavefunction of particle moving in free space is given by,   e ikx  2e ikx The
energy of the particle is
5 2 k 2 3 2 k 2 2k 2 2k 2
(a) (b) (c) (d)
2m 4m 2m m
Q3. A particle is confined in a one dimensional box with impenetrable walls at x   a . Its
energy eigenvalue is 2 eV and the corresponding eigenfunction is as shown below.

a 0 a

The energy for second excited energy of the particle is


(a) 5eV (b) 2eV (c) 4eV (d) 4.5eV
Q4. A particle of mass m is confined into one dimensional infinite rigid box of width a the
quantum state of the system at t  0 given by  ( x, 0)  ( 3  1 ) the average value of

energy on state  ( x, 0 is given by

5 2  2 5 2  2 3 2  2 3 2  2
(a) (b) (c) (d)
2ma 2 ma 2 2ma 2 ma 2

                     
Head office  Branch office 
 
fiziks, H.No. 23, G.F, Jia Sarai,  Anand Institute of Mathematics, 
 
Near IIT, Hauz Khas, New Delhi‐16  28‐B/6, Jia Sarai, Near IIT 
 
Phone: 011‐26865455/+91‐9871145498 Hauz Khas, New Delhi‐16 
              
                   Website: www.physicsbyfiziks.com                        
                           Email: fiziks.physics@gmail.com                       160 
fiziks
Institute for NET/JRF, GATE, IIT‐JAM, JEST, TIFR and GRE in PHYSICAL SCIENCES 
 
Q5. A particle of mass m is confined into one dimensional infinite rigid box of width a the

4 1
quantum state of the system at t  0 given by  ( x, 0)  2  1 the
5 5
2 2  2
probability of measurement of energy on state  ( x, 0 is given by if n is
ma 2
orthonormal eigen state of Hamiltonian
1 4 1
(a) 0 (b) (c) (d)
2 5 5

4 1
Q6. If state of system is define as  ( x, 0)  1  2 for the potential
5 5

 a a
0 for  x
V x    2 2 where 1 , 2 are eigen state of Hamiltonian then
 otherwise

which of the following is expression for probability density   x, 0 


2

8 x 2 2 x 8 x 2 x
(a) sin 2  sin 2  sin sin
5a a 5a a 5a a a
8 x 2 2 x 8 x 2 x
(b) cos 2  sin 2  cos sin
5a a 5a a 5a a a
8 x 2 2 x 8 x 2 x
(c) cos 2  cos 2  cos cos
5a a 5a a 5a a a
8 x 2 2 x 8 x 2 x
(d) sin 2  cos 2  sin cos
5a a 5a a 5a a a
Q7. A particle is in the ground state of an infinite square well potential given by,
0 for  a  x  a
V x   
 otherwise

a a
The probability to find the particle in the interval between  and is
2 2
1 1 1 1 1 1
(a) (b)  (c)  (d)
2 2  2  

                     
Head office  Branch office 
 
fiziks, H.No. 23, G.F, Jia Sarai,  Anand Institute of Mathematics, 
 
Near IIT, Hauz Khas, New Delhi‐16  28‐B/6, Jia Sarai, Near IIT 
 
Phone: 011‐26865455/+91‐9871145498 Hauz Khas, New Delhi‐16 
              
                   Website: www.physicsbyfiziks.com                        
                           Email: fiziks.physics@gmail.com                       161 
fiziks
Institute for NET/JRF, GATE, IIT‐JAM, JEST, TIFR and GRE in PHYSICAL SCIENCES 
 
Q8. A particle in the infinite square well
0 0xa
V x   
 otherwise

 3  x 
 A sin   0 xa
is prepared in a state with the wave function   x    a
0
 otherwise
The expectation value of A such that  is normalized .

2 8 16 32
(a) A  (b) A  (c) A  (d) A 
a 5a 5a 5a
Q9. A particle in the infinite square well
0 0xa
V x   
 otherwise

 3  x 
 A sin   0 xa
is prepared in a state with the wavefunction   x    a
0
 otherwise
if energy is measured on state  the measurement

 2 2 9 2 2  2 2 9 2 2 9 2  2
(a) (b) (c) or (d)
2ma 2 2ma 2 2ma 2 2ma 2 10ma 2
Q10. A particle is in the ground state of an infinite square well potential given by,
0 for  a  x  a
V x   
 otherwise

a a
The probability to find the particle in the interval between  and is
4 4
1 2 1 1 1 1 1 2
(a)  (b)  (c)  (d) 
2  4 2 4 2 2 

                     
Head office  Branch office 
 
fiziks, H.No. 23, G.F, Jia Sarai,  Anand Institute of Mathematics, 
 
Near IIT, Hauz Khas, New Delhi‐16  28‐B/6, Jia Sarai, Near IIT 
 
Phone: 011‐26865455/+91‐9871145498 Hauz Khas, New Delhi‐16 
              
                   Website: www.physicsbyfiziks.com                        
                           Email: fiziks.physics@gmail.com                       162 
fiziks
Institute for NET/JRF, GATE, IIT‐JAM, JEST, TIFR and GRE in PHYSICAL SCIENCES 
 
Q11. Consider a particle in a one dimensional potential that satisfies V  x   V  x  . Let  0

and 1 denote the ground and the first excited states, respectively, and let

   0  0   1  1 be a normalized state with  0 and  1 being real constants. The

expectation value x of the position operator x in the state  is given by

(a)  02  0 x  0   12  1 x  1 (b)  0 1   0 x  1   1 x  0 
(c)  02   12 (d) 2 0 1

 a a
0  x
A particle of mass m is in potential V  x   
2 2
Q12. then no. of bound state is
16
2
otherwise
 ma 2

(a) One (b) Two (c) Three (d) Infinite


Q13. A quantum mechanical particle in a harmonic oscillator potential has the initial wave
function  0 x    1  x , where  0 and  1 are the real wavefunctions in the ground and
first excited state of the harmonic oscillator Hamiltonian. For convenience we take
m      1 for the oscillator. What is the probability density of finding the particle at
x at time t   ?
(a)  1 x    0 x  (b)  1  x    0  x 
2 2 2

(c)  1  x    0  x  (d)  1  x    0  x 
2 2 2

 1
A particle of mass m is confined in the potential V x    2 m x for x  0
2 2
Q14.
 for x  0

2i
Let the wavefunction of the particle be given by   x  
1
0  1
5 5
where  0 and  1 are the eigenfunctions of the ground state and the first excited state
respectively. The expectation value of the energy is
31 25 13 11
(a)  (b)  (c)  (d) 
10 10 10 10

                     
Head office  Branch office 
 
fiziks, H.No. 23, G.F, Jia Sarai,  Anand Institute of Mathematics, 
 
Near IIT, Hauz Khas, New Delhi‐16  28‐B/6, Jia Sarai, Near IIT 
 
Phone: 011‐26865455/+91‐9871145498 Hauz Khas, New Delhi‐16 
              
                   Website: www.physicsbyfiziks.com                        
                           Email: fiziks.physics@gmail.com                       163 
fiziks
Institute for NET/JRF, GATE, IIT‐JAM, JEST, TIFR and GRE in PHYSICAL SCIENCES 
 
The energy eigenvalues of a particle in the potential V x   m 2 x 2  ax are
1
Q15.
2

 1 a2  1 a2
(a) En   n    (b) En   n   
 2 2m 2  2 2m 2

 1 a2  1
(c) En   n    (d) En   n  
 2 m 2  2
Q16. A particle of mass m is in a cubic box of size a . The potential inside the box
0  x  a,0  y  a,0  z  a  is zero and infinite outside. If the particle is in an

14  2
eigenstate of energy E  , its wavefunction is
2ma 2

3x 5y 6z 7x 4y 3z


3/ 2 3/ 2
2 2
(a)     sin sin sin (b)     sin sin sin
a a a a a a a a

4x 8y 2z x 2y 3z


3/ 2 3/ 2
2 2
(c)     sin sin sin (d)     sin sin sin
a a a a a a a a

Q17. The energy of the first excited quantum state of a particle in the two-dimensional

potential V  x, y  
1
2

m 2 x 2  4 y 2 is 
3 5
(a) 2 (b) 3 (c)  (d) 
2 2

V  x, y   0 0  x  a, 0  y  b
Q18
 otherwise
If b  2a , the energy of first excited state is given by
5  2 2  2 2  2 2 13  2  2
(a) (b) (c) (d)
8 ma 2 ma 2 8ma 2 8 ma 2

                     
Head office  Branch office 
 
fiziks, H.No. 23, G.F, Jia Sarai,  Anand Institute of Mathematics, 
 
Near IIT, Hauz Khas, New Delhi‐16  28‐B/6, Jia Sarai, Near IIT 
 
Phone: 011‐26865455/+91‐9871145498 Hauz Khas, New Delhi‐16 
              
                   Website: www.physicsbyfiziks.com                        
                           Email: fiziks.physics@gmail.com                       164 
fiziks
Institute for NET/JRF, GATE, IIT‐JAM, JEST, TIFR and GRE in PHYSICAL SCIENCES 
 
A particle of mass m in the potential V  x, y   m 2 4 x 2  y 2  , is in an eigenstate of
1
Q19.
2
5
energy E   . The corresponding un-normalized eigen function is
2
 m  m
(a) y exp  2 x 2  y 2 

(b) x exp  2 x 2  y 2 

 2   2 
 m 2  m 2
(c) y exp  x  y 2 

(d) xy exp  x  y 2 

 2   2 
Q20. A quantum mechanical particle in a harmonic oscillator potential has the initial wave

 0 x    1  x , where  0 and  1 are the real wavefunctions in the ground


5 1
function
6 6
and first excited state of the harmonic oscillator Hamiltonian.if angular frequency of the
harmonic oscillator is   then average value of energy is given by
3 2 8 4
(a) (b) (c) (d)
2 3 3 3

NAT (Numerical Answer Type)


Q21. The wavefunction of particle moving in free space is given by,   e ikx  2e ikx . The
probability current density for the real part of the wavefunction is …………..in unit of
k
m
Q22. The wavefunction of particle of mass m moving in free space is given by,
  4e ikx  e  ikx . The probability current density for the wavefunction is………..in unit
k
of
m
Q23. A particle of mass m is confined in one dimensional box of width a if width of the box
will increase twice then ratio of energy of bigger to smaller box for given quantum
number ………..

                     
Head office  Branch office 
 
fiziks, H.No. 23, G.F, Jia Sarai,  Anand Institute of Mathematics, 
 
Near IIT, Hauz Khas, New Delhi‐16  28‐B/6, Jia Sarai, Near IIT 
 
Phone: 011‐26865455/+91‐9871145498 Hauz Khas, New Delhi‐16 
              
                   Website: www.physicsbyfiziks.com                        
                           Email: fiziks.physics@gmail.com                       165 
fiziks
Institute for NET/JRF, GATE, IIT‐JAM, JEST, TIFR and GRE in PHYSICAL SCIENCES 
 
Q24. A proton is confined to a cubic box, whose sides have length 10 12 m . The minimum
kinetic energy of the proton is …………..  10 17 J The mass of proton is 1.67  1027 kg

and Planck’s constant is 6.63  10 34 Js .


0 0xa
Q25. A particle in the infinite square well V  x    is prepared in a state with
 otherwise

x 3x
the wave function  x  
3 2 1 2
sin  sin the average value of energy
10 a a 10 a a

 2 2
is given by …………..
ma 2
Q26. Particle of same mass confined in the other infinite box where potential is defined as
 a a
0  x
V2  x    2 2
 otherwise
     
The expectation value of position on eigen state is ……………… a

 0 x

Q27. A particle of mass m is in potential V  x   0 0 xa then no. of bound state
16 2
 xa
 ma 2

is ………………
Q28. Consider the wave function Ae i k r r0 / r  , where A is the normalization constant.

For r  2r0 , the magnitude of probability current density up to two decimal places, in


units of A2 k / m is………… 

                     
Head office  Branch office 
 
fiziks, H.No. 23, G.F, Jia Sarai,  Anand Institute of Mathematics, 
 
Near IIT, Hauz Khas, New Delhi‐16  28‐B/6, Jia Sarai, Near IIT 
 
Phone: 011‐26865455/+91‐9871145498 Hauz Khas, New Delhi‐16 
              
                   Website: www.physicsbyfiziks.com                        
                           Email: fiziks.physics@gmail.com                       166 
fiziks
Institute for NET/JRF, GATE, IIT‐JAM, JEST, TIFR and GRE in PHYSICAL SCIENCES 
 
Q29. Let  1 and  2 denote the normalized eigenstates for potential is defined as

0 0xa
V1  x    of a particle with energy eigenvalues E1 and E2 respectively,
 otherwise

 t  0   1   2 
1
with E 2  E1 . At time t  0 the particle is prepared in a state
2

The shortest time T at which  t  T  will be orthogonal to t  0 is then the

 2 2
value of  is given by …………….(unit of )
2ma 2
Q30. A one dimensional harmonic oscillator is in the superposition of number state n given

1 3
by   2  3
2 2
The average energy of the oscillator in the given state is …………  .
Q31. The motion of a particle of mass m in one dimension is described by the
p2 1
Hamiltonian H   m 2 x 2  x . The difference between the (quantized) energies
2m 2
of the first two levels   the value of  ……….(In the following, x is the

expectation value of x in the ground state.)


Q32. If particle of mass m of two dimension infinite box is defined by
V  x, y   0 0  x  a, 0  y  b
     otherwise
If b  2a , the ratio of first excited to second excited state is ………….
Q33. If particle of m is confined into three dimensional box whose potential is defined as
V  x, y, z   0 0  x  a , 0  y  b, 0  z  c

 otherwise
If b  2a and c  3a , then energy eigenvalue first excited state is given by……….. in
 22
the unit of .
ma 2

                     
Head office  Branch office 
 
fiziks, H.No. 23, G.F, Jia Sarai,  Anand Institute of Mathematics, 
 
Near IIT, Hauz Khas, New Delhi‐16  28‐B/6, Jia Sarai, Near IIT 
 
Phone: 011‐26865455/+91‐9871145498 Hauz Khas, New Delhi‐16 
              
                   Website: www.physicsbyfiziks.com                        
                           Email: fiziks.physics@gmail.com                       167 
fiziks
Institute for NET/JRF, GATE, IIT‐JAM, JEST, TIFR and GRE in PHYSICAL SCIENCES 
 
Q34. A particle of mass m is subjected to a potential

V  x, y  
1
2
 
m 2 x 2  y 2 ,    x  ,  y  

The state with energy 4  is g  fold degenerate. The value of g  is …………
Q35. A particle of mass m is confined into two dimensional harmonic oscillator. If
 m
un-normalized wave function is given by y exp   2 x 2  y 2  , what will

be
 2 
corresponding energy eigen value is …………. 

                     
Head office  Branch office 
 
fiziks, H.No. 23, G.F, Jia Sarai,  Anand Institute of Mathematics, 
 
Near IIT, Hauz Khas, New Delhi‐16  28‐B/6, Jia Sarai, Near IIT 
 
Phone: 011‐26865455/+91‐9871145498 Hauz Khas, New Delhi‐16 
              
                   Website: www.physicsbyfiziks.com                        
                           Email: fiziks.physics@gmail.com                       168 
fiziks
Institute for NET/JRF, GATE, IIT‐JAM, JEST, TIFR and GRE in PHYSICAL SCIENCES 
 
MSQ (Multiple Select Questions)
Q36. A system defined as particle of mass m confined in the infinite square well
0 0xa
V1  x   
 otherwise
Where another system defined particle of same mass confined in the other infinite box
 a a
0  x
where potential is defined as V2  x    2 2
 otherwise

then which of the following statements is /are correct

(a)The energy eigen value of both is system will same

(b) The energy Eigen function of both the system will same

(c) The expectation value of position is center of potential in both the system.

(d) The eigen functions are either symmetric or anti symmetric about center of potential

Q37. A system defined as particle of mass m confined in the infinite square well
0 0  x  2a
V x   
     otherwise
then which of following is/are correct statement
 2 2
(a) The ground state energy eigen value is
2ma 2
1 x
(b) The ground state eigen function is given by sin
a 2a
a
(c) The expectation value of position at any eigen state of Hamiltonian is
2
 2 2
(d) The average kinetic energy if particle in ground state is given by
8ma 2

                     
Head office  Branch office 
 
fiziks, H.No. 23, G.F, Jia Sarai,  Anand Institute of Mathematics, 
 
Near IIT, Hauz Khas, New Delhi‐16  28‐B/6, Jia Sarai, Near IIT 
 
Phone: 011‐26865455/+91‐9871145498 Hauz Khas, New Delhi‐16 
              
                   Website: www.physicsbyfiziks.com                        
                           Email: fiziks.physics@gmail.com                       169 
fiziks
Institute for NET/JRF, GATE, IIT‐JAM, JEST, TIFR and GRE in PHYSICAL SCIENCES 
 
Q38. In a one-dimensional harmonic oscillator, 0 , 1 and 2 are respectively the ground, first
and the second excited states. These three states are normalized and are orthogonal to one
another. 1 and 2 are two states defined by  1  0  21  32 ,

 2  0  1  2 ,where α is a constant


(a)The value of α which  1 is orthogonal to 2 is 1

(b)The value of α which  1 is orthogonal to 2 is 1

(c) For the value of  determined  1 and  2 are orthogonal average value on state 2 is

3
(d) ) For the value of  determined  1 and  2 are orthogonal average value on state 2 is

3

2
Q39. A particle is constrained to move in a truncated harmonic potential well x  0 as shown
in the figure and potential is defined as A particle of mass m is confined in the potential
1
V  x    m 2 x 2 for -   x  
2
Then which of the following is correct statement
(a) The energy spacing between two consecutive energy levels is constant.
(b) There is finite probability to find the particle outside the harmonic oscillator.
(c) The wave functions are symmetric for n is even
(d) The average value of position at any state is zero.
Q40. A particle is constrained to move in a truncated harmonic potential
well x  0 as shown in the figure and potential is defined as a V  x
particle of mass m is confined in the potential
 1
V x    2 m x for x  0
2 2

 for x  0
   

                     
Head office  Branch office 
 
fiziks, H.No. 23, G.F, Jia Sarai,  Anand Institute of Mathematics, 
 
Near IIT, Hauz Khas, New Delhi‐16  28‐B/6, Jia Sarai, Near IIT 
 
Phone: 011‐26865455/+91‐9871145498 Hauz Khas, New Delhi‐16 
              
                   Website: www.physicsbyfiziks.com                        
                           Email: fiziks.physics@gmail.com                       170 
fiziks
Institute for NET/JRF, GATE, IIT‐JAM, JEST, TIFR and GRE in PHYSICAL SCIENCES 
 
Which one of the following statements is correct
(a) The parity of the first excited state is odd
(b) The parity of the ground state is even
1
(c) the ground state energy is 
2
7
(d) The first excited state energy is 
2
Q41. If particle of mass m is confined three dimensional box is whose potential is given by
V  x, y, z   0 0  x  a , 0  y  b, 0  z  c
   
 otherwise
For a  b  c , then which of the following is correct
(a) The ground state eigen value corresponds to nx  1, n y  0, nz  0 which is triply

degenerate.
(b) The first excited state is triply degenerate.
a
(c) The average value of position in x direction is on any state  nx ,ny ,nz
2
(d) One of the normalized eigen state of second excited state is

8 2 x y z
sin .sin sin
a 3
a a a
Q42. If the
potential of two dimensional harmonic oscillator is
1 1
V  x, y   m 2 x 2  m 4 2 y 2 then which of the following statement is /are correct
2 2
3
(a) The ground state energy eigen value is  it is non degenerate .
2
5
(b) The first excited state is  and its degeneracy is 2
2
7
(c) The second excited state is  and its degeneracy is 2
2
m x 2 m y 2
(d) The ground state wave function is  ( x, y )  exp .exp
2 2

                     
Head office  Branch office 
 
fiziks, H.No. 23, G.F, Jia Sarai,  Anand Institute of Mathematics, 
 
Near IIT, Hauz Khas, New Delhi‐16  28‐B/6, Jia Sarai, Near IIT 
 
Phone: 011‐26865455/+91‐9871145498 Hauz Khas, New Delhi‐16 
              
                   Website: www.physicsbyfiziks.com                        
                           Email: fiziks.physics@gmail.com                       171 
fiziks
Institute for NET/JRF, GATE, IIT‐JAM, JEST, TIFR and GRE in PHYSICAL SCIENCES 
 
Q43. If particle of mass m is confined three dimensional box is whose potential is given by
V  x, y, z   0 0  x  a , 0  y  b, 0  z  c
               
 Otherwise
If a  b  2c , then write down energy eigenvalue for ground state, first excited state and
second excited state.
Then which of the following is /are correct?
3 2  2
(a) The ground state energy eigen value is
4mc 2
9 2  2
(b) The first excited state energy is
8mc 2
(c) The ground state is non degenerate and first excited state is doubly degenerate.
(d) The one of the normalized wave function corresponds to first excited state is

2 x y z
sin sin sin
c 3
c 2c c

                     
Head office  Branch office 
 
fiziks, H.No. 23, G.F, Jia Sarai,  Anand Institute of Mathematics, 
 
Near IIT, Hauz Khas, New Delhi‐16  28‐B/6, Jia Sarai, Near IIT 
 
Phone: 011‐26865455/+91‐9871145498 Hauz Khas, New Delhi‐16 
              
                   Website: www.physicsbyfiziks.com                        
                           Email: fiziks.physics@gmail.com                       172 
fiziks
Institute for NET/JRF, GATE, IIT‐JAM, JEST, TIFR and GRE in PHYSICAL SCIENCES 
 
Solutions
MCQ (Multiple Choice Questions)
Ans. 1: (b)
Solution:  E  V0 , so there is decaying wave function.

Ans. 2: (c)

  2  2   2
Solution: H  E , H   ik ik e ikx  2 ik  ik e ikx
2m x 2 2m

 2 k 2 ikx 2k 2
 H 
2m
 e  2e   2m 
 ikx

Ans. 3: (d)
Solution: For A particle is confined in a one dimensional box with impenetrable walls at x = ±a.
The given state

a 0 a

is representation of first exited state whose energy is 2eV . If En is energy of nth state

and E0 is energy of ground state then En  n 2 E0 .

So E2  4 E0 and E0  0.5eV energy of second exited state is 3  E0  4.5eV


Ans. 4: (a)
1
Solution: If some one measure energy on state in normalized  ( x, 0)  ( 3  1 ) then
2
1 1
measurement is eigen value 9E0 and E0 with probability and respectively where
2 2
 2 2
E0 
2ma 2
1 1 5 2  2
Then average value of energy E  .9 E0  .E0  5 E0 
2 2 2ma 2
                     
Head office  Branch office 
 
fiziks, H.No. 23, G.F, Jia Sarai,  Anand Institute of Mathematics, 
 
Near IIT, Hauz Khas, New Delhi‐16  28‐B/6, Jia Sarai, Near IIT 
 
Phone: 011‐26865455/+91‐9871145498 Hauz Khas, New Delhi‐16 
              
                   Website: www.physicsbyfiziks.com                        
                           Email: fiziks.physics@gmail.com                       173 
fiziks
Institute for NET/JRF, GATE, IIT‐JAM, JEST, TIFR and GRE in PHYSICAL SCIENCES 
 
Ans. 5: (c)

4 1
Solution: If some one measure energy on state in normalized  ( x, 0)  2  1 ) then
5 5
4 1
measurement is eigen value 4E0 and E0 with probability and respectively where
5 5
2
 2 2 n 
E0  by using formula P  En  
2ma 2

Ans. 6: (b)

2 x 2 2 x
Solution: 1  cos and 2  sin
a a a a
Ans. 7: (b)
a a
Solution: The probability to find the particle in the interval between  and is
2 2

x x 1 1  2x  
a/2
2 2 a/2
1 x a/2
 
a / 2

2 a 2a
cos  cos dx   cos 2
2a 2a a / 2
a 2a
dx     1  cos
a 2 a / 2 
dx 
2a  
a/2
1  a x  1 a a a  1  2a   1 1 
  x  sin      1  1   a    
2a   a   a / 2 2a  2 2   2a    2  

Ans. 8: (c)
 3  x  
0 0 xa  A sin   0 xa 
Solution: V  x     x    a 
 otherwise , 0 otherwise

x  3 x 1 3 x
  x   A sin 3    A sin  A sin sin 3 A  3sin A  4sin 3 A
 a  4 a 4 a

3 x 1 3 x A  a 2 x a 2 3 x 
 A sin  A sin    3sin  sin 
4 a 4 a 4 2 a a 2 a a 

A a a 
   x  3 1  x   3  x  
4 2 2 

a 2 a 2 10a 2 32
  1 9 A  A 1  A 1  A 
32 32 32 10a
                     
Head office  Branch office 
 
fiziks, H.No. 23, G.F, Jia Sarai,  Anand Institute of Mathematics, 
 
Near IIT, Hauz Khas, New Delhi‐16  28‐B/6, Jia Sarai, Near IIT 
 
Phone: 011‐26865455/+91‐9871145498 Hauz Khas, New Delhi‐16 
              
                   Website: www.physicsbyfiziks.com                        
                           Email: fiziks.physics@gmail.com                       174 
fiziks
Institute for NET/JRF, GATE, IIT‐JAM, JEST, TIFR and GRE in PHYSICAL SCIENCES 
 
Ans. 9: (c)
 3  x  
0 0 xa  A sin   0 xa 
Solution: V  x     x    a 
 otherwise , 0 otherwise

x  3 x 1 3 x
  x   A sin 3    A sin  A sin sin 3 A  3sin A  4sin 3 A
 a  4 a 4 a

3 x 1 3 x A  a 2 x a 2 3 x 
 A sin  A sin    3sin  sin 
4 a 4 a 4 2 a a 2 a a 

A a a 
   x  3 1  x   3  x  
4 2 2 

 2 2 9 2  2
Now, E1  , E2   E  an P  an 
2ma 2 2ma 2
Ans. 10: (b)
a a
Solution: The probability to find the particle in the interval between  and is
4 4

x x 1 1  2 x  
a/4
2 2 a/4
1 x a/4
 
a / 4

2a 2a
cos
2a
 cos
2a
dx   cos 2
a / 4
a 2 a
dx      1  cos
a 2 a / 4  2 a
 dx 
 
a/4
1  a  x 1 a a a  1 1  1  a 2a   1 1 
  x  sin              
2a   a   a / 4 2a  4 4   2 2   2a  2   4 2 

Ans. 11: (b)


Solution: Since V  x   V  x  so potential is symmetric.

 0 x 0  0 ,  1 x 1  0

 x   0  0   1  1   0  0   1  1    01   0 x  1   1 x  0 

                     
Head office  Branch office 
 
fiziks, H.No. 23, G.F, Jia Sarai,  Anand Institute of Mathematics, 
 
Near IIT, Hauz Khas, New Delhi‐16  28‐B/6, Jia Sarai, Near IIT 
 
Phone: 011‐26865455/+91‐9871145498 Hauz Khas, New Delhi‐16 
              
                   Website: www.physicsbyfiziks.com                        
                           Email: fiziks.physics@gmail.com                       175 
fiziks
Institute for NET/JRF, GATE, IIT‐JAM, JEST, TIFR and GRE in PHYSICAL SCIENCES 
 
Ans. 12: (b)
mVo a 2
Solution: We compare the result  2   2   R2
2 2

16 2 
Put V0   2   2  8  R 2 , R  2 2 so  R      tan  and   cot 
ma 2 2
So there is 2 bound state.
Ans. 13: (a)
E0t Et
Solution:   x    0  x    1  x     x, t    0  x  e  i  1  x  ei 1
 
Now probability density at time t
t
  x, t    *  x, t   x, t    0  x    1  x   2 Re 0*  x  1  x  cos  E1  E0 
2 2 2


Putting t  

  x, t    0  x    1  x   2 Re 0*  x  1  x  cos 
2 2 2
 E1  E0    1

  x, t    0  x    1  x   2 Re 0*  x  1  x    1  x   0  x  
2 2 2 2

Ans. 14: (a)


3 7 1 3 4 7 31
Solution: For half parabolic potential E0   , E1    E       .
2 2 5 2 5 2 10
Ans. 15: (a)
p x2 1
Solution: Hamiltonian  H  of Harmonic oscillator, H   m 2 x 2
2m 2
 1
Eigen value of this, E n   n  
 2

p x2 1 px2 1 2  2 2ax a2  a2
But here, H   m x  ax  H 
2 2
 m  x   
2m 2 2m 2  m 2 m2 4  2m 2
2
p2 1  a  a2  1 a2
H  x  m 2  x    E   n    
m 2  2m 2
n
2m 2  2 m 2  2

                     
Head office  Branch office 
 
fiziks, H.No. 23, G.F, Jia Sarai,  Anand Institute of Mathematics, 
 
Near IIT, Hauz Khas, New Delhi‐16  28‐B/6, Jia Sarai, Near IIT 
 
Phone: 011‐26865455/+91‐9871145498 Hauz Khas, New Delhi‐16 
              
                   Website: www.physicsbyfiziks.com                        
                           Email: fiziks.physics@gmail.com                       176 
fiziks
Institute for NET/JRF, GATE, IIT‐JAM, JEST, TIFR and GRE in PHYSICAL SCIENCES 
 
Ans. 16: (d)
 2 2 14 2  2
Solution: E nx ,n y ,nz  n x2  n y2  n z2  
2ma 2 2ma 2
 n x2  n y2  n z2  14  n x  1, n y  2, n z  3 .

Ans. 17: (d)


 1  1
Solution: V  x, y   m 2 x 2  4 y 2   m2 x 2  m42 y 2 , E   n x     n y   2
1 1 1
2 2 2  2  2
 1 3
For ground state energy n x  0, n y  0  E   2 
2 2 2
3 5
First exited state energy n x  1, n y  0    
2 2
Ans. 18: (b)

 2   nx 2
ny 2   2  2  nx 2 ny 2 
Solution: Enx ,ny     E       b  2a
2m  a 2 b 2  2m  a 2  2a 2 
nx , n y

    
5  22 8  22
For ground state: E1,1  For first excited state: E 1,2 
8 ma 2 8 ma 2
Ans. 19: (a)

Solution: V  x, y   m 2 4 x 2  y 2  , E  
1 5
2 2
1 1
 V  x, y   m  2  x 2  m 2 y 2
2

2 2
 1  1  1  1
Now, E n   n x   x   n y   y   n x  2   n y  
 2  2  2  2

 3
 E n   2 n x  n y   
 2
5
 En   when nx  0 and n y  1 .
2
Ans. 20: (b)
5  1 3 8 2
Solution: E  .  .  
6 2 6 2 12 3
                     
Head office  Branch office 
 
fiziks, H.No. 23, G.F, Jia Sarai,  Anand Institute of Mathematics, 
 
Near IIT, Hauz Khas, New Delhi‐16  28‐B/6, Jia Sarai, Near IIT 
 
Phone: 011‐26865455/+91‐9871145498 Hauz Khas, New Delhi‐16 
              
                   Website: www.physicsbyfiziks.com                        
                           Email: fiziks.physics@gmail.com                       177 
fiziks
Institute for NET/JRF, GATE, IIT‐JAM, JEST, TIFR and GRE in PHYSICAL SCIENCES 
 
NAT (Numerical Answer Type)
Ans. 21: 0
Solution: The real part of the wave function real  cos kx  2 cos kx
Current density for real part of wave function = 0
Ans. 22: 15
16k
Solution: Probably current density associated with 4e ikx is J   and probably density
m
k 15k
associated with e  ikx J    so total current density is J  J   J  
m m  
Ans. 23: 0.25  
n 2 2  2 n 2 2 2
Solution: For width a En  for width 2a En 
2ma 2 2m(2a )2

n 2 2  2 n 2 2  2 1
Ratio of energy of bigger to smaller box= /   .25
2m4a 2 2ma 2 4
Ans. 24: 9.9
3 2  2
Solution:  9.9 1017
2ma 2
Ans. 25: 0.9  
x 3x
Solution:  x     x   1 x   3  x 
3 2 1 2 3 1
sin  sin
10 a a 10 a a 10 10
 2 2 9 2  2
E1  , E2   E  an P  an 
2ma 2 2ma 2
2 2
1  9 2  1
Probably P  E1    , P  E2   
 10  10

9  2  2 1 9 2  2 9 2  2
E      E 
10 2ma 2 10 2ma 2 10ma 2

                     
Head office  Branch office 
 
fiziks, H.No. 23, G.F, Jia Sarai,  Anand Institute of Mathematics, 
 
Near IIT, Hauz Khas, New Delhi‐16  28‐B/6, Jia Sarai, Near IIT 
 
Phone: 011‐26865455/+91‐9871145498 Hauz Khas, New Delhi‐16 
              
                   Website: www.physicsbyfiziks.com                        
                           Email: fiziks.physics@gmail.com                       178 
fiziks
Institute for NET/JRF, GATE, IIT‐JAM, JEST, TIFR and GRE in PHYSICAL SCIENCES 
 
Ans. 26: 0 
a

2
Solution: X   x dx  0
2
n
a

2

Ans. 27: 2
2mVo a 2
Solution: We compare the result  2   2   R 2 and    cot 
2
16 2 3 5
Put V0   2   2  32  R 2 , R  5.6 so R . So there is 2 bound state.
ma 2 2 2
Ans. 28: 0.25
2
 2 k 2 r
2
k 2 r0 k 2 k
Solution: J    A 0 J A J A
m r m 2r0 m 4m

Ans. 29:  3
1  iE1T 1  iE2T
 1   2 
1
Solution:   t  0  and   t  T   e 1  e 2
2 2 2

1  iE1 T 1  iE2 T iE
 1T
iE
 2T
T
i  E2  E1 
  0  T  dx  0  2 e  2 e  0  e  e  e   1
*  

T    
Equate real part  cos   E2  E1   1  T  cos 1  1  =
   E2  E1   E2  E1  
n 2 2  2 3 2  2
where En  so   E2  E1 
2ma 2 2ma 2
Ans. 30: 3.25
1 5 3 7
.  .
Solution: Average energy will 4 2 4 2  3.25
1 3

4 4
Ans. 31: 1
p2 1 1
Solution: H   m 2 x 2   x  V  x   m 2 x 2   x
2m 2 2

                     
Head office  Branch office 
 
fiziks, H.No. 23, G.F, Jia Sarai,  Anand Institute of Mathematics, 
 
Near IIT, Hauz Khas, New Delhi‐16  28‐B/6, Jia Sarai, Near IIT 
 
Phone: 011‐26865455/+91‐9871145498 Hauz Khas, New Delhi‐16 
              
                   Website: www.physicsbyfiziks.com                        
                           Email: fiziks.physics@gmail.com                       179 
fiziks
Institute for NET/JRF, GATE, IIT‐JAM, JEST, TIFR and GRE in PHYSICAL SCIENCES 
 
1 2  2 2  1 2 2  2 2 
V  x   m  x  x  m   x  2  x    
2  m 2  2  m 2 m 2 4 m 2 4 
  2 2
2
  1
V x  
1
m 2  x     En   n    
2  m 2  2 m 2  2 2m 2
3 1
 E1  E0      
2 2
Ans. 32: 1.62

 2   nx 2 ny 2   2  2  nx 2 ny 2 
Solution: Enx ,ny      E      b  2a
2m  a 2 b 2  2m  a 2  2a 2 
nx , n y

5  22 8  22
For ground state: E1,1  , For first excited state: E1,2 
8 ma 2 8 ma 2
13  2  2 E1,3 13
For second excited stste E1,3    1.62
8 ma 2 E1,2 8

Ans. 33: 0.84


49  2  2
Solution: For ground state: nx  1, n y  1, nz  1  E1,1,1  
36 2ma 2
61  2  2
For first excited state: nx  1, n y  1, nz  2  E1,1,2  
36 2ma 2
Ans. 34: 4
Ans. 35: 2.5
Solution: The energy eigen value for two dimensional harmonic oscillator whose potential is
1 1  1  1
V ( x, y )  m x2 x 2  m y2 y 2 is given by E   nx    x   n y    y .
2 2  2  2

 m 
If the eigen function is given by y exp  
 2
 2 x 2  y 2   then

5
nx  0, n y  1,  x  2 ,  y   so E  
2  

                     
Head office  Branch office 
 
fiziks, H.No. 23, G.F, Jia Sarai,  Anand Institute of Mathematics, 
 
Near IIT, Hauz Khas, New Delhi‐16  28‐B/6, Jia Sarai, Near IIT 
 
Phone: 011‐26865455/+91‐9871145498 Hauz Khas, New Delhi‐16 
              
                   Website: www.physicsbyfiziks.com                        
                           Email: fiziks.physics@gmail.com                       180 
fiziks
Institute for NET/JRF, GATE, IIT‐JAM, JEST, TIFR and GRE in PHYSICAL SCIENCES 
 
MSQ (Multiple Select Questions)

Ans. 36: (a), (c) and (d)

Solution: Hence width of the potential is same, so eigen value ,normalization constant will same

but eigen function will such that it is either symmetric or anti symmetric about mid center

of potential .
Ans. 37: (b) and (d)

n 2 2  2
Solution: For given potential energy eigen value is and eigen function is
8ma 2

2 n x
sin n  1, 2,3.... the expectation value of position is center of box ie a .
a 2a
Ans. 38: (b) and (d)
Solution: For orthogonal condition scalar product For orthogonal condition scalar product
 1 , 2   0 so 1  2  3  0    1

 3 5
2 H 2  
 2  0  1  2 put   1 , H   2 2 2  3 
2 2 3 2

Ans. 39: (a), (b), (c) and (d)


 1
Solution: The eigen value of harmonic oscillator  n    . So spacing between consecutive
 2
energy levels are constant i.e.  and there is finite probably to find the particle out side
the well where wave function are symmetric of n  0, 2, 4,.... and antisymmetric for

n  1,3,5,.... so x  0 .

Ans. 40: (a) and (d)


3 7
Solution: There is only odd parity. Ground state is  and first excited 
2 2

                     
Head office  Branch office 
 
fiziks, H.No. 23, G.F, Jia Sarai,  Anand Institute of Mathematics, 
 
Near IIT, Hauz Khas, New Delhi‐16  28‐B/6, Jia Sarai, Near IIT 
 
Phone: 011‐26865455/+91‐9871145498 Hauz Khas, New Delhi‐16 
              
                   Website: www.physicsbyfiziks.com                        
                           Email: fiziks.physics@gmail.com                       181 
fiziks
Institute for NET/JRF, GATE, IIT‐JAM, JEST, TIFR and GRE in PHYSICAL SCIENCES 
 
Ans. 41: (b) and (c).  
3 2  2
Solution: For ground state: nx  1, n y  1, nz  1  E1,1,1 
2ma 2
nx  1, n y  1 nz  2 
  6 2  2
For first excited state: nx  1, n y  2, nz  1   E1,1,2  E1,2,1  E2,1,1  (Triply
n  2 n  1 n  1 2ma 2
 x y z 
degenerate)

8 n x n y y n z
 n ,n y , nz
 sin x .sin sin z
x
a 3
a a a
second excited state is triply degenerate which can be discussed below .

 nx  1, n y  2 nz  2 
  9 2  2
For second excited state:  nx  2, n y  1, nz  2   E1,2,2  E2,1,2  E2,2,1 
 n  2 n  2 n  1 2ma 2
 x y z 

Ans. 42: (a) and (c)


1 1
Solution: V  x. y   m 2 x 2  m  2  y 2 i.e. 1   , 2  2
2

2 2
 1  1
 En x , n y   n x       n y   2  
 2  2
3
For ground state: E0,0   and wave function is proportional to
2
m x 2 m2 y 2
 ( x, y )  exp .exp 
    2 2
5
For first excited state: E1,0  
2
7
For second excited state: E0,1  E2,0   (Doubly degenerate)
2

                     
Head office  Branch office 
 
fiziks, H.No. 23, G.F, Jia Sarai,  Anand Institute of Mathematics, 
 
Near IIT, Hauz Khas, New Delhi‐16  28‐B/6, Jia Sarai, Near IIT 
 
Phone: 011‐26865455/+91‐9871145498 Hauz Khas, New Delhi‐16 
              
                   Website: www.physicsbyfiziks.com                        
                           Email: fiziks.physics@gmail.com                       182 
fiziks
Institute for NET/JRF, GATE, IIT‐JAM, JEST, TIFR and GRE in PHYSICAL SCIENCES 
 
Ans. 43: (a), (b), (c) and d)

 2  2  nx2 n y 
2

Solution: E n ,n     n 2
  
y ,n z 
2mc  4
2 z
x
4 

6  22
For ground state: nx  1, n y  1, nz  1  E1,1,1  
4 2mc 2
For first excited state: nx  1, n y  2, nz  1 and nx  2, n y  1, nz  1

9  22
 E1,2,1  E2,1,1   (Doubly degenerate)
4 2mc 2  

2 2 2 x 2 y z
 1,2,1  . . .sin .sin . sin
2c 2c c 2c 2c c

2 2 2 x y z
 2,1,1  . . .sin .sin . sin
2c 2c c c 2c c

                     
Head office  Branch office 
 
fiziks, H.No. 23, G.F, Jia Sarai,  Anand Institute of Mathematics, 
 
Near IIT, Hauz Khas, New Delhi‐16  28‐B/6, Jia Sarai, Near IIT 
 
Phone: 011‐26865455/+91‐9871145498 Hauz Khas, New Delhi‐16 
              
                   Website: www.physicsbyfiziks.com                        
                           Email: fiziks.physics@gmail.com                       183 
fiziks
Institute for NET/JRF, GATE, IIT‐JAM, JEST, TIFR and GRE in PHYSICAL SCIENCES 
 
5. Statistical Physics
5.1 Basic Definition, Mathematical Tools and Postulates of Statistical Mechanics
Statistical mechanics is a branch of physics that applies probability theory, which
contains mathematical tools for dealing with large population to study of the
thermodynamic behavior of systems composed of a large number particles.
It provides a framework for relating the microscopic properties of individual atoms and
molecules to the macroscopic bulk properties of materials that can be observed in
everyday life.
 Micro state: A microstate is a specific microscopic configuration of a
thermodynamic system that occupy with a certain property in the course of its thermal
fluctuation. The position (x), momentum (p), energy (E), and spin ( s, sz ) of individual

atom are the example of microstate of system.


 Macro state: A macro state refers to macroscopic properties of system such as
temperature (T), pressure (P), free energy (F), entropy (S). A macro state is
characterized by a probability distribution of possible state across a certain statistical
ensemble of all microstates, and distribution describes the probability of finding the
system in certain microstate.
 Accessible state: Any microstate in which a system can be found without
contradicting the macroscopic information available about the system.
 Statistical Ensemble: an assembly of large number of mutually non interacting
systems, each of which satisfies the same conditions as those known to be satisfied by
a particular system under condition. There are three type of ensemble (a) micro
canonical ensemble, (b) canonical ensemble, (c) grand canonical ensemble. An
ensemble is said to be time independent ensemble if number of system exhibiting any
particular property is the same at a time.

                     
Head office  Branch office 
 
fiziks, H.No. 23, G.F, Jia Sarai,  Anand Institute of Mathematics, 
 
Near IIT, Hauz Khas, New Delhi‐16  28‐B/6, Jia Sarai, Near IIT 
 
Phone: 011‐26865455/+91‐9871145498 Hauz Khas, New Delhi‐16 
              
                   Website: www.physicsbyfiziks.com                        
                           Email: fiziks.physics@gmail.com                       184 
fiziks
Institute for NET/JRF, GATE, IIT‐JAM, JEST, TIFR and GRE in PHYSICAL SCIENCES 
 
 Probability: The probability pr of occurrence of an event r in a system is defined

with respect to statistical ensemble of N such a systems. If N r systems in the

Nr
ensemble exhibit the event r then pr 
N
 Probability density: The probability density  (u ) is defined by the property that
 (u )du yields the probability of finding the continuous variable u in the range
between u and u  du .
 Mean value : The mean value of u is denoted by u as defined as

u    pr ur where the sum is over all possible value values ur of the variable u and
r

pr is denotes the probability of occurrence of the particular value ur .Above definition


is for discrete variable .
For continuous variable u ; u =  u  (u )du

 Dispersions (or variance): The dispersion of u is defined


as  2   (  u ) 2    p r (u r   u  ) 2 which is equivalent to
r

 2   ( u ) 2    ( u 2    u  2 )
r

 Stirling formula: Stirling’s approximation (or Stirling’s formula) is an


approximation for large factorials. It is named after James Stirling.
The formula as typically used in applications is
ln N  N ln N  N
5.2 Postulates of statistical mechanics
If an isolated system is found with equal probability in each of its accessible state, it is in
equilibrium, which is popularly known as postulates of equal a priori probabilities.
Suppose that we were asked to pick a card at random from a well-shuffled pack. It is
accepted that we have an equal probability of picking any card in the pack. There is
nothing which would favor one particular card over all of the others. So, since there are

                     
Head office  Branch office 
 
fiziks, H.No. 23, G.F, Jia Sarai,  Anand Institute of Mathematics, 
 
Near IIT, Hauz Khas, New Delhi‐16  28‐B/6, Jia Sarai, Near IIT 
 
Phone: 011‐26865455/+91‐9871145498 Hauz Khas, New Delhi‐16 
              
                   Website: www.physicsbyfiziks.com                        
                           Email: fiziks.physics@gmail.com                       185 
fiziks
Institute for NET/JRF, GATE, IIT‐JAM, JEST, TIFR and GRE in PHYSICAL SCIENCES 
 
fifty-two cards in a normal pack, we would expect the probability of picking the Ace of
1
Spades, say, to be .
52
We could now place some constraints on the system. For instance, we could only count
red cards, in which case the probability of picking the Ace of Hearts, say, would be
1 1 1 1
  2  , by the same reasoning. In both cases, we have used the principle
52 52 52 26
of equal a priori probabilities. People really believe that this principle applies to games
of chance such as cards, dice.
In statistical mechanics, we treat a many particle system a bit like an extremely large
game of cards. Each accessible state corresponds to one of the cards in the pack. The
interactions between particles cause the system to continually change state. This is
equivalent to constantly shuffling the pack. Finally, an observation of the state of the
system is like picking a card at random from the pack. The principle of equal a
priori probabilities then boils down to saying that we have an equal chance of choosing
any particular card.

Example: If there are four identical molecule in one dimensional container and it is given that
molecule can be found only either right or left end of container .
(a) What are possible configuration and no of ways to arrange these configuration? what
are corresponding probability of each configuration?
(b) What is most probable configuration?
(c) If some one is doing the experiment in which he observed molecule position to right
of container what is mean value of particle being in right?
(d) How postulates of a priori probability apply on the experiment?

Solution: The number of different ways of arranging N molecules with n on one side and N  n
N!
on the other side is given by , where ! represents the factorial function. The
N ! N  n  !
total number of possible ways of arranging the molecules is 2 N  24  16
                     
Head office  Branch office 
 
fiziks, H.No. 23, G.F, Jia Sarai,  Anand Institute of Mathematics, 
 
Near IIT, Hauz Khas, New Delhi‐16  28‐B/6, Jia Sarai, Near IIT 
 
Phone: 011‐26865455/+91‐9871145498 Hauz Khas, New Delhi‐16 
              
                   Website: www.physicsbyfiziks.com                        
                           Email: fiziks.physics@gmail.com                       186 
fiziks
Institute for NET/JRF, GATE, IIT‐JAM, JEST, TIFR and GRE in PHYSICAL SCIENCES 
 
(a)
Configuration No. of ways to arrange given configuration Probability

1 1
1. (L,L,L,L) 1  1
16 16
1 4
2. (L,L,L) and (R) 4 4 
16 16
1 6
3. (L,L) and(R,R) 6 6 
16 16
1 4
4. (L) and (R,R,R) 4 4 
16 16
1 1
5. only (R,R,R,R) 1 1 
16 16
(b) Most probable configuration is the one in which half the molecules are on one side
and half on the other, i.e. the molecules are uniformly distributed over the space. Most
probable configuration is configuration (L,L) and(R,R) which has maximum probability .
1 4 6 4 1
(c)  R   pr Rr = 0 R   1R   2 R   3R   4 R  = 2R
r 16 16 16 16 16
(d) We will now apply a fundamental postulate of statistical mechanics which states that
an isolated system which can be in any one of a number of accessible states (=16 in this
example) is equally likely to be in any one of these states at equilibrium. Therefore, the
probability that the molecules are distributed in any one of these 16 possible ways is
simply 1/16. But there are 4 ways in which the molecules can be arranged so that 3 are on
the left side and 1 on the right side, and therefore, the probability of finding that
configuration is 4/16. Similarly other configuration can be weighted.

                     
Head office  Branch office 
 
fiziks, H.No. 23, G.F, Jia Sarai,  Anand Institute of Mathematics, 
 
Near IIT, Hauz Khas, New Delhi‐16  28‐B/6, Jia Sarai, Near IIT 
 
Phone: 011‐26865455/+91‐9871145498 Hauz Khas, New Delhi‐16 
              
                   Website: www.physicsbyfiziks.com                        
                           Email: fiziks.physics@gmail.com                       187 
fiziks
Institute for NET/JRF, GATE, IIT‐JAM, JEST, TIFR and GRE in PHYSICAL SCIENCES 
 
Example: Suppose we know 3 particle being spin 1/2 kept into homogeneous magnetic B field at
temperature T.
(a) Show all possible microstate and corresponding probability .
(b) Find average value of z component of spin .
(c) If 0 is magnetic moment which configuration has maximum energy what is

corresponding probability.
Solution: (a)There is total 8 microstate is possible they are
configuration Microstate probability
All are in up state 1
  
8
Two are up and one is down   
3
  
8
  
One up and two down   
3
  
8
  
All three are down 1
  
8
3 1  3  3 3 1
(b) Average value of  sz          0
2 8 2 8 2 8 2 8
(c) The energy is given by E   0 B the magnetic moment of configuration in which

all three are down  have magnetic moment 30 so this configuration has

1
maximum energy which is equal to 30 B the corresponding probability is given by
8

                     
Head office  Branch office 
 
fiziks, H.No. 23, G.F, Jia Sarai,  Anand Institute of Mathematics, 
 
Near IIT, Hauz Khas, New Delhi‐16  28‐B/6, Jia Sarai, Near IIT 
 
Phone: 011‐26865455/+91‐9871145498 Hauz Khas, New Delhi‐16 
              
                   Website: www.physicsbyfiziks.com                        
                           Email: fiziks.physics@gmail.com                       188 
fiziks
Institute for NET/JRF, GATE, IIT‐JAM, JEST, TIFR and GRE in PHYSICAL SCIENCES 
 
5.3 Ensemble is collection of particle
5.2.1 Micro canonical ensemble  
Micro canonical ensemble is theoretical tool used to analyze an isolated thermo
dynamic system. The microstate of the system has fixed given energy(E), fixed number
of particle (N)and fixed volume (V).All accessible micro state has same
probability .popularly it is known as NVE ensemble .
Schematically the system can be shown as
NVE NVE NVE NVE
NVE NVE NVE NVE
NVE NVE NVE NVE
NVE NVE NVE NVE
NVE NVE NVE NVE
NVE NVE NVE NVE
In above table each cell consider as each microstate energy , volume and no of particle is
fixed in each cell.  
If  is the number of accessible microstates, the probability that a system chosen at
1
random from the ensemble would be in a given microstate is simply .

No. of accessible microstate in phase space which has energy between E to E  dE is
dp.dr
given by  
sates

h3
which is given in term of energy is

3/ 2
 2m 
n  E  dE  2 g V  2  E1/ 2 dE where g is degeneracy of the particle.
h 
5.2.1.1 Entropy:
From the number of accessible microstates,  , we can obtain the entropy(S) of the
system via S  k B ln 

where kB is the Boltzmann constant. or, equivalently,


s

( E ,V , N )  e kB
  is equivalent to “micro canonical partition function”
                     
Head office  Branch office 
 
fiziks, H.No. 23, G.F, Jia Sarai,  Anand Institute of Mathematics, 
 
Near IIT, Hauz Khas, New Delhi‐16  28‐B/6, Jia Sarai, Near IIT 
 
Phone: 011‐26865455/+91‐9871145498 Hauz Khas, New Delhi‐16 
              
                   Website: www.physicsbyfiziks.com                        
                           Email: fiziks.physics@gmail.com                       189 
fiziks
Institute for NET/JRF, GATE, IIT‐JAM, JEST, TIFR and GRE in PHYSICAL SCIENCES 
 
Example: The energy of Einstein oscillator is given by Ei  ni h if there is N no. of oscillator in

the 3 Dimensional system which has total energy E at temperature T.


(a) Write down micro canonical partition function  .
(b) What is entropy of the system?
3N 3N
Ei
Solution: (a)  is no. of microstate which is equivalent to    ni  
1 1 h
3N 3N
Ei E
  where E E
h h
i
1 1

3
Example: If there is N no of particle which have spin which will interact with magnetic field B
2
which are in equilibrium at temperature T
(a) How many no. of microstate for each particle
(b) What is entropy of the system.
3 3 1 1 3
Solution: (a) if s= then z component of spin ie. sZ  , , , so there is 4 microstate for
2 2 2 2 2
each particle
For the N no. of particle there will be 4 N no of state .
(b) S  k B ln  , where  = 4 N for given system. so S  Nk B ln 4

1
Example: A solid contain N magnetic atoms having spin . At sufficiently high temperature
2
each spin is completely random oriented .at sufficiently low temperature all the spin
become oriented in same direction let the heat capacity as a function of temperature T by
2T T1
c(T )  a (  1) If  T  T1
T1 2
0 Otherwise
Find the value of " a "  

Solution: at very low temperature all spins are oriented in only one direction so there is only

one possible microstate for each atoms . hence entropy is S1  0 , at high temperature all
                     
Head office  Branch office 
 
fiziks, H.No. 23, G.F, Jia Sarai,  Anand Institute of Mathematics, 
 
Near IIT, Hauz Khas, New Delhi‐16  28‐B/6, Jia Sarai, Near IIT 
 
Phone: 011‐26865455/+91‐9871145498 Hauz Khas, New Delhi‐16 
              
                   Website: www.physicsbyfiziks.com                        
                           Email: fiziks.physics@gmail.com                       190 
fiziks
Institute for NET/JRF, GATE, IIT‐JAM, JEST, TIFR and GRE in PHYSICAL SCIENCES 
 
the spin are randomly oriented and they can be either in up or down microstate so there
are two microstate for each atom hence for N no of atom entropy is given by
S2  Nk B ln 2 .

so S2  S1  Nk B ln 2 which is determined by theoretical calculation .


Tds
now from the given expression of heat capacity we have relation c 
dT .
 T1
c 2T dT Nk ln 2
S ()  S (o)  S2  S1   dT   a(  1) a
0
T T1 T1 T 1  ln 2
2

5.2.2 Canonical Ensemble


The canonical ensemble occurs when a system with fixed volume (V) and number of
particle (N) at constant temperature (T) . In other words we will consider an assembly of
systems closed to others by rigid, diathermal, impermeable walls. The energy of the
microstates can be fluctuate, the system is kept in equilibrium by being in contact with
the heat bath at temperature T. It is also referred to as the NVT ensemble
Schematically the system can be shown as
NVT  NVT  NVT  NVT 
NVT  NVT  NVT  NVT 
NVT  NVT  NVT  NVT 
NVT  NVT  NVT  NVT 
In above table each cell considers as each microstate temperature, volume and no of
particle is fixed in each cell. . Only value of energy is different in different cell which can
be exchanged in the process.
 

                     
Head office  Branch office 
 
fiziks, H.No. 23, G.F, Jia Sarai,  Anand Institute of Mathematics, 
 
Near IIT, Hauz Khas, New Delhi‐16  28‐B/6, Jia Sarai, Near IIT 
 
Phone: 011‐26865455/+91‐9871145498 Hauz Khas, New Delhi‐16 
              
                   Website: www.physicsbyfiziks.com                        
                           Email: fiziks.physics@gmail.com                       191 
fiziks
Institute for NET/JRF, GATE, IIT‐JAM, JEST, TIFR and GRE in PHYSICAL SCIENCES 
 
5.2.2.1 Partition Function for Canonical Ensemble
According to Gibbs, the probability of finding the system in any of its ith state at
1
temperature T where energy of that state is Ei is given by p ( Ei )  e  Ei  where  
k BT

p ( Ei )  ce  Ei  where c is proportionality constant . hence p ( Ei ) is probability then

1 1 1
 p ( E )   ce  Ei 
 1 so c  c 
i
i
i  e Ei 
i
ei
 Ei 
Z

The letter Z stands for the German word Zustandssumme, "sum over states" and is
popularly known as partition function for canonical ensemble which is given by
Z   e  Ei 
i

In systems with multiple quantum, we can write the partition function in terms of the
contribution from energy levels (indexed by i ) as follows:
Z   gi e  Ei  ,
i

where gj is the degeneracy factor, or number of quantum states which have the same
energy level defined by Ei .

In classical statistical mechanics, it is not really correct to express the partition function
as a sum of discrete terms, as we have done.
In classical mechanics, the position and momentum variables of a particle can vary
continuously, so the set of microstates is actually uncountable. In this case we must
describe the partition function using an integral rather than a sum. For instance, the
partition function of a gas of N identical classical particles is
1
3N 
ZN  exp   H ( p1... pN , x1..xN )d 3 x1...d 3 xN d 3 p1...d 3 pN
Nh

where pi  indicate particle momenta xi  indicate particle positions

d 3  is a shorthand notation serving as a reminder that the pi and xi are vectors in three
dimensional space, and H is the classical Hamiltonian.

                     
Head office  Branch office 
 
fiziks, H.No. 23, G.F, Jia Sarai,  Anand Institute of Mathematics, 
 
Near IIT, Hauz Khas, New Delhi‐16  28‐B/6, Jia Sarai, Near IIT 
 
Phone: 011‐26865455/+91‐9871145498 Hauz Khas, New Delhi‐16 
              
                   Website: www.physicsbyfiziks.com                        
                           Email: fiziks.physics@gmail.com                       192 
fiziks
Institute for NET/JRF, GATE, IIT‐JAM, JEST, TIFR and GRE in PHYSICAL SCIENCES 
 
The reason for the (N factorial): However, there is a well-known exception to this rule. If
the sub-systems are actually identical particles, in the quantum mechanical sense that
they are impossible to distinguish even in principle, the total partition function must be
divided by a N ! (N factorial):. For simplicity, we will use the discrete form of the
partition function in this article. Our results will apply equally well to the continuous
form.
The extra constant factor introduced in the denominator was introduced because, unlike
the discrete form, the continuous form shown above is not dimensionless. To make it into
a dimensionless quantity, we must divide it by where h is some quantity with units
of action (usually taken to be Planck's constant).
5.2.2.2 Relation Between Macroscopic Variable and Canonical Partition Function Z
 Relation between total energy and partition function for large no for particle
average of total energy E is equivalent to average of internal energy U.

Ee  i
 Ei
    ln Z
E  U    Ei Pi  i
 ln   e   Ei   
i e  i
 Ei
  i  
               
  ln Z  1 Z
E  U  kBT 2    E  U  k BT 2
 T  Z T
 Relation between partition function and specific heat at constant volume CV
 U 
CV   
 T V
 Relation between partition function and Helmholtz free Energy:
 F 
 F   U   T  S  and S     
 T V

 F    F / T  
U  F T    T 2  
 T V  T 

equating the coefficient of T 2 between relation

                     
Head office  Branch office 
 
fiziks, H.No. 23, G.F, Jia Sarai,  Anand Institute of Mathematics, 
 
Near IIT, Hauz Khas, New Delhi‐16  28‐B/6, Jia Sarai, Near IIT 
 
Phone: 011‐26865455/+91‐9871145498 Hauz Khas, New Delhi‐16 
              
                   Website: www.physicsbyfiziks.com                        
                           Email: fiziks.physics@gmail.com                       193 
fiziks
Institute for NET/JRF, GATE, IIT‐JAM, JEST, TIFR and GRE in PHYSICAL SCIENCES 
 
 F /T   2   ln Z 
U   T 2   and so E  U  kBT   so F   k BT ln Z
  T    T 
 relation between partition function and other thermodynamical variable
once internal energy(U) and Helmholtz free energy(F) is obtained one can find
U F
(a) entropy (S) S 
T
 F 
(b) pressure (P) P    
 V T
(c) enthalpy(H) H = U + PV
(d) Gibbs free energy G = H – TS
5.2.2.3 Relation Between Entropy and Probability . 
e   Ei F
Pi   ln Pi    Ei  ln Z  F   k BT ln Z  ln Z 
Z kBT

 F   Ei F  Ei  F
ln Pi    E i     ln pi    ln Pi 
 k BT  kBT k BT k BT

U  F  S
 ln Pi    F  U  TS   S  k B ln Pi
k BT kB

 S   k B  pi ln pi
i

Example: A system in thermal equilibrium has energies 0 and E. Calculate partition function of
system.
Then calculate
(i) Helmholtz Free energy (F)
(ii) entropy (S)
(iii) internal energy (U)
(iv) Specific heat at constant volume cV discuss the trend of specific heat at (a) low

temperature and
(b) high temperature

                     
Head office  Branch office 
 
fiziks, H.No. 23, G.F, Jia Sarai,  Anand Institute of Mathematics, 
 
Near IIT, Hauz Khas, New Delhi‐16  28‐B/6, Jia Sarai, Near IIT 
 
Phone: 011‐26865455/+91‐9871145498 Hauz Khas, New Delhi‐16 
              
                   Website: www.physicsbyfiziks.com                        
                           Email: fiziks.physics@gmail.com                       194 
fiziks
Institute for NET/JRF, GATE, IIT‐JAM, JEST, TIFR and GRE in PHYSICAL SCIENCES 
 
Solution: Let T be the temperature of the system. The partition function Z of the system is
Z  e 0 / k B T  e  E / k BT  1  e  E / k BT
(i) Free energy F of the system is

F  k B T ln Z  k B T ln 1  e  E / k BT 
(ii) Entropy S of the system is
 F 
S    
 k B ln 1  e  E / k BT 
E 1

 T  N ,V 1
E / k BT
T e

(iii) Internal energy U is


U  F  ST

  
 k B T ln 1  e  E / kT  k B T ln 1  e  E / k BT   e
E
E / k BT
1

e
E
E / k BT
1
(iv) Specific heat at constant volume cV is
2
 U   E  e E / k BT
cV     k B  
 T  N ,V  k BT 
 e E / k BT  1
2

(a) At a low temperature (E/kT) >> 1, and equation (3.35) reduces to
2
 E   E / k BT
CV  k B   e
k T
 B 

Since with the decrease of T, the function  E / k BT  , therefore


2

CV  0 when T  0

(b) At a high temperature (E/k BT) << 1 and equation reduces to


2
 E  1
CV  k   Hence, CV  0 when T  
 k BT  4

                     
Head office  Branch office 
 
fiziks, H.No. 23, G.F, Jia Sarai,  Anand Institute of Mathematics, 
 
Near IIT, Hauz Khas, New Delhi‐16  28‐B/6, Jia Sarai, Near IIT 
 
Phone: 011‐26865455/+91‐9871145498 Hauz Khas, New Delhi‐16 
              
                   Website: www.physicsbyfiziks.com                        
                           Email: fiziks.physics@gmail.com                       195 
fiziks
Institute for NET/JRF, GATE, IIT‐JAM, JEST, TIFR and GRE in PHYSICAL SCIENCES 
 
One Dimensional Free Particle:
Example: The Hamiltonian of one dimensional N free particle is confine in box of length L given
N
 p2 
by E  q, p     i  write down
i 1  2m 

(a) Expression of partition function


(b) Internal energy of system
(c) Specific heat at constant volume .

1   N  pi2   
Solution: Z N  N 
exp       k BT  dqi dpi
h   i 1  2m   

  
N
1  L
e  pi / 2 mkBT dpi  dqi
2

hN i 1
 0

pi2 p dp
For evaluation of the first integral of equation let us put  u and i i  du
2mk BT mk BT  
Using equations in the first integral equation we have

   
 

exp pi2 / 2mk BT dpi  2  exp  pi2 / 2mk BT dpi
0


 2mk BT  eu u 1/ 2  2 mk BT and integration of second integral is L
0

1
 2 mkBT   L 
1/ 2
Partition function of one particle is Z 
h
1
 2 mkBT   L 
N /2 N
Partition function of N particle is Z N  Z N  N
h N
1 Z
(b) the internal energy E  U  k BT 2
Z T
NkBT
E  U 
2
 U  Nk B
(c) CV    
 T V 2  

                     
Head office  Branch office 
 
fiziks, H.No. 23, G.F, Jia Sarai,  Anand Institute of Mathematics, 
 
Near IIT, Hauz Khas, New Delhi‐16  28‐B/6, Jia Sarai, Near IIT 
 
Phone: 011‐26865455/+91‐9871145498 Hauz Khas, New Delhi‐16 
              
                   Website: www.physicsbyfiziks.com                        
                           Email: fiziks.physics@gmail.com                       196 
fiziks
Institute for NET/JRF, GATE, IIT‐JAM, JEST, TIFR and GRE in PHYSICAL SCIENCES 
 
Classical Harmonic Oscillator:
Total energy of the system of N one dimensional classical oscillators is given by
 pi2 1 N

E q, p      m 2 qi2 
i 1  2m 2 
where qi and pi are position and momentum of the i-th oscillator, respectively.
(a) Write down partition
(b) Helmholtz Free energy
(c) Internal energy
(d) Specific heat at constant volume
The partition function of the system is where  

1   N  pi2 1   
Z N  N  exp      m 2 qi2   k BT  dqi dpi
h   i 1  2m 2   
    

  
N
1  
e  pi / 2 mkBT dpi  e  m qi / 2 kBT dqi
2 2 2

hN i 1
 

pi2 p dp
For evaluation of the first integral of equation let us put  u and i i  du
2mk BT  
mk BT
Using equations in the first integral equation we have

   
  
 
exp pi2 / 2mk BT dpi  2 exp  pi2 / 2mk BT dpi  2mk BT  eu u 1/ 2  2 mk BT
0 0

For evaluation of the second integral of equation, let us put


m 2 qi2 m 2 qi dqi
u and  du
2 k BT k BT

   
 
 
exp m 2 qi2 / 2k BT dqi  2 exp m 2 qi / 2k BT
0
dqi

k BT  2 k BT

m 2 

eu u 1/ 2 du 
m 2
N /2
1 N / 2  2 k BT 
Z N  N  2 mk BT   
h  h 
                     
Head office  Branch office 
 
fiziks, H.No. 23, G.F, Jia Sarai,  Anand Institute of Mathematics, 
 
Near IIT, Hauz Khas, New Delhi‐16  28‐B/6, Jia Sarai, Near IIT 
 
Phone: 011‐26865455/+91‐9871145498 Hauz Khas, New Delhi‐16 
              
                   Website: www.physicsbyfiziks.com                        
                           Email: fiziks.physics@gmail.com                       197 
fiziks
Institute for NET/JRF, GATE, IIT‐JAM, JEST, TIFR and GRE in PHYSICAL SCIENCES 
 
(a) Free energy F of the system is
 2 k BT 
F   k BT ln Z N   Nk BT ln  
 h 
Once the free energy of the system is known, we can calculate other thermo dynamical
quantities of the system.
(b) Entropy S of the system is
 F   2 k BT 
S     Nk B ln    Nk B
 T  N ,V  h 
(c) Internal energy U is
U  F  ST  Nk BT

Thus, the mean energy per oscillator is . k BT

(d) Specific heat at constant volume CV is

U
CV   NkB The specific heat at constant volume CV is independent of the
T
temperature
Quantum Harmonic Oscillator:
Example: In quantum mechanics, energy of an oscillator is quantized and the energy of the N
such system is given by
N
 1
E ni     ni  
i 1  2
where ni is an integer; ni = 0, 1, 2, 3, ….. then find
(a) The partition function of the system .
(b) entropy
(c) Helmholtz free energy
(d) Internal energy

                     
Head office  Branch office 
 
fiziks, H.No. 23, G.F, Jia Sarai,  Anand Institute of Mathematics, 
 
Near IIT, Hauz Khas, New Delhi‐16  28‐B/6, Jia Sarai, Near IIT 
 
Phone: 011‐26865455/+91‐9871145498 Hauz Khas, New Delhi‐16 
              
                   Website: www.physicsbyfiziks.com                        
                           Email: fiziks.physics@gmail.com                       198 
fiziks
Institute for NET/JRF, GATE, IIT‐JAM, JEST, TIFR and GRE in PHYSICAL SCIENCES 
 
Solution: (a) Specific heat at constant volume ,also discuss the case for lower temperature and
higher temperature.
 N  1 
ni
 ni  i 1 

a) Z N   exp  Eni / k BT   exp    ni   k BT   
2 
N  
  1 
    exp   ni   k BT 
i 1  ni  0   2 

  1 
We know that  exp   n  2 
ni  0
i k BT   exp   / 2k BT   exp  3 / 2k BT  ……..

 1  exp   / 2k BT 
 exp   / 2k BT   
1  exp   / k BT   1  exp   / k BT 
Thus, the partition function is
N
 exp   / 2k BT  
ZN   
1  exp   / k BT  
(b) Free energy F of the system is
 exp   / 2k BT  
F  k BT ln Z N   Nk BT ln  
1  exp   / k BT  
N 
  Nk BT ln 1  exp   / kBT  
2
Once the free energy of the system is known, we can calculate other thermodynamical
quantities of the system.
(c) Entropy S of the system is
 F  N  / T
S      Nk B ln 1  exp   / k BT   
 T  N ,V exp   / k BT   1

   1 
 Nk B    ln 1  exp   / k BT   
 k BT  exp   / k BT   1 

                     
Head office  Branch office 
 
fiziks, H.No. 23, G.F, Jia Sarai,  Anand Institute of Mathematics, 
 
Near IIT, Hauz Khas, New Delhi‐16  28‐B/6, Jia Sarai, Near IIT 
 
Phone: 011‐26865455/+91‐9871145498 Hauz Khas, New Delhi‐16 
              
                   Website: www.physicsbyfiziks.com                        
                           Email: fiziks.physics@gmail.com                       199 
fiziks
Institute for NET/JRF, GATE, IIT‐JAM, JEST, TIFR and GRE in PHYSICAL SCIENCES 
 
1  
(d) Internal energy U is U  F  ST  N    
 2 exp   / k BT   1 

(e) Specific heat at constant volume cV is

exp   / k BT     exp   / k BT 
2
U   
CV   N  2  2 
 Nk B  
T
exp   / k BT   1  B   k BT  exp   / k BT   1
2
kT

 At a low temperature, we have  / k B T   1 , and therefore, equation reduces to


2
     
CV  Nk B   exp   
 k BT   k BT 
Since with the decrease of T, the function e   / k BT reduces much faster than the increase
of the function  / k B T  , therefore when T  0
2
CV  0

 At a high temperature, we have  / k B T   1 , and therefore, equation reduces to


1   / k BT  .....
CV  Nk B
1   / 2kBT  .....
2

It gives CV  Nk B when T   it shows that the classical result for CV is valid at high

temperature.
Example: In quantum mechanics, energy of an oscillator is quantized and the energy of the N
 1
such system is given by En    n   where n = 0, 1, 2, 3, ….. then
 2
N
  
(a) Prove that partition function of the system is Z N   2sinh 
 2 k BT 

From the expression used in (a) then find


(a) Internal energy
(b) Specific heat at constant volume, also discuss the case for lower temperature and
higher temperature.
(c) Helmholtz free energy
(d) Entropy.

                     
Head office  Branch office 
 
fiziks, H.No. 23, G.F, Jia Sarai,  Anand Institute of Mathematics, 
 
Near IIT, Hauz Khas, New Delhi‐16  28‐B/6, Jia Sarai, Near IIT 
 
Phone: 011‐26865455/+91‐9871145498 Hauz Khas, New Delhi‐16 
              
                   Website: www.physicsbyfiziks.com                        
                           Email: fiziks.physics@gmail.com                       200 
fiziks
Institute for NET/JRF, GATE, IIT‐JAM, JEST, TIFR and GRE in PHYSICAL SCIENCES 
 
Solution:
 1
(a) In quantum mechanics, the energy of an oscillator is En   n    . Thus, the
 2
quantum mechanical partition function for one oscillator is

Z1   e  En / kBT   e  n 1/ 2  / kBT  
n n 0

e   / 2 k B T 1 1
   / 2 k B T  Since the partition function ZN of
1 e   / 2 k B T
e e   / 2 k B T
2sinh   / 2k BT 

a system of N independent particles is equal to the product of the partition function Z1 of


individual particle, we have
N
   1
Z N  Z   2sinh
1
N
 Using   ,
 2 k BT  k BT

(b) the internal energy U of the system is


     
U  ln Z N  N ln sinh
   2 

       
 N   coth N   / k B T 
 2  2 k BT  2 e  1

(c) Specific heat at constant volume CV is


2
 U  e / kBT   
CV     Nk  
 T  N ,  
2
e / kBT  1  2kBT 

(d) The Helmohltz free energy F is


  
F  k BT ln Z n  Nk BT  2sinh 
 2 k BT 

  
N
 2

 k BT ln 1  e   / kBT 


U F       
(e) The entropy S is S  Nk B  coth  ln  2sinh 
T  2k BT 2k BT  2k BT  

                     
Head office  Branch office 
 
fiziks, H.No. 23, G.F, Jia Sarai,  Anand Institute of Mathematics, 
 
Near IIT, Hauz Khas, New Delhi‐16  28‐B/6, Jia Sarai, Near IIT 
 
Phone: 011‐26865455/+91‐9871145498 Hauz Khas, New Delhi‐16 
              
                   Website: www.physicsbyfiziks.com                        
                           Email: fiziks.physics@gmail.com                       201 
fiziks
Institute for NET/JRF, GATE, IIT‐JAM, JEST, TIFR and GRE in PHYSICAL SCIENCES 
 
Quantum Mechanical Treatment of Spin Half Paramagnetic Substance:
Example: Suppose a system comprising N identical particles is placed in a uniform magnetic
1
field H and is kept at a temperature T . When a particle having spin is placed in a
2
magnetic field H , its each energy level splits into two with changes in energies by  H
and the particle has a magnetic moment  or   along the direction of the magnetic
field, respectively. Find expressions for internal energy, entropy, specific heat and total
magnetic moment M of this system with the help of the canonical distribution.
Solution: As the spins of particles are independent of each other, the partition function of the
total system Z N is equal to the product of the partition functions for spins of individual

particle. The partition function for spins of individual particle is


Z i  e  H / kBT  e   H / kBT  2 cosh   H / k BT 

Thus,

Z N  Z iN   2 cosh   H / k BT  
N

The Helmholtz free energy is


F   k BT ln Z N   Nk BT ln  2 cosh   H / k BT  
 
The entropy is
 F 
S     Nk B ln 2 cosh   H / k BT     H / k BT  tanh   H / k BT  
 T V
Total energy is U  F  TS   NH tanh H / k B T 
Total magnetic moment is
F
M   N  tanh   H / kBT 
H
The specific heat at constant volume CV is

 U 
 Nk B   H / k BT  sec h 2   H / k BT 
2
CV   
 T V

                     
Head office  Branch office 
 
fiziks, H.No. 23, G.F, Jia Sarai,  Anand Institute of Mathematics, 
 
Near IIT, Hauz Khas, New Delhi‐16  28‐B/6, Jia Sarai, Near IIT 
 
Phone: 011‐26865455/+91‐9871145498 Hauz Khas, New Delhi‐16 
              
                   Website: www.physicsbyfiziks.com                        
                           Email: fiziks.physics@gmail.com                       202 
fiziks
Institute for NET/JRF, GATE, IIT‐JAM, JEST, TIFR and GRE in PHYSICAL SCIENCES 
 
 1
Example: If Z is partition of one dimensional harmonic oscillator with energy  n    where
 2
n=0,1,2,3.... at equilibrium temperature T .

(a) what is probability that system has energy
2
(b) what is probability that system has energy lower than 4
(c) what is probability that system has energy greater than 4 

Solution: (a) If Z is partition of system what will be probability that system has energy
2
equilibrium temperature T.
Ei 
exp exp
k BT    2 k BT
p  Ei   p 
Z ,  2  Z
 3 5 7
(b) System has smaller thus energy 4 possible energy is , , , ,
2 2 2 2
 3 5 7 
exp  exp   exp   exp 
2 k BT 2 k BT 2 k BT 2 k BT
so p  E  4  
Z
(c) 

p  E  4  = 1  p  E  4   

  3 5 7 


 exp  2k T  exp  2k T  exp 2k T  exp 2k T 
p  E  4   1   B B B B

 Z 
 

                     
Head office  Branch office 
 
fiziks, H.No. 23, G.F, Jia Sarai,  Anand Institute of Mathematics, 
 
Near IIT, Hauz Khas, New Delhi‐16  28‐B/6, Jia Sarai, Near IIT 
 
Phone: 011‐26865455/+91‐9871145498 Hauz Khas, New Delhi‐16 
              
                   Website: www.physicsbyfiziks.com                        
                           Email: fiziks.physics@gmail.com                       203 
fiziks
Institute for NET/JRF, GATE, IIT‐JAM, JEST, TIFR and GRE in PHYSICAL SCIENCES 
 
Example: A particle is confined to the region x  0 by a potential which increases linearly as
U ( x)  u0 x . find the mean position of the particle at temperature T.
p2 u0 x
1  
Solution: Partition function is given by Z   e 2 mkBT dp  e kBT dx
h
x   xp  x dxdp x

u0 x 2 
p2 u0 x
   k BT 
 xe    te
 k BT t
dx dt
  xe u 0 
2 mk BT k BT
dp e dx  k BT
  0
u x
 0


 k BT 
2


p

u0 x  0 u0
  e
t
e dt
k BT
e dx
2 mk bT k BT
dp e dx
0  u0  0

5.2.3 Grand canonical ensemble:


In grand canonical ensemble, each element is in contact with reservoir where exchange of
energy and particles is feasible .so in such type of ensemble energy  E  and number of

particle  N  of system vary . This is an extension of the canonical but instead the grand

canonical ensemble being modeled is allowed to exchange energy and particles with its
environment. The chemical potential    (or fugacity ) is introduced to specify the

fluctuation of the number of particles as chemical potential and particle numbers are
thermodynamic conjugates. Popularly grand canonical ensemble is also known as T ,V ,  .
Schematically the grand canonical ensemble can be represented as
T V µ  T V µ  T V µ  T V µ 
T V µ  T V µ  T V µ  T V µ 
T V µ  T V µ  T V µ  T V µ 
T V µ  T V µ  T V µ  T V µ 
 
In above table each cell considers as each microstate temperature, volume and chemical
potential which is fixed in each cell. Only value of energy and no of particle is different
in different cell which can be exchanged in the process.

                     
Head office  Branch office 
 
fiziks, H.No. 23, G.F, Jia Sarai,  Anand Institute of Mathematics, 
 
Near IIT, Hauz Khas, New Delhi‐16  28‐B/6, Jia Sarai, Near IIT 
 
Phone: 011‐26865455/+91‐9871145498 Hauz Khas, New Delhi‐16 
              
                   Website: www.physicsbyfiziks.com                        
                           Email: fiziks.physics@gmail.com                       204 
fiziks
Institute for NET/JRF, GATE, IIT‐JAM, JEST, TIFR and GRE in PHYSICAL SCIENCES 
 
Grand canonical partition function is defined :
In grand canonical ensemble for the system of interest having constant value of T ,V , 
the partition function in classical system is given by

1 1  N    E N ( q, p ) 
Z   3N
exp    exp  dqdp
N N h  k BT   k BT 
  N  En , N 
here in quantum mechanical system Z    g n exp  
n N  k BT 
WThermo dynamical quantities in grand canonical ensemble:
Relation between Helmholtz free energy and grand canonical partition function
  N  En , N 
exp  
 k BT 
According to Gibbs distribution function  ( E , N ) 
Z

If  is statistical weight which is equivalent to no of microstate  in micro canonical


1
ensemble then  ( E, N )  1 and  
 (E, N )
1
So entropy is given by S  k B ln   k B ln
 ( E, N )
    N  En, N   En , N  N
So entropy S  k B ln exp   Z     k B ln Z  .
  k BT   T T

 En , N  ST   N   k B ln Z   F   N   k B ln Z 

Where F   N is popularly known as grand potential popularly represented by  .


    k B ln Z 

  
 Pressure of the system is P    
 V T , 

  
 The entropy of system is S    
 T V , 

                     
Head office  Branch office 
 
fiziks, H.No. 23, G.F, Jia Sarai,  Anand Institute of Mathematics, 
 
Near IIT, Hauz Khas, New Delhi‐16  28‐B/6, Jia Sarai, Near IIT 
 
Phone: 011‐26865455/+91‐9871145498 Hauz Khas, New Delhi‐16 
              
                   Website: www.physicsbyfiziks.com                        
                           Email: fiziks.physics@gmail.com                       205 
fiziks
Institute for NET/JRF, GATE, IIT‐JAM, JEST, TIFR and GRE in PHYSICAL SCIENCES 
 
  
 The average number is N    
  V ,T
 Helmholtz free energy F   N  k B ln Z 

  ln Z  
 Internal energy U  F  TS , U   N  k BT 2  
 T V , 

5.3 Maxwell-Boltzmann distribution 


In statistical mechanics, the Maxwell–Boltzmann distribution describes particle speeds
in gases, where the particles move freely without interacting with one another, except for
very brief elastic collision in which they may exchange momentum and kinetic energy,
but do not change their respective states of intermolecular excitation, as a function of
the temperature of the system, the mass of the particle, and speed of the particle. Particle
in this context refers to the gaseous atoms or molecules – no difference is made between
the two in its development and result.
Maxwell –Boltzmann system constituent identical particles who are distinguishable in
nature and there is not any restriction on no of particles which can occupy any energy
level. The wave function of particle will not overlap to each other because mean
separation of particles is more than the thermal wavelength , which is identified by  .
h
(where   is defined as the thermal wavelength )
2mk B T

Suppose there are l states with energies, E1 , E2 , E3 .......El and degeneracy

g1 , g 2 , g3 .......gl . Respectively, in which the particles are distributed. If there is

N numbers of distinguishable particles out of these n1 , n2 , n3 .......nl particles is adjusted in


i l i l
energy level E1 , E2 , E3 .......El respectively. So  ni  N
i 1
, En
i 1
i i U .

                     
Head office  Branch office 
 
fiziks, H.No. 23, G.F, Jia Sarai,  Anand Institute of Mathematics, 
 
Near IIT, Hauz Khas, New Delhi‐16  28‐B/6, Jia Sarai, Near IIT 
 
Phone: 011‐26865455/+91‐9871145498 Hauz Khas, New Delhi‐16 
              
                   Website: www.physicsbyfiziks.com                        
                           Email: fiziks.physics@gmail.com                       206 
fiziks
Institute for NET/JRF, GATE, IIT‐JAM, JEST, TIFR and GRE in PHYSICAL SCIENCES 
 
The total number of arrangements of the particles in the given distributions is given by
N i lgi ni
W g1n1 g 2 n2 ......g l nl , W  N
n1 n2 n3 ....... nl i 1 n
i

The Maxwell-Boltzmann distribution law for the particles in the states is


ni  gi exp     Ei  , ni  gi (exp  )(exp  Ei )
3/ 2
 N  h2  1
After using the values e    where  
V  2 mk BT  k BT
3/ 2
N  h2 
We get ni  gi   e Ei / kBT
V  2 mk BT 

5.3.1 Energy Distribution Function


Energy distribution function f ( Ei ) is the average number of particles per level in the
3/ 2
n N  h2 
energy states Ei . Therefore f  Ei   i    e Ei / kBT
gi V  2 mk BT 

Energy Distribution in Different Dimension


3/ 2
N  h2 
 f E    e E / kBT distribution function in three dimension where
V  2 mk BT 

V is volume .

N  h2   E / k BT
 f E   e distribution function in two dimension where A is
A  2 mk BT 

area.
1
N  h 2  2  E / k BT
 f E    e distribution function in one dimension. Where L
L  2 mkBT 

is length.

                     
Head office  Branch office 
 
fiziks, H.No. 23, G.F, Jia Sarai,  Anand Institute of Mathematics, 
 
Near IIT, Hauz Khas, New Delhi‐16  28‐B/6, Jia Sarai, Near IIT 
 
Phone: 011‐26865455/+91‐9871145498 Hauz Khas, New Delhi‐16 
              
                   Website: www.physicsbyfiziks.com                        
                           Email: fiziks.physics@gmail.com                       207 
fiziks
Institute for NET/JRF, GATE, IIT‐JAM, JEST, TIFR and GRE in PHYSICAL SCIENCES 
 
The number of particles n( E )dE having energies in the range from E to E  dE is 
n( E )dE  f ( E ) g ( E )dE where f ( E) is distribution function and g ( E )dE is number of
level(quantum state ) in the range of E to E  dE
3
1
 2m  2
 g ( E )dE in three dimension g ( E )dE  2 V  2  E 2 dE where V is volume of
h 
three dimensional space
 2m 
 g ( E )dE in two dimension g ( E )dE   A  2  dE where A is area of the two
h 
dimensional space
1
1
 2m  2
 g ( E )dE in one dimension g ( E )dE  L  2  E 2 dE where L is area of the one
h 
dimensional space
The number of particles n( E )dE having energies in the range from E to E  dE in three
dimensional space   0 .5

N(E)/(2N/kTπ1/2
0 .4
3/ 2 3
N h  2
 2m  2 0 .3
n  E  dE    e E / kBT 2 V  2  E1/ 2 dE
V  2 mk BT  h  0 .2
0 .1
2 N 1/ 2  E / k BT
 E e dE 0 .0
 kBT 
3/ 2 0 1 2 3 4 5 6 7
E / kT 

This is known as the Maxwell-Boltzmann energy distribution law for an-ideal gas.
h
Where   is defined as the thermal wavelength.
2 mk BT

                     
Head office  Branch office 
 
fiziks, H.No. 23, G.F, Jia Sarai,  Anand Institute of Mathematics, 
 
Near IIT, Hauz Khas, New Delhi‐16  28‐B/6, Jia Sarai, Near IIT 
 
Phone: 011‐26865455/+91‐9871145498 Hauz Khas, New Delhi‐16 
              
                   Website: www.physicsbyfiziks.com                        
                           Email: fiziks.physics@gmail.com                       208 
fiziks
Institute for NET/JRF, GATE, IIT‐JAM, JEST, TIFR and GRE in PHYSICAL SCIENCES 
 
Average Energy
For the Maxwell-Boltzmann energy distribution law, average energy  E  of the particles

 E n  E  dE 1 2 N 
 E    E   E 3/ 2 e  E / kBT dE
0
is 
N  k BT 3/ 2
 n  E  dE
0
0

Let E  kBTx and therefore, dE  k BTdx Then we have

2  2 k BT  3
 k Tx  
 k T  
3/ 2
E B e  x k BTdx  x3/ 2 e x dx  k BT
B
3/ 2 0
 0 2

1
Hence, the average of a particle is kBT per degree of freedom,
2
3
for three degree of freedom it is kBT
2
Example: Two distinguishable particles have to be adjusted in a state whose degeneracy is three
(a) How many ways the particles can be adjusted?
(b) Show all arrangement .
Ngn
Solution: (a) N  2, n  2, g  3 and no of microstate is W   9 , 9 ways .
n
(b) Total no of arrangement for 2 distinguishable in state whose degeneracy is 3.

First level Second level Third level


 
AB  0  0 
0  AB  0 
0  0  AB 
A  B  0 
B  A  0 
A  0  B 
B  0  A 
0  A  B 
0  B  A 

                     
Head office  Branch office 
 
fiziks, H.No. 23, G.F, Jia Sarai,  Anand Institute of Mathematics, 
 
Near IIT, Hauz Khas, New Delhi‐16  28‐B/6, Jia Sarai, Near IIT 
 
Phone: 011‐26865455/+91‐9871145498 Hauz Khas, New Delhi‐16 
              
                   Website: www.physicsbyfiziks.com                        
                           Email: fiziks.physics@gmail.com                       209 
fiziks
Institute for NET/JRF, GATE, IIT‐JAM, JEST, TIFR and GRE in PHYSICAL SCIENCES 
 
Example: If N no of distinguishable particle is kept into two dimensional box of area A what is
average energy at temperature A .
 2m 
Solution: for two dimensional system g ( E )dE   A  2  dE and distribution
h 

N  h2   E / k BT
Function is given by f  E    e
A  2 mk BT 

 E 
 E n  E  dE
0
where n  E  dE  f ( E ) g ( E )dE 
N  h2   E / k BT  2m 
 A  2  dE
 e

A  2 mk BT
 n  E  dE
0
 h 

 E   kBT

5.4 Bose Einstein distribution


In quantum statistics, Bose–Einstein statistics (or more colloquially B–E statistics) is
one of two possible ways in which a collection of indistinguishable particles may occupy
a set of available discrete energy state . The aggregation of particles in the same state,
which is a characteristic of particles obeying Bose–Einstein statistics .who recognized
that a collection of identical and indistinguishable particles can be distributed in this
way.
The Bose–Einstein statistics apply only to those particles not limited to single occupancy
of the same state—that is, particles that do not obey the Pauli exclusion restrictions.
Such particles have integer values of spin and are named boson , after the statistics that
correctly describe their behavior.
The wave function of particle will overlap to each other because mean separation of
particles is less than the thermal wavelength , which is identified by  .
h
(where   is defined as the thermal wavelength )
2mk B T

Suppose there are l states with energies, E1 , E2 , E3 .......El and degeneracy

g1 , g 2 , g3 .......gl . Respectively, in which the particles are distributed. If there is

N numbers of indistinguishable boson particles out of these n1 , n2 , n3 .......nl particles is

                     
Head office  Branch office 
 
fiziks, H.No. 23, G.F, Jia Sarai,  Anand Institute of Mathematics, 
 
Near IIT, Hauz Khas, New Delhi‐16  28‐B/6, Jia Sarai, Near IIT 
 
Phone: 011‐26865455/+91‐9871145498 Hauz Khas, New Delhi‐16 
              
                   Website: www.physicsbyfiziks.com                        
                           Email: fiziks.physics@gmail.com                       210 
fiziks
Institute for NET/JRF, GATE, IIT‐JAM, JEST, TIFR and GRE in PHYSICAL SCIENCES 
 
i l
adjusted in energy level E1 , E2 , E3 .......El respectively. It is given n
i 1
i N ,

i l

En
i 1
i i U .

The total no. of arrangements of the particles in the given distributions is given by
ni  gi  1 i l ni  g i  1
W , W 
ni  gi  1 i 1 ni  g i  1

If ni and gi are large numbers , we can omit 1 in comparison to them , so we have

i l ni  gi
W 
i 1 ni gi

gi
The Bose Einstein distribution of the particle among various states ni     E 
e 1
The Bose-Einstein energy distribution is
ni 1 1
f (E)  = f E     E   E
gi e  1 Ae  1
3/ 2
1 V  2 mk BT 
where  and A  e  e   / kBT   2 
k BT N  h 
Here, μ is chemical potential which is general a function of T. when A  1 , Bose-
Einstein gas reduces to the Maxwell-Boltzmann gas. The chemical potential for Bose gas
is negative, but for photon gas is zero.

                     
Head office  Branch office 
 
fiziks, H.No. 23, G.F, Jia Sarai,  Anand Institute of Mathematics, 
 
Near IIT, Hauz Khas, New Delhi‐16  28‐B/6, Jia Sarai, Near IIT 
 
Phone: 011‐26865455/+91‐9871145498 Hauz Khas, New Delhi‐16 
              
                   Website: www.physicsbyfiziks.com                        
                           Email: fiziks.physics@gmail.com                       211 
fiziks
Institute for NET/JRF, GATE, IIT‐JAM, JEST, TIFR and GRE in PHYSICAL SCIENCES 
 
Example: Two indistinguishable boson particles have to be adjusted in a state whose degeneracy
is three.
a) How many ways the particles can be adjusted?
b) Show all arrangement.
ni  gi  1
Solution: (a) ni  2, g i  3 , Wi  = 6 ways
ni gi  1

(b) Total no of arrangement for 2 indistinguishable boson particles in state whose


degeneracy is 3.
 
First level Second level Third level
AA  0  0 
0  AA  0 
0  0  AA 
A  A  0 
0  A  A 
A  0  A 

Examples: (a) write down distribution function of photon at temperature T ,if average energy in
each state is given by   h .
(b) what is density of state of photon gas between frequency  to   d
(c) write down expression of no of particle for photon gas at temperature T .
(d) write down expression of average energy for photon gas at temperature T .
1 1
Solution: (a) the Bose Einstein distribution is given by f E     E   E
e 1 Ae 1
3/ 2
V  2 mk BT  1
where A  e  e   / kBT    , for boson   0 . So f  E   E
for if
N  h
2
 e 1
1
average energy in each state is given by   h then f E  h

e k BT
1

                     
Head office  Branch office 
 
fiziks, H.No. 23, G.F, Jia Sarai,  Anand Institute of Mathematics, 
 
Near IIT, Hauz Khas, New Delhi‐16  28‐B/6, Jia Sarai, Near IIT 
 
Phone: 011‐26865455/+91‐9871145498 Hauz Khas, New Delhi‐16 
              
                   Website: www.physicsbyfiziks.com                        
                           Email: fiziks.physics@gmail.com                       212 
fiziks
Institute for NET/JRF, GATE, IIT‐JAM, JEST, TIFR and GRE in PHYSICAL SCIENCES 
 
(b) if j is quantum number associate with frequency  then total no of frequency
between  to   d is same as the number of points between j to j  dj . The volume of

spherical shell of radius j and thickness dj is 4 j 2 dj .


Hence all three component of j is positive (same as particle in box) and there are two
direction of polarization so degeneracy g  2 .

1
So no of standing wave g ( j ) dj  (2)   4 j 2 dj   j 2 dj
8
2 L 2L 2Ld 8 L3 2
It is given j   and dj  g ( )d  d
 c c c3
g ( )
So density of standing wave in cavity is given by g ( )d  d
L3
8 2
g ( )d  d
c3
 
(c) N   f E g E dE   f   g   d
0 0

3
8 V   2 d h 8 V  k T   x 2 dx
N 3
c 
0 h
put x 
k BT
N 3  B 
c  hc  
0 ex 1
e k BT
1
The integral have value
3
k T 
N  1.92V  B 
 hc 
 
(d) U   Ef E g E dE   h f   g   d
0 0

8 Vh   3d h
U
c 3 0 h put x 
k BT
e k BT  1
4
k T   x3dx 8 5Vk 4 BT 4
U  8 Vc  B 
 hc 
 0 ex 1
U
15c3 h3
.

                     
Head office  Branch office 
 
fiziks, H.No. 23, G.F, Jia Sarai,  Anand Institute of Mathematics, 
 
Near IIT, Hauz Khas, New Delhi‐16  28‐B/6, Jia Sarai, Near IIT 
 
Phone: 011‐26865455/+91‐9871145498 Hauz Khas, New Delhi‐16 
              
                   Website: www.physicsbyfiziks.com                        
                           Email: fiziks.physics@gmail.com                       213 
fiziks
Institute for NET/JRF, GATE, IIT‐JAM, JEST, TIFR and GRE in PHYSICAL SCIENCES 
 
Example: A system consisting of two boson particles each of which can be any one of three
quantum state of respective energies o,  ,3 is in equilibrium at temperature T .write the
expression of partition function.
Two boson can be distributed in three given state with their respective energy level
shown in table
S.N.  Energy 0  Energy  Energy 3 Total energy  
1  A A  0  0  0 
2  0  AA  0  2  
3  0  0  AA  6  
4  A  A  0   
5  A  0  A  3  
6  0  A  A  4  
 
Z  1  exp(   )  exp( 2  )  exp( 3 )  exp( 4  )  exp( 6  )
5.5 Fermi Dirac Distribution
In quantum statistics, Fermi–Dirac statistics describes distribution of particles in
a system comprising many identical particles that obey the Pauli Exclusion Principle
(The Pauli Exclusion Principle is the quantum mechanical principle that no two
identical fermions (particles with half-integer spin) may occupy the same quantum state
simultaneously.)
Fermi–Dirac (F–D) statistics applies to identical particles with half odd integer spin in
a system in thermal equilibrium. Additionally, the particles in this system are assumed to
have negligible mutual interaction. This allows the many-particle system to be described
in terms of single-particle energy states . The result is the F–D distribution of particles
over these states and includes the condition that no two particles can occupy the same
state, which has a considerable effect on the properties of the system. Since F–D statistics
applies to particles with half-integer spin, these particles have come to be
called fermions.

                     
Head office  Branch office 
 
fiziks, H.No. 23, G.F, Jia Sarai,  Anand Institute of Mathematics, 
 
Near IIT, Hauz Khas, New Delhi‐16  28‐B/6, Jia Sarai, Near IIT 
 
Phone: 011‐26865455/+91‐9871145498 Hauz Khas, New Delhi‐16 
              
                   Website: www.physicsbyfiziks.com                        
                           Email: fiziks.physics@gmail.com                       214 
fiziks
Institute for NET/JRF, GATE, IIT‐JAM, JEST, TIFR and GRE in PHYSICAL SCIENCES 
 
1
It is most commonly applied to electrons , which are fermions with spin
2
No. of ways W in which ni indistinguishable particles to place in gi level with the
condition that only one particle or no particle can be placed in each level i.e identical
i l
particles that obey the 5.5.1Pauli exclusion principle. ( it is given It is given n
i 1
i N

i l l gi
En i i  U .) is given by W 
i 1 i 1 ni gi  ni

Fermi-Dirac distribution of the particles among various states is given by


gi
ni 
exp(   E )  1
ni 1 1
So Fermi Dirac distribution f  E       E   E
gi e  1 Ae  1
3/ 2
1 V  2 mk BT 
where   and A  e   
k BT N  h2 
when A.>> 1, Fermi-Dirac gas reduces to the Maxwell-Boltzmann gas. Fermi-Dirac gas
is said to be weakly degenerate when A>1, degenerate when A<1 and strongly
degenerate when A = 0.Strongly degenerate Fermi gas A  1
The Fermi-Dirac energy distribution is
1
f E 
exp  E    / k BT  1

where  is the chemical potential which is a function of T, i.e;    (T ) . the gas is


strongly degenerate (A = 0) at T = 0. at T = 0, where    (0)  EF . The limiting
chemical potential is known as the Fermi energy EF of the gas and the distribution
1
function can be written as f  E    E  EF  / k BT
e 1

                     
Head office  Branch office 
 
fiziks, H.No. 23, G.F, Jia Sarai,  Anand Institute of Mathematics, 
 
Near IIT, Hauz Khas, New Delhi‐16  28‐B/6, Jia Sarai, Near IIT 
 
Phone: 011‐26865455/+91‐9871145498 Hauz Khas, New Delhi‐16 
              
                   Website: www.physicsbyfiziks.com                        
                           Email: fiziks.physics@gmail.com                       215 
fiziks
Institute for NET/JRF, GATE, IIT‐JAM, JEST, TIFR and GRE in PHYSICAL SCIENCES 
 
Strongly degenerate Fermi gas at T = 0 1
1 1 T 0
At T = 0, when E < EF, we have f E    1
e  1 0 1 
f E 
1 1
At T = 0, when E > EF, we have f  E   
 0
e 1  1 T 0
Fermi function f E  versus E atT 0 EF E 

The number of energy states in the energy range from E to E  dE


3/ 2
 2m 
g E dE  2 gV  2  E1 / 2 dE Here, g is the spin degeneracy, g  (2 s  1) , where s is
h 
the spin quantum number of a particle. The number of particles in the energy range from
E to E + dE at T = 0 is
  2m 
3/ 2

nE dE   f E g E dE  2 gV  2  E 1 / 2 dE for E  E F


 h 
0 for E  E F
3/ 2 3/ 2
 2m  4 gV  2m 
N   nE  dE
EF EF

0
 2 gV  2 
h 
 0
E 1/ 2 dE 
3
 2 
h 
E F3 / 2

2/3
 3N  h2
Thus, the Fermi energy is EF (0)    and the Fermi
 4 gV  2m
EF
E
temperature TF is defined as TF  F
kB 

The Fermi momentum pF is given by
0
 3N 
1/ 3 0 T  TF
pF  2mEF 
1/ 2
   h Fig: Variation of chemical
 4gV  potential μ with T.
Total energy of the gas at T  0 is
3/ 2 3/ 2
  2m  4 gV  2m  2 gV 5
EnE  dE  2 gV  2 
EF
U   E 3/ 2
dE    EE5 F/ 2  pF
0
h  0 5  h2  5mh 3
U 3
Thus, at T  0 we have  EF
N 5

                     
Head office  Branch office 
 
fiziks, H.No. 23, G.F, Jia Sarai,  Anand Institute of Mathematics, 
 
Near IIT, Hauz Khas, New Delhi‐16  28‐B/6, Jia Sarai, Near IIT 
 
Phone: 011‐26865455/+91‐9871145498 Hauz Khas, New Delhi‐16 
              
                   Website: www.physicsbyfiziks.com                        
                           Email: fiziks.physics@gmail.com                       216 
fiziks
Institute for NET/JRF, GATE, IIT‐JAM, JEST, TIFR and GRE in PHYSICAL SCIENCES 
 
Example: What is no. of ways if two fermions have to adjust in energy state whose degeneracy is
three.
gi
Solution: g i  3 , ni  2 W  3
ni gi  ni

Two indistinguishable particles is shown by A,A


Possible 1  2  3 
selection 
1  A  A  0 
2  0  A  A 
3  A  0  A 
 
Example: Fermions of mass m are kept in two dimensional box of area A at temperature T  0
(a) What is total number of particle if EF is Fermi energy.

(b) What is the energy of the system if EF is Fermi energy.

c) Write expression of energy in term of EF and N

 2m 
Solution: For two dimensional systems density of state g ( E )dE   A  2  dE and distribution
h 
function at temperature T  0 for is given by f ( E )  1 if E  EF  0 if E  EF
EF  2mAEF
a) N   g ( E ) f ( E )dE 
0 h2
EF
EF  A2m  mAEF2
b) E   Eg ( E ) f ( E )dE E  EdE , E
0 h2 0
h2

NEF
c) E 
2

                     
Head office  Branch office 
 
fiziks, H.No. 23, G.F, Jia Sarai,  Anand Institute of Mathematics, 
 
Near IIT, Hauz Khas, New Delhi‐16  28‐B/6, Jia Sarai, Near IIT 
 
Phone: 011‐26865455/+91‐9871145498 Hauz Khas, New Delhi‐16 
              
                   Website: www.physicsbyfiziks.com                        
                           Email: fiziks.physics@gmail.com                       217 
fiziks
Institute for NET/JRF, GATE, IIT‐JAM, JEST, TIFR and GRE in PHYSICAL SCIENCES 
 
Example: (a) If Fermi gas is at temperature T  0 what will f  EF  .

(b) At E = E F + x, find the fraction of occupied levels .

(c) At E = E F - x, find fraction of unoccupied levels.

Solution: (a) It is also interesting to note that at T > 0, when E  EF we have

1 1 1 1
f E  E  EF / kT
 
  
e 1 e 0 / kT
1 11 2
(b) At T > 0 , fraction of levels above EF are occupied and a fraction of levels below EF
are vacant. The fraction of occupied levels at the energy E is
1
f E    E  EF  / k T
e 1
1
At E = E F + x, , the fraction of occupied levels is f EF  x  
e x / kT
1
(c) The fraction of unoccupied levels at the energy E is
1
1  f E   1   E  EF  / k T
e 1
1
At E  EF  x , the fraction of unoccupied levels is 1  f EF  x  
1  ex/ kT

                     
Head office  Branch office 
 
fiziks, H.No. 23, G.F, Jia Sarai,  Anand Institute of Mathematics, 
 
Near IIT, Hauz Khas, New Delhi‐16  28‐B/6, Jia Sarai, Near IIT 
 
Phone: 011‐26865455/+91‐9871145498 Hauz Khas, New Delhi‐16 
              
                   Website: www.physicsbyfiziks.com                        
                           Email: fiziks.physics@gmail.com                       218 
fiziks
Institute for NET/JRF, GATE, IIT‐JAM, JEST, TIFR and GRE in PHYSICAL SCIENCES 
 
Questions
MCQ (Multiple Choice Questions)
3
Q1. If spin of a fermions is s   then no of microscope is given
2
(a) 1 (b) 2 (c) 3 (d) 4
Q2. If 2 classical particle have to to adjusted in 2 different non degenerate quantum state
where one particle is in ground state and another is in upper state the no of possible
microstate is
(a) 1 (b) 2 (c) 3 (d) 4
Q3. If 3 classical particle have to to adjusted in 2 different quantum state where one
particle is in ground state and other is in upper state where ground state is non degenerate
and upper state is doubly degenerate
a) 4 (b) 6 (c) 10 (d) 12
Q4. If 2 boson have to to be adjusted in 2 different non degenerate quantum state where
one particle is in ground state and another is in upper state the no of possible microstate is
(a) 1 (b) 2 (c) 3 (d) 4
Q5. If 3 boson particle have to adjusted in 2 different quantum state where one particle is
in ground state and other is in upper state where ground state is non degenerate and upper
state is doubly degenerate
(a) 2 (b) 3 (c) 6 (d) 12
Q6. If 2 fermions have to to be adjusted in 2 different non degenerate quantum state
where one particle is in ground state and another is in upper state the no of possible
microstate is
(a) 1 (b) 2 (c) 3 (d) 4
Q7. If 3 fermions particle have to adjusted in 2 different quantum state where one
particle is in ground state and other is in upper state where ground state is non degenerate
and upper state is doubly degenerate
(a) 1 (b) 2 (c) 3 (d) 4

                     
Head office  Branch office 
 
fiziks, H.No. 23, G.F, Jia Sarai,  Anand Institute of Mathematics, 
 
Near IIT, Hauz Khas, New Delhi‐16  28‐B/6, Jia Sarai, Near IIT 
 
Phone: 011‐26865455/+91‐9871145498 Hauz Khas, New Delhi‐16 
              
                   Website: www.physicsbyfiziks.com                        
                           Email: fiziks.physics@gmail.com                       219 
fiziks
Institute for NET/JRF, GATE, IIT‐JAM, JEST, TIFR and GRE in PHYSICAL SCIENCES 
 
Q8. If ground state of single particle confined in one dimensional potential box of width
a have ground state energy is E0 then if 4 such type of fermions have to adjusted in same
box then energy of ground state of configuration is
(a) E0 (b) 4 E0 (c) 10 E0 (d) 12 E0

Q9. For a two-dimensional free electron gas, the electronic density n , and the Fermi energy
EF at temperature T , are related by

2mE F  2
3
mE F
(a) n  (b) n 
3  2 3
 2

mE F 23
2
mE F  3 2
(c) n  (d) n 
2 2 
Q10. A system has two energy levels with energies  and 2 . The lower level is 4 -fold
degenerate while the upper level is doubly degenerate. If there are N non-interacting
classical particles in the system, which is in thermodynamic equilibrium at a temperature
T , the fraction of particles in the upper level is
1 1
(a) (b)
1  e  / k BT 1  2e  / k BT
1 1
(c)  / k BT
(d)
2e  4e 2  / k B T 2e  / k BT
 4e 2  / k B T
Q11. For an ideal Fermi gas in three dimensions, the electron velocity VF at the Fermi surface

is related to electron concentration n as,


(a) VF  n 2 / 3 (b) VF  n (c) VF  n1 / 2 (d) V F  n1 / 3
Q12. Consider a system whose three energy levels are given by 0 ,  and 2 . The energy level
 is two-fold degenerate and the other two are non-degenerate. The partition function of
1
the system with   is given by
k BT

(a) 1  2e   (b) 2e    e 2  (c) (1  e   ) 2 (d) 1  e    e 2 

                     
Head office  Branch office 
 
fiziks, H.No. 23, G.F, Jia Sarai,  Anand Institute of Mathematics, 
 
Near IIT, Hauz Khas, New Delhi‐16  28‐B/6, Jia Sarai, Near IIT 
 
Phone: 011‐26865455/+91‐9871145498 Hauz Khas, New Delhi‐16 
              
                   Website: www.physicsbyfiziks.com                        
                           Email: fiziks.physics@gmail.com                       220 
fiziks
Institute for NET/JRF, GATE, IIT‐JAM, JEST, TIFR and GRE in PHYSICAL SCIENCES 
 
Q13. Consider a collection of N two-level systems in thermal equilibrium at temperature T .
Each system has only two states: a ground state of energy 0 and excited state of energy
E . What is probability that system will be found in the excited state.
1 e E/kT 1 eE/kT
(a) (b) (c) (d)
1  e E/ kT 1  e E/kT 1  e  E/kT 1  e E/kT
Q14. For a free electron gas in two dimensions the variations of the density of states. N E  as a
function of energy E , is best represented by
(a) (b)

N E  N E 

E E

(c) (d)

N E  N E 

E E

Q15. If 6 fermions of spin half have to be adjusted in two dimensional harmonic isotropic
oscillator with angular frequency  the energy of ground state configuration is given by

(a) (b)  (c) 6 (d) 10
2

                     
Head office  Branch office 
 
fiziks, H.No. 23, G.F, Jia Sarai,  Anand Institute of Mathematics, 
 
Near IIT, Hauz Khas, New Delhi‐16  28‐B/6, Jia Sarai, Near IIT 
 
Phone: 011‐26865455/+91‐9871145498 Hauz Khas, New Delhi‐16 
              
                   Website: www.physicsbyfiziks.com                        
                           Email: fiziks.physics@gmail.com                       221 
fiziks
Institute for NET/JRF, GATE, IIT‐JAM, JEST, TIFR and GRE in PHYSICAL SCIENCES 
 
NAT (Numerical Answer Type)
1
Q16. 6 spin fermions and 6 boson of spin 1 confined in one dimensional box of width a
2
the ratio of ground state energy configuration of fermions to ground state bosons
are ………
Q17. Consider a linear collection of N independent spin 1/ 2 particles, each at a fixed location.
The entropy of this system is ( k B is the Boltzmann constant) Nk B then value  is
given by ……..
Q18. Consider a system of 3 fermions which can occupy any of the 4 available energy states
with equal probability. The entropy of the system is  k B ln  where value of

 is ………..and  ………..
Q19. For three dimensional system, the energy density is proportional by E then value of
 is given by …………..
Q20. If fermions are confined in three dimensional energy if energy per particle is  EF at

temperature T  0 K the value of  ........ if EF is Fermi energy at temperature T  00 K

Q21. For photon gas in three dimension no density n is proportional to T  where T is


temperature at Kelvin then the value of  is given by ............
Q22. For photon gas in three dimension energy E is proportional to T  at constant V where
T is temperature at Kelvin then the value of  is given by ............
Q23. For photon gas in three dimension energy E per particle is proportional to T  at
constant V where T is temperature at Kelvin then the value of  is given by ............
Q24. For one dimensional classical harmonic oscillator total energy per particle at equilibrium

temperature T is given by kBT then value of  is ………..
2

                     
Head office  Branch office 
 
fiziks, H.No. 23, G.F, Jia Sarai,  Anand Institute of Mathematics, 
 
Near IIT, Hauz Khas, New Delhi‐16  28‐B/6, Jia Sarai, Near IIT 
 
Phone: 011‐26865455/+91‐9871145498 Hauz Khas, New Delhi‐16 
              
                   Website: www.physicsbyfiziks.com                        
                           Email: fiziks.physics@gmail.com                       222 
fiziks
Institute for NET/JRF, GATE, IIT‐JAM, JEST, TIFR and GRE in PHYSICAL SCIENCES 
 
MSQ (Multiple Select Questions)
Q25. Which of the following statements is correct ?
(a) Indistinguishable particles obey Maxwell-Boltzmann statistics
(b) All particles of an ideal Bose gas occupy a single energy state at T = 0
(c) The integral spin particles obey Bose-Einstein statistics
(d) Protons obey Fermi-Dirac statistics
Q26. Which of the following statements is/are correct ?
(a) Distinguishable particles obey Maxwell-Boltzmann statistics
(b) In any quantum state two fermions can be adjusted
(c) The half integral spin particles obey Fermi Dirac statistics
(d) Photon obey Bose Einstein statistics
Q27. Which of the following is correct
(a) fermions and bosons are indistinguishable particle
(b) Electrons are fermions while protons are bosons
(c) Electrons are fermions while photons are bosons
(d) protons are fermions while photons are bosons
Q28. The chemical potential of an ideal Bose gas at any temperature is
(a) necessarily negative (b) zero
(c) necessarily positive (d) negative
3
Q29. If six fermions of spin have to adjusted in one dimensional quantum mechanical
2
harmonic oscillator then which of the following is correct .
(a) In one particular energy state four fermions can be adjusted
(b) In one particular quantum state only one fermions can be adjusted.

(c) The energy of ground state configuration is .
2
(d) The energy of ground state configuration is 5

                     
Head office  Branch office 
 
fiziks, H.No. 23, G.F, Jia Sarai,  Anand Institute of Mathematics, 
 
Near IIT, Hauz Khas, New Delhi‐16  28‐B/6, Jia Sarai, Near IIT 
 
Phone: 011‐26865455/+91‐9871145498 Hauz Khas, New Delhi‐16 
              
                   Website: www.physicsbyfiziks.com                        
                           Email: fiziks.physics@gmail.com                       223 
fiziks
Institute for NET/JRF, GATE, IIT‐JAM, JEST, TIFR and GRE in PHYSICAL SCIENCES 
 
Q30. Which one of the following is correct
(a) In Micro canonical ensemble the volume and temperature are constant but energy is
allow to exchange
(b) In canonical ensemble the volume and temperature and number of particle are
constant but energy is allow to exchange
(c) In canonical ensemble the volume and temperature is constant but energy and
number of particle are allow to exchange
(d) In Grand canonical ensemble the volume and temperature are constant but energy
and number of Particles are allow to exchange
Q31. Which one of the following system has average energy particle per particle is k BT where
T is equilibrium temperature.
(a) Two dimensional classical free particle .
(b) Classical particle confined in one dimensional Harmonic oscillator.
(c) Photon confined in one dimensional Box of length L
(d) Photon confined in two dimensional Box of of area A
Q32. Which of the following is correct if N particle confine in harmonic oscillator potential
m 2 x 2
defined as V ( x)  at equilibrium temperature T
2
(a) for classical particle average energy is Nk BT

   
(b) For quantum mechanical particles average energy is U  N    / k B T
 2 e  1 

   
(c) For quantum mechanical particles average energy is U  N    / k B T
 2 e  1 
 
(d) For quantum mechanical particle average energy is U  N coth
2 2  

                     
Head office  Branch office 
 
fiziks, H.No. 23, G.F, Jia Sarai,  Anand Institute of Mathematics, 
 
Near IIT, Hauz Khas, New Delhi‐16  28‐B/6, Jia Sarai, Near IIT 
 
Phone: 011‐26865455/+91‐9871145498 Hauz Khas, New Delhi‐16 
              
                   Website: www.physicsbyfiziks.com                        
                           Email: fiziks.physics@gmail.com                       224 
fiziks
Institute for NET/JRF, GATE, IIT‐JAM, JEST, TIFR and GRE in PHYSICAL SCIENCES 
 
Solutions
MCQ (Multiple Choice Questions)
Ans. 1: (d)
3
Solution: If spin of fermion then no of microstate is 2s  1 put s  so no of microstate is 4
2
Ans. 2: (b)
N ni
Solution: W   gi N  2, n1  1, n2  1 and g1  1 , g 2  1 So W  2
i ni

Ans. 3: (d)
N ni
Solution: W   gi N  3, n1  1, n2  2 and g1  1 , g 2  2  So W  12
i ni

Ans. 4: (a)
ni  gi  1
Solution: W   N  2, n1  1, n2  1 and g1  1 , g 2  1 So W  1
i ni gi  1

Ans. 5: (b)
ni  gi  1
Solution: W   N  3, n1  1, n2  2 and g1  1 , g 2  2 W 3
i ni gi  1

Ans. 6: (a)
gi
Solution: W   N  2, n1  1, n2  1 and g1  1 , g 2  1 So W  1
i gi  ni ni

Ans. 7: (a)
gi
Solution: W   N  3, n1  1, n2  2 and g1  1 , g 2  2 W 1
i gi  ni ni

Ans. 8: (c)
 2 2
Solution: E  2  E0  2  4 E0  10 E0 where E0 
2ma 2

                     
Head office  Branch office 
 
fiziks, H.No. 23, G.F, Jia Sarai,  Anand Institute of Mathematics, 
 
Near IIT, Hauz Khas, New Delhi‐16  28‐B/6, Jia Sarai, Near IIT 
 
Phone: 011‐26865455/+91‐9871145498 Hauz Khas, New Delhi‐16 
              
                   Website: www.physicsbyfiziks.com                        
                           Email: fiziks.physics@gmail.com                       225 
fiziks
Institute for NET/JRF, GATE, IIT‐JAM, JEST, TIFR and GRE in PHYSICAL SCIENCES 
 
Ans. 9: (c)
EF
A2m f E   1 if E  E F
Solution: n  g(E)f (E)dE , g(E)dE 
0
h2
dE , at T  0
0 if E  E F

2mE F mE F
n 
h2 2 2
Ans. 10: (b)
Solution: Partition function Z  4e / kT  2e 2/ kT
2e 2/ kT 1
P2   / kT  2/ kT

4e  2e 1  2e/ kT
Ans. 11: (d)
1
Solution: E F  mV F2  E F  n 2 / 3  V F2  n 2 / 3  VF  n1 / 3 .
2
Ans. 12: (c)
Solution: E1  0, E 2   , E3  2 ; g1  1, g 2  2, g 3  1 where g1 , g 2 and g 3 are degeneracy.

The partition function Z  g 1e   E1  g 2 e   E2  g 3 e   E3  1  2e    e   2  1  e   


2

Ans. 13: (a)


e  E / kT 1
Solution:  E / kT

1 e 1  e E / kT
Ans. 14: (c)
Ans. 15: (d)
Solution: nth energy state have n  1 degeneracy and due to spin s particle the total degeneracy
g  (n  1)   2s  1 according to Pauli exclusion principle maximum no of fermions are

no of degeneracy .
So energy of ground state configuration is 2    4  2  10

                     
Head office  Branch office 
 
fiziks, H.No. 23, G.F, Jia Sarai,  Anand Institute of Mathematics, 
 
Near IIT, Hauz Khas, New Delhi‐16  28‐B/6, Jia Sarai, Near IIT 
 
Phone: 011‐26865455/+91‐9871145498 Hauz Khas, New Delhi‐16 
              
                   Website: www.physicsbyfiziks.com                        
                           Email: fiziks.physics@gmail.com                       226 
fiziks
Institute for NET/JRF, GATE, IIT‐JAM, JEST, TIFR and GRE in PHYSICAL SCIENCES 
 
NAT (Numerical Answer Type)
Ans. 16: 4.66
 2 2
6 2  2
Solution: Energy of bosons are Eboson  6  all bosons are in ground state
2ma 2 2ma 2
1
Fermions obey Pauli exclusion principle if spin of fermions are then maximum
2
2 fermions will adjusted in one energy state
 2 2 4 2  2 9 2  2 28 2  2
E fermoins  2   2  2  
2ma 2 2ma 2 2ma 2 2ma 2
E fermions 28
  4.66
Eboson 6

Ans. 17: 0.693


Solution: No of microstate is 2 so entropy is Nk B ln 2

Ans. 18:   2,   2
Solution: Number of ways that 3 fermions will adjust in 4 available energy is 4
C 3  4 so

entropy is k B ln 4 = 2k B ln 2
Ans. 19: 0.5
Solution: For three dimensional system g ( E )  E1/ 2 so   .5
U 3
Ans. 20:  EF   0.6
N 5
Ans. 21:  3
Solution: Photon in three dimensional N  VT 3
Ans. 22:  4
Solution: Photon in three dimensional E  VT 4
Ans. 23:  1
E
Solution: Photon in three dimensional N  VT 3 , E  VT 4  T
N
                     
Head office  Branch office 
 
fiziks, H.No. 23, G.F, Jia Sarai,  Anand Institute of Mathematics, 
 
Near IIT, Hauz Khas, New Delhi‐16  28‐B/6, Jia Sarai, Near IIT 
 
Phone: 011‐26865455/+91‐9871145498 Hauz Khas, New Delhi‐16 
              
                   Website: www.physicsbyfiziks.com                        
                           Email: fiziks.physics@gmail.com                       227 
fiziks
Institute for NET/JRF, GATE, IIT‐JAM, JEST, TIFR and GRE in PHYSICAL SCIENCES 
 
Ans. 24:  2
Solution: The partition function of one dimensional harmonic oscillator at equilibrium
2 kBT
temperature T is Z
h
 ln Z
The internal energy U  kBT 2  k BT
T
MSQ (Multiple Select Questions)
Ans. 25: (b), (c) and (d)
Solution: Distinguishable particles obey Maxwell-Boltzmann statistics
Ans. 26: (a), (c) and (d)
Solution: In any quantum state only one fermions can be adjusted
Ans. 27: (a), (c) and (d)
Solution: Fermions and bosons are indistinguishable ,electrons and protons are fermions but
photons are bosons .
Ans. 28: (b) and (d)
Solution: The chemical potential of an ideal Bose gas at any temperature either zero or
negative.(zero in case of photon).
Ans. 29: (a), (b) and (d)
3 3 1 1 3
Solution: If fermions have spin then sz   ,  , , .
2 2 2 2 2
Due to Pauli exclusion principle only one fermions can be adjusted in one quantum sate
n, s, sz

so degeneracy of one particular energy state is 4 ,so 4 out of 6 fermions will adjust in
ground state and rest two will in first excited state .
 3
so energy is 4   2  5
2 2
Ans. 30: (b) and (d) 

                     
Head office  Branch office 
 
fiziks, H.No. 23, G.F, Jia Sarai,  Anand Institute of Mathematics, 
 
Near IIT, Hauz Khas, New Delhi‐16  28‐B/6, Jia Sarai, Near IIT 
 
Phone: 011‐26865455/+91‐9871145498 Hauz Khas, New Delhi‐16 
              
                   Website: www.physicsbyfiziks.com                        
                           Email: fiziks.physics@gmail.com                       228 
fiziks
Institute for NET/JRF, GATE, IIT‐JAM, JEST, TIFR and GRE in PHYSICAL SCIENCES 
 
Ans. 31: (a), (b) and (c)
2 mkBT
Solution: (a)Two dimensional free particle has partition function Z ,
h2
 ln Z
U  kBT 2  k BT
T
(b) The partition function of one dimensional harmonic oscillator at equilibrium
2 kBT  ln Z
temperature T is Z  The internal energy U  kBT 2  k BT
h T
Lk BT  ln Z
(c) the partition function for photon in one dimension is Z  U  kBT 2  k BT
hc T
2
k T   ln Z
(d) the partition function for two dimension box Z  A  B  U  k BT 2  2kBT
 hc  T
Ans. 32: (a), (b) and (d)

                     
Head office  Branch office 
 
fiziks, H.No. 23, G.F, Jia Sarai,  Anand Institute of Mathematics, 
 
Near IIT, Hauz Khas, New Delhi‐16  28‐B/6, Jia Sarai, Near IIT 
 
Phone: 011‐26865455/+91‐9871145498 Hauz Khas, New Delhi‐16 
              
                   Website: www.physicsbyfiziks.com                        
                           Email: fiziks.physics@gmail.com                       229 
fiziks
Institute for NET/JRF, GATE, IIT‐JAM, JEST, TIFR and GRE in PHYSICAL SCIENCES 
 
6. Basic Nuclear Properties
An ordinary hydrogen atom has as its nucleus a single proton, whose charge is +e and
whose mass is 1836 times that of the electron. All other elements have nuclei that contain
neutrons as well as protons. As its name suggests, the neutron is uncharged; its mass is
slightly greater than that of the proton. Neutrons and protons are jointly called nucleons.
The atomic number of an element is the number of
protons in each of its nuclei, which is the same as the number of electrons in a neutral
atom of the element. Thus atomic number of hydrogen is 1, of helium 2, of lithium 3, and
of uranium 92. All nuclei of a given element do not necessarily have equal numbers of
neutrons. For instance, although over 99.9 percent of hydrogen nuclei are just single
protons, a few also contain a neutron, and a very few two neutrons, along with the
protons. The varieties of an element that differ in the numbers of neutrons their nuclei
contain are called isotopes.
Ordinary Hydrogen Deuterium Tritium

Proton Neutron Electron


Figure: The isotope of hydrogen
A
The conventional symbols for nuclear species, or nuclides, follow the pattern Z X ,
where
X  Chemical symbol of the element
Z  Atomic number of the element
 Number of protons in the nucleus
A  Mass number of the nuclide
 Number of nucleons in the nucleus

                     
Head office  Branch office 
 
fiziks, H.No. 23, G.F, Jia Sarai,  Anand Institute of Mathematics, 
 
Near IIT, Hauz Khas, New Delhi‐16  28‐B/6, Jia Sarai, Near IIT 
 
Phone: 011‐26865455/+91‐9871145498 Hauz Khas, New Delhi‐16 
              
                   Website: www.physicsbyfiziks.com                        
                           Email: fiziks.physics@gmail.com                       230 
fiziks
Institute for NET/JRF, GATE, IIT‐JAM, JEST, TIFR and GRE in PHYSICAL SCIENCES 
 
Nuclear Terminology
 Isotopes
If two nuclei have same atomic number Z (proton), then they are called as isotopes.
13 14 16 17
Example: 6 C& 6 C, 8 O& 8 O and 11 H, 12 H, 13 H
 Isotones
If two nuclei have same neutron number N (proton), then they are called as isotones.
13 14
Example: 6 C and 7 N
 Isobars
If two nuclei have same mass number A, then they are called as isobars.
14 14
Example: 6 C and 7 N
 Mirror nuclei
Nuclei with same mass number A but with proton and neutron number interchanged and
their difference is 1 .
Example: 11 11
6 C & 5 B and
13
7 N & 13
6 C

Atomic masses: Atomic masses refer to the masses of neutral atoms, not of bare nuclei.
Thus an atomic mass always includes the masses of Z electrons. Atomic masses are
12
expressed in mass units (u), which are so defined that the mass of a 6 C atom is exactly

12u. The value of mass unit is 1u  1.66054  10 27 kg  931.4 MeV .

Some rest masses in various units are:

Particle Mass(kg) Mass(u) Mass(MeV/c2)

Proton 1.6726 × 10-27 1.007276 938.28

Neutron 1.6750 × 10-27 1.008665 939.57

Electron 9.1095 × 10-31 5.486 × 10-4 0.511

1
1H 1.6736 × 10-27 1.007825 938.79

                     
Head office  Branch office 
 
fiziks, H.No. 23, G.F, Jia Sarai,  Anand Institute of Mathematics, 
 
Near IIT, Hauz Khas, New Delhi‐16  28‐B/6, Jia Sarai, Near IIT 
 
Phone: 011‐26865455/+91‐9871145498 Hauz Khas, New Delhi‐16 
              
                   Website: www.physicsbyfiziks.com                        
                           Email: fiziks.physics@gmail.com                       231 
fiziks
Institute for NET/JRF, GATE, IIT‐JAM, JEST, TIFR and GRE in PHYSICAL SCIENCES 
 
6.1 Size and Density
Majority of atomic nuclei have spherical shape and only very few show departure from
spherical symmetry. For spherically symmetrical nuclei, nuclear radius is given by
R  R0 A1/ 3

where A is the mass number and R0  1.2  0.1  1015 m  1.2 fm .

R varies slightly from one nucleus to another but is roughly constant for A  20 .
12
The radius of 6 C nucleus is

R  1.2 12 
1/ 3
 2.7 fm

Example: The radius of Ge nucleus is measured to be twice the radius of 94 Be . How

many nucleons are there in Ge nucleus?

Solution: R  Ro  A   RGe  2 RBe  Ro  A   2 Ro  9 


1/ 3 1/ 3 1/ 3
 A  72

Nuclear Density
Assuming spherical symmetry, volume of nucleus is given by
4 3  4 R 3 A
V  R 0 .
3 3

Mass of one proton = 1.67 × 10-27 kg, Nuclear Mass = A × 1.67 × 10-27 kg.

A 1.67 1027
Nuclear density   1017 kg / m3
4 3
R 0  A
3

Nuclear Mass Density 1017 Kg/m3


Nuclear Particle Density = 
Nuclear Mass 1.67 1027 Kg/Nucleon

= 1044 Nucleons/m3

                     
Head office  Branch office 
 
fiziks, H.No. 23, G.F, Jia Sarai,  Anand Institute of Mathematics, 
 
Near IIT, Hauz Khas, New Delhi‐16  28‐B/6, Jia Sarai, Near IIT 
 
Phone: 011‐26865455/+91‐9871145498 Hauz Khas, New Delhi‐16 
              
                   Website: www.physicsbyfiziks.com                        
                           Email: fiziks.physics@gmail.com                       232 
fiziks
Institute for NET/JRF, GATE, IIT‐JAM, JEST, TIFR and GRE in PHYSICAL SCIENCES 
 
6.2 Spin and Magnetic Moment

1
Proton and neutrons, like electrons, are fermions with spin quantum numbers of s  .
2

This means they have spin angular momenta S of magnitude

11  3
S  ss  1    1  
22  2

1
and spin magnetic quantum number of m s   .
2

As in the case of electrons, magnetic moments are associated with the spins of protons
and neutrons. In nuclear physics magnetic moments are expressed in nuclear
magnetons  N  , where

e
Nuclear magneton  N   5.051  10 27 J / T  3.152  10 8 eV / T where m p is the
2m p

proton mass.

e
In atomic physics we have defined Bohr magneton  B  where me is the electron
2me
mass.

The nuclear magneton is smaller than the Bohr magneton by the ratio of the proton mass
to the electron mass which is 1836.

B mp
  1836 .
 N me

The spin magnetic moments of the proton and neutron have components in any direction
of

Proton  pz  2.793 N

Neutron  nz  1.913 N

                     
Head office  Branch office 
 
fiziks, H.No. 23, G.F, Jia Sarai,  Anand Institute of Mathematics, 
 
Near IIT, Hauz Khas, New Delhi‐16  28‐B/6, Jia Sarai, Near IIT 
 
Phone: 011‐26865455/+91‐9871145498 Hauz Khas, New Delhi‐16 
              
                   Website: www.physicsbyfiziks.com                        
                           Email: fiziks.physics@gmail.com                       233 
fiziks
Institute for NET/JRF, GATE, IIT‐JAM, JEST, TIFR and GRE in PHYSICAL SCIENCES 
 
There are two possibilities for the signs of  pz and  nz , depending on whether ms is

1 1
 or  . The  sign is used for  pz because  pz is in the same direction as the spin S ,
2 2

whereas  is used for  nz because  nz is opposite to spin S .

S S
p

n

Figure: Spin magnetic moment (  ) and spin angular momentum ( S )


directions for neutron and protons.
Note: For neutron, magnetic moment is expected to be zero as e  0 but  nz  1.913 N .

At first glance it seems odd that the neutron, with no net charge, has spin magnetic
moment. But if we assume that the neutron contains equal amounts of positive and
negative charge, a spin magnetic moment arise if these charges are not uniformly
distributed. Thus we can say that neutron has physical significance of negative charges
because magnetic moment is opposite to that of its intrinsic spin angular momentum.

6.3 Angular Momentum of Nucleus

The hydrogen nucleus 11 H consists of a single proton and its total angular momentum is

3
given by S   . A nucleon in a more complex nucleus may have orbital angular
2
momentum due to motion inside the nucleus as well as spin angular momentum. The total
angular momentum of such a nucleus is the vector sum of the spin and orbital angular
momenta of its nucleons, as in the analogous case of the electrons of an atom.

When a nucleus whose magnetic moment has z component  z is in a constant magnetic

field B , the magnetic potential energy of the nucleus is


                     
Head office  Branch office 
 
fiziks, H.No. 23, G.F, Jia Sarai,  Anand Institute of Mathematics, 
 
Near IIT, Hauz Khas, New Delhi‐16  28‐B/6, Jia Sarai, Near IIT 
 
Phone: 011‐26865455/+91‐9871145498 Hauz Khas, New Delhi‐16 
              
                   Website: www.physicsbyfiziks.com                        
                           Email: fiziks.physics@gmail.com                       234 
fiziks
Institute for NET/JRF, GATE, IIT‐JAM, JEST, TIFR and GRE in PHYSICAL SCIENCES 
 
Magnetic energy U m   z B

The energy is negative when  z is in the same direction as B and positive when  z is

opposite to B . In a magnetic field, each angular momentum state of the nucleus is


therefore split into components, just as in the Zeeman Effect in atomic electron states.
Figure below shows the splitting when the angular momentum of the nucleus is due to the
spin of a single proton. Spin down
1 E 0   pz B
ms  
2
E0  E  2  pz B
Spin up
E 0   pz B
1
ms  
2
B 0 B 0

Figure: The energy levels of a proton in a magnetic field are split into spin-up
and spin-down sublevels.
The energy difference between the sublevels is E  2  pz B

A photon with this energy will be emitted when a proton in the upper state flips its spin to
fall to the lower state. A proton in the lower state can be raised to upper one by absorbing
a photon of this energy. The photon frequency  L that corresponds to E is

E 2  pz B
Larmor frequency for photons L  
h h

This is equal to the frequency with which a magnetic dipole precesses around a magnetic
field.

                     
Head office  Branch office 
 
fiziks, H.No. 23, G.F, Jia Sarai,  Anand Institute of Mathematics, 
 
Near IIT, Hauz Khas, New Delhi‐16  28‐B/6, Jia Sarai, Near IIT 
 
Phone: 011‐26865455/+91‐9871145498 Hauz Khas, New Delhi‐16 
              
                   Website: www.physicsbyfiziks.com                        
                           Email: fiziks.physics@gmail.com                       235 
fiziks
Institute for NET/JRF, GATE, IIT‐JAM, JEST, TIFR and GRE in PHYSICAL SCIENCES 
 
6.4 Stable Nuclei

Not all combination of neutrons and protons


Stable nuclei

Neutron number (N)


form stable nuclei. In general, light nuclei
 A  20 contain equal numbers of neutrons
N=Z line
and protons, while in heavier nuclei the
proportion of neutrons becomes
progressively greater. This is evident in
figure as shown below, which is plot of N
versus Z for stable nuclides. Proton number (Z)
Figure: Neutron-proton diagram for
The tendency for N to equal Z follows from
stable nuclides.
the existence of nuclear energy levels.
Nucleons, which have spin ½, obey exclusion principle. As a result, each energy level
can contain two neutrons of opposite spins and two protons of opposite spins. Energy
levels in nuclei are filled in sequence, just as energy levels in atoms are, to achieve
configurations of minimum energy and therefore maximum stability. Thus the boron
12
isotope 5 B has more energy than the carbon isotope 12
6 C because one of its neutrons is in

12
a higher energy level, and 5 B is accordingly unstable. If created in a nuclear reaction, a
12 12
5 B nucleus changes by beta decay into a stable 6 C nucleus in a fraction of second.

Proton Neutron

Stable Stable Unstable Stable Stable

Energy

10
5 B 11
5 B  B
12
5
12
6C 13
6 C
Figure: Simplified energy level diagrams of some boron and carbon isotopes.

                     
Head office  Branch office 
 
fiziks, H.No. 23, G.F, Jia Sarai,  Anand Institute of Mathematics, 
 
Near IIT, Hauz Khas, New Delhi‐16  28‐B/6, Jia Sarai, Near IIT 
 
Phone: 011‐26865455/+91‐9871145498 Hauz Khas, New Delhi‐16 
              
                   Website: www.physicsbyfiziks.com                        
                           Email: fiziks.physics@gmail.com                       236 
fiziks
Institute for NET/JRF, GATE, IIT‐JAM, JEST, TIFR and GRE in PHYSICAL SCIENCES 
 
The preceding argument is only part of the story. Protons are positively charged and repel
one another electrically. This repulsion becomes so great in nuclei with more than 10
protons or so that an excess of neutrons, which produce only attractive forces is required
for stability. Thus the curve departs more and more from N  Z line as Z increases.

Sixty percent of stable nuclides have both even Z and even N;


these are called “even-even” nuclides. Nearly all the others have either even Z and odd N
(“even-odd” nuclides) or odd Z and even N (“odd-even” nuclides) with the numbers of
both kinds being about equal. Only five stable odd-odd nuclides are known:
2
1 H, 36 Li, 10
5 B, 14
7 N and 180
73 Ta . Nuclear abundances follow a similar pattern of favoring

even numbers for Z and N.

These observations are consistent with the presence of nuclear energy levels that can
each contain two particles of opposite spin. Nuclei with filled levels have less tendency to
pick up other nucleons than those with partially filled levels and hence were less likely to
participate in the nuclear reactions involved in the formation of elements.

Nuclear forces are limited in range, and as a result nucleons interact strongly only with
their nearest neighbors. This effect is referred to as the saturation of nuclear forces.
Because the coulomb repulsion of protons is appreciable throughout the entire nucleus,
there is a limit to the ability of neutrons to prevent the disruption of large nucleus. This
209
limit is represented by the bismuth isotope 83 Bi , which is the heaviest stable nuclide.

All nuclei with Z  83 and A  209 spontaneously transform themselves lighter ones
through the emission of one or more alpha particles, which are 42 He nuclei:

Alpha decay A
Z X  AZ24 Y  42 He

Since an alpha particle consists of two protons and two neutrons, an alpha decay reduces
the Z and N of the original nucleus by two each. If the resulting daughter nucleus has
either too small or too large a neutron/proton ratio for stability, it may beta-decay to a
more appropriate configuration.

                     
Head office  Branch office 
 
fiziks, H.No. 23, G.F, Jia Sarai,  Anand Institute of Mathematics, 
 
Near IIT, Hauz Khas, New Delhi‐16  28‐B/6, Jia Sarai, Near IIT 
 
Phone: 011‐26865455/+91‐9871145498 Hauz Khas, New Delhi‐16 
              
                   Website: www.physicsbyfiziks.com                        
                           Email: fiziks.physics@gmail.com                       237 
fiziks
Institute for NET/JRF, GATE, IIT‐JAM, JEST, TIFR and GRE in PHYSICAL SCIENCES 
 
In negative beta decay, a neutron is transformed into a proton and an electron is emitted:

Negative beta decay n0  p   e

In positive beta decay, a proton becomes a neutron and a positron is emitted:

Positron emission p   n0  e

Thus negative beta decay decreases the proportion of neutrons and positive beta decay
increases it. A process that competes with positron emission is the capture by a nucleus of
an electron from its innermost shell. The electron is absorbed by a nuclear proton which
is thereby transformed into neutron:

Electron Capture p   e  n0

Figure below shows how alpha and beta decays enable stability to be achieved.

Alpha decay
N decreases by 2
Z decreases by 2
Neutron number (N)

Stability curve
Positive beta decay or
electron capture
Negative beta decay
Z decreases by 1
N decreases by 1 N increases by 1
Z increases by 1

Proton number (Z)

Figure: Alpha and beta decays permit an unstable nucleus to reach a stable configuration.

                     
Head office  Branch office 
 
fiziks, H.No. 23, G.F, Jia Sarai,  Anand Institute of Mathematics, 
 
Near IIT, Hauz Khas, New Delhi‐16  28‐B/6, Jia Sarai, Near IIT 
 
Phone: 011‐26865455/+91‐9871145498 Hauz Khas, New Delhi‐16 
              
                   Website: www.physicsbyfiziks.com                        
                           Email: fiziks.physics@gmail.com                       238 
fiziks
Institute for NET/JRF, GATE, IIT‐JAM, JEST, TIFR and GRE in PHYSICAL SCIENCES 
 
6.5 Binding Energy

When nuclear masses are measured, it is found that they are less than the sum of the
masses of the neutrons and protons of which they are composed. This is in agreement
with Einstein’s theory of relativity, according to which the mass of a system bound by
energy B is less than the mass of its constituents by B / c 2 (where c is the velocity of
light).

The Binding energy B of a nucleus is defined as the difference between the energy of the
constituent particles and of the whole nucleus. For a nucleus of atom ZA X ,

B   ZM p  NM N – ZA M  c 2  [ ZM H  NM N – M ( ZA X )]c 2

If mass is expressed in atomic mass unit

B   ZM p  NM N – ZA M   931.5 MeV  [ ZM H  NM N – M ( ZA X )]  931.5 MeV

M p : Mass of free proton, M N : MN: Mass of free neutron,

M H : mass of hydrogen atom A


Z M : mass of the nucleus,

Z : Number of proton, N : Number of neutron,

M  ZA X  : mass of atom.

6.5.1 Binding Energy per Nucleon


The binding energy per nucleon for a given nucleus is found by dividing its total
binding energy by the number of nucleon it contains. Thus binding energy per nucleon is

B c2 c2
  ZM p  NM N – ZA M   [ ZM H  NM N – M ( ZA X )]
A A A

2.224
The binding energy per nucleon for 12 H is  1.112 MeV / nucleon and for 209
63 Bi it
2
1640 MeV
is  7.8 MeV / nucleon .
209
                     
Head office  Branch office 
 
fiziks, H.No. 23, G.F, Jia Sarai,  Anand Institute of Mathematics, 
 
Near IIT, Hauz Khas, New Delhi‐16  28‐B/6, Jia Sarai, Near IIT 
 
Phone: 011‐26865455/+91‐9871145498 Hauz Khas, New Delhi‐16 
              
                   Website: www.physicsbyfiziks.com                        
                           Email: fiziks.physics@gmail.com                       239 
fiziks
Institute for NET/JRF, GATE, IIT‐JAM, JEST, TIFR and GRE in PHYSICAL SCIENCES 
 
Figure below shows the binding energy per nucleon against the number of nucleons in
various atomic nuclei.
Fusion Fission

  E   E
Binding energy per nucleon, MeV

56
26 Fe

Mass number, A

Figure: Binding energy per nucleon as function of mass number.

The greater the binding energy per nucleon, the more stable the nucleus is. The graph has
the maximum of 8.8 MeV / nucleon when the number of nucleons is 56. The nucleus that
56
has 56 protons and neutrons is 26 Fe an iron isotope. This is the most stable nucleus of

them all, since the most energy is needed to pull a nucleon away from it.
Two remarkable conclusions can be drawn from the above graph.
(i) If we can somehow split a heavy nucleus into two medium sized ones, each of the new
nuclei will have more binding energy per nucleon than the original nucleus did. The extra
235
energy will be given off, and it can be a lot. For instance, if the uranium nucleus 92 U is

broken into two smaller nuclei, the binding energy difference per nucleon is about
0.8 MeV. The total energy given off is therefore
 MeV 
 0.8   235 nucleon   188 MeV
 nucleon 
This process is called as nuclear fission.

                     
Head office  Branch office 
 
fiziks, H.No. 23, G.F, Jia Sarai,  Anand Institute of Mathematics, 
 
Near IIT, Hauz Khas, New Delhi‐16  28‐B/6, Jia Sarai, Near IIT 
 
Phone: 011‐26865455/+91‐9871145498 Hauz Khas, New Delhi‐16 
              
                   Website: www.physicsbyfiziks.com                        
                           Email: fiziks.physics@gmail.com                       240 
fiziks
Institute for NET/JRF, GATE, IIT‐JAM, JEST, TIFR and GRE in PHYSICAL SCIENCES 
 
(ii) If we can somehow join two light nuclei together to give a single nucleus of medium
size also means more binding energy per nucleon in the new nucleus. For instance, if two
2
1 H deuterium nuclei combine to form a 42 He helium nucleus, over 23 MeV is released.

Such a process, called nuclear fusion, is also very effective way to obtain energy. In fact,
nuclear fusion is the main energy source of the sun and other stars.

Example: The measured mass of deuteron atom  12 H  , Hydrogen atom  11 H  , proton and

neutron is 2.01649 u , 1.00782 u , 1.00727 u and 1.00866 u . Find the binding energy of
the deuteron nucleus (unit MeV / nucleon ).

Solution: Here A  2, Z  1, N  1

B.E.  [ ZM H  NM N – M (12 H )]  931.5 MeV


 [1 1.00782  1 1.00866  2.01649]  931.5 MeV
 [0.00238]  931.5 MeV  2.224 MeV
20
Example: The binding energy of the neon isotope 10 Ne is 160.647 MeV. Find its atomic

mass.
Solution: Here A  10, Z  10, N  10
B
M ( AZ X)   ZM H  NM N  -
931.5 MeV / u
160.647
20
M (10 Ne)  10 1.00782   10 1.00866     19.992 u
931.5 MeV / u
Example:
(a) Find the energy needed to remove a neutron from the nucleus of the calcium
42
isotope 20 Ca .
(b) Find the energy needed to remove a proton from this nucleus.
(c) Why are these energies different?

Given: atomic masses of 20 Ca  41.958622 u , 20 Ca  40.962278 u , K  40.961825 u ,


42 41 41
19

and mass of 0 n  1.008665 u , 1 p  1.007276 u .


1 1

                     
Head office  Branch office 
 
fiziks, H.No. 23, G.F, Jia Sarai,  Anand Institute of Mathematics, 
 
Near IIT, Hauz Khas, New Delhi‐16  28‐B/6, Jia Sarai, Near IIT 
 
Phone: 011‐26865455/+91‐9871145498 Hauz Khas, New Delhi‐16 
              
                   Website: www.physicsbyfiziks.com                        
                           Email: fiziks.physics@gmail.com                       241 
fiziks
Institute for NET/JRF, GATE, IIT‐JAM, JEST, TIFR and GRE in PHYSICAL SCIENCES 
 
Solution:

(a)
42
20 Ca 41
20 Ca  0 n ;
1

20 Ca and 0 n  41.970943 u
Total mass of the 41 1

Mass defect m  41.970943  41.958622  0.012321 u


So, B.E. of missing neutron  m  931.5  11.48 MeV

(b) 42
20 Ca 19
41
K 11 p ;
41
Total mass of the 19 K and 11 p  41.919101 u

Mass defect m  41.919101  41.958622  0.010479 u


So, B.E. of missing protron  m  931.5  10.27 MeV

(c) The neutron was acted upon only by attractive nuclear forces whereas the proton was
also acted upon by repulsive electric forces that decrease its binding energy.

6.6 Salient Features of Nuclear Forces


Nucleus is bounded by nuclear forces. The basic properties of nuclear forces are
(i) It is a short range attractive force.
(ii) It is in general non-central force.
(iii) They have property of saturation i.e. each nucleon interacts only with its nearest
neighbors and not with all the constituents in the nucleus. This is apparent from the fact
that average B.E. per nuclear remains approximately constant i.e.
B.E.  A
A  A  1
When all interactions are possible then interaction may take place;
2
B.E .  A2 is not valid.
(iv) They are charge independent i.e. n  n, p  p and n  p have same nuclear force.
(v) They are spin dependent force as shown by deuteron.
(vi) They are exchange forces proposed by Yukawa(Meson Theory).

                     
Head office  Branch office 
 
fiziks, H.No. 23, G.F, Jia Sarai,  Anand Institute of Mathematics, 
 
Near IIT, Hauz Khas, New Delhi‐16  28‐B/6, Jia Sarai, Near IIT 
 
Phone: 011‐26865455/+91‐9871145498 Hauz Khas, New Delhi‐16 
              
                   Website: www.physicsbyfiziks.com                        
                           Email: fiziks.physics@gmail.com                       242 
fiziks
Institute for NET/JRF, GATE, IIT‐JAM, JEST, TIFR and GRE in PHYSICAL SCIENCES 
 
7. Radio Active Decay

Despite the strength of the forces that holds nucleons together to form an atomic nucleus,
many nuclides are unstable and spontaneously change into other nuclides by radioactive
decay.

Five kinds Example Reason for instability

A
Z X ZA42 Y  42 He
Alpha decay Nucleus is too large.
Emission of -particle reduces
size of nucleus.

A
Z X ZA1 Y  e 

Beta decay Emission of electron by neutron Nucleus has too many neutrons
in nucleus changes the neutron relative to number of protons.
to a proton.

A
Z X*  AZ X  

Gamma decay Emission of -ray reduces Nucleus has excess energy.


energy of the nucleus.
A
Z X  e  A
Y
Z 1

Electron capture Capture of electron by protons


Nucleus has too many protons
changes the proton to a neutron.
relative to number of neutrons.

A
Z X A
Y  e
Z 1

Positron emission Emission of positron by proton Nucleus has too many protons
in nucleus changes the proton to relative to number of neutrons.
a neutron.

                     
Head office  Branch office 
 
fiziks, H.No. 23, G.F, Jia Sarai,  Anand Institute of Mathematics, 
 
Near IIT, Hauz Khas, New Delhi‐16  28‐B/6, Jia Sarai, Near IIT 
 
Phone: 011‐26865455/+91‐9871145498 Hauz Khas, New Delhi‐16 
              
                   Website: www.physicsbyfiziks.com                        
                           Email: fiziks.physics@gmail.com                       243 
fiziks
Institute for NET/JRF, GATE, IIT‐JAM, JEST, TIFR and GRE in PHYSICAL SCIENCES 
 
7.1 Alpha Decay

Nuclei which contain 210 or more nucleons are so large that the short range nuclear
forces that hold them together are barely able to counterbalance the mutual repulsion of
their protons. Alpha decay occurs in such nuclei as a means of increasing their stability
by reducing their size
A
Z X  ZA 42 Y  42 He

To escape from nucleus, a particle must have K.E., and only the alpha particle mass is
sufficiently smaller than that of its constituent nucleons for such energy to be available
(-particle have high B.E. as compared to proton or 32 He nuclei).

The energy Q-released when various particles are emitted by a heavy nucleus is,

i.e.

Disintegration energy Q   mi  m f  mx  c 2 where

mi  Mass of initial nuclei,

m f  mass of final nuclei,

mx  -particle mass

The KE of the emitted -particle is never quite equal to Q, since momentum must be
conserved, the nucleus recoils with a small amount of kinetic energy when the -particle
emerges. Thus

 A4
KE   Q
 A 

since A  210 , most of the disintegration energy appears as the K.E. of the -particle.

                     
Head office  Branch office 
 
fiziks, H.No. 23, G.F, Jia Sarai,  Anand Institute of Mathematics, 
 
Near IIT, Hauz Khas, New Delhi‐16  28‐B/6, Jia Sarai, Near IIT 
 
Phone: 011‐26865455/+91‐9871145498 Hauz Khas, New Delhi‐16 
              
                   Website: www.physicsbyfiziks.com                        
                           Email: fiziks.physics@gmail.com                       244 
fiziks
Institute for NET/JRF, GATE, IIT‐JAM, JEST, TIFR and GRE in PHYSICAL SCIENCES 
 
7.1.1 Tunnel Theory of -decay: (How -particle can actually escape the nucleus)

The height of the potential barrier is  25 MeV , which is equal to the work that must be
done against the repulsive electric force to bring an -particle from infinity to a position
adjacent to the nucleus but just outside the range of its attractive forces.

We may therefore regard an  -particle in Energy


such a nucleus as being inside a box whose
2 Ze 2
U 
box requires energy of 25 MeV to be 4 0 r

E
surmounted. However, decay -particles have
energies that range from 4 to 9 MeV , 0 r
R0 2 Ze 2
R
depending on the particular nuclide involved, 4 0 E

16 to 21 MeV short of the energy needed for


escape.

Although  -decay is inexplicable classically, quantum mechanics provides a straight


forward explanation. The basic notions of this theory are:

An  -particle may exist as an entity within a heavy nucleus. Such a particle is in


constant motion and is held in the nucleus by potential barrier. There is a small but
definite-likelihood that the particle may tunnel through the barrier (despite its height)
each time a collision with it occurs.

The decay probability per unit time, i.e decay constant    T

Where  = number of times per second an -particle within a nucleus strikes the
potential barrier around it and

T  e 2k2 L = Probability that the particle will be transmitted through the barrier.

2m U  E 
L is the width of the barrier, wave number inside the barrier k2  , where

E is the K.E. , U is height of the barrier and m is the mass of -particle.

                     
Head office  Branch office 
 
fiziks, H.No. 23, G.F, Jia Sarai,  Anand Institute of Mathematics, 
 
Near IIT, Hauz Khas, New Delhi‐16  28‐B/6, Jia Sarai, Near IIT 
 
Phone: 011‐26865455/+91‐9871145498 Hauz Khas, New Delhi‐16 
              
                   Website: www.physicsbyfiziks.com                        
                           Email: fiziks.physics@gmail.com                       245 
fiziks
Institute for NET/JRF, GATE, IIT‐JAM, JEST, TIFR and GRE in PHYSICAL SCIENCES 
 
7.2 Beta Decay

It is a means whereby a nucleus can alter its composition to become more stable. The
conservation principles of energy, linear momentum and angular momentum are all
apparently violated in beta decay: n  p  e

(i) The electron energies observed in the


   decay 83 Bi 210 
 decay of a particular nuclide are found to

Relative no. of electrons


vary continuously from 0 to maximum value Energy equivalent of
KEmax characteristic of the nuclide. The mass lost by nucleus

maximum energy 
Emax  m0 c 2  KEmax 
carried by the decay electron is equal to the
energy equivalent of the mass difference
between the parent and daughter nuclei.
0 0.4 0.6 0.8 1.17 1.2
However an emitted electron is rarely found
Electron energy in MeV
with energy of KEmax .

(ii) When the directions of emitted electron and of the recoiling nuclei are observed, they
are almost never exactly opposite as required for linear momentum to be conserved.

(iii) The spins of the neutron, proton and electron are all ½. If beta decay involved just a
neutron becoming a proton and an electron, spin (and hence angular momentum) is not
conserved.

In 1930 Pauli proposed a "desperate remedy": If an uncharged particle of small or zero


rest mass and spin ½ is emitted in    decay together with the electron, the above
discrepancies would not occur. This particle is called neutrino which would carry off
energy equal to the difference between KEmax and actual K .E of the electron (the

recoiling nucleus carry away negligible K .E ). The neutrino's linear momentum also
exactly balances those of the electron and the recoiling daughter nucleus.

                     
Head office  Branch office 
 
fiziks, H.No. 23, G.F, Jia Sarai,  Anand Institute of Mathematics, 
 
Near IIT, Hauz Khas, New Delhi‐16  28‐B/6, Jia Sarai, Near IIT 
 
Phone: 011‐26865455/+91‐9871145498 Hauz Khas, New Delhi‐16 
              
                   Website: www.physicsbyfiziks.com                        
                           Email: fiziks.physics@gmail.com                       246 
fiziks
Institute for NET/JRF, GATE, IIT‐JAM, JEST, TIFR and GRE in PHYSICAL SCIENCES 
 
Thus in ordinary   decay n  p  e   (also possible outside the nucleus)
 

The interaction of neutrinos with matter is extremely feeble. The only interaction with
matter a neutrino can experience is through a process called inverse beta decay with
extremely low probability p    n  e  and n    p  e  .

Note: Parity violates in    decay.

7.2.1 Positron emission

It is the conversion of a nuclear proton into a neutron, a positron and a neutrino:

p  n  e  (Possible only within a nucleus)

7.2.2 Electron capture

It is closely connected with positron emission. In electron capture a nucleus absorbs one
of its inner atomic electron, with the result that a nuclear proton becomes a neutron and
neutrino is emitted:

p  e  n  .

Usually the absorbed electron comes from the K-shell, and an X-ray photon is emitted
when one of the atoms outer electrons falls into the resulting vacant state. The
wavelength of the photon will be one of those characteristic of daughter element, not of
the original one, and the process can be recognized on that basis.

Note:

1. Electron capture is competitive with positron emission since both processes lead to the
same nuclear transformation.

2. Electron capture occurs more often than positron emission in heavy nuclides because
the electrons in such nuclides are relatively close to the nucleus, which promotes their
interaction with it.

                     
Head office  Branch office 
 
fiziks, H.No. 23, G.F, Jia Sarai,  Anand Institute of Mathematics, 
 
Near IIT, Hauz Khas, New Delhi‐16  28‐B/6, Jia Sarai, Near IIT 
 
Phone: 011‐26865455/+91‐9871145498 Hauz Khas, New Delhi‐16 
              
                   Website: www.physicsbyfiziks.com                        
                           Email: fiziks.physics@gmail.com                       247 
fiziks
Institute for NET/JRF, GATE, IIT‐JAM, JEST, TIFR and GRE in PHYSICAL SCIENCES 
 
7.3 Gamma Decay

Nuclei can exist in definite energy levels just as an atom can. Due to  or -emission,
nuclei get into an excited state. These excited nuclei return to their ground state by
emitting photons whose energies correspond to energy difference between the various
initial and final states in the transition involved called -ray.
226
88 Ra

  4.61 eV 
  4.8 eV 
0.19 eV  Excited State 
  0.19eV 
222
0 eV  Ground State 
86 Rn

-rays characteristics

1. It is an electromagnetic wave.

2. Very short wavelength (  400A to 0.4A ).


o o

3. No electric charge and so not detected by magnetic and electric field.

When a beam of -rays photons passes through matter, the intensity of beam decreases
exponentially i.e. I  I 0 e   x where I 0 : Initial Intensity, µ: absorption coefficient of
substance, x : thickness of absorber.

7.3.1 Various processes by which -rays can lose its energy

Three separate processes responsible for the decrease in intensity of -rays.

1. Photoelectric absorption

In this all the energy of -ray photon is transferred to a bound electron and -ray photon
ceases to exist. The ejected electron may either escape from the absorber or may get
reabsorbed due to collision. At low photon energies (8 KeV for Al and 500 KeV for Pb)
the photoelectric effect is chiefly responsible for the -ray absorption.

                     
Head office  Branch office 
 
fiziks, H.No. 23, G.F, Jia Sarai,  Anand Institute of Mathematics, 
 
Near IIT, Hauz Khas, New Delhi‐16  28‐B/6, Jia Sarai, Near IIT 
 
Phone: 011‐26865455/+91‐9871145498 Hauz Khas, New Delhi‐16 
              
                   Website: www.physicsbyfiziks.com                        
                           Email: fiziks.physics@gmail.com                       248 
fiziks
Institute for NET/JRF, GATE, IIT‐JAM, JEST, TIFR and GRE in PHYSICAL SCIENCES 
 
2. Compton Scattering

At energies in neighborhood of 1 MeV, Compton Scattering becomes the chief cause of


removal of photons form the -ray beam.

3. Pair production

At high enough energies pair production becomes important. In this a -ray photon
passing close to an atomic nucleus in the absorbing matter disappears and an electron-

positron pair is created:   e –  e

The charge is conserved in the reaction. The rest mass m0 and hence the rest mass energy
of e- and e+ are same i.e. 0.51 MeV . The energy of the -ray photon must be at least
2  0.51 MeV  1.02 MeV for pair production to be possible. If h greater
than 1.02 MeV , the balance of the energy appears as K.E. of particles.

7.3.2 Internal Conversion

“Process of Internal Conversion is an alternative to -decay”. Internal conversion is a


process which enables an excited nuclear state to come down to some lower state without
the emission of -photon. The energy E involved in this nuclear transition gets
transferred directly to a bound electron of the atom. Such an electron gets knocked out of
the atom. Electrons like this are called “internal conversion” electrons.

This probability is highest for the K-shell electrons which are closest to the nucleus. For
such a case, the nucleus may not de-excite by -emission but by giving the excitation
energy E directly to a K-shell electron. Internal conversion is also possible (though less,
as compared to K-shell) for higher atomic shells L, M etc.

The kinetic energy of the converted electron is K e  E  Be ,

where E  Ei  E f  Nuclear excitation energy between initial state i (higher) and final

state f (lower) and Be = atomic binding energy of electron.

                     
Head office  Branch office 
 
fiziks, H.No. 23, G.F, Jia Sarai,  Anand Institute of Mathematics, 
 
Near IIT, Hauz Khas, New Delhi‐16  28‐B/6, Jia Sarai, Near IIT 
 
Phone: 011‐26865455/+91‐9871145498 Hauz Khas, New Delhi‐16 
              
                   Website: www.physicsbyfiziks.com                        
                           Email: fiziks.physics@gmail.com                       249 
fiziks
Institute for NET/JRF, GATE, IIT‐JAM, JEST, TIFR and GRE in PHYSICAL SCIENCES 
 
We know that the -spectrum is continuous; usually this continuous   spectra are
superimposed by discrete lines due to conversion electrons. These lines are called
‘internal conversion’ lines.

-ray emission and internal conversion are competing process for de-excitation of nucleus.

If we neglect the small recoil energy of the -emitter nucleus, the energy of the -ray is
given by h  E  Ei  E f ; where  is the frequency of the -photon.

7.3.3 Pair Production (Energy into matter)

In a collision a photon can give an electron all of its energy (the photoelectric effect) or
only part (the Compton Effect). It is also possible for a photon to materialize into an
electron and a positron. In this process, electromagnetic energy is converted into matter.
This process is called pair production.

No conservation principles are violated when an electron-positron pair is created near an


atomic nucleus.

The rest energy m0 c 2 of an electron or positron is 0.51 MeV , hence pair production

requires photon energy of at least 1.02 MeV . Any additional photon energy becomes K.E.
of the electron and positron.

photon
 Electron

Nucleus
 positron

                     
Head office  Branch office 
 
fiziks, H.No. 23, G.F, Jia Sarai,  Anand Institute of Mathematics, 
 
Near IIT, Hauz Khas, New Delhi‐16  28‐B/6, Jia Sarai, Near IIT 
 
Phone: 011‐26865455/+91‐9871145498 Hauz Khas, New Delhi‐16 
              
                   Website: www.physicsbyfiziks.com                        
                           Email: fiziks.physics@gmail.com                       250 
fiziks
Institute for NET/JRF, GATE, IIT‐JAM, JEST, TIFR and GRE in PHYSICAL SCIENCES 
 
7.3.4 Pair Annihilation

The inverse of pair production occurs when a positron is near an electron and the two
come together under the influence of their opposite electric charges. Both particles vanish
simultaneously with the lost mass becoming energy in the form of two gamma ray photon.

e  e     

The total mass of the positron and electron is equivalent to 1.02 MeV , and each photon
has energy h of 0.51 MeV plus half the K.E. of the particles relative to their center of
mass.

Note:

1. The directions of the photons are such as to conserve both energy and linear
momentum.

2. No nucleus or other particles is needed for this pair annihilation to take place.

7.2.5 Massbauer Effect

“It is the recoilless emission and absorption of photon”

The emission of gamma rays is generally accompanied by the emission of an  or 


particle. If after the emission of an  or  particle the product nucleus is left in an excited
state, it reaches the ground state by releasing or emitting photons called -rays. When a
nucleus emits a photon it recoils in the opposite direction. This reduces the energy of the
-ray from its usual transition energy E0 to E0  R , where R is the recoil energy.

Mossbauer Effect almost eliminates the energy of recoil by using solid state properties of
a crystal lattice. Also, such recoil-less emission of -rays makes it possible to construct a
source of essentially mono-energetic and hence monochromatic photons.

The isotope of iron, Fe57 is the most often used nucleus to study Mossbauer Effect.

                     
Head office  Branch office 
 
fiziks, H.No. 23, G.F, Jia Sarai,  Anand Institute of Mathematics, 
 
Near IIT, Hauz Khas, New Delhi‐16  28‐B/6, Jia Sarai, Near IIT 
 
Phone: 011‐26865455/+91‐9871145498 Hauz Khas, New Delhi‐16 
              
                   Website: www.physicsbyfiziks.com                        
                           Email: fiziks.physics@gmail.com                       251 
fiziks
Institute for NET/JRF, GATE, IIT‐JAM, JEST, TIFR and GRE in PHYSICAL SCIENCES 
 
7.4 Activity

The activity of a sample of any radioactive nuclide is the rate at which the nuclei of its
constituent atoms decay. If N is the number of nuclei present in the sample at a certain
time, its activity R is given by

dN
R , SI unit is Becquerel.
dt

1 Becquerel = 1 Bq = 1 decay/sec.

The traditional unit of activity is the curie (Ci),

1 Curie = 3.7 × 1010 decay/sec = 37 GBq (1Ci is activity of 1 g of radium 226


88 Ra )

Let  be the probability per unit time for the decay of each nucleus of given nuclide.

Then  dt is the probability that any nucleus will undergo decay in a time interval dt . If a
sample contains N undecayed nuclei, the number dN that decay in a time dt is

dN   N  dt

dN
R  N
dt
N0 t
dN

0
N
    dt  N  N 0 e  t
0

ln 2 0.693  N0 
Half Life: T1/ 2    At t  T1/ 2 , N  2 
   

1
Mean Time: T   1.44T1/ 2

                     
Head office  Branch office 
 
fiziks, H.No. 23, G.F, Jia Sarai,  Anand Institute of Mathematics, 
 
Near IIT, Hauz Khas, New Delhi‐16  28‐B/6, Jia Sarai, Near IIT 
 
Phone: 011‐26865455/+91‐9871145498 Hauz Khas, New Delhi‐16 
              
                   Website: www.physicsbyfiziks.com                        
                           Email: fiziks.physics@gmail.com                       252 
fiziks
Institute for NET/JRF, GATE, IIT‐JAM, JEST, TIFR and GRE in PHYSICAL SCIENCES 
 
7.4.1 Successive Growth and Decay Process
In a successive growth/decay process A  B  C , element C is a stable nucleus. The
following parameters are given for the process:
1 2
A B C
t  0 N0 0 0
t t N1 N2 N3

where 1 and 2 are decay constant, N 0 is the concentration of A at t  0 and N 1 , N 2 ,

N 3 are concentration of A, B, C at any time t . N


Thus
N1  N 0e  1t N0
C

dN 2 B
Rate equation for B:  1 N1  2 N 2
dt A

dN 2 t
  2 N 2  0 N 0 e  1t
dt

t 1 N 0 e   
2 1

Multiply both side by e 2 dt and then integrate  N 2 e2t  K


2  1
1 N 0 1 N 0   t   t
At t  0, N 2  0  K   , thus N2  e  e  1 2

2  1 2  1 
dN 3
If C is a stable nucleus, the rate of decay of atoms of B into C i.e is given by,
dt
dN 3
 2 N 2  At t  0, N3  0 
dt

  e  2t 2 e  1t 
 N 3  N 0 1  1  
  2  1   2  1  

Then the time at which concentration of intermediate member B  will reach maxima is:

 dN 2  ln  2 / 1 
  0 t'
 dt t t ' 2  1

                     
Head office  Branch office 
 
fiziks, H.No. 23, G.F, Jia Sarai,  Anand Institute of Mathematics, 
 
Near IIT, Hauz Khas, New Delhi‐16  28‐B/6, Jia Sarai, Near IIT 
 
Phone: 011‐26865455/+91‐9871145498 Hauz Khas, New Delhi‐16 
              
                   Website: www.physicsbyfiziks.com                        
                           Email: fiziks.physics@gmail.com                       253 
fiziks
Institute for NET/JRF, GATE, IIT‐JAM, JEST, TIFR and GRE in PHYSICAL SCIENCES 
 
7.4.2 Branching
A given type of of nuclei will normally decay by one particular mode; say by emission of
 -particles. But many cases have been found in which a smaller percentage of nuclei
will decay by a different mode such as  -emission.
Let us denote the probability of  -emission by one nucleus, in dime dt emission by
 dt and that of  -emission by  dt .

Then the probability of decay of a nucleus in time dt by either  or  -emission is:

     dt .

       N  N  N 0 e     .
dN    t
Hence the activity is
dt
1 
Giving mean life   and Branching Ratio  
     

7.4.3 Determination of the Age of the Earth


Let us consider successive growth decay process
238
U
U X Y P b 20 6
 S tab le P roduct,  P b  0 

The half life of U 238 is 4.5  109 Years . Hence after sufficient time the only element
present in any appreciable amount will be uranium and lead.
1 N 0   t   t
 N2  e  e 
1 2

2  1 
Here 1  U , 2  Pb  0 , N 2  N Pb and N 0  NV

Thus N Pb  NV 1  e U t 

N 0  NV  Present no. of Pb atoms+Present no. of U atoms  NV  N Pb  NU

1  N  NU 
t ln  Pb 
U  NU 

                     
Head office  Branch office 
 
fiziks, H.No. 23, G.F, Jia Sarai,  Anand Institute of Mathematics, 
 
Near IIT, Hauz Khas, New Delhi‐16  28‐B/6, Jia Sarai, Near IIT 
 
Phone: 011‐26865455/+91‐9871145498 Hauz Khas, New Delhi‐16 
              
                   Website: www.physicsbyfiziks.com                        
                           Email: fiziks.physics@gmail.com                       254 
fiziks
Institute for NET/JRF, GATE, IIT‐JAM, JEST, TIFR and GRE in PHYSICAL SCIENCES 
 
Example: Half life of P is 14.3 days. If you have 1.00 g of P today, then what would
be the amount remaining in 10 days.
0.693 0.693
Solution: t1/ 2     0.04847 per day
 14.3
N  N o e  t  N  1.00 e 0.04847 10  N  0.616 g or N  616 mg
Example: A radioactive nucleus has a half life of 100 years. If the number of nuclei
t  0 is N 0 , then find the number of nuclei that have decayed in 300 years.

Solution:
T / T1/2 3
1 1 7
Number of nuclei present after 300 year N=N o    N'  N 0 -N 0    N 0
2 2 8
238 234
Example: The atomic ratio between the uranium isotopes U and U in a mineral
sample is found to be 1.8 10 4 . The half life of U is 4.5 109 years , then find the half
238

234
life of U.
N A B T1/2 A
Solution: N A A  N B B   
N B A T1/2 B
NB 1
 T1/2 B  T1/2 A  4.5 109  2.5 105 years
NA 1.8  104

Example: A radioactive sample contains 1.00 mg of radon 222 Rn , whose atomic mass
is 222 u . The half life of the radon is 3.8 day . Then find the activity of the radon.
0.693 0.693
Solution: Decay constant     2.1 106 sec 1
T1 3.8  24  60  60
2

1.00 106 kg
Number of atoms in 1.00 mg is N   2.7  1018 atoms
 222u  1.66 10 kg / u
27

103
or N   6.023 1023 =2.7 1018 atoms
222
Hence, activity R   N  2.1106  2.7 1018  5.7 1012 decay / sec

                     
Head office  Branch office 
 
fiziks, H.No. 23, G.F, Jia Sarai,  Anand Institute of Mathematics, 
 
Near IIT, Hauz Khas, New Delhi‐16  28‐B/6, Jia Sarai, Near IIT 
 
Phone: 011‐26865455/+91‐9871145498 Hauz Khas, New Delhi‐16 
              
                   Website: www.physicsbyfiziks.com                        
                           Email: fiziks.physics@gmail.com                       255 
fiziks
Institute for NET/JRF, GATE, IIT‐JAM, JEST, TIFR and GRE in PHYSICAL SCIENCES 
 
8. Nuclear Reaction
All the nuclear reactions are of the form
x X Y  y or X  x, y  Y

The equation and notation both means that particle x strikes nucleus X to produce
nucleus Y and particle y . The particles x and y may be elementary particles or γ-rays
or they may themselves be nuclei, eg.  -particles or deuteron. In more general nuclear
X  x
X *x

equations are shown x  X  
Y  y
 Z  z

The first equation x  X  X  x represents elastic scattering in which the total K. E. of


the system, projectile plus target, is the same before the collision as after.
The second reaction x  X  X *  x represents inelastic scattering, in which the target
nucleus X is raised into an excited state X*, and the total K.E. of the system is decreased
by the amount of excited energy given to target nucleus.
The last two equations show a general nuclear equation.
8.1 Conservation Laws
In any nuclear reaction certain quantities must be conserved. The following conservation
laws hold well during a nuclear reaction.
1. Conservation of Energy 2. Conservation of Linear Momentum
3. Conservation of charges 4. Conservation of Nucleons
5. Conservation of Angular Momentum 6. Conservation of Parity
7. Conservation of spin 8. Conservation of statistics
9. Conservation of Isobaric spins
8.1.1 The Quantities not conserved
These are magnetic dipoles moments and the electrical quadrupole moment of the
reacting nuclei. These moments depends upon the internal distribution of mass, charge
and current within the nuclei involved and are not subjected to conservation laws.

                     
Head office  Branch office 
 
fiziks, H.No. 23, G.F, Jia Sarai,  Anand Institute of Mathematics, 
 
Near IIT, Hauz Khas, New Delhi‐16  28‐B/6, Jia Sarai, Near IIT 
 
Phone: 011‐26865455/+91‐9871145498 Hauz Khas, New Delhi‐16 
              
                   Website: www.physicsbyfiziks.com                        
                           Email: fiziks.physics@gmail.com                       256 
fiziks
Institute for NET/JRF, GATE, IIT‐JAM, JEST, TIFR and GRE in PHYSICAL SCIENCES 
 
8.2 Nuclear Reaction Kinematics  Q  Value 

Consider the reaction


a A Bb
Now according to Law of conservation of energy the total energy of the reactant is equal
to the total energy of the product.

M ca
2
 Ea    M Ac 2  E A    M B c 2  EB    M b c 2  Eb 

where M a , M A , M b , M B are the masses of the particles a, A, B and b respectively and

Ea , E A , Eb , EB are their respective kinetic energies.

If we suppose that the target is at rest then E A  0 ,

Q  EB  Eb  Ea   M A  M a    M B  M b   c 2

Here masses are the nuclear masses. The quantity Q is called the energy balance of the
reaction or more commonly Q -value of the reaction. Thus Q is energy appearing due to
disappearance of masses or mass defect in a nuclear reaction.

(i) If Q is  ve Q  0
The Kinetic energy of the products is greater than that of the reactants, the reaction is
then said to be exothermic or exoergic. The total mass of the reactants is greater than that
of the products in this case.

(ii) If Q is  ve Q  0
The reaction is said to be endothermic or endoergic, i.e. energy must be supplied usually
as K. E. of the incident particles.

                     
Head office  Branch office 
 
fiziks, H.No. 23, G.F, Jia Sarai,  Anand Institute of Mathematics, 
 
Near IIT, Hauz Khas, New Delhi‐16  28‐B/6, Jia Sarai, Near IIT 
 
Phone: 011‐26865455/+91‐9871145498 Hauz Khas, New Delhi‐16 
              
                   Website: www.physicsbyfiziks.com                        
                           Email: fiziks.physics@gmail.com                       257 
fiziks
Institute for NET/JRF, GATE, IIT‐JAM, JEST, TIFR and GRE in PHYSICAL SCIENCES 
 
8.2.1 General solution of Q -Eequation B

M B , EB , PB

a A

M a , Ea , Pa

b
M b , Eb , Pb
Let the particle a strike with target A and the product nuclei B and b are observed at
angle  and  to the direction of incident particle respectively.
Then applying the law of conservation of linear momentum
       
P a  P B cos   P b cos   P B cos   P a  Pb cos  and P B sin   Pb sin 

Squaring and adding above two equations PB2  Pb2  Pa2  2 Pa Pb cos  , where
   
P a , P A , P B , Pb are the momenta of the particles a, A, B and b respectively.
 
 P a  2M a Ea , P A  2M A E A
 
P B  2M B EB , P b  2M b Eb

Putting these values in the equation PB2  Pb2  Pa2  2 Pa Pb cos 

2M B EB  2M b Eb  2 M a Ea  2 2M a Ea 2M b Eb cos 

Mb M 2
 M a M b Ea Eb  cos 
1/ 2
or, EB  Eb  a Ea 
MB MB MB

But Q  EB  Eb  Ea

M   M  2
 M a M b Ea Eb  cos  ,
1/ 2
Q  Ea  a  1  Eb 1  b 
 MB   MB  MB

2  M a M b Ea 
1/ 2
 M  MB   QM B 
Eb  Eb1/ 2 cos    a  Ea   0
M B  Mb   Mb  M B   Mb  M B 
This equation is of the form x 2  2ux  v  0

                     
Head office  Branch office 
 
fiziks, H.No. 23, G.F, Jia Sarai,  Anand Institute of Mathematics, 
 
Near IIT, Hauz Khas, New Delhi‐16  28‐B/6, Jia Sarai, Near IIT 
 
Phone: 011‐26865455/+91‐9871145498 Hauz Khas, New Delhi‐16 
              
                   Website: www.physicsbyfiziks.com                        
                           Email: fiziks.physics@gmail.com                       258 
fiziks
Institute for NET/JRF, GATE, IIT‐JAM, JEST, TIFR and GRE in PHYSICAL SCIENCES 
 
 M a M b Ea  cos  QM B  Ea  M B  M a 
1/ 2

where x  Eb , u and v  .
M B  Mb  M B  Mb 
The solution of the above equation is given by

2u  4u 2  4v
x or x  u  4u 2  4v .
2

Thus Eb  u  u 2  v

The energetically possible reactions are those for which Eb is real and  ve .

8.2.2 Exothermic Reaction  Q  0 

For exothermic reaction Q  0 and these reactions are possible even for Ea  0

 QM B 
 Q  EB  Eb  Ea for Ea  0 , u  0 and v    .
 M B  Mb 
 QM B 
Thus, Eb   .
 M B  Mb 
8.2.3 Exothermic Reaction  Q  0 

All endothermic reactions have negative Q -Values.

When Ea  0 , u 2  v  ve and hence Eb is imaginary.

It means that these reactions are not possible. The smallest value of Ea at which reaction

can take place is called threshold energy. The reaction first becomes possible when Ea is
large enough to make
u2  v  0 ,
M a M b Ea cos 2  QM B  Ea  M B  M a 
  0
Mb  M B  M B  Mb
2

M a M b Ea cos 2 
  QM B  Ea  M B  M a   0
Mb  M B 
 QM B  M b  M B   M a M b Ea cos 2   Ea  M B  M a  M b  M B 

                     
Head office  Branch office 
 
fiziks, H.No. 23, G.F, Jia Sarai,  Anand Institute of Mathematics, 
 
Near IIT, Hauz Khas, New Delhi‐16  28‐B/6, Jia Sarai, Near IIT 
 
Phone: 011‐26865455/+91‐9871145498 Hauz Khas, New Delhi‐16 
              
                   Website: www.physicsbyfiziks.com                        
                           Email: fiziks.physics@gmail.com                       259 
fiziks
Institute for NET/JRF, GATE, IIT‐JAM, JEST, TIFR and GRE in PHYSICAL SCIENCES 
 
 QM B  M b  M B    M a M b Ea 1  cos    Ea  M B2  M b M B  M a M B 
2

Q  M B  M b 
 Ea 
 M aMb 2 
 M B  M b  M a  M sin  
 B 
At   0 , Ea is minimum and is the threshold energy Eth .

Q  M B  M b   M 
 Eth   Eth   1  a  Q
M B  Mb  Ma   MA 
NOTE:
Isotopic masses can be used to compute Q except in positive beta decay: (Q in mass units)
(i) 238
92 U 90
234
Th  24 He

Q  [m(U )  92me )]  [m(Th)  90me +m( He)  2me ]


Q  m(U )  [m(Th)  m( He)]

7 N  e 
C 14
14
(ii) 6

Q  m(C )  6me  [m( N )  7me  me ]


Q  m (C )  m ( N )

29 Cu  e  

(iii) 63
30 Zn 63

Q  m( Zn)  30me  [m(Cu )  29me  me ]

Q  m( Zn)  [m(Cu )  2me ]

                     
Head office  Branch office 
 
fiziks, H.No. 23, G.F, Jia Sarai,  Anand Institute of Mathematics, 
 
Near IIT, Hauz Khas, New Delhi‐16  28‐B/6, Jia Sarai, Near IIT 
 
Phone: 011‐26865455/+91‐9871145498 Hauz Khas, New Delhi‐16 
              
                   Website: www.physicsbyfiziks.com                        
                           Email: fiziks.physics@gmail.com                       260 
fiziks
Institute for NET/JRF, GATE, IIT‐JAM, JEST, TIFR and GRE in PHYSICAL SCIENCES 
 
Example: The Thermal Neutrons are captured by 5 B10 to form 5 B11 which further

decays into  - particle and 3 Li 7 . Then find the Q - value.

(Given mn  1.008665 u, mB  10.01611 u, m  4.003879 u , mLi  7.01823 u )

Solution:

5 B10  0 n1 5 B11  2 He 4     3 Li 7  Q

Q   M B  M N  M   M Li   931.5 MeV

Q  10.01611  1.008665  4.003879  7.018231  931.5  2.78 MeV  2.8 MeV

Example: Consider the nuclear reaction 7 N 14  , p  8O17 which occurred in Rutherford’s

 -range in nitrogen experiment. The mass of N 14  14.0031 u , He 4  4.0026 u ,

O17  16.9994 u and p  1.0078 u p. Then find the Q -value of the reaction.

Solution: a  A  B  b ;   7 N 14  8O17  p

Q   M A  M a    M B  M b    931.5 MeV

Q  14.003  4.0026   16.9994  1.0078    931.5 MeV

Q  0.0013  931.5 MeV  1.49 MeV


Example: A neutron beam is incident on a stationary target of fluorine atoms. The
reaction F 17  n, p  O19 has a Q -value of 4.0 MeV . Then find the lowest neutron energy

which will make this reaction possible.


Solution:
a A Bb; n  F 17  O19  p

 M  Ma   19  1 
Eth  Q  A   4   4.2 MeV
 MA   19 

                     
Head office  Branch office 
 
fiziks, H.No. 23, G.F, Jia Sarai,  Anand Institute of Mathematics, 
 
Near IIT, Hauz Khas, New Delhi‐16  28‐B/6, Jia Sarai, Near IIT 
 
Phone: 011‐26865455/+91‐9871145498 Hauz Khas, New Delhi‐16 
              
                   Website: www.physicsbyfiziks.com                        
                           Email: fiziks.physics@gmail.com                       261 
fiziks
Institute for NET/JRF, GATE, IIT‐JAM, JEST, TIFR and GRE in PHYSICAL SCIENCES 
 
8.2.3 Nuclear Fission
Nuclear fission is a process in which a heavy nucleus, after capturing a neutron splits up
into two lighter nuclei of comparable masses. The process is accompanied by release of a
few fast neutrons and a huge amount of energy in form of K.E. of fission fragments and
γ-rays.
235
92 U  0 n1  92
236
U *  92
36 Kr  56 Ba  3 0 n  Q
141 1

Energy released by 1 kg of Uranium


235
Mass of 92 U  235.035315 u , Mass of n  1.008665 u

Total initial mass  236.04398 u

56 Ba  140.9177 u , Mass of 36 Kr  91.8854 u , Mass of 3n  3.0259 u


Mass of 141 92

Total final mass  235.8290 u


Mass defect  m   initial mass- final mass  0.21498 u

Energy released i.e Q -value  m  931.5 MeV  200 MeV

Number of uranium nucleus in 1 kg of uranium  6.023  1023  1000/235


Energy released by 1 kg of uranium
 6.023  1023  1000/235  200 MeV

 5.13  10 26 MeV

                     
Head office  Branch office 
 
fiziks, H.No. 23, G.F, Jia Sarai,  Anand Institute of Mathematics, 
 
Near IIT, Hauz Khas, New Delhi‐16  28‐B/6, Jia Sarai, Near IIT 
 
Phone: 011‐26865455/+91‐9871145498 Hauz Khas, New Delhi‐16 
              
                   Website: www.physicsbyfiziks.com                        
                           Email: fiziks.physics@gmail.com                       262 
fiziks
Institute for NET/JRF, GATE, IIT‐JAM, JEST, TIFR and GRE in PHYSICAL SCIENCES 
 
8.3 Nuclear Fusion in Stars
Fusion is a process, in which the lighter nuclei fuse together and produce a heavier
nucleus. The sum of the masses of the individual light nuclei is more than would be the
mass of the nucleus formed by their fusion, and thus the fusion process result in liberation
of energy.
Stellar energy was liberated in the formation of helium from hydrogen because large
amounts of hydrogen and helium exist in the sun. Such processes are called
thermonuclear reactions because energy is liberated due to very high stellar temperature.
In order to interact two nuclei, that must have enough K.E. to permit them to overcome
the electrostatic repulsion barrier which tends to keep them apart.
The basic energy producing process in the sun is the fusion of hydrogen nuclei into
helium nuclei. This can take place in several different reaction sequences, the most
common of which, the proton-proton cycle. The total evolved energy is 24.7 MeV

per 42 He nucleus formed.

Since 24.7 MeV is 4  1012 Joules , the sun’s power output of 4  1026 Watt means each

sequence of reactions must occur 1038 times per second.


p
2 He3 p
n n
p p p p p p p p 2 He 4
1 H2 n
e 
p

 e 1 H2 n
p p p p p p

2 He3
p

Proton-Proton Cycle

                     
Head office  Branch office 
 
fiziks, H.No. 23, G.F, Jia Sarai,  Anand Institute of Mathematics, 
 
Near IIT, Hauz Khas, New Delhi‐16  28‐B/6, Jia Sarai, Near IIT 
 
Phone: 011‐26865455/+91‐9871145498 Hauz Khas, New Delhi‐16 
              
                   Website: www.physicsbyfiziks.com                        
                           Email: fiziks.physics@gmail.com                       263 
fiziks
Institute for NET/JRF, GATE, IIT‐JAM, JEST, TIFR and GRE in PHYSICAL SCIENCES 
 
Energy is given off at each step. The net result is the combination of four hydrogen nuclei
to form a helium nucleus and two positrons.
Self-sustaining fusion reactions can occur only under conditions of extreme temperature
and density. The high temperature ensures that the some nuclei have the energy needed to
come close enough together to interact, and the high density ensures that such collisions
are frequent. A further condition for the proton-proton cycles is a large reacting mass,
such as that of the sun, since much time may elapse between the initial fusion of a
particular proton and its eventual incorporation in an  -particle.
Carbon-Cycle
It also involves the combination of four hydrogen nuclei (Stars hotter than sun) to form a
helium nucleus with the evolution of energy. The net result again is the formation of an
 -particle and two positrons from four protons, with the evolution of 24.7 MeV . The
12
initial 6 C acts as a kind of catalyst for the process, since it reappears at its end.
p

4
2 e
12
6 C

13
 e
p 15
7  7

e 15
8 O 13
6 C p

14
7 

Carbon Cycle

                     
Head office  Branch office 
 
fiziks, H.No. 23, G.F, Jia Sarai,  Anand Institute of Mathematics, 
 
Near IIT, Hauz Khas, New Delhi‐16  28‐B/6, Jia Sarai, Near IIT 
 
Phone: 011‐26865455/+91‐9871145498 Hauz Khas, New Delhi‐16 
              
                   Website: www.physicsbyfiziks.com                        
                           Email: fiziks.physics@gmail.com                       264 
fiziks
Institute for NET/JRF, GATE, IIT‐JAM, JEST, TIFR and GRE in PHYSICAL SCIENCES 
 
Multiple Choice Questions (MCQ)
Q1. The radius of Ge nucleus is measured to be twice the radius of 49 Be . How many

nucleons are there in Ge nucleus?


(a) 64 (b) 72 (c) 82 (d) 86
Q2. The radius of a 64
29 X nucleus is measured to be 4.8 1013 cm . The radius of a 27
Y
12

nucleus can be estimated to be


(a) 2  1013 cm (b) 4 1013 cm
(c) 6 1013 cm (d) 8 1013 cm
16
Q3. According to the empirical observations of charge radii, a 8 X nucleus is spherical

4
and has charge radius R and a volume V   R 3 . Then the volume of the 128
Y nucleus,
54
3
is
(a) 1.5V (b) 2V (c) 6.5V (d) 8V

Q4. Assume spherical symmetry of the nucleus ZA X , where Z is atomic number and A is
mass number of the nucleus. Then the nuclear density and nuclear particle density of


nucleus is of the order of: m p  mn  1.67 1027 kg and R0  1.2 fermi 
(a) 1015 kg / m3 and 1040 nucleons / m3 (b) 1017 kg / m3 and 1044 nucleons / m3

(c) 1017 kg / m3 and 1040 nucleons / m3 (d) 1015 kg / m3 and 1044 nucleons / m3
Q5. Consider a nucleus with N neutrons and Z protons. If m p , mn and B.E. represents

the mass of the proton, the mass of the neutron and binding energy of the nucleus
respectively. Then mass of the nucleus is given by (and c is the velocity of light in free
space)
(a) Nmn  Zm p (b) Nm p  Zmn

B.E. B.E.
(c) Nmn  Zm p  (d) Nm p  Zmn 
c2 c2

                     
Head office  Branch office 
 
fiziks, H.No. 23, G.F, Jia Sarai,  Anand Institute of Mathematics, 
 
Near IIT, Hauz Khas, New Delhi‐16  28‐B/6, Jia Sarai, Near IIT 
 
Phone: 011‐26865455/+91‐9871145498 Hauz Khas, New Delhi‐16 
              
                   Website: www.physicsbyfiziks.com                        
                           Email: fiziks.physics@gmail.com                       265 
fiziks
Institute for NET/JRF, GATE, IIT‐JAM, JEST, TIFR and GRE in PHYSICAL SCIENCES 
 
Q6. If M e , M p and M H are the rest masses of electron, proton and hydrogen atom in

the ground state (with energy 13.6 eV ) respectively. Which of the following is exactly
true? ( c is the speed of light in free space)
(a) M H  M p  M e

13.6 eV
(b) M H  M p  M e 
c2
13.6 eV
(c) M H  M p  M e 
c2
13.6 eV
(d) M H  M p  M e  K , where K   or zero
c2
20
Q7. Let m p and mn be the mass of proton and neutron. M 1 is the mass of 10 Ne nucleus
40
and M 2 is the mass of a Ca nucleus. Then find the correct relation:
20

(a) M 1  10  m p  mn  , M 2 = 20  m p  mn  and M 2 = 2M 1

(b) M 1  10  m p  mn  , M 2  20  m p  mn  and M 2 = 2M 1

(c) M 1  10  m p  mn  , M 2  20  m p  mn  and M 2  2M 1

(d) M 1  10  m p  mn  , M 2  20  m p  mn  and M 2  2M 1

 
Q8. The measured mass of deuteron atom 12 H , Hydrogen atom 11H , proton  p  and  
neutrons  n  are 2.0141 u , 1.0078 u , 1.0073 u and 1.0087 u . Then the binding energy of

the deuteron nucleus is:


(a) 1.11 MeV (b) 2.22 MeV (c) 3.33 MeV (d) 4.44 MeV
238
Q9. The masses of a hydrogen atom, neutron and U atom are given by 1.0078 u ,
92

238
1.0087 u and 238.0508 u respectively. The binding energy per nucleon of 92 U nucleus is
therefore approximately equal to
(a) 6.6 MeV / nucleons (b) 7.6 MeV / nucleons
(c) 8.6 MeV / nucleons (d) 9.6 MeV / nucleons

                     
Head office  Branch office 
 
fiziks, H.No. 23, G.F, Jia Sarai,  Anand Institute of Mathematics, 
 
Near IIT, Hauz Khas, New Delhi‐16  28‐B/6, Jia Sarai, Near IIT 
 
Phone: 011‐26865455/+91‐9871145498 Hauz Khas, New Delhi‐16 
              
                   Website: www.physicsbyfiziks.com                        
                           Email: fiziks.physics@gmail.com                       266 
fiziks
Institute for NET/JRF, GATE, IIT‐JAM, JEST, TIFR and GRE in PHYSICAL SCIENCES 
 
5
Q10. Total binding energy of 2 He is approximately equal to:


where m  52 He   5.01220 u , m  1 H   1.007825 u, mn =1.008665 u 
(a) 16.5 MeV (b) 40.1 MeV
(c) 8.00 MeV (d) 27.4 MeV

20
Q11. The binding energy of the neon isotope 10 Ne is160.647 MeV . The atomic mass of

 
hydrogen atom 11H , mass of proton and neutron is 1.0078 u , 1.0073 u and 1.0087 u .
20
Then the atomic mass of 10 Ne is

(a) 18.00 u (b) 18.99 u


(c) 19.99 u (d) 20.99 u

 B.E. 
Q12. The following histogram represents the binding energy per particle   in
 A 
MeV as a function of the mass
number  A of a nucleus. A

nucleus with mass number A  180 8

fission into two nuclei of equal 6


B.E.
masses. In the process A 4
(a) 180 MeV of energy is released
2
(b) 180 MeV of energy is absorbed
40 80 A
(c) 360 MeV of energy is released 120 160 200
(d) 360 MeV of energy is absorbed

                     
Head office  Branch office 
 
fiziks, H.No. 23, G.F, Jia Sarai,  Anand Institute of Mathematics, 
 
Near IIT, Hauz Khas, New Delhi‐16  28‐B/6, Jia Sarai, Near IIT 
 
Phone: 011‐26865455/+91‐9871145498 Hauz Khas, New Delhi‐16 
              
                   Website: www.physicsbyfiziks.com                        
                           Email: fiziks.physics@gmail.com                       267 
fiziks
Institute for NET/JRF, GATE, IIT‐JAM, JEST, TIFR and GRE in PHYSICAL SCIENCES 
 
 B.E. 
Q13. The following histogram represents the binding energy per particle   in MeV
 A 
as a function of the mass number  A  of a nucleus. A nucleus with mass number A  170

fission into two nuclei of equal


masses. In the process 8
(a) 340 MeV of energy is released
6
(b) 340 MeV of energy is absorbed B.E.
A 4
(c) 360 MeV of energy is released
(d) 360 MeV of energy is absorbed 2

40 80 A
120 160 200
Q14. Binding energy per nucleon
Vs mass number curve for nuclei
8.5 Y
W , X , Y and Z is indicated on the
curve. The process that would    /  8 X
MeV 
release energy is: 7.5 W
(a) Y  2 Z
5 Z
(b) W  X  Z
(c) W  2Y 30 60 90 120
(d) X  Y  Z 

Q15. Six α-decay and four β--decay occurs before 232


90 X achieves stability. The final
product in the chain is
210 208 210 208
(a) 82 Y (b) Y
80 (c) Y
80 (d) 82 Y

Q16. A radioactive sample containing N 0 nuclei emits N α-particle per second on

N
decaying. The half life of the sample is 0.693 .
N0
N N0 N N0
(a) 0.693 (b) 0.693 (c) 1.44 (d) 1.44
N0 N N0 N

                     
Head office  Branch office 
 
fiziks, H.No. 23, G.F, Jia Sarai,  Anand Institute of Mathematics, 
 
Near IIT, Hauz Khas, New Delhi‐16  28‐B/6, Jia Sarai, Near IIT 
 
Phone: 011‐26865455/+91‐9871145498 Hauz Khas, New Delhi‐16 
              
                   Website: www.physicsbyfiziks.com                        
                           Email: fiziks.physics@gmail.com                       268 
fiziks
Institute for NET/JRF, GATE, IIT‐JAM, JEST, TIFR and GRE in PHYSICAL SCIENCES 
 
Q17. According to measurements by Rutherford and Geiger, one gram of radium 226
emits in one second 3.7 1010 alpha particles. The half life of radium is
(a) 400 years (b) 800 years
(c) 1600 years (d) 3200 years

Q18. A radioactive sample contains 3 109 kg of active gold 200 Au , whose half life
is 48 min . Then the activity of the radon sample is
(a) 55 Ci (b) 57 Ci (c) 59 Ci (d) 61 Ci

Q19. The radio isotope 14C maintains a fixed proportion in a living entity by exchanging
14
carbon with the atmosphere. After it dies exchange ceases and proportion of C
decreases continuously as 14C beta decays with half life of 5500 years . Estimate the age
of the dead tree whose present activity is 1/ 3 of initial activity.
(a) 8717 years (b) 6520 years
(c) 5500 years (d) 4500 years

Q20. A radioactive sample emits n β-particles in 2 sec . In next 2 sec it emits 0.75n

β-particles, then the mean life of the sample is  ln 2  0.693, ln 3  1.0986 

(a) 2 sec (b) 5 sec (c) 7 sec (d) 9 sec

Q21. A radioactive substance is initially absent, is formed at constant rate P nuclei per
second. If the decay constant of the nuclei formed is  , then the number of nuclei N
present after time t seconds is
P P
(a) (b) e  t
 

(c)
P

1  e   t
(d)
P

e  t
 1

                     
Head office  Branch office 
 
fiziks, H.No. 23, G.F, Jia Sarai,  Anand Institute of Mathematics, 
 
Near IIT, Hauz Khas, New Delhi‐16  28‐B/6, Jia Sarai, Near IIT 
 
Phone: 011‐26865455/+91‐9871145498 Hauz Khas, New Delhi‐16 
              
                   Website: www.physicsbyfiziks.com                        
                           Email: fiziks.physics@gmail.com                       269 
fiziks
Institute for NET/JRF, GATE, IIT‐JAM, JEST, TIFR and GRE in PHYSICAL SCIENCES 
 
Q22. In a successive growth/decay process A  B  C , element C is a stable nucleus.
The following parameters are given for the process:
1 2
A B C
t  0 N0 0 0
t t N1 N2 N3

where 1 and 2 are decay constant, N 0 is the concentration of A at t  0 and N 1 , N 2 ,

N 3 are concentration of A, B, C at any time t . Then the concentration of intermediate

member B  will be:


1 N 0  t 1 N 0   t
(a) N 2 
2  1
e  e  t 
1 2
(b) N 2 
2  1
e  e  t 
1 2

 2 N 0  t 2 N 0  t
(c) N 2 
2  1
e  e  t 
1 2
(d) N 2 
2  1
e  e t 
1 2

Q23. In a successive growth/decay process A  B  C , element C is a stable nucleus.


The following parameters are given for the process:
1 2
A B C
t  0 N0 0 0
t t N1 N2 N3

where 1 and 2 are decay constant, N 0 is the concentration of A at t  0 and N 1 , N 2 ,

N 3 are concentration of A, B, C at time t . Then the time at which concentration of

intermediate member B  will reach maxima is:


ln 1 /  2  ln  2 / 1 
(a) t '  (b) t ' 
1  2  2  1 
ln 1 2  ln 1 2 
(c) t '  (d) t ' 
1  2  2  1 

                     
Head office  Branch office 
 
fiziks, H.No. 23, G.F, Jia Sarai,  Anand Institute of Mathematics, 
 
Near IIT, Hauz Khas, New Delhi‐16  28‐B/6, Jia Sarai, Near IIT 
 
Phone: 011‐26865455/+91‐9871145498 Hauz Khas, New Delhi‐16 
              
                   Website: www.physicsbyfiziks.com                        
                           Email: fiziks.physics@gmail.com                       270 
fiziks
Institute for NET/JRF, GATE, IIT‐JAM, JEST, TIFR and GRE in PHYSICAL SCIENCES 
 
Q24. The disintegration energy is defined to be the difference in the rest energy between
the initial and final states. Consider the following process:
240
94 Pu  236
92U  24 He

The emitted α-particle has a kinetic energy 5.17 MeV . The value of the disintegration
energy is
(a) 5.26 MeV (b) 5.17 MeV (c) 5.08 MeV (d) 2.59 MeV
210
Q25. The polonium isotope 84 Po is unstable and emits a 5.30 MeV α-particle. The atomic
210
mass of 84 Po is 209.9829 u and that of 24 He is 4.0026 u , then the atomic mass of its
daughter nuclei is
(a) 203.9723 u (b) 204.9052 u
(c) 205.9754 u (d) 206.1053 u
Q26. The Thermal Neutrons are captured by 5 B10 to form 5 B11 which further decays into

 - particle and 3 Li 7 , the kinetic energy of Li is

(Given mn  1.008665 u, mB  10.01611 u, m  4.003879 u , mLi  7.01823 u )

(a) 1.78 MeV (b) 2.5 MeV (c) 1.00 MeV (d) 2 MeV

Q27. Neutrons are observed in a nuclear reaction Li 7  p, n  Be7 . Then the bombarding

energy of proton at which neutrons of zero energy is obtained, will be ( Q -value of


reaction is 1.65 MeV ):
(a) 1.7 MeV (b) 1.9 MeV (c) 2.1 MeV (d) 5.2 MeV
Q28. A nuclear decay process is given

Z XA Y A  e
Z 1

The atomic masses of X and X are 51.9648 u and 51.9571 u . Then the Q -value of
the reaction is:
(a) 2.7 MeV (b) 3.7 MeV (c) 4.7 MeV (d) 6.2 MeV

                     
Head office  Branch office 
 
fiziks, H.No. 23, G.F, Jia Sarai,  Anand Institute of Mathematics, 
 
Near IIT, Hauz Khas, New Delhi‐16  28‐B/6, Jia Sarai, Near IIT 
 
Phone: 011‐26865455/+91‐9871145498 Hauz Khas, New Delhi‐16 
              
                   Website: www.physicsbyfiziks.com                        
                           Email: fiziks.physics@gmail.com                       271 
fiziks
Institute for NET/JRF, GATE, IIT‐JAM, JEST, TIFR and GRE in PHYSICAL SCIENCES 
 
Q29. For nuclear fusion reaction to take place, which one of the following is true?
(a) Only very high temperature is required
(b) Normal temperature and comparatively high pressure is required.
(c) Very high temperature and comparatively high pressure is required.
(d) Very high temperature and very low pressure is required.
Q30. Which of the following fusion reaction give more energy? The nuclear mass of the
different nuclei is as follows
M  1 H 1   1.00783 u , M  1 H 2   2.01410 u , M  1 H 3   3.01605 u ,

M  2 He3   3.01603 u , M  2 He4   4.02603 u , M  3 Li 6   6.01512 u ,

M  e    0.00055 u

(a) 1 H 1  1 H 1  1 H 2  e   (b) 1 H 2  1 H 1  2 He3

(c) 1 H 3  2 He3  3 Li 6 (d) 1 H 2  2 He 4  3 Li 6

Numerical Answer Type Question (NAT)


208 26
Q31. The ratio of the sizes of 82 Pb and 12 Mg nuclei is approximately……….
27
Q32. If the nuclear radius of Al is 3.6 Fermi, the approximate nuclear radius of 64Cu
in Fermi is…………
Q33. The atomic masses of 42
20 Ca  41.958622 u , 41
20 Ca  40.962278 u and mass

of 10 n  1.008665 u , 11 p  1.007276 u . Then the energy needed to remove a neutron from

the nucleus of the calcium isotope 42


20 Ca is………… MeV

Q34. The atomic masses of 42


20 Ca  41.958622 u , 41
19 K  40.961825 u and mass of
1
0 n  1.008665 u , 11 p  1.007276 u . Then the energy needed to remove a proton from the

nucleus of the calcium isotope 42


20 Ca is………… MeV

                     
Head office  Branch office 
 
fiziks, H.No. 23, G.F, Jia Sarai,  Anand Institute of Mathematics, 
 
Near IIT, Hauz Khas, New Delhi‐16  28‐B/6, Jia Sarai, Near IIT 
 
Phone: 011‐26865455/+91‐9871145498 Hauz Khas, New Delhi‐16 
              
                   Website: www.physicsbyfiziks.com                        
                           Email: fiziks.physics@gmail.com                       272 
fiziks
Institute for NET/JRF, GATE, IIT‐JAM, JEST, TIFR and GRE in PHYSICAL SCIENCES 
 
Q35. The energy required to break 6 C into 3 2 He4 particle is………… MeV
12
 
(Given m  6 C12   12.0 u, m  42 He   4.0026 u )

Q36. A 280 day old radioactive substance shows an activity of 6000 dps , 140 day later
its activity becomes 3000 dps . Then the initial activity of the sample was………… dps

Q37. A radioactive sample contains 1.00 g of radim 226 Ra , whose half life is1622 years .
Then the activity of the radon sample is ………… Curie

Q38. If 92 U 235 captures a thermal neutron a releases 160 MeV and if the resulting fission

fragments have mass numbers 138 and 95 , the kinetic energy of the lighter fragment
is………… MeV

238
Q39. In the uranium radioactive series, the initial nucleus is 92 U and the final nucleus is
206
82 Pb . When the uranium nucleus decays to lead, the number of  -particles emitted

are ………. and the number of  -particles emitted are ……….

Q40. The binding energies per nucleon for deuteron  H  and


1
2
helium  2 He 4  are
1.1 MeV and 7.0 MeV respectively. The energy released when two deuterons fuse to

form a helium nucleus  2 He 4  is ………..

                     
Head office  Branch office 
 
fiziks, H.No. 23, G.F, Jia Sarai,  Anand Institute of Mathematics, 
 
Near IIT, Hauz Khas, New Delhi‐16  28‐B/6, Jia Sarai, Near IIT 
 
Phone: 011‐26865455/+91‐9871145498 Hauz Khas, New Delhi‐16 
              
                   Website: www.physicsbyfiziks.com                        
                           Email: fiziks.physics@gmail.com                       273 
fiziks
Institute for NET/JRF, GATE, IIT‐JAM, JEST, TIFR and GRE in PHYSICAL SCIENCES 
 
Multiple Select Type Questions (MSQ)
Q41. A radioactive sample emits n β-particles in 2 sec . In next 2 sec it emits 0.75n

β-particles, then which of the following statements are true  ln 2  0.693, ln 3  1.0986 

(a) Decay constant of the sample is 0.14sec 1


(b) Decay constant of the sample is 0.28sec 1
(c) Mean life of the sample is 3.5 sec
(d) Mean life of the sample is 7.0 sec
Q42. Which of the following statement is true?
(a) Six α-decay and four β--decay occurs before 232
90 Th achieves stability; the final product
208
in the chain being 82 Pb .
(b) A radioactive nucleus has a half life of 100 years. If the number of nuclei t  0 is
7
N 0 , then N 0 number of nuclei have decayed in 300 years.
8
238 234
(c) The atomic ratio between the uranium isotopes U and U in a mineral sample is
found to be 1.8 10 4 . Then the half life of 234
U and 238
U is 2.5 105 years and

4.5 109 years .


(d) A radioactive sample containing N 0 nuclei emits N α-particle per second on

N
decaying. The half life of the sample is 0.693 .
N0
Q43. A Uranium nucleus decays at rest into a Thorium nucleus and a Helium nucleus as
shown below U 235  Th 231  He 4
Which of the following is not true?
(a) Each decay product has the same kinetic energy
(b) Each decay product has the same speed
(c) The Thorium nucleus has more momentum than the Helium nucleus
(d) The Helium nucleus has more kinetic energy than the Thorium nucleus

                     
Head office  Branch office 
 
fiziks, H.No. 23, G.F, Jia Sarai,  Anand Institute of Mathematics, 
 
Near IIT, Hauz Khas, New Delhi‐16  28‐B/6, Jia Sarai, Near IIT 
 
Phone: 011‐26865455/+91‐9871145498 Hauz Khas, New Delhi‐16 
              
                   Website: www.physicsbyfiziks.com                        
                           Email: fiziks.physics@gmail.com                       274 
fiziks
Institute for NET/JRF, GATE, IIT‐JAM, JEST, TIFR and GRE in PHYSICAL SCIENCES 
 
Q44. Which of the following statement is true?
(a) Electron capture occurs more often than positron emission in heavy nuclides
(b) Proton outside the nucleus decays into neutron
(c) Neutron outside the nucleus decays into proton
(d) Positron emission leads to daughter nucleus of lower atomic number
Q45. Which of the following statement is not true regarding a negative beta decay
(a) an atomic electron is ejected
(b) an electron which is already present within the nucleus is ejected.
(c) a neutron in the nucleus decays emitting an electron.
(d) a part of the binding energy of the nucleus is converted into an electron.
Q46. From the following equations pick out the possible nuclear fusion reactions:
(a) 6 C13 1 1 6 C14  4.3 MeV

(b) 6 C12 1 1 7 N 13  2 MeV

(c) 7 N 14 1 1 8O15  7.3 MeV

(d) 92 U 235  0 n1 54 Xe140 38 Sr 94  0 n1  0 n1  200 MeV


Q47. Which of the following statement is not true regarding a nuclear fusion reaction
(a) a heavy nucleus breaks into two fragments by itself
(b) a light nucleus bombarded by thermal neutrons breaks up
(c) a heavy nucleus bombarded by thermal neutrons breaks up
(d) two light nuclei combine to give a heavier nucleus and possibly other products
Q48. Which of the following statement(s) is (are) correct?
(a) The rest mass of a stable nucleus is less than the sum of the rest masses of its
separated nucleons
(b) The rest mass of a stable nucleus is greater than the sum of the rest masses of its
separated nucleons.
(c) In nuclear fission, energy is released by fusing two nuclei of medium mass
(approximately 100 amu).
(d) In nuclear fission, energy is released by fragmentation of a very heavy nucleus.
                     
Head office  Branch office 
 
fiziks, H.No. 23, G.F, Jia Sarai,  Anand Institute of Mathematics, 
 
Near IIT, Hauz Khas, New Delhi‐16  28‐B/6, Jia Sarai, Near IIT 
 
Phone: 011‐26865455/+91‐9871145498 Hauz Khas, New Delhi‐16 
              
                   Website: www.physicsbyfiziks.com                        
                           Email: fiziks.physics@gmail.com                       275 
fiziks
Institute for NET/JRF, GATE, IIT‐JAM, JEST, TIFR and GRE in PHYSICAL SCIENCES 
 
20
Q49. Let m p be the mass of proton, mn the mass of neutron, M 1 the mass of a 10 Ne
40
nucleus and M 2 the mass of a 20 Ca nucleus. Then

(a) M 2  2 M 1 (b) M 2  2 M 1
(c) M 2  2 M 1 (d) M 1  10 mn  m p 

Q50. Which of the following statement is not true for 42


20 Ca ?

(a) Energy needed to remove a neutron is greater than to remove a proton


(b) Energy needed to remove a proton is greater than to remove a neutron
(c) Energy needed to remove a neutron and proton is same
(d) Energy needed to remove a neutron or proton cannot be predicted

                     
Head office  Branch office 
 
fiziks, H.No. 23, G.F, Jia Sarai,  Anand Institute of Mathematics, 
 
Near IIT, Hauz Khas, New Delhi‐16  28‐B/6, Jia Sarai, Near IIT 
 
Phone: 011‐26865455/+91‐9871145498 Hauz Khas, New Delhi‐16 
              
                   Website: www.physicsbyfiziks.com                        
                           Email: fiziks.physics@gmail.com                       276 
fiziks
Institute for NET/JRF, GATE, IIT‐JAM, JEST, TIFR and GRE in PHYSICAL SCIENCES 
 
Solution
Multiple Choice Questions (MCQ)

Ans.1: (b)  R  Ro  A  and given that RGe  2 RBe  Ro  AGe   2 Ro  9 


1/3 1/3 1/3
 AGe  72 .
1/ 3 1/ 3
R A   27 
Ans.2: (b) Since R  R0  A  Y   Y 
1/ 3
 
R X  AX   64 
RY 3 3
   RY  RX  RY  3.6 1013 cm  4 1013 cm
RX 4 4
4 4 4 4
Ans.3: (d) V   R 3   R03 A   R03 16; V '   R03 128  8V
3 3 3 3
Ans.4: (b)
Ans.5: (c)
Ans.6: (b)
Ans.7: (d)
Since nuclear mass is always less than their constituent particles so
M 1  10  m p  mn  and M 2  20  m p  mn 

Since B.E. of 40
20 Ca  B.E. of 20
10 Ne

  20  m p  mn  – M 2  c 2  10  m p + mn  – M1  c 2  M 2  10  m p + mn   M1   2 M1

Ans.8: (b) B.E.   ZmH  Nmn  m  12 H    931.5 MeV

 B.E.  1 1.0078  1 1.0087  2.0141  931.5 MeV

 B.E.  0.0024  931.5 MeV  2.2356 MeV


Ans.9: (b)

B.E.   ZmH  Nmn  m  238


92 U    931.5 MeV

 B.E.  92  1.0078  146  1.0087  238.0508  931.5 MeV

 B.E.  1.937  931.5 MeV  1804 MeV


B.E. 1804
   7.6 MeV / nucleons
A 238
                     
Head office  Branch office 
 
fiziks, H.No. 23, G.F, Jia Sarai,  Anand Institute of Mathematics, 
 
Near IIT, Hauz Khas, New Delhi‐16  28‐B/6, Jia Sarai, Near IIT 
 
Phone: 011‐26865455/+91‐9871145498 Hauz Khas, New Delhi‐16 
              
                   Website: www.physicsbyfiziks.com                        
                           Email: fiziks.physics@gmail.com                       277 
fiziks
Institute for NET/JRF, GATE, IIT‐JAM, JEST, TIFR and GRE in PHYSICAL SCIENCES 
 
Ans.10: (d)
21 H+3n 52 He

B.E.   2m  1 H   3m  n   m  52 He    931.5 MeV

 B.E.   2 1.007825  3  1.008665  5.01220   931.5 MeV

 B.E.   0.02944   931.5 MeV  27.4 MeV

Ans.11: (c)
160.647
m  Z   
X A   Zm 1 H 1  N mn  
Eb
931.5 MeV / u
 10 1.0078  10 1.0087  
931.5
 19.99 u

Ans.12: (c)
Since a nucleus with mass number A  180 fission into two nuclei of equal masses thus
180  90  90 .
So B.E. of the heavier nucleus is  180  4  720 MeV .
Total B.E. of the lighter nuclei is  90  6  90  6  1080 MeV .
Since product nuclei have higher B.E. so in this process energy is released
i.e  1080  720  360 MeV .

Ans.13: (a)
Since a nucleus with mass number A  170 fission into two nuclei of equal masses thus
180  85  85 .
So B.E. of the heavier nucleus is  170  4  680 MeV .
Total B.E. of the lighter nuclei is  85  6  85  6  1020 MeV .
Since product nuclei have higher B.E. so in this process energy is released
i.e  1020  680  340 MeV .

                     
Head office  Branch office 
 
fiziks, H.No. 23, G.F, Jia Sarai,  Anand Institute of Mathematics, 
 
Near IIT, Hauz Khas, New Delhi‐16  28‐B/6, Jia Sarai, Near IIT 
 
Phone: 011‐26865455/+91‐9871145498 Hauz Khas, New Delhi‐16 
              
                   Website: www.physicsbyfiziks.com                        
                           Email: fiziks.physics@gmail.com                       278 
fiziks
Institute for NET/JRF, GATE, IIT‐JAM, JEST, TIFR and GRE in PHYSICAL SCIENCES 
 
Ans.14: (c)
Let us write B.E. in MeV of both sides.
(a) Y  2 Z ; 8.5  60  510  2  5  30  300
(b) W  X  Z ; 7.5 120  900  8  90  5  30  870
(c) W  2Y ; 7.5  120  900  2  8.5  60  1020
(d) X  Y  Z ; 8  90  720  8.5  60  5  30  660
In W  2Y , product have higher B.E. than reactant. So energy will release.
Ans.15: (d)
A
Z X A 4
Z 2Y  24 He and ZA X  Y 
A
Z 1

Change in mass number  232  24  208 .


Change in atomic number after 6 α-decay  90  12  78 .
Final products mass number  82 ; 4 β--decay
0.693 N0
Ans.16: (b) R  N  N  N 0  T1 / 2   0.693
 N
Ans.17: (c)
1
Number of radium atoms in one gram of radium   6.02 1023  2.7 1021 .
226
dN 3.7  1010
Decay constant    1     1.37 1011 sec 1
N 2.7  1021
0.693 0.693 5 1010
Thus Half Life T1/2    5  1010
sec   1600 years
 1.37 1011 365  24  60  60
Ans.18: (a)
0.693 0.693
Decay constant     2.406  104 sec 1
T1 48  60
2

3 106 g
Number of atoms in 3  109 kg is N   6.023 1023 =9.04  1015 atoms
200
Hence, activity R   N  2.406 104  9.04 1015  2.18 1012 decay / sec  59 Ci

1.0 Ci  3.7 1010 decay / sec

                     
Head office  Branch office 
 
fiziks, H.No. 23, G.F, Jia Sarai,  Anand Institute of Mathematics, 
 
Near IIT, Hauz Khas, New Delhi‐16  28‐B/6, Jia Sarai, Near IIT 
 
Phone: 011‐26865455/+91‐9871145498 Hauz Khas, New Delhi‐16 
              
                   Website: www.physicsbyfiziks.com                        
                           Email: fiziks.physics@gmail.com                       279 
fiziks
Institute for NET/JRF, GATE, IIT‐JAM, JEST, TIFR and GRE in PHYSICAL SCIENCES 
 
Ans.19: (a)
 R  R0 e  t

1 R  T  R  5500  3 
The age of the dead tree is t  ln  0   1/2 ln  0   ln  8717 years
  R  0.693  R  0.693  1 
Ans.20: (c)
Let N 0 be the number of initial number of nuclei. Then

n  N 0  N 0 e2   N 0 (1  e2 )

0.75n  N 0 e 2   N 0 e2  e 2   N 0 e2 (1  e2 )

0.75n N o e 1  e

-2ë 2 
 3 
 e 2   2  2 ln 2  ln 3    0.1438sec 1
n No 1  e 2 
 4 
1
  7 sec

Ans.21: (c)
N t
dN dN 1
 P  N     dt   ln  P   N 0  t
N

dt 0
P  N 0 
P  N
 e  t  N  1  e  t 
P
 ln  P   N   ln  P    t 
P 
Ans.22: (a)
dN 2 dN 2
 1 N1  2 N 2   2 N 2  0 N 0 e  1t
dt dt

t 1 N o e   
2 1

Multiply both side by e 2 dt and then integrate  N 2 e 2t  K.


2  1
1 N 0 N
At t  0, N 2  0  K   , thus N 2  1 o e  t  e  t . 1 2

2  1 2  1

                     
Head office  Branch office 
 
fiziks, H.No. 23, G.F, Jia Sarai,  Anand Institute of Mathematics, 
 
Near IIT, Hauz Khas, New Delhi‐16  28‐B/6, Jia Sarai, Near IIT 
 
Phone: 011‐26865455/+91‐9871145498 Hauz Khas, New Delhi‐16 
              
                   Website: www.physicsbyfiziks.com                        
                           Email: fiziks.physics@gmail.com                       280 
fiziks
Institute for NET/JRF, GATE, IIT‐JAM, JEST, TIFR and GRE in PHYSICAL SCIENCES 
 
Ans.23: (b)
dN 2 dN 2
 1 N1  2 N 2   2 N 2  0 N 0 e  1t
dt dt

2t 2 t 1 N o e   
2 1

Multiply both side by e dt and then integrate  N 2 e  K.


2  1
1 N 0 N
At t  0, N 2  0  K   , thus N 2  1 o e  t  e  t 
1 2

2  1 2  1

 dN 2  ln  2 / 1 
  0t' .
 dt t t ' 2  1
Ans.24: (a)
 A4   A  240
K .E   Q  Q    KE   5.17  5.26 MeV
 A   A4  236
Ans.25: (c)
 A  210
Q  KE   5.30  5.40 MeV
 A4  206
5.40MeV
The mass equivalent of this Q-value, mQ   0.0058 u
931.5MeV / u
Hence m f  mi – m - mx  209.9829 – 4.026 – 0.0058  m f  205.9754 u

Ans.26: (c)

5 B10  0 n1 5 B11  2 He4     3 Li 7  Q

Q   M B  M N  M   M Li   931.5 MeV

Q  10.01611  1.008665  4.003879  7.018231  931.5  2.78 MeV  2.8 MeV

Energy released in the process for  - particle ( Q  0 and it is an exothermic reaction).


Q  M Li 2.78  7.018221
E    E  1.78 MeV
M Li  M  4.003879  7.018221

We know that ELi  E  Q  ELi  Q  E  2.78  1.78  1.00 MeV

                     
Head office  Branch office 
 
fiziks, H.No. 23, G.F, Jia Sarai,  Anand Institute of Mathematics, 
 
Near IIT, Hauz Khas, New Delhi‐16  28‐B/6, Jia Sarai, Near IIT 
 
Phone: 011‐26865455/+91‐9871145498 Hauz Khas, New Delhi‐16 
              
                   Website: www.physicsbyfiziks.com                        
                           Email: fiziks.physics@gmail.com                       281 
fiziks
Institute for NET/JRF, GATE, IIT‐JAM, JEST, TIFR and GRE in PHYSICAL SCIENCES 
 
Ans.27: (b)
a A Bb; p  Li 7  Be7  n

 M  Ma   7 1 
Eth  Q  A   1.65    1.9 MeV
 MA   7 
Ans.28: (d)
Q  X  Y  2me    X  Y   2me   51.9648 u  51.9571 u   931.5MeV  1.02 MeV

Q  7.17 MeV  1.02 MeV  6.2 MeV


Ans.29: (c)
Ans.30: (d)
The energy  Q  released in all fusion reaction

(a) 1 H 1  1 H 1  1 H 2  e  

Q   M  1 H 1   M  1 H 1   M  1 H 2   M  e     931.5 MeV

 Q  1.00783 u  1.00783 u  2.01410 u  0.00055 u   931.5 MeV


 Q  1.07 103   931.5 MeV  Q  0.99 MeV

(b) 1 H 2  1 H 1  2 He3

Q   M  1 H 2   M  1 H 1   M  2 He3    931.5 MeV

 Q   2.01410 u  1.00783 u  3.01603 u   931.5 MeV

 Q   5.9  103   931.5 MeV  Q  5.5 MeV

(c) 1 H 3  2 He3  3 Li 6

Q   M  1 H 3   M  2 He3   M  3 Li 6    931.5 MeV

 Q  3.01605 u  3.01603 u  6.01512 u   931.5 MeV

 Q  .01696 u   931.5 MeV  Q  15.8 MeV

                     
Head office  Branch office 
 
fiziks, H.No. 23, G.F, Jia Sarai,  Anand Institute of Mathematics, 
 
Near IIT, Hauz Khas, New Delhi‐16  28‐B/6, Jia Sarai, Near IIT 
 
Phone: 011‐26865455/+91‐9871145498 Hauz Khas, New Delhi‐16 
              
                   Website: www.physicsbyfiziks.com                        
                           Email: fiziks.physics@gmail.com                       282 
fiziks
Institute for NET/JRF, GATE, IIT‐JAM, JEST, TIFR and GRE in PHYSICAL SCIENCES 
 
(d) 1 H  2 He  3 Li
2 4 6

Q   M  1 H 2   M  2 He 4   M  3 Li 6    931.5 MeV

 Q   2.01410 u  4.02603 u  6.01512 u   931.5 MeV

 Q   0.02501 u   931.5 MeV  Q  23.3 MeV

Numerical Answer Type Question (NAT)


Ans.31: 2
1/3
R A   208 
1/3

Since R  R0  A   8
1/3 1/3
 Pb   Pb    2
RMg  AMg   26 

Ans.32: 1.33
1/3 1/3
R A   64  4
Since R  R0  A 
1/3
 Cu   Cu      1.33
RAl  AAl   27  3

Ans.33: 11.48
42
20 Ca 41
20 Ca  0 n ;
1

Total mass of the 41


20 Ca and 10 n  41.970943 u .

Mass defect m  41.970943  41.958622  0.012321 u


So, B.E. of missing neutron= m  931.5  11.48 MeV
Ans.34: 10.27
42
20 Ca 19
41
K 11 p ;

Total mass of the 41


19 K and 11 p  41.969101 u .

Mass defect m  41.969101  41.958622  0.010479 u


So, B.E. of missing proton= m  931.5  10.27 MeV .
Ans.35: 7.3

6 C12  32 He 4

Mass defect m   3  4.0026  12  u  0.0078 u  0.0078  931.5  7.2657 MeV

                     
Head office  Branch office 
 
fiziks, H.No. 23, G.F, Jia Sarai,  Anand Institute of Mathematics, 
 
Near IIT, Hauz Khas, New Delhi‐16  28‐B/6, Jia Sarai, Near IIT 
 
Phone: 011‐26865455/+91‐9871145498 Hauz Khas, New Delhi‐16 
              
                   Website: www.physicsbyfiziks.com                        
                           Email: fiziks.physics@gmail.com                       283 
fiziks
Institute for NET/JRF, GATE, IIT‐JAM, JEST, TIFR and GRE in PHYSICAL SCIENCES 
 
Ans.36: 24000
1 R0 1 R 1 R
  ln  ln 0  ln 0  R0  24000 dps .
t R 280 6000  280  140  3000

Ans.37: 1.0
0.693 0.693
Decay constant     1.36 1011 sec1
T1 1622  365  24  60  60
2

1
Number of atoms in 1.00 g is N   6.023 1023 =2.7 1021 atoms
226
Hence, activity R   N  1.36 1011  2.7 1021  3.7 1010 decay / sec  1.0 Ci
Ans.38: 95
Since the reaction is due to thermal neutron Ea  0

a A Bb;
MB 138
 Ea  Q  160   95MeV
M B  ma 138  95
Ans.39: 8 and 6

Z X A Z  2 Y A4  2 He 4 and Z X A Z 1 Y A  e 

Change in A occurs only due to   particle and change in Z occurs due to  and  both.
Let number of   particles emitted  n1 and number of   particles emitted  n2

 n1  4  238  206  32 or n1  8

For Z ,  n1  2    n2 1  92  82 or  8  2   10  n2 or n2  6

   Particle emitted are 8 and   particles emitted are 6.


Ans.40: 23.6
2  1 H 2  2 He 4

Binding energy of two deuterons  E1 E1  2  2  1.1  4.4 MeV

Binding energy of helium nucleus  E2 E2  4  7.0   28.0 MeV

 Energy released E  E2  E1  28  4.4  23.6 MeV

                     
Head office  Branch office 
 
fiziks, H.No. 23, G.F, Jia Sarai,  Anand Institute of Mathematics, 
 
Near IIT, Hauz Khas, New Delhi‐16  28‐B/6, Jia Sarai, Near IIT 
 
Phone: 011‐26865455/+91‐9871145498 Hauz Khas, New Delhi‐16 
              
                   Website: www.physicsbyfiziks.com                        
                           Email: fiziks.physics@gmail.com                       284 
fiziks
Institute for NET/JRF, GATE, IIT‐JAM, JEST, TIFR and GRE in PHYSICAL SCIENCES 
 
Multiple Select Type Questions (MSQ)
Ans.41: (a), (d)
Let N 0 be the number of initial number of nuclei. Then

n  N 0  N 0 e2   N 0 (1  e2 )

0.75n  N 0 e 2   N 0 e2  e 2   N 0 e2 (1  e2 )

0.75n N o e 1  e

-2ë 2 
 3  1
 e 2     0.1438sec 1     7 sec
n No 1  e 2 
 4  

Ans.42: (a), (b) and (c)


(a) ZA X  A 4
Y  24 He and ZA X 
Z 2 Y 
A
Z 1

24
Change in mass number  232  208  24   6 α-decay.
4
Change in atomic number after 6 α-decay  90  12  78 .
Final products mass number  82 ; 4 β--decay
T / T1/2
1
(b) Number of nuclei present after 300 year N=N o  
2
3
1 7
 N'  N 0 -N 0    N 0
2 8
N A  B T1 / 2 A 4.5  10 9
(c) N A  A  N B  B      1.8  10 4
N B  A T1 / 2 B 2.5  10 5

0.693 N0
(d) R  N  N  N 0  T1 / 2   0.693
 N
Ans.43: (a), (b) and (c)
For momentum conservation momentum of Th and He must be same and opposite.
mTh vTh  mHe vHe  vTh  vHe  mTh  mHe

1 1
Also mTh vTh2  mHe v 2He
2 2

                     
Head office  Branch office 
 
fiziks, H.No. 23, G.F, Jia Sarai,  Anand Institute of Mathematics, 
 
Near IIT, Hauz Khas, New Delhi‐16  28‐B/6, Jia Sarai, Near IIT 
 
Phone: 011‐26865455/+91‐9871145498 Hauz Khas, New Delhi‐16 
              
                   Website: www.physicsbyfiziks.com                        
                           Email: fiziks.physics@gmail.com                       285 
fiziks
Institute for NET/JRF, GATE, IIT‐JAM, JEST, TIFR and GRE in PHYSICAL SCIENCES 
 
Ans.44: (a), (c) and (d)
Mass of the neutron is more than the mass of the proton. Outside the nucleus decay of
proton into neutron will led to violation of conservation of mass and energy.
Ans.45: (a), (b) and (d)
For beta decay, n  p  e  
A neutron disrupts into a proton and a beta particle. The beta particle is ejected from the
nucleus. The beta particles are fast moving electrons.
Ans.46: (b), (c)
Nuclear fusion reaction occurs when two or lighter nuclei combine to produce a heavier
nucleus.
Ans.47: (a), (b) and (c)
Ans.48: (a), (d)
Ans.49: (c), (d)
M 1 is the mass of 10 Ne 20 nucleus, M 2 is the mass of 20 Ca 40 nucleus.

Ne means 10 protons + 10 neutrons, Ca means 20 protons + 20 neutrons.


Due to mass defect, which is necessary for binding the nucleus, mass of nucleus is always
less than the sum of masses of protons and neutrons
For 10 Ne 20 nucleus, M 1  10  m p  mn 

The mass defect is more in case of heavier nucleus where in binding energy needed is
more  20  m p  mn   M 2  B.E. for 20Ca 40

10  m p  mn   M 1  B.E.for 10 Ne20

 20  m p  mn   M 2  10  m p  mn   M 1

10  m p  mn    M 2  M 1  or M 2  M 1  10  m p  mn 

 M 1  10  m p  mn   M 2  2M1

Ans.50: (b), (c) and (d)

                     
Head office  Branch office 
 
fiziks, H.No. 23, G.F, Jia Sarai,  Anand Institute of Mathematics, 
 
Near IIT, Hauz Khas, New Delhi‐16  28‐B/6, Jia Sarai, Near IIT 
 
Phone: 011‐26865455/+91‐9871145498 Hauz Khas, New Delhi‐16 
              
                   Website: www.physicsbyfiziks.com                        
                           Email: fiziks.physics@gmail.com                       286 

You might also like